Вы находитесь на странице: 1из 533

( yz z + b = 0

y
[
y +
)] x y z



E z =

3
y

X l

2X
+ + + b =
( +


xz


y +
0 = 1 3 ,33
)]

X l
z
x y z

2
3
[

2
]
z
10 5 r r r

0
z =

l
t (n ) =n

3
x
)

l 2 3
( +
+ b = 0
3

2
X

,3 323x1 a r4ax 2 4ax 2 z


E
( =
z

2X

n ) = 0 2 x1 a
2

2,31
X

]
X 2

x 10 5 (

z )= xz y = 1 = x +
l

t = n

3

0 8
4ax 2 2ax1 x +
)]
xy x
+ + b =
l2

2 4 ax 2 ax 1x
0 10 5


y =

2 3

X 0
G

2X
2
, 3 x y z y =
8 (2 x1a )(2ax1 ) 16
x
1
PROBLEMAS RESUELTOS DE
+ x(22ax a=0 )(2ax )2,16
2 2
4,
)] xy 1 xyzy = 1 1 2 x1 a5x120a 5=40ax 2 348

X 2
2 2
10 5 x xy

3
y =
0

4,13 ( ) (4ax 2+) +


y
+G2by = 0 2 =0 10 6 + +

l2


2
= 2
x =
2 x a y (2 x1xaz ) =xy(4=ax02 )x41 ax 2 2ax1
48
[ x x y
1

=
E x 10 6
0 5 dx1 t
(
2 x1 a = 4ax

1yz +
= 2x x
y =
a 1 4 ax
(2x11 a==2 xd1)(
a 41ax2 ) 16 x 2 a = 0 2 2
xy y


1 2
=xb4ax t 2ax

0
2 + 2

ij

y = 1
xz
= x a z
G
+
+ = = 0 X1 x1 + +
[ y + 4ax2 x x =l2n1


)] y2 2 x1za 4ax 2 0 ( =02x )a2 + ( ) [ x1 x y x
t 1 y z 2
2 x a = 4 ax
2 x a 4 ax = t
MECNICA DEL MEDIO CONTINUO
dt E
1
z = = 0 x 21 1 2 2
y x(4ax 2 2ax1 3,3
( E
2
dx2 Xx yz
0 1 1=2 x1 a 4ax 2 x1 =
t
ln 1 33
z = 1
[ = d + +
xz
x =+ t = 1


X 1 2 x a z ) =2ax1
( t )( ] )
10 52 2 t (n ) = n X
16 x a =
r
0
x22 x1 a = 4ax 2
0 2t + 3
y
[
E lyn2=x22 x1 a + 4axz2 = 3
y +
)]

X 1xexp t y
E z
1 (
2

2 2xx1 =a X 2 4ax
dt ( = ln

(
x =

= 1 ,33
x (+2x1 a )2 = (,43ax 1182)2 1 exp t X3
X2

2t=+ 13
3

3
] ) ( )
2
=0
)] [ x + 2t +

z
10 5 r

1
l n x 4 ax 0 2 ax ) 3 t(

==ln4
l 2

l
(
5
G xy = y n
E
( =
2
z
,3 +2 ( 1= )(
2
2 x1 a r4axr2
1
2

l 2
z

2 3
) ( )
0

2,5 X
2

+ 1b =)0 16 x 22 a 2 = 0 +xb


X

10 25x21a = 24t4ax
r r
2 3

8 3 2xln1a = 22xax
0

,31
3

1
X

l 2 3 = = 1
0 + x

8
+
)]


1 a 4 ax

y
2X

G xy = x1 0 6 23 2 10 5

2X


1 2
(2 x1 a ) = (4ax 2x ) G xy = =
X

2 y


0 dx
X

2 = X 4,3
l

2,5

X l
2

t
48

2
l2 3

= 1
X 0
2X

1 r r
1

0
= + x
2 3

3
1 = 23x1 a 14ax 2
2 y

0+ b = 0
X 0
2X

x
X
10 6

l 2 3
dt 5

2
X1 x1 2 x a = 4ax
2

y =
ln x1 1 x + 2 xy +2 xz= 2+x1 ab+=4ax
Eduardo
G xy = x1
0xz dx W. V. Chaves 2 x1 a 4a

2X
X 2

0 x2
X 2 X

3
0

= t x x 0x= 2+1

xy
+ + 1b = 0
0

l2

y z y 4ax 2 2ax1
l

dx2 X1
x zX x =x dt
0

x xy

2 x1 a

2 3


X 0

=
2X

dt = G
ij =

y = x (

X 2 x2 xy r 1 y X1exyzp t 1
2 )(2a2
ij =

l
1

+ by = xy 0 n x1 a4ax

0 + y + xyz2 + 2
00

r
X 2

0
3

0 2t +13 x + +b = 0 + bx =0 xy 2x1xza =4tax


1

3

2

= l n
[ x =x1 y x z d
l 2 3

yt+ + X
+ b1 =20x a
x3 = X x
([ ) z4ax 2 x2(ax (2x1a=)02 =
0

x z x2=
2

2
l
2X

x y

E x X 2 = ln 2 dty 1
l


ij =

(
3

(( )
X

t +3 x
X 2

E ln 3z + b =2x0xz1 a yz 4ax
1
xz +x yz
X2
z2 2 xy0 2t +3y (2x1 a=)

y + y = 1
+ += 0b z += 0 + (yz2 x+1 aln bx)( 4

y = 1
2 3

[

X l

=2x01ax
X 0
2X

+ +
0

)] [ y y +z 22 a1
(

z
x3 = X

x

)] [
r1 rxx
0

=
3

, x yxy xz xz+==b = 0 42 ax = 2X y 2ax 2 1 z 3x X ln


y

z = y x
X 2

E y 3E
2

( (z2 x1 a 2)2= 2(4xaxa 22)t 2+


l
3

l 2 3

2 x=1 a 1 4ax 3 +3 y ( zr n + + = +

b
2X

3 t
0


l2

2
3 0
[
2
x =+0 z = 1x
( E
x ,33x 3(2 x a 3)(2ax ) 16 x2 ayz2 = 0
1 1
=

] [
z
1 5 x +t 1
z )=
0
0

1

]
4Eax 2 2ax1
( )
= n
1 r xz
+ 2
+ z
+ b =
)x2n y z2 x1 a z = 4
0
2xyz y yzyz =) = (2 x a y + )2 t= (4ax
1 0 5
[
n

( =
( )
z
)]


2 3

4a
( E ax
0
2X

+ Eby2=,y30
= (12x x=1 a 1 )(2axx 1 + )xy 16= x 221a 2,3=108 +x +
2 x1 a
0

2
z =
ij

xy + 1 2

X

)] x=z x1 xy 18 (xz
X 2

,33 = 2 x a 4ax 4ax


[

)] [ x 10y5

3 1 2 ax


)2 = (4ax )2 = G x + + 10x ++5 b = 0
0

l2

G(2 xx1yaE= 2

z
]
y 1 r 2 1
4,3 y = 2 yz y E=x 4 y 2xz1 a z =)x=4ax 2 10=52 x a t+(n4)ax
2
= n


x5
(


2 (2 x1 a )(2a

l 2
,
0


(
10 5y xy+ =1 = 8 1 +,5 1 + + b,z3z4=80
2
xy = 1

4 2

2
2

23 X
xz z

0
1
y = 1
[

3
ij =

X ll 2 3
,31

)]
2x1ax 04ax62y 0 xy5 =z 1 xy

y 10 +
0

G x2yEx1=a x1 = 4ax 2 Gz = 8 (2 x1 a )2 =
)] 1 5
1 yz
+ by = 0

+ + x 6

2
0 y dx t x =12 =xy32,=x10a +x14ax 2 0

1( [

0
=

[
xy = 1 3 G x x y x y
=xxy+d= 1E 33 1dx1 = t r (n ) y = 2,5

l 2 3
X
(

2
z 00 0l 2 2 X
z = 1 4,3
]

2X
y =dtt =xz n 4


G xyE = 1 1 Gz ) x t 0 1
X 2 l 2 3
0 + b = 0 8 10 6
0
2

x
5
x 1 2 x1 a = 4
xy = ln 2Xx
X
x
0

x111 a21 4ax


( 0y = 1 =
X3

yz
+

0 z x2 + 5z
X

2 XX

[ +x a 4 ax x l 22 X3332
)] l

0 G 2
= n
dx
2 ,31 Xx2 = t y 0
yx = 0 1=y 0 =z
0 z

1 x 2
1

xy = 1 2
+
)
E y 1d1x02xy524=ax
]
8
= 1
0

4 ax 2 ax dx t t


X l

(+ = t12dt3,323ax3x11=XX1xeXx1p1 =t2r 2dt= x1 = X + br = 0r


x z

X

G xy =X 2 x2 =z = y d1t =
2
2


l
l ij =


X 2


0
3

X 2x x2(2x1 aG )( =)(12ax10)1516
X

l
l 2 3

[
3
X 2

x 1 ex p t
3

2,5 4,
] (2 x a xy2ax ) x16 2 a(n x )2 a02 =
X 0

3 0
2X

0
l2
0 2t +
x3 = X 10 5 E 3z 48 x1ln x2 z)0 =21t+ 23 0 1x2 2 ln x0t2 2 = n ln 1 =
3 1

2X

xy = 1

(
X l

( )

(620
rx1 a=ln2), = (24ax 2 )dx 2 t
2

G xy = x1 x3y = 1 3
r0=XX
( )
23 (2 x1 a 2 3t1+)8 = (4ax22 ) X = ln X 1 t
0

0

x = 1
[
3



ij 2=
l 2 3

+ b =
0

+
( )

)]


2 3

0 dx =
( )
3 2 + x
X 0

=
0

x d
2X


1X 20 lxn5231 = 2 x1 a 4ax 2 3 ln t


2X

X1 E
[(

2
2= 1 e tx
0

t 1
x
X l

x1=a2
=

p
0 21t x xy y = xz1 x =
=+2 x1 a 4ax 2 3

=0 = G 1
1

y
x =
2

2,5 2 =
X 2

G xy = X1 x1 dt y x41,a34 4=axx24ax
l

3 x

l
[
X

x n x+ = X + 2bEx = 0x y +
l 2 3

= +

2+ =
8 3 = X 3 2 22 x=1 a2 x1 4aax X 22 2 = +
(
0

2
0

0 x2 xy = 01
l2

ij

2 2

3

ln1 0x15

24 ax
X

Xt 2+1x ln x 2=ty +1 2 E

10 6
)
2X


[ ( )[ z 2 y t+
0

=
2

y = 31 1(

=


2
X

dx2 Gt xy = x1 X1 xy t 3
X 2

3 ln

l

0 dxx + t + xz + bx == 0 xy E
3

x1 a 4ax 2 zy = x1
[ ( 3 +

= d

l2

+ +E b y y =0 x

1
0

X 2 yx2=
1 t
3

x
ij

yz
X 0

+
2X

X1 exp t

x 1x
(2
x =
y z r r
+ b = 0 x y = 1y Ez x=z z 1
[ [ ( )]
0 2t + x = 1
x1 a + 4ax 2 X x dt
[ y +
2
X 2

x3 = X G xy = 30 1 1xy ln0x2y x
l


( l n E x
( Exy = y1E x + = 3x
X 2 20 3

= ln yz + b 1 = =0
0

( )]
2

z xz= +1 yzG+ x(z+ zb


0


+ +
) ( )[
x2
y ,

dx X t
X l X 2

x = 1
2x t y2 x z 2t +X31y y = z =0
[
0

1
[
2lX32

+
=

l + =
n 3 x1 r= Xr e tz )] x y xy = z1 2,
]
2

=
5 z ) = x +
y y x
3

dt
E x
0

y+b = 01 xp
2X

X 2 x2
X

( = Exy = z 1
(

E
0

1
ij

(x2 = X 2 ,33
l 2 3

+ lnbz = 3 x Gxx =+xy1 = 2 0 5 2,3


xz 0+ t + 3 + 2 yz

z xz 20= 1
y = 1
[ y + x3 = X
[( + = 3 1 G [)]

)] 3 t + 1 10 xy5 = 1t=(n0) E =dy


,5
t 10 5
x
y z
]
r
2

2 y
3x X 2 E= ln 2t +
x y x
E y )
() ( )
n
1
X

z =
(
l

z 3 = G y = xy1 = x x =
1
(
z
ln 3 2,3
Xl 2 0 3

x x
3,33 2,G
0


2X

r= 1r = x1= 4d,3
t4yt8+
X 22 X3

5
xy =y =10 X15 01
1
[ 3 1 5
[ x
y
z b = 0 x + xy = 1 x + xy = 118 G
X


]
r
z )=
t = n
)]
d x
32

0 n 1x0y =5 21 0 E x2
E z
( )

G xy = x + x + +xyy +=xz+xzb+xG =bx20x==y X


01

( = d
l

xy
y
ij =

3
0

l 2

x1
(

2 = x
0 0

t
l2

0 G d 1
2 4 0
t + = x
2
,3 x
5x , y z ,
xyz=341 z3 = x1 1
X t
[ d 1
x x
= 1 x + 18 y y
0 x2 = xdt0+

2
xy = 1

t

)] 8
0 0

10 5 1


0 5 1 6 =X x
G xy = xyx1+xy y + y yz+yzGb yx=y 00= X1 x1E 2z dt2 d0
ij =

(
y =
G
x22t +t z
xy xy = xz2, 4

, 0 + + + b =0 = l3nd

x = 1 x dx1 yt x z
5+ b x = 0 348
[ [ 0 x 3 = X
0
1 + xy x2= 1 X 2 x2x +
y z 1 0 5 10 6 E x = 1 x y = 1 x
X x = dt
y x
y
dx2 t )] 0 y2t + X
G xy = x1
3
x G
z

y 0 yz dx t 1
y =
E x ( (
x y =
0 x2 +xz +0 +yz + + z bl+nz=bx10=xy 0=X 2 1x2
1 xz 1 yz z G =xy = xdt =
23,5 X 3
x y
+
Xx1 z x1
+1 =b y = 0
dt E
y = 1
[ [
y +
y +
z )] )]
=

x
dx2x t y y z z X 1
z = t
x3G= x 0 2 t + 3 10 5
ln
y = x E y( 3,3=33 = X 3y = x1 =x1

y z d
0 l n z 1= 1 (+ X x32,3 33
t 0 X
dx1 expt t
[
0
x 1 1
Nomenclature III

Problemas Resueltos de

Mecanica del Medio Continuo
EDUARDO WALTER VIEIRA CHAVES
IV MECNICA DEL MEDIO CONTINUO
Presentacin


Presentacion

s
eratur

Co

lo
nv

Sue
ec a
ci ic
ul
Temp

n
-di
dr
. de
fus
i n Hi
Mec
Flu

as
Vi g
jo

ras
os

Estructu Placa
s
uid
Fl

s
li do
S

PCVI y tratamiento numrico


Mov. Slido Rgido
Ecuaciones Constitutivas

Ecuaciones Fundamentales de MMC

Tensiones

Cinemtica del continuo

Tensores
VI MECNICA DEL MEDIO CONTINUO

para el Alumno
Guia

1) NO SE MEMORIZA EJERCICIO.

2) Una vez que la teora haya sido estudiada, intentar resolver los ejercicios sin mirar la
solucin. Es importante que el alumno ante un nuevo problema desarrolle la habilidad de
dar la solucin al problema con los conocimientos adquiridos.

3) Tener en cuenta que, en general, un ejercicio es un caso particular de la teora. Es muy


importante saber reconocer cuando estamos ante una aproximacin del caso general.

4) A veces, la solucin de un ejercicio se puede obtener por varios caminos. Una vez
resuelto el ejercicio, intentar verificar si existe otra forma de resolverlo.

5) Cuidado, puede haber erratas, seis crticos...


Contenido

Contenido

PRESENTACIN V
CONTENIDO VII
NOMENCLATURA IX
OPERADORES XIII
UNIDADES XIV

1 TENSORES1
1.1 EJERCICIOS RESUELTOS 1
1.1.1 Vectores, Notacin Indicial 1
1.1.2 Operaciones con Tensores de Orden Superior 9
1.1.3 Transpuesta 14
1.1.4 Simetra y Antisimetra 14
1.1.5 Cofactor. Adjunta. Traza. Tensores Particulares. Determinante 18
1.1.6 Descomposicin Aditiva de Tensores 24
1.1.7 Ley de Transformacin. Invariantes. 25
1.1.8 Autovalores y Autovectores 31
1.1.9 Representacin Espectral 38
1.1.10 Teorema de Cayley-Hamilton 42
1.1.11 Tensores Istropos y Anistropos 44
1.1.12 Descomposicin Polar 44
1.1.13 Tensor Esfrico y Desviador 45
1.1.14 Otros 46
1.1.15 Funcin de Tensores. Campo de Tensores. 47
1.1.16 Teoremas con Integrales 57
1.2 EJERCICIOS PROPUESTOS 59

2 CINEMTICA DEL CONTINUO 65


2.1 EJERCICIOS RESUELTOS 65
2.1.1 Descripcin del Movimiento, Derivada Material, Velocidad, Aceleracin 65
2.1.2 Tensores de Deformacin Finita, Deformacin Homognea 89
2.1.3 Descomposicin Polar del Gradiente de Deformacin 121
2.1.4 Deformacin Infinitesimal 142
2.2 EJERCICIOS PROPUESTOS 152

3 TENSIONES 157
3.1 EJERCICIOS RESUELTOS 157
3.1.1 Fuerza, Tensor de Tensiones, Vector Tensin 157
3.1.2 Ecuacin de Equilibro, Tensiones y Direcciones Principales 162
3.1.3 Otras Medidas de Tensin 170
3.1.4 Mxima Tensin de Corte, Crculo de Mohr 171
3.1.5 Particularidades del Tensor de Tensiones 179
3.1.6 Estado Tensional en Dos Dimensiones 192
VIII PROBLEMAS RESUELTOS DE MECNICA DEL MEDIO CONTINUO

3.1.7 Tensiones En Coordenadas Cilndricas y Esfricas 198


3.2 EJERCICIOS PROPUESTOS 202

4 LEYES FUNDAMENTALES DE LA MECNICA DEL MEDIO CONTINUO 207


4.1 EJERCICIOS RESUELTOS 207
4.2 EJERCICIOS PROPUESTOS 218

5 INTRODUCCIN A LAS ECUACIONES CONSTITUTIVAS 219


5.1 EJERCICIOS RESUELTOS 219
5.2 EJERCICIOS PROPUESTOS 224

7 ELASTICIDAD LINEAL 225


7.1 EJERCICIOS RESUELTOS 225
7.2 EJERCICIOS PROPUESTOS 280

11 FLUIDOS 283
11.1 Ejercicios Resueltos283
11.2 Ejercicios Propuestos 296
Abbreviations

Abreviaturas

PVCI Problema de Valor de Contorno Inicial


PVC Problema de Valor de Contorno
MEF Mtodo de los Elementos Finitos
MEC Mtodo de los Elementos de Contorno
MDF Mtodo de las Diferencias Finitas

Latin

i.e. id est es decir


et al. et alii y otros
e.g. exempli gratia por ejemplo
etc. et cetera y as sucesivamente
Q.E.D. Quod Erat Demonstrandum lo que se quera demostrar
v., vs. versus versus
viz. vidilicet a saber

Alfabeto griego

(a) - alfa (n) - nu


(b) - beta (o) - micron
(c) - ji (p) - pi
(d) - delta (q) - theta
(e) - psilon (r) - ro (rho)
(f) - fi (s) - sigma
(g) - gamma (t) - tau
(h) - eta (u) - ypsilon
(i) - iota (v) - sigma
(j) - fi (w) - omega
(k) - kappa (x) - xi
(l) - lambda (y) - psi
(m) - mu (z) - dseta
Operadores

Operadores

+
= parntesis de MacAuley
2
norma Euclidiana de
Tr () traza de ()
() T transpuesta de ()
() 1 inversa de ()
() T inversa de la transpuesta de ()
() sym parte simtrica de ()
() anti parte antisimtrica de ()
() esf parte esfrica de () o parte hidrosttica
() dev parte desviadora de ()
mdulo de
[[]] salto de
producto escalar
det() determinante de ()
cof () Cofactor de ;
Adj() adjunta de ()
Tr () traza de ()
: doble producto escalar
2 operador diferencial escalar (Laplaciano)
producto tensorial
grad() gradiente de
div () divergencia de
producto vectorial
I , II , III Primer, segundo y tercer invariantes del tensor
D
& Derivada material de
Dt
r Vector

Vector unitario (versor)


1 Tensor identidad de segundo orden
I Tensor identidad de cuarto orden
I sym I Parte simtrica del tensor identidad de cuarto orden
Unidades

Unidades (SI)
longitud m - metro energa, trabajo, calor J = Nm - Joules
J
masa kg - kilogramo potencia W Vatio
s
tiempo s - segundo permeabilidad m2
temperatura K - Kelvin viscosidad dinmica Pa s
m kg
velocidad flujo de masa
s m2s
m J
aceleracin flujo de energa
s2 m2s
W
energa J = Nm - Joules conductividad trmica:
mK
1
fuerza N - Newton frecuencia Hz Hertz
s
N kg
presin, tensin Pa - Pascal densidad de masa
m2 m3
J
densidad de energa
m3

Prefijo Smbolo Potencia Prefijo Smbolo Potencia


10 10
pico p 10 12 kilo k 10 3
nano 10 9 Mega M 10 6
micro 10 6 Giga G 10 9
mili m 10 3 Tera T 1012
centi c 10 2
deci d 10
XII PROBLEMAS RESUELTOS DE MECNICA DEL MEDIO CONTINUO
Nomenclatura


Notacion

r r r r m
A( X , t ) a ( X , t ) Aceleracin (configuracin de referencia)
s2
A Matriz de transformacin de base
r r m
a ( x, t ) Aceleracin (configuracin actual)
s2
B0 Medio continuo en la configuracin de referencia - t = 0
B Medio continuo en la configuracin actual - t
B Contorno de B
r r N
b( x , t ) Fuerzas msicas (por unidad de masa)
m3
b Tensor izquierdo de deformacin de Cauchy-Green,
tensor de deformacin de Finger
B Tensor de deformacin de Piola
J
B Entropa creada interiormente
sK
Manantial de entropa local por unidad de masa y por J
b unidad de tiempo kg s K
Ce Tensor constitutivo elstico Pa
[C ] Matriz elstica (notacin de Voigt) Pa
C in Tensor constitutivo inelstico Pa
c Tensor de deformacin de Cauchy
Cv Calor especfico a volumen constante
Cp Calor especfico a presin constante
c Cohesin Pa
mol
cc Concentracin
m3
C Tensor derecho de deformacin de Cauchy-Green
m
DV Deformacin volumtrica
m
D Tensor velocidad de deformacin o tensor tasa de
deformacin o tensor tasa de deformacin Euleriana o
tensor estiramiento
r
dA Diferencial de rea en la configuracin de referencia m2
r
da Diferencial de rea en la configuracin actual m2
dV Diferencial de volumen m3
XIV PROBLEMAS RESUELTOS DE MECNICA DEL MEDIO CONTINUO

E Tensor material de deformacin Green-Lagrange, tensor


m
de deformacin de Green, tensor de deformacin
Green-St. Venant m
Tensor de deformacin finita Euleriana o tensor de m
e
deformacin de Almansi m
E Mdulo de elasticidad longitudinal o mdulo de Young Pa
e i Base Cartesiana en notacin simblica
i , j, k Base Cartesiana
m
F Gradiente de deformacin
m
G Mdulo de elasticidad transversal Pa
H Tensor de deformacin de Biot
J
H Entropa total
K
r kgm 2
HO Momento angular = Js
s
m3
J Determinante del Jacobiano
m3
r m
J ( X , t) Tensor gradiente material de los desplazamientos
m
r m
j ( x, t ) Tensor gradiente espacial de los desplazamientos
m
r mol
J Tensor de difusividad
m2s
W J
K Tensor de conductividad trmica =
mK smK
K Energa cintica J
r kg m
L Cantidad de movimiento lineal
s
m
l Tensor gradiente espacial de velocidad
sm
m Masa total kg
M Tensor de tensiones de Mandel Pa
Vector unitario normal a una superficie (configuracin
n
actual)
Vector unitario normal a una superficie (configuracin
N de referencia)
r
NO Momento angular
r N
p Fuerza msicas por unidad de volumen
m3
P Primer tensor de tensiones de Piola-Kirchhoff, tensor
de tensiones nominales o tensor de tensiones Pa
Lagrangiano
p Presin media Pa
p Presin termodinmica Pa
NOTACIN XV

r r J
q( x , t ) Flujo de calor o vector del flujo no convectivo
m2s
Q Tensor ortogonal
Q Potencia calorfica J
r
r ( x, t ) Funcin escalar que describe en forma espacial el calor J
generado por las fuentes internas por unidad de masa kg s
R Tensor ortogonal de la descomposicin polar
S Segundo tensor de tensiones de Piola-Kirchhoff Pa
r J
s Flujo de entropa
kg s m 2
T Tensor de tensiones de Biot Pa
r (n ) r
t ( x , t , n ) Vector traccin (configuracin de referencia) Pa
r (N )
t0 Pseudo vector tensin (configuracin de referencia) Pa
r
T ( x, t ) Temperatura K
t Tiempo s
t0 t = 0 Tiempo inicial s
J
U& Tasa de la energa interna =W
s
J
u Energa interna especfica
kg
r r
u( x , t ) Vector desplazamiento (Euleriana) m
r r
u( X , t ) Vector desplazamiento (Lagrangiana) m
r
U( X , t Tensor derecho de estiramiento, o tensor de
estiramiento Lagrangiano, o tensor de estiramiento
material
r
V ( x, t ) Tensor izquierdo de estiramiento, o tensor de
estiramiento Euleriano, o tensor de estiramiento espacial
r r r r m
V ( X , t ) v ( X , t ) Velocidad (configuracin de referencia)
s
r r m
v ( x, t ) Velocidad (configuracin actual)
s
m rad
W Tensor spin o tensor velocidad de rotacin =
ms s
J
w int Potencia tensorial =W
s
r
X Vector posicin coordenada material m
r
x Vector posicin coordenada espacial m
1
Coeficiente de expansin trmica
K
ij Delta de Kronecker
1 , 2 , 3 Deformaciones principales
m
Alargamiento unitario
m
ijk Smbolo de permutacin, componentes del tensor Levi-
Civita
XVI PROBLEMAS RESUELTOS DE MECNICA DEL MEDIO CONTINUO

Deformacin volumtrica (para pequeas m


V
deformaciones) m
m
Tensor de deformacin infinitesimal
m
J
Entropa especfica
kg K
Mdulo de deformacin volumtrico Pa
m2
Difusividad trmica
s
m
Estiramiento
m
, Constante de Lam Pa
Coeficiente de Poisson
kg
Densidad de masa
m3
kg
S Densidad de masa de la solucin
m3
kg
f Densidad de masa del fluido
m3
r kg
0 ( x, t ) Densidad de masa en la configuracin de referencia
m3
r kg
( x, t ) Densidad de masa en la configuracin actual
m3
Tensor de tensiones de Cauchy o tensor de tensiones
Pa
verdaderas
r
N Componente normal del vector traccin Pa
r
S Componente tangencial del vector traccin Pa
m Tensin media Pa
1 , 2 , 3 Tensiones principales Pa
r
oct Tensin normal octadrica Pa
r Tensin tangencial octadrica o tensin de corte
oct Pa
octadrica
max Tensin de corte mximo Pa
Tensor de tensiones de Kirchhoff Pa
ngulo de friccin interno
J
Energa libre de Helmholtz por unidad de masa
kg
Energa libre de Helmholtz por unidad de volumen J
(densidad de energa) m3
J
( ) = e Densidad de energa de deformacin
m3
ngulo de dilatancia
Tensor tasa del tensor de rotacin material
r rad
Tensor de vorticidad
s
1 Tensores
La notacin indicial fue introducida por Einstein (1916, sec. 5),
who later jested to a friend, "I have made a great discovery in
mathematics; I have suppressed the summation sign every
time that the summation must be made over an index which
occurs twice..." (Kollros 1956; Pais 1982, p. 216).
Ref. (Wolfram MathWorld (Einstein Summation))

1.1 Ejercicios Resueltos

1.1.1 Vectores, Notacin Indicial

Ejemplo 1.1
r r
Probar que si a y b son vectores se cumple que:
(ar br ) (ar br ) = (ar ar )(br br ) (ar br )
2

Solucin:

(ar br ) (ar br )
r r 2 r r
= a b = a b sin ( 2
)
r 2 r 2 r 2 r 2 r 2 r r 2
( ) r
2 2
= a b sin 2 = a b 1 cos 2 = a b a b cos 2
r 2 r 2 r r
( 2
= a b a b cos = a b a b
r 2 r 2 r r 2
) ( )
r r r r
( ) ( )
r r 2
= (a a) b b a b
r r r 2 r r r 2
donde hemos considerado que a a = a y b b = b .

Ejemplo 1.2
r r r r
Probar que: si c = a + b , el mdulo de c puede ser expresado a travs de la siguiente relacin:
r r 2 r r r 2
c = a + 2 a b cos + b
r r
donde es el ngulo que forman los dos vectores a y b .
Solucin:
2 PROBLEMAS RESUELTOS DE MECNICA DEL MEDIO CONTINUO

Partiendo de la definicin del mdulo de un vector se cumple que:


r r
a+b
2
(
r r
= a+b ) (ar + br ) = ar ar + ar br + br ar + br br
r r r r r r 2 r r r r
Teniendo en cuenta que a a = a , b b = b y que a b = b a (conmutativo), concluimos
2

que:
r r 2 r r r r r r r r r
a+b = aa + ab + b a + b b b
r r r r 2
+ 2a b + b
2
= a r
r 2 r r r 2 a
= a + 2 a b cos + b

r r r 2 r r r 2
con lo cual demostramos que a+b = a + 2 a b cos + b . Luego es de fcil
r r r 2 r r r 2
demostracin que a b = a 2 a b cos + b .
r r 2 r r r r 2
NOTA: Partiendo de la expresin a + b = a + 2a b + b
2
podemos concluir que el
r r 2
valor a + b ser mximo cuando = 0 resultando que
r r 2 r r r r 2
a+b = a
2
+ 2a b + b = 0
r
r 2 r r r 2
r b
= a +2a b + b a r r r r
a+b = a + b
r
( r
= a + b )
2

r r r r
Luego para cualquier otro valor de 0 < 180 el valor a + b ser menor que a + b .
r r r r
luego, a + b a + b :

r r r r r r
b r r r c = a+b a + b
c = a+b
r
b
r
a

r r r r r r
De forma anloga se puede demostrar que a c + b y b a + c que es la conocida
desigualdad triangular, donde se cumple que:

a<c+b
c b<a+c
b
c<a+b

Universidad de Castilla- La Mancha Draft Por: Eduardo W. V. Chaves (2012)


Ciudad Real - Espaa
1 TENSORES 3

Ejemplo 1.3
1
Verificar si para las siguientes transformaciones () = E y () = E 2 son
2
transformaciones lineales.
Solucin:
( 1 + 2 ) = E [1 + 2 ] = E 1 + E 2 = (1 ) + ( 2 ) (transformacin lineal)

()

( 1 + 2 ) = ( 1 ) + ( 2 )

( 2 )

( 1 )

1 2 1 + 2

1
La transformacin () = E 2 se demuestra fcilmente que no es una transformacin lineal
2
ya que:
(1 + 2 ) = E[1 + 2 ]2 = E [12 + 21 2 + 22 ]
1 1
2 2
1 2 1 2 1
= E1 + E 2 + E 2 1 2
2 2 2
= ( 1 ) + ( 2 ) + E 1 2 ( 1 ) + ( 2 )

()

(1 + 2 )

(1 ) + ( 2 )
( 2 )

(1 )

1 2 1 + 2

Universidad de Castilla- La Mancha Draft Por: Eduardo W. V. Chaves (2012)


Ciudad Real - Espaa
4 PROBLEMAS RESUELTOS DE MECNICA DEL MEDIO CONTINUO

Ejemplo 1.4
Considrense los puntos A(1,3,1) , B (2,1,1) , C (0,1,3) y D(1,2,4 ) .
Se pide:

1) Encontrar el rea del paralelogramo definido por AB y AC ;

2) Encontrar el volumen del paraleleppedo definido por: AB , AC y AD ;

3) Encontrar el vector proyeccin del vector AB sobre el vector BC .
Solucin:

1) Primero se calculan los vectores AB y AC :
r
( ) ( )
a = AB = OB OA = 2i 1j + 1k 1i + 3j + 1k = 1i 4j + 0k

b = AC = OC OA = (0i + 1j + 3k ) (1i + 3j + 1k ) = 1i 2j + 2k
r

Utilizando la definicin del producto vectorial se obtiene el producto vectorial:


i j k
r r
ab= 1 4 0 = ( 8)i 2j + ( 6)k
1 2 2

El rea del paralelogramo ser igual al mdulo del vector resultante del producto
vectorial:
r r
A = a b = (8) 2 + (2) 2 + ( 6) 2 = 104 (unidades cuadradas)


2) Calculando vector AD :
r
( ) (
c = AD = OD OA = 1i + 2j + 4k 1i + 3j + 1k = 0i 1j + 3k )
Utilizando la definicin:
r r r r r r
V (a, b, c ) = c a b ( ) = (0i 1j + 3k ) ( 8i 2j 6k )
= 0 + 2 18 = 16 (unidades cbicas)

3) A continuacin calculamos el vector BC :

( ) (
BC = OC OB = 0i + 1j + 3k 2i 1j + 1k = 2i + 2j + 2k )

Luego el vector proyeccin de AB sobre BC viene dado por:

proj

AB =

BC AB


BC =
( 2i + 2j + 2k ) (1i 4j + 0k ) ( 2i + 2j + 2k )

BC
BC
1
42 4
BC
3

( 2i + 2j + 2k ) ( 2i + 2j + 2k )
2
BC

=
( 2 8 + 0 ) ( 2i + 2j + 2k )
(4 + 4 + 4 )

Universidad de Castilla- La Mancha Draft Por: Eduardo W. V. Chaves (2012)


Ciudad Real - Espaa
1 TENSORES 5

5 5 5
proj
AB = i j k
BC 3 3 3

Ejemplo 1.5
Reescribir en notacin indicial las siguientes expresiones:
1) a1 x1 x 3 + a 2 x 2 x 3 + a 3 x3 x 3
Solucin: a i xi x 3 (i = 1,2,3)
2) x1 x1 + x2 x2
Solucin: xi x i (i = 1,2)

a11 x + a12 y + a13 z = b x



3) a 21 x + a 22 y + a 23 z = b y

a 31 x + a 32 y + a 33 z = b z
Solucin:
a11 x1 + a12 x 2 + a13 x 3 = b1 a1 j x j = b1

ndice ndice

a 21 x1 + a 22 x 2 + a 23 x 3 = b2
mudo j
a 2 j x j = b2 libre
i a ij x j = bi
a x + a x + a x = b
31 1 32 2 33 3 3 a 3 j x j = b3

Ejemplo 1.6
a) Demostrar que: 3 p v p = v3 ;
b) Demostrar que: 3i A ji = A j 3 ;
c) Obtener el resultado de ij ijk ;
d) Obtener el resultado de i 2 j 3 Aij .
Solucin:
Las componentes de la delta de Kronecker son:
11 12 13 1 0 0
ij = 21 22 23 = 0 1 0 (1.1)
31 32 33 0 0 1
a) La expresin ( 3 p v p ) no tiene ndice libre, luego el resultado es un escalar:
3 p v p = 31v1 + 32 v 2 + 33 v 3 = v3 (1.2)
b) La expresin 3i A ji tiene un ndice libre ( j ), luego el resultado es un vector:
3i A ji = 31 A j1 + 32 A j 2 + 33 A j 3 = A j 3 (1.3)
c) La expresin ij ijk tiene un ndice libre ( k ), luego el resultado es un vector:

Universidad de Castilla- La Mancha Draft Por: Eduardo W. V. Chaves (2012)


Ciudad Real - Espaa
6 PROBLEMAS RESUELTOS DE MECNICA DEL MEDIO CONTINUO

ij ijk = 1 j 1 jk + 2 j 2 jk + 3 j 3 jk
123 1424 3 123
1111k + 21 21k + 31 31k
+ + + (1.4)
12 12 k + 22 22 k + 32 32 k
+ + +
13 13k + 23 23k + 33 33k

luego, ij ijk = 0 k (vector nulo).


d)
i 2 j 3 Aij = A23 (1.5)

Ejemplo 1.7
Expandir la expresin: Aij x i x j (i, j = 1,2,3)

Solucin: Los ndices i, j son ndices mudos (indican suma), no hay ndice libre, y como
resultado tenemos un escalar. Expandimos primero el ndice mudo i y a continuacin el
ndice j , resultando as:
Aij x i x j expandiend
oi
A1 j x1 x j + A2 j x 2 x j + A3 j x 3 x j
1424 3 1 424 3 1 424 3
A11 x1 x1 A21 x 2 x1 A31 x 3 x1
+ + +
expandiendo j

A12 x1 x 2 A22 x 2 x 2 A32 x 3 x 2


+ + +
A13 x1 x 3 A23 x 2 x 3 A33 x 3 x 3
Reagrupando los trminos anteriores obtenemos:
Aij x i x j = A11 x1 x1 + A12 x1 x 2 + A13 x1 x3 + A21 x 2 x1 + A22 x 2 x 2 +
A23 x 2 x3 + A31 x3 x1 + A32 x 3 x 2 + A33 x 3 x3

Ejemplo 1.8
Desarrollar las siguientes expresiones y obtener el valor numrico correspondiente:
1) ii jj
Solucin: ii jj = ( 11 + 22 + 33 )( 11 + 22 + 33 ) = 3 3 = 9
2) 1 1
Solucin: 1 1 = 1 1 = 11 = 1
NOTA: Observar que es incorrecto hacer la siguiente operacin
1 1 = 3 11 = 1 , ya que lo que se reemplaza es el ndice repetido

Universidad de Castilla- La Mancha Draft Por: Eduardo W. V. Chaves (2012)


Ciudad Real - Espaa
1 TENSORES 7

Ejemplo 1.9
a) Probar que a) ijk pjk = 2 ip ; b) ijk ijk = 6 c) ijk a j a k = 0 i ; d) Obtener el valor numrico
de la siguiente expresin ijk 2 j 3k 1i .
Solucin:
a) Utilizando la expresin:
ijk pqk = ip jq iq jp
y haciendo q = j , resulta:
ijk pjk = ip jj ij jp
= ip 3 ip = 2 ip
b) Partiendo del resultado anterior, es trivial la siguiente comprobacin:
ijk ijk = 2 ii = 6
c) Observemos que ijk = ikj , es decir, es antisimtrico en jk y observemos que a j a k
resulta un tensor de segundo orden simtrico. Como sabemos el doble producto escalar de un
tensor simtrico y otro antisimtrico es cero luego:
r r
ijk a j a k = ijk (a a ) jk = 0 i
r r
= (a a ) i = 0 i
d) ijk 2 j 3k 1i = 123 = 1

Ejemplo 1.10
Obtener el valor de las siguientes expresiones:
a) ijk i1 j 2 3k
b) ijk pqk = ip jq iq jp para los siguientes casos
b.1) i = 1, j = q = 2, p = 3
b.2) i = q = 1, j = p = 2
c) ( ijk A jp c p A kq c q + i1 )( ist A sa c a A tb c b + i1 )
donde ijk es el smbolo de permutacin y ij es la Delta de Kronecker.
Solucin: ijk = 1
a) ijk i1 j 2 3k = 123 = 1 1
12 k 32 k = 121 321 + 122 322 + 123 323 =
b.1) ijk = 1
= 0 (1) + 0 0 + 0 0 = 0
3 2
12 k 21k = 121 211 + 122 212 + 123 213
b.2)
= 0 0 + 0 0 + 1 (1) = 1
c) Observemos que la operacin A jp c p = b j resulta un vector y verificamos tambin que
r r
ijk A jp c p A kq c q = [( A c) ( A c )]i = (b b) i = 0 i , con lo cual resulta que:
r r

Universidad de Castilla- La Mancha Draft Por: Eduardo W. V. Chaves (2012)


Ciudad Real - Espaa
8 PROBLEMAS RESUELTOS DE MECNICA DEL MEDIO CONTINUO

( ijk A jp c p A kq c q + i1 )( ist A sa c a A tb c b + i1 ) = (0 i + i1 )(0 i + i1 ) = i1 i1 = 11 = 1

Ejemplo 1.11
r
Escribir en notacin indicial: a) el mdulo del vector a ; b) cos , donde es el ngulo que
r r
forman los vectores a y b .
Solucin:
r
r r r
= a a = a i e i a j e j = a i a j ij = a i a i = a j a j
2
a a = ai ai
r
luego, tambin cumple que b = b i b i .
r r r r
Por definicin a b = a b cos , donde:
r
a b = a i e i b j e j = a i b j ij = a i b i = a j b j

Teniendo en cuenta que un ndice no puede aparecer ms que dos veces en un trmino de la
expresin, podemos expresar cos como:
r r
ab a jb j
cos = r r =
a b ai ai b k b k

Ejemplo 1.12
Demostrar la desigualdad de Schwarz:
r r r r
ab a b Desigualdad de Schwarz (1.6)

Solucin:
Consideremos un escalar , y la siguiente operacin:
r r 2 r r r r
a b = (a b) (a b) 0
r r r r r r r r
= a a 2 a b b a + b b 0
r 2 r r r 2
= a 2 2a b + b 0
r r
r 2 2 r r r 2 ab
f ( ) = a 2a b + b 0 , si ahora consideramos el valor de = r 2 podemos
a
obtener que:
r r r r 2 r r
(a b) r 2 (a b ) r r ( a b) r 2
f = r 2 = a r 2 2(a b) r 2 + b 0
a a a

r r r r r r r r
r 2 ( a b) 2 r r (a b) r 2 (a b) 2 ( a b) 2 r 2
= a r 4 2 ( a b) r 2 + b = r 2 2 r 2 + b 0
a a a a
r r 2
(a b ) r 2
= r 2 + b 0
a

Universidad de Castilla- La Mancha Draft Por: Eduardo W. V. Chaves (2012)


Ciudad Real - Espaa
1 TENSORES 9

r r 2
r 2 (a
b r 2
b)
r 2 r 2 r r
a b ( a b) 2
r r r r
a b ab
a
c.q.d.
Solucin alternativa
r r r r r r
Teniendo en cuenta que 0 cos 1 la relacin a b = a b cos a b se cumple, luego
r r r r
concluimos que a b a b .

Ejemplo 1.13
(r r ) (r r )
Escribir la siguiente relacin a b c d sin emplear el producto vectorial.
(r r )
Solucin: Observemos que el producto vectorial a b lo podemos expresar de la siguiente
(r r )
forma: a b = a j e j b k e k = ijk a j b k e i , cuyo resultado ser un vector. De esta forma hemos
utilizado la definicin del smbolo de permutacin. Anlogamente podemos expresar el
r r
( ) r r
( )
producto vectorial c d como c d = nlm c l d m e n , por lo tanto:
(ar br ) (cr dr ) = ( nlm c l d m e n )
ijk a j b k e i )
= ijk nlm a j b k c l d m e i e n
= ijk nlm a j b k c l d m in
= ijk ilm a j b k c l d m

Teniendo en cuenta que ijk ilm = jki lmi y aplicando la relacin


jki lmi = jl km jm kl = jki ilm , concluimos que:
ijk ilm a j b k c l d m = ( jl km jm kl ) a j b k c l d m = a l b m c l d m a m b l c l d m

Puesto que el subndice mudo indica el producto escalar: a l c l = (a c ) y b m d m = b d , luego:


r r
(r r )
(ar br ) (cr dr ) = (ar cr ) (br dr ) (ar dr )(br cr )
r r r r
Observemos que, cuando c = a y d = b obtenemos que:
(ar br ) (ar br ) = (ar ar ) (br br ) (ar br )(br ar ) = (ar ar ) (br br ) (ar br )
2

Que es la misma expresin obtenida en el Ejemplo 1.1.


Ejemplo 1.14
Probar que
a) ijk a i a j b k = 0
r r
b) ijk (a k b 3 i1 j 2 + a j b 2 i1 k 3 + a i b1 j 2 k 3 ) = a b
c) Aij A ji es un invariante
Solucin:
a) ijk a i a j b k = ij1a i a j b1 + ij 2 a i a j b 2 + ij 3 a i a j b 3 . Para el trmino ij1a i a j b1 tenemos que:

Universidad de Castilla- La Mancha Draft Por: Eduardo W. V. Chaves (2012)


Ciudad Real - Espaa
10 PROBLEMAS RESUELTOS DE MECNICA DEL MEDIO CONTINUO

ij1a i a j b1 = 1 j1 a1 a j b1 + 2 j1a 2 a j b1 + 3 j1 a 3 a j b1
= 111 a1 a1b1 + 211 a 2 a1b1 + 311 a 3 a1b1 +
+ 121 a1a 2 b1 + 221 a 2 a 2 b1 + 321 a 3 a 2 b1 +
+ 131 a1a 3b1 + 231 a 2 a 3 b1 + 331 a 3 a 3 b1
= 321 a 3 a 2 b1 + 231 a 2 a 3 b1 = a 3 a 2 b1 + a 2 a 3b1
=0
Anlogamente para los trminos ij 2 a i a j b 2 = ij 3 a i a j b 3 = 0 . Es interesante observar que
ijk a i a j b k representa el determinante con dos filas iguales:
a1 a2 a3
ijk a i a j b k = a1 a2 a3 = 0
b1 b 2 b3
b)
ijk a k b 3 i1 j 2 + ijk a j b 2 i1 k 3 + ijk a i b1 j 2 k 3 =
r r
12 k a k b 3 + 1 j 3 a j b 2 + i 23 a i b1 = a 3b 3 + a 2 b 2 + a1b1 = a i b i = a b

Ejemplo 1.15
(r r ) (r r ) r (r
Probar que: a b c d = c d a b d c a b
r r
) r (r r r
)
Solucin: Expresaremos en notacin indicial el segundo miembro de la expresin:

[
r r r
cr d (a


] [
rr r r
] [ (
b) d c (a b) = c p d i ijk a j b k d p c i ijk a j b k
p
)] [ ( )]
ijk a j b k c p di ijk a j b k c i d p ijk a j b k (c p di c i d p )
Si utilizamos la propiedad de la delta de Kronecker:
ijk a j b k ( ni im c m d n np )
pm c m d n ( ijk a j b k )c mdn ( ni im np )
pm

y si consideramos que pm ni im np = pil mnl . Reemplazamos en la expresin anterior y


obtenemos:
( )
ijk a j b k c m d n pil mnl ( )
pil ijk a j b k ( mnl c m d n ) [( ) ]
r r
( )
Dado que las componentes de a b son ijk a j b k y las componentes de c d son
r r
( )
mnl c m d n , obtenemos que:

[(r r ) (r r )]
pil [( ijk a j b k ) ( mnl c m d n )] = a b c d p

Ejemplo 1.16
r r r
a) Si a , b , c son vectores linealmente independientes y que se cumple que:
r r r r
v = a + b + c componente
s v i = a i + b i + c i 0 i
Probar que los escalares , , son dados por:

Universidad de Castilla- La Mancha Draft Por: Eduardo W. V. Chaves (2012)


Ciudad Real - Espaa
1 TENSORES 11

ijk v i b j c k ijk a i v j c k ijk a i b j v k


= ; = ; =
pqr a p b q c r pqr a p b q c r pqr a p b q c r
b) Dados tres vectores linealmente independientes, demostrar que al intercambiar 2 filas 2
r r r
columnas el signo del determinante a (b c ) cambia.
r r r
Solucin: a) Haciendo el producto escalar del vector v por el vector ( b c ) obtenemos que:
r r r r r r r r r r r r
v (b c ) = a (b c ) + b (b c ) + c (b c )
14243 14243
=0 =0
Obtenemos entonces el valor de como:
r r r
v (b c )
= r r r
a (b c )
En componentes:
v1 v2 v3 v1 b1 c1
b1 b2 b3 v2 b2 c2
c1 c2 c3 v3 b3 c3 ijk v i b j c k
= = =
a1 a2 a3 a1 b1 c1 pqr a p b q c r
b1 b2 b3 a2 b2 c2
c1 c2 c3 a3 b3 c3

Anlogamente podemos obtener los parmetros , , es decir, hacemos el producto escalar


r r r r r
del vector v por los vectores a c y a b , respectivamente, i.e.:
r r r r r r r r r r r r
v (a c ) = a (a c ) + b ( a c ) + c ( a c )
14243 14243
=0 =0
r r r r r r
v (a c ) ijk v i a j c k jik a j v i c k a (v c)
= r r r = = = r r r
b (a c ) pqr b p a q c r qpr a q b p c r a (b c )
r r r r r r r r r r r r
v (a b ) = a ( a b ) + b ( a b ) + c (a b )
14243 14243
=0 =0
r r r r r r
v (a b) ijk v i a j b k jki a j b k v i a (b v )
= r r r = = = r r r
c (a b) pqr c p a q b r qrp a q b r c p a (b c )
r
NOTA 1: Podemos reestructurar las componentes del vector v de la siguiente forma:
v 1 a1 b1 c 1 a1 b1 c 1 z1

v i = v 2 = a 2 b2 c 2 = a 2 b2 c 2 z 2 = B ij z j
v a b3 c 3 a 3 b3 c 3 z 3
3 3
donde hemos denotado por z1 = , z 2 = , z 3 = . Teniendo en cuenta que:
v1 b1 c1 a1 v1 c1
v2 b2 c2 a2 v2 c2
vb c v b3 c3 B (1) ijk a i v j c k a3 v3 c3 B (2)
=z1 = ijk i j k = 3 = ; =z 2 = = =
pqr a p b q c r a1 b1 c1 B pqr a p b q c r a1 b1 c1 B
a2 b2 c2 a2 b2 c2
a3 b3 c3 a3 b3 c3

Universidad de Castilla- La Mancha Draft Por: Eduardo W. V. Chaves (2012)


Ciudad Real - Espaa
12 PROBLEMAS RESUELTOS DE MECNICA DEL MEDIO CONTINUO

a1 b1 v1
a2 b2 v2
ab v a b3 v3 B (3)
=z 3 = ijk i j k = 3 =
pqr a p b q c r a1 b 1 c1 B
a2 b2 c2
a3 b3 c3

donde B (i ) es el determinante de la matriz resultante al reemplazar la columna (i) de la


r
matriz B por las componentes del vector v . Con eso, podemos decir que:
B (i )
Dado v i = B ij z j zi = Regla de Cramer
B

NOTA 2: Aunque hemos demostrado para una matriz 3 3 , este procedimiento es vlido
para matrices de n-dimensiones y es conocido en la literatura como Regla de Cramer.
NOTA 3: La solucin ( z i ) solo es posible si B 0 .

NOTA 4: Si v i = 0 i tenemos que B ij z j = 0 i y B (i ) = 0 i , con eso, segn la regla de Cramer


tenemos que:
z i B = B (i ) = 0 i

Notar que, la solucin non-trivial z i 0 i solo es posible si y solo si B = 0 , (ver Ejemplo


1.51).
r r r r r r
b) El determinante definido por a (b c ) = [a, b, c ] en notacin indicial queda ijk a i b j c k ,
adems sabiendo que se cumple que:
ijk = jki = kij
ijk = ikj = kji = jik
i

k j

r r r r r r r r r
ijk a i b j c k = [a, b, c ] = ikj a i b j c k = [a, c , b] = jki a i b j c k = [b, c, a]
Luego
a1 a2 a3 a1 a2 a3
ijk a i b j c k = b 1 b 2 b 3 = c 1 c2 c 3 = ikj a i b j c k
c1 c2 c3 b1 b2 b3

b1 b2 b3
= c1 c2 c 3 = jki a i b j c k
a1 a2 a3

Universidad de Castilla- La Mancha Draft Por: Eduardo W. V. Chaves (2012)


Ciudad Real - Espaa
1 TENSORES 13

Ejemplo 1.17
Probar las relaciones:
r r r r r r r r r r r r r r
( ) (
a (b c ) = (a c ) b a b c = b c c b a
r r
)
r r r
a (b a) = [(a a)1 a a] b
r r r

r r
(
Solucin: Representando el producto vectorial b c i = ijk b j c k , luego: )
[ar (br cr )]r = rsi a s ( ijk b j c k )
= rsi ijk a s b j c k = rsi jki a s b j c k
( )
= rj sk rk sj a s b j c k
= rj sk a s b j c k rk sj a s b j c k = asb r c s asb s c r
= ak br c k a jb j c r = (b r c s b s c r )a s
r r r r
( )
= b r (a c ) c r a b [(
r r r r r
= b c c b a r ) ]
[
r r r rr r
= b(a c ) c a b r ( )]
Comprobando que:
r r r
( )
r r r r r r r r r r r
( ) (
a b c = (a c ) b a b c = b c c b a )
r r
En el caso particular cuando a = c podemos decir que:

[ (
r r r
)]
a b a r = (a k a k )b r (a j b j )a r = (a j a j )b p rp (a j b p jp )a r
[ ] [
= (a j a j ) rp (a j jp )a r b p = (a j a j ) rp a p a r b p ]
{ r r r
= [(a a)1 a a] b r
r r
}
Ejemplo 1.18
Demostrar la identidad de Jacobi:
(
r r r r r r r r r r
)
a b c + b (c a) + c a b = 0 ( )
r
Solucin: A travs del ejercicio anterior demostramos que a b c = (a c ) b a b c , luego, (r r ) r r r
(r r ) r
tambin es vlido que:
r r r
b (c a) = (br rar )cr (br cr )arr
(
r r r
c ab = ) (cr b)ar (cr ar )b
Luego, teniendo en cuenta que el producto escalar entre dos vectores es conmutativo, es decir,
r r r r
( ) ( )( ) ( )
r r r r r r r r
(a c ) = (c a) , a b = b a , b c = c b , concluimos que:
(a c )b (a b )c
r r r r r r
+
(
r r r r
)
r r r r r r r
a b c + b (c a) + c a b = b a ( ) ( )cr + (br cr )ar = 0r
r r
c b ( )ar (cr ar )br
1.1.2 Operaciones con Tensores de Orden Superior

Universidad de Castilla- La Mancha Draft Por: Eduardo W. V. Chaves (2012)


Ciudad Real - Espaa
14 PROBLEMAS RESUELTOS DE MECNICA DEL MEDIO CONTINUO

Ejemplo 1.19
Cul es el orden de los tensores representados por sus componentes: v i , ijk , Fijj , ij , C ijkl ,
ij ? Determinar cuantas componentes independientes tiene el tensor C .

Solucin: El orden del tensor viene dado por el nmero de subndices libres, luego:
r r
Tensores de orden uno: v , F
Tensores de segundo orden: ,
Tensor de tercer orden:
Tensor de cuarto orden: C
El nmero de componentes de un tensor viene dado por el mximo valor del rango del
subndice, 3 si ( i = 1,2,3 ), elevado al nmero de subndices libres. Es decir, para el tensor de
cuarto orden, el nmero de ndices libres es 4, luego:
3 4 = (i = 3) ( j = 3) (k = 3) (l = 3) = 81
El tensor de cuarto orden C ijkl tiene 81 componentes independientes.

Ejemplo 1.20
(r r ) r (r r ) r
Demostrar que a) a b c = b c a ; b) a b c d = b c a d(r r ) (r r ) (r r ) r r

Solucin:
(ar br ) cr = (a e
i i b j e j ) c k e k
= a i e i b j c k jk
a)
= (b k c k )a i e i
r r r r r r
= (b c )a (b c ) a
r r r r
( )( )
b) La expresin a b c d , que resulta un tensor de segundo orden, expresamos
directamente en notacin indicial:

[(ar br ) (cr dr )] ij ( )
= (a i b k ) c k d j = a i b k c k d j
= b k c k ai d j
r r r
= (b k c k )(a i d j ) = (b c )(a d) ij
123
escalar

Ejemplo 1.21
Desarrollar y simplificar lo posible la expresin A ij xi x j para los siguientes casos:
a) A ij = A ji
b) A ij = A ji
Solucin:
Expandiendo A ij xi x j obtenemos:

Universidad de Castilla- La Mancha Draft Por: Eduardo W. V. Chaves (2012)


Ciudad Real - Espaa
1 TENSORES 15

A ij xi x j = A 1 j x1 x j + A 2 j x 2 x j + A 3 j x3 x j =
= A 11 x1 x1 + A 21 x 2 x 1 + A 31 x3 x 1 +
(1.7)
A 12 x1 x 2 + A 22 x 2 x 2 + A 32 x 3 x 2 +
A 13 x1 x 3 + A 23 x 2 x 3 + A 33 x3 x 3

a) A ij = A ji (simetra)

A ij xi x j = A 11 x12 + 2 A 12 x1 x 2 +2 A 13 x1 x 3 +
(1.8)
A 22 x 22 + 2 A 23 x 2 x 3 + A 33 x32

b) A ij = A ji (antisimetra)

A ij xi x j = 0 (1.9)
lo que era de esperar ya que:
r r r r
A ij xi x j = x A x = A : ( x x ) (1.10)
r r
Si A antisimtrico y ( x x ) resulta simtrico, el doble producto escalar de un tensor
simtrico y uno antisimtrico resulta ser siempre igual a cero.

Ejemplo 1.22
Si las componentes de los tensores de segundo orden y T son representadas
respectivamente por:
5 2 4 3 1 2
ij = 1 2 1 ; Tij = 4 2 1 (1.11)
4 3 6 1 3 8

Obtener T : .
Solucin:
T : = Tij ij (1.12)
Tij ij = T1 j 1 j + T2 j 2 j + T3 j 3 j
123 123 123
T1111 + T21 21 + T31 31
+ + + (1.13)
T12 12 + T22 22 + T32 32
+ + +
T13 13 + T23 23 + T33 33

luego,
Tij ij = 5 3 + 2 1 + 4 2 + (1) 4 + 2 2 + 1 1 + 4 1 + 3 3 + 6 8 = 87 (1.14)

Ejemplo 1.23
Dadas las componentes del tensor B en el sistema de coordenadas cartesianas:

Universidad de Castilla- La Mancha Draft Por: Eduardo W. V. Chaves (2012)


Ciudad Real - Espaa
16 PROBLEMAS RESUELTOS DE MECNICA DEL MEDIO CONTINUO

3 2 4
B ij = 1 5 3 (1.15)
5 7 9

Obtener:
a) C ij = B ik B kj ; b) D ij = B ik B jk ; c) E ij = B ki B kj ; d) C ii , D ii , E ii
Solucin:

3 2 4 3 2 4 31 44 54
C = B B C ij = B ik B kj = 1 5 3 1 5 3 = 23 48 46 (1.16)
5 7 9 5 7 9 67 108 122
T
3 2 4 3 2 4 29 25 65
D = B BT D ij = B ik B jk
= 1 5 3 1 5 3 = 25 35 67 (1.17)
5 7 9 5 7 9 65 67 155
T
3 2 4 3 2 4 35 46 60
E = BT B E ij = B ki B kj = 1 5 3 1 5 3 = 46 78 86 (1.18)
5 7 9 5 7 9 60 86 106

Luego:
C ii = C 11 + C 22 + C 33 = 31 + 48 + 122 = 201
D ii = D11 + D 22 + D 33 = 29 + 35 + 155 = 219 (1.19)
E ii = E11 + E 22 + E 33 = 35 + 78 + 106 = 219

NOTA: Verificamos que se cumple que: Tr (B B T ) = Tr (B T B) = B : B

Ejemplo 1.24
Dadas las componentes cartesianas del tensor de segundo orden B :
1 0 2
B ij = 0 1 2
3 0 3

Obtener: a) B kk b) B ij B ij c) B jk B kj
Solucin:
a) B kk = B 11 + B 22 + B 33 = 1 + 1 + 3 = 5

Universidad de Castilla- La Mancha Draft Por: Eduardo W. V. Chaves (2012)


Ciudad Real - Espaa
1 TENSORES 17

b) B ij B ij = B 1 j B 1 j + B 2 jB 2 j + B 3 jB 3 j
123 123 123
B 11B 11 + B 21B 21 + B 31B 31
+ + +
B 12B 12 + B 22B 22 + B 32B 32
+ + +
B 13B 13 + B 23B 23 + B 33B 33
Resultando:
B ij B ij = 1 1 + 0 0 + 2 2 + 0 0 + 1 1 + 2 2 + 3 3 + 0 0 + 3 3 = 28

c) B jk B kj = B 1k B k1 + B 2k B k 2 + B 3k B k 3
123 123 123
B 11B 11 + B 21B 12 + B 31B 13
+ + +
B 12B 21 + B 22B 22 + B 32B 23
+ + +
B 13B 31 + B 23B 32 + B 33B 33
B jk B kj = B 11B 11 + B 22B 22 + B 33B 33 + 2B 21B 12 + 2B 31B 13 + 2B 32B 23
= 1 1 + 1 1 + 3 3 + 2(0 0) + 2(3 2 ) + 2(0 2 ) = 23

Ejemplo 1.25
Obtener las componentes del tensor D resultante de la siguiente operacin D = A : B , para
los siguientes casos:
2 3 2 2 3 1
a) con Aij = 4 1 1 ; Bij = 1 2 1
1 1 5 1 2 5

7 13 14 13 9 17
b) con Aik Bkj = 11 18 11 ; Aik B jk = 15 9 13
16 27 31 18 12 32

Solucin:
a) A : B = 2 2 + 3 3 + 2 1 + 4 1 + 1 2 + 1 1 + 1 1 + 1 2 + 5 5 = 50
b) Teniendo en cuenta la expresin Tr ( A B T ) = Tr ( AT B) = A : B y que Aik B jk = A B T ,
concluimos que A : B = Tr ( A B T ) = 13 + 9 + 32 = 54 .
Ejemplo 1.26
Considrese un tensor de segundo orden T = Tr ( E )1 + ( F : E ) E o en notacin indicial
Tij = E kk ij + ( Fkp E kp ) E ij . Si las componentes de los tensores E y F vienen dadas por:

2 1 4 4 3 1
E ij = 1 5 0 ; Fij = 2 0 3
2 0 1 2 0 0

Universidad de Castilla- La Mancha Draft Por: Eduardo W. V. Chaves (2012)


Ciudad Real - Espaa
18 PROBLEMAS RESUELTOS DE MECNICA DEL MEDIO CONTINUO

a) Obtener las componentes del tensor T . b) Son los tensores T y E coaxiales?


Demustralo.
Solucin:
Obtenemos primero los siguientes escalares:
Tr ( E ) = 2 + 5 + 1 = 8
F : E = 2 4 + 1 3 + 4 1 + 1 2 + 5 0 + 0 3 + 2 2 + 0 0 + 1 0 = 21
Luego
1 0 0 2 1 4 50 21 84

Tij = 80 1 0 + 211 5 0 = 21 113 0
0 0 1 2 0 1 42 0 29

Dos tensores son coaxiales cuando presentan los mismos autovectores o cuando se cumple
que T E = E T :
50 21 84 2 1 4 289 155 284
Tik E kj = 21 113 0 1 5 0 = 155 586 84
42 0 29 2 0 1 142 42 197
2 1 4 50 21 84 289 155 284
E ik Tkj = 1 5 0 21 113 0 = 155 586 84
2 0 1 42 0 29 142 42 197
Con lo cual concluimos que son coaxiales.

Ejemplo 1.27
Obtener el resultado de las siguientes operaciones: I : I , I : I , I : I , I : I , I : I , I : I , I : I ,
I : I , I sym : I sym , I sym : I , I : I sym , donde

I = 11 = I ijkl e i e j e k e l con I ijkl = ik jl (1.20)


I = 11 = I ijkl e i e j e k e l con I ijkl = il jk (1.21)

I = 1 1 = I ijkl e i e j e k e l con I ijkl = ij kl (1.22)

Solucin:
(I : I ) ijkl = I ijpq I pqkl = ip jq pk ql = ik jl = I ijkl

( I : I ) ijkl = I ijpq I pqkl = iq jp pl qk = ik jl = I ijkl

( I : I ) ijkl = I ijpq I pqkl = ij pq pq kl = qq ij kl = 3I ijkl

( I : I ) ijkl = I ijpq I pqkl = iq jp pk ql = il jk = I ijkl

(I : I ) ijkl = I ijpq I pqkl = ip jq pl qk = il jk = I ijkl

(I : I ) ijkl = I ijpq I pqkl = ip jq pq kl = iq jq kl = ij kl = I ijkl

( I : I ) ijkl = I ijpq I pqkl = iq jp pq kl = iq jq kl = ij kl = I ijkl

Universidad de Castilla- La Mancha Draft Por: Eduardo W. V. Chaves (2012)


Ciudad Real - Espaa
1 TENSORES 19

Resumiendo lo anterior en notacin tensorial:


I : I = (11) : (11) = 11 = I

I : I = (11) : (11) = 11 = I

I : I = (1 1) : (1 1) = 3(1 1) = 3I

I : I = (11) : (11) = 11 = I

I : I = (11) : (11) = 11 = I

I : I = (11) : (1 1) = 1 1 = I

I : I = (11) : (1 1) = 1 1 = I

Teniendo en cuenta la definicin: I sym =


1
2
( 1
) (
I + I = 11 + 11 , concluimos que:
2
)
I sym : I sym =
1
4
( )(
11 + 11 : 11 + 11 )
1
[( ) (
= 11 : 11 + 11 : 11 + 11 : 11 + (11 : 11)
4
) ( ) ]
1
[
= 11 + 11 + 11 + 11
4
]
1
(
= 11 + 11
2
)
= I sym

I sym : (1 1) = I sym : I =
1
2
( 1
2
) 1
(
I + I : I = I : I + I : I = I + I = I =1 1
2
) ( )
(1 1) : I sym = I : I sym
1
2
( 1
2
) 1
(
= I : I + I = I :I + I : I = I + I = I =1 1
2
) ( )
1.1.3 Transpuesta

Ejemplo 1.28
Demostrar que la siguiente propiedad es vlida:
( )
A : (B C ) = B T A : C = A C T : B ( )
donde A , B , C son tensores de segundo cualesquiera.
Solucin: Demostraremos esta identidad a travs de sus componentes:
(
A : (B C ) = A ij e i e j : B lk e l e k C pq e p e q )
= A ij B lk C pq e i e j : kp e l e q ( )
= A ij B lk C pq kp il jq = A ij B ik C kj

Observemos que cuando trabajamos en notacin indicial el orden no importa, es decir:


A ij B ik C kj = B ik A ij C kj = A ij C kj B ik

Universidad de Castilla- La Mancha Draft Por: Eduardo W. V. Chaves (2012)


Ciudad Real - Espaa
20 PROBLEMAS RESUELTOS DE MECNICA DEL MEDIO CONTINUO

Podemos ahora observar que la operacin B ik A ij resultar un tensor de segundo orden cuyas
componentes son (B T A ) kj luego, B ik A ij C kj = (B T A ): C . Anlogamente podemos decir que
( )
A ij C kj B ik = A C T : B .

Ejemplo 1.29
r r
Demostrar que, si u , v son vectores y A un tensor de segundo orden, la siguiente relacin es
vlida:
r r r r
u AT v = v A u
Solucin:
r r r r
u AT v = v A u
u i e i A jl e l e j v k e k = v k e k A jl e j e l u i e i
u i A jl il v k jk = v k kj A jl u i il
u l A jl v j = v j A jl u l
c.q.d.

1.1.4 Simetra y Antisimetra

Ejemplo 1.30
Si es un tensor de segundo orden simtrico y W es un tensor de segundo orden
antisimtrico. Demostrar que : W = 0 .
Solucin:
:W = ij (e i e j ) : Wlk (e l e k )
= ij Wlk il jk
= ij Wij

Desarrollando
ij Wij = 1 j W1 j + 2 j W2 j + 3 j W3 j
123 1424 3 1
424 3
11W11 21W21 31W31
+ + +
12 W12 22 W22 32 W32
+ + +
13W13 23W23 33W33

Considerando la propiedad de un tensor simtrico 12 = 21 , 31 = 13 , 32 = 23 y


antisimtrico W11 = W22 = W33 = 0 , w 21 = w 12 , W31 = W13 , W32 = W23 , resultando:
:W =0
c.q.d.

Universidad de Castilla- La Mancha Draft Por: Eduardo W. V. Chaves (2012)


Ciudad Real - Espaa
1 TENSORES 21

Ejemplo 1.31
Demostrar que:
r r r r
a) M Q M = M Q sym M ;
b) A : B = A sym : B sym + A anti : B anti ;
r
donde, M es un vector, y Q , A , y B son tensores de segundo orden arbitrarios.
c) Demostrar que si se cumple que ijk T jk = 0 i , T es simtrico, es decir, Tij = T ji .
Solucin:
a)
( )
r r r r
M Q M = M Q sym + Q anti M
r r r r
= M Q sym M + M Q anti M
r r r r
(
Ya que el producto: M Q anti M = Q anti : M M = 0 , resulta que: )
r r r r
M Q M = M Q sym M c.q.d.
r r
NOTA: Podemos hacer la interpretacin geomtrica de M Q anti M = 0 . Notar que la
r r r r r r r r
operacin algebraica Q anti M = q (M) resulta un vector, luego M Q anti M = M q (M) = 0 , que
r r r
implica que M y q (M) son vectores ortogonales. Con eso, concluimos que: la proyeccin de
r r r
un tensor antisimtrico de segundo grado segn una direccin ( M ) resulta un vector ( q (M) )
r
que es ortogonal a M , ver figura abajo:

r
Q M r r r
q (M ) M = 0

r r r r
M q (M) = Q anti M r
M

b)
A :B = ( A sym + A anti ) : (B sym + B anti )
= A sym : B sym + 1
A sym
42
anti
43 + 1
:B A anti
42
sym
43 + A
:B anti
: B anti
=0 =0
= A sym : B sym + A anti : B anti
Luego como consecuencia tenemos que:
A : B sym = A sym : B sym ; A : B anti = A anti : B anti
c.q.d.
c)
ijk T jk = ij1 T j1 + ij 2 T j 2 + ij 3 T j 3 = 0 i
= i11 T11 + i 21 T21 + i 31 T31 + i12 T12 + i 22 T22 + i 32 T32 + i13 T13 + i 23 T23 + i 33 T33
= i 21 T21 + i 31 T31 + i12 T12 + i 32 T32 + i13 T13 + i 23 T23 = 0 i

Universidad de Castilla- La Mancha Draft Por: Eduardo W. V. Chaves (2012)


Ciudad Real - Espaa
22 PROBLEMAS RESUELTOS DE MECNICA DEL MEDIO CONTINUO

Luego, las componentes del vector resultante quedan:


i =1 1 jk T jk = 132 T32 + 123 T23 = T32 + T23 = 0 T32 = T23
i=2 2 jk T jk = 231 T31 + 213 T13 = T31 T13 = 0 T31 = T13
i=3 3 jk T jk = 321 T21 + 312 T12 = T21 + T12 = 0 T21 = T12

con lo cual demostrando que si ijk T jk = 0 i , T es simtrico, T = T T .

Ejemplo 1.32
Dado un tensor de segundo orden arbitrario A donde se conocen las componentes de su
parte simtrica en el sistema Cartesiano:
4 2 0
A ijsym = 2 1 0
0 0 3

, donde las componentes del versor N son N = [1 0 0] .


A N
Obtener N i

Solucin:
A N
En el Ejemplo 1.31 se ha demostrado que N A sym N
=N con lo cual:

4 2 0 1
A N
N A
=N sym
= N A sym N
N i ij i = [1 0 0] 2 1 0 0 = 4
0 0 3 0

Ejemplo 1.33
Si W es un tensor antisimtrico. a) Demostrar que W W resulta un tensor de segundo
orden simtrico. b) Demostrar tambin que (W T W W) : 1 = 0
Solucin:
a) Si demostramos que (W W ) anti = 0 , demostramos que W W resultar ser simtrico:
1
(W W ) anti =
2
[ 1
] [
( W W) ( W W) T = (W W) W T W T
2
]
1
= [(W W ) W W ]
2
=0
donde hemos aplicado la propiedad del tensor antisimtrico W = W T .
Soluciones alternativas a) Teniendo en cuenta la definicin de un tensor antisimtrico donde se
cumple que W = W T :
W W = W T W = W T W T = (W W ) T
Tambin se puede comprobar a travs de sus componentes:

Universidad de Castilla- La Mancha Draft Por: Eduardo W. V. Chaves (2012)


Ciudad Real - Espaa
1 TENSORES 23

0 W12 W13 0 W12 W13



( W W ) ij = W12 0
W23 W12 0 W23
W13 W23 0 W13 W23 0
W122 W132 W13 W23 W12 W23
2 2
= W13 W23 W12 W23 W12 W13
W12 W23 W12 W13 2
W132 W23

b) (W T W W ) : 1 = (W pi W pk Wkj ) ij = W pi (W pk Wki ) = W : (W W) = 0 , ya que el doble


producto escalar entre un tensor simtrico (W W ) y uno antisimtrico ( W ) resulta cero.

Ejemplo 1.34
1
Sea B un tensor de segundo orden tal que B pq = pqs a s con a i = ijk B jk . Demostrar que B
2
es un tensor antisimtrico.
Solucin:
1 1 1
B pq = pqs a s = pqs sjk B jk = pqs sjk B jk = pqs jks B jk
2 2 2
Recurriendo a la relacin pqs jks = pj qk pk qj
1 1
B pq = pqs jks B jk = ( pj qk pk qj )B jk
2 2
1
= ( pj qk B jk pk qj B jk )
2
1
= (B pq B qp )
2
= B anti
pq

Solucin Alternativa:
Teniendo en cuenta que B qp = qps a s , y que por definicin se cumple que pqs = qps ,
concluimos que:
B pq = pqs a s = qps a s = B qp B = B T (antisimtrico)

Ejemplo 1.35
Demostrar que la operacin A anti A sym + A sym A anti resulta un tensor antisimtrico.
Solucin:
Denominando por B = A anti A sym + A sym A anti , y teniendo en cuenta que se cumple que
A anti = (A anti ) T , A sym = (A sym ) T , concluimos que:

B = A anti A sym + A sym A anti


= A anti A sym A sym ( A anti ) T
= A anti A sym ( A anti A sym ) T
= 2( A anti A sym ) anti

Universidad de Castilla- La Mancha Draft Por: Eduardo W. V. Chaves (2012)


Ciudad Real - Espaa
24 PROBLEMAS RESUELTOS DE MECNICA DEL MEDIO CONTINUO

Ejemplo 1.36
r r r
La relacin n T = T n es vlida siempre? Siendo T un tensor de segundo orden y n un
vector. En el supuesto de que la relacin no sea vlida, para qu caso particular lo sera?
Solucin:
r r
n T = n i e i Tkl (e k e l ) T n = Tlk (e l e k ) n i e i
= n i Tkl ik e l y = n i Tlk ki e l
= n k Tkl e l = n k Tlk e l
Con lo que comprobamos que n k Tkl n k Tlk , luego:
r r
n T T n
r r r r
La relacin n T = T n slo ser vlida cuando el tensor sea simtrico, i.e. n T sym = T sym n .

Ejemplo 1.37
r r r r
Obtener el vector axil w asociado al tensor antisimtrico ( x a ) anti . Expresar w en funcin
r r
de x y a .
r
Solucin: Sea z un vector arbitrario, se cumple que:
r r r r r
( x a ) anti z = w z
r r r
donde w es el vector axil asociado a ( x a ) anti . Teniendo en cuenta que:
r r 1 r r
[ r r
]
1 r r r r
( x a ) anti = ( x a ) ( x a ) T = [ x a a x ]
2 2
podemos an decir que:
1 r r r r r r r
[x a a x ] z = w z [xr ar ar xr ] zr = 2wr zr
2
r r r r r r r r r r r
Recordar que, dados tres vectores a , b , c se cumple que: a (b c ) = (b c c b) a ,
r r r r r r r r
ver Ejemplo 1.17. Luego, se cumple que [x a a x ] z = z ( x a ) . Retomando nuestra
expresin anterior:
[xr ar ar xr ] zr = zr ( xr ar ) = (ar xr ) zr = 2wr zr
con lo cual, concluimos que:
r 1 r r r r
w = (a x ) es el vector axil asociado al tensor ( x a ) anti
2

Ejemplo 1.38
Consideremos dos tensores antisimtricos W (1) y W ( 2) y sus vectores axil representados,
r r
respectivamente, por w (1) y w ( 2) . Demostrar que:
r r r r
W (1) W ( 2 ) = ( w ( 2 ) w (1) ) ( w (1) w ( 2) )1
[ ] r r
Tr W (1) W ( 2 ) = 2( w (1) w ( 2 ) )

Universidad de Castilla- La Mancha Draft Por: Eduardo W. V. Chaves (2012)


Ciudad Real - Espaa
1 TENSORES 25

Solucin:
Teniendo en cuenta las propiedades de tensor antisimtrico, podemos decir que:
r r r
W (1) a = w (1) a
r r r
a W (1) = a w (1)
T
r r r
r r r y W ( 2) a = w ( 2 ) a
a W (1) = a w (1)
r r r
a W (1) = a w (1)
A continuacin hacemos el producto escalar (a W (1) ) (W ( 2) a), obteniendo que:
r r

(ar W ) (W
(1) ( 2)
a) = (a w (1) ) ( w ( 2) a)
r r r r r

Continuaremos el desarrollo en notacin indicial:


(a i Wij(1) )(W jk(1) a k ) = ( ijk a j wk(1) )( ipq w (p2) a q )
a i (Wij(1) W jk(1) )a k = a j ( ijk ipq wk(1) w (p2 ) )a q
[
= a j ( jp kq jq kp ) wk(1) w (p2 ) a q ]
=aj [ jp (1) ( 2 )
kq wk w p jq (1) ( 2 )
kp wk w p ]a q

=aj [w (1) ( 2 )
q wj (1) ( 2 )
jq wk wk ]a q

En notacin tensorial la expresin anterior queda:


r
[ r r r
] [
r r r
a W (1) W ( 2 ) a = a ( w ( 2 ) w (1) ) ( w (1) w ( 2 ) )1 ] ar
r r r r
con lo cual demostramos que: W (1) W ( 2) = ( w ( 2) w (1) ) ( w (1) w ( 2) )1 .
b)
[
Tr W (1) W ( 2) ] [ r r r r
= Tr ( w ( 2) w (1) ) ( w (1) w ( 2 ) )1 ]
[ r r
] r
[ r
= Tr ( w ( 2) w (1) ) Tr ( w (1) w ( 2 ) )1 ]
r r r r
= ( w ( 2 ) w (1) ) ( w (1) w ( 2) ) 1 [3
Tr21]
=3
r r
= 2( w (1) w ( 2) )
Solucin alternativa
En esta solucin alternativa vamos utilizar las componentes. Para ellos consideremos:
0 W12(1) W13(1) 0 w3(1) w2(1)
(1) (1)
Wij(1) = W12(1) 0 W23 = w3 0 w1(1)
W (1) W12(1) 0 w2(1) w1(1) 0
12

0 W12( 2 ) W13( 2 ) 0 w3( 2 ) w2( 2 )


(2) (2)
Wij( 2 ) = W12( 2 ) 0 W23 = w3 0 w1( 2 )
W ( 2 ) W12( 2 ) 0 w2( 2) w1( 2 ) 0
12

Con eso podemos obtener que:

Universidad de Castilla- La Mancha Draft Por: Eduardo W. V. Chaves (2012)


Ciudad Real - Espaa
26 PROBLEMAS RESUELTOS DE MECNICA DEL MEDIO CONTINUO

0 w3(1) w2(1) 0 w3( 2 ) w2( 2 )


[W (1)
W ( 2) ]ij = Wik(1) Wkj( 2 )

= w3(1) 0

w1(1) w3( 2 ) 0

w1( 2 )
w (1) w1(1) 0 w2( 2) w1( 2 ) 0
2
w3(1) w3( 2 ) w2(1) w2( 2 ) w2(1) w1( 2 ) w3(1) w1( 2 )

Wik(1) Wkj( 2 ) = w1(1) w2( 2) w3( 2 ) w3(1) w1(1) w1( 2 ) w2( 2) w3(1)
w3( 2 ) w1(1) w3( 2 ) w2(1) (1) ( 2 )
w2 w2 w1 w1 ( 2 ) (1)

En el trmino (11) sumamos y restamos el trmino w1( 2) w1(1) , en el trmino (22) sumamos y
restamos el trmino w2( 2) w2(1) y en el trmino (33) sumamos y restamos el trmino w3( 2) w3(1) .
Con lo cual quedamos con:
w1( 2) w1(1) w1( 2) w2(1) w1( 2 ) w3(1)

Wik(1) Wkj( 2 ) = w2( 2) w1(1) w2( 2) w2(1) w2( 2 ) w3(1) +
w ( 2) w (1) w3( 2) w2(1) w3( 2 ) w3(1)
3 1
w1(1) w1( 2 ) w2(1) w2( 2) w3(1) w3( 2) 0 0

+ 0 w1(1) w1( 2 ) w2(1) w2( 2 ) w3(1) w3( 2 ) 0
0 0 w1(1) w1( 2 ) w2(1) w2( 2 ) w3(1) w3( 2 )

Que es igual a:
Wik(1) Wkj( 2 ) = wi( 2) w (j1) ( w1(1) w1( 2) + w2(1) w2( 2) + w3(1) w3( 2) ) ij = wi( 2) w (j1) ( wk(1) wk( 2) ) ij

Dejamos al lector comprobar la traza.


NOTA: La solucin alternativa fue realizada solo como comprobacin. El lector debe dar
preferencia a la solucin tensorial o indicial, porque ni siempre la solucin a travs de
componentes es tan sencilla.

1.1.5 Cofactor. Adjunta. Traza. Tensores Particulares.


Determinante
Ejemplo 1.39
r r r r
Demostrar que Tr (a b) = a b .
Solucin:
r r
[
Tr (a b) = Tr (a i e i ) (b j e j ) ]
[
= a i b j Tr e i e j ]
= a i b j (e i e j )
= a i b j ij
= aibi
r r
= ab
Ejemplo 1.40
1
Dado que Tij = Ekk ij + 2 Eij , W = Tij E ij , y P = Tij Tij . Demostrar que:
2

Universidad de Castilla- La Mancha Draft Por: Eduardo W. V. Chaves (2012)


Ciudad Real - Espaa
1 TENSORES 27


W = E : E + [Tr( E )]2
2
P = 4 E : E + (3 + 4 )[Tr ( E )]
2 2

Solucin 1: (Notacin indicial)

W=
1
2
1
2
( 1
2
) ( 1
)
Tij Eij = Ekk ij + 2 Eij Eij = Ekk ij Eij + 2 Eij Eij = Ekk Eii + 2 Eij Eij
2
( )

Como E kk = E ii = Tr (E ) , y Eij Eij = E : E , concluimos que W = E : E + [Tr ( E )]2 .
2
( )(
P = TijTij = Ekk ij + 2 Eij Eqq ij + 2 Eij )
= Ekk ijEqq ij + Ekk ij 2 Eij + 2 EijEqq ij + 2 Eij 2 Eij
= 2 Ekk ii Eqq + 2 Ekk Eii + 2 Eii Eqq + 4 2 Eij Eij
= 3 2 Ekk Eqq + 4 Ekk Eii + 4 2 Eij Eij
= (3 + 4 )Ekk Eqq + 4 2 Eij Eij

Con lo cual demostramos que P = 4 2 E : E + (3 + 4 )[Tr ( E )]2 .


Solucin 2: (Notacin tensorial)
En notacin tensorial tenemos que:
1
T = Tr ( E )1 + 2 E , W = T : E , y P = T : T
2
Luego:
1 1 1
W = T : E = (Tr ( E )1 + 2 E ) : E = (Tr ( E )1 : E + 2 E : E )
2 2 2
1
= (Tr ( E ) Tr ( E ) + 2 E : E )
2

= [Tr( E )]2 + E : E
2
P = T : T = (Tr ( E )1 + 2 E ) : (Tr ( E )1 + 2 E )
= [Tr ( E )] 1 : 1 + 2 Tr ( E ) 1 : E + 2 Tr ( E ) {
E : 1 + (2 ) 2 E : E
2
{ {
=3 = Tr ( E ) = Tr ( E )

= 3 [Tr ( E )] + 4 [Tr ( E )] + 4 E : E
2 2 2 2

= (3 + 4 )[Tr ( E )] + 4 2 E : E
2

Ejemplo 1.41
Sea un tensor de segundo orden ij que es una funcin del tensor ij , ij = ij ( ij ) , y viene
dado por:
ij = kk ij + 2 ij Tensorial
= Tr ( )1 + 2

donde , son constantes positivas. Partiendo de la expresin anterior, obtener la expresin


de ij en funcin de ij , es decir, ij = ij ( ij ) . Expresar el resultado en notacin indicial y
tensorial.

Universidad de Castilla- La Mancha Draft Por: Eduardo W. V. Chaves (2012)


Ciudad Real - Espaa
28 PROBLEMAS RESUELTOS DE MECNICA DEL MEDIO CONTINUO

Solucin:
Notacin Indicial Notacin Tensorial
ij = kk ij + 2 ij = Tr ( )1 + 2
2 ij = ij kk ij 2 = Tr ( )1
1 1
ij = ij kk ij = Tr ( )1
2 2 2 2

Tenemos que obtener la siguiente traza kk , para ello obtenemos la traza de ij :

Notacin Indicial Notacin Tensorial


ij = kk ij + 2 ij
ii = kk ii + 2 ii = kk 3 + 2 kk : 1 = Tr ( )1 : 1 + 2 : 1
kk = (3 + 2 ) kk Tr ( ) = Tr ( )3 + 2 Tr ( )
1 1
kk = kk Tr ( ) = Tr ( )
(3 + 2 ) (3 + 2 )

Luego:
Notacin Indicial Notacin Tensorial
1 1
ij = ij kk ij = Tr ( )1
2 2 2 2
1 1 1
= ij kk ij = Tr ( )1
2 2 (3 + 2 ) 2 2 (3 + 2 )

Ejemplo 1.42
Consideremos T un tensor de segundo orden, demostrar las siguientes identidades:
(T ) = (T )
m T T m
y ( )
Tr T T
m
( )
= Tr T m .
Solucin:
(T )m T
= (T T L T ) = T T T T L T T = T T
T
( ) m
c.q.d.
Para la segunda demostracin utilizaremos la propiedad de la traza Tr (T T ) = Tr (T )

( )
Tr T T
m
= Tr T m ( )T
= Tr T m( ) c.q.d.

Ejemplo 1.43
Demostrar que: T : 1 = Tr (T ) .
Solucin:
T : 1 = Tij e i e j : kl e k e l
= Tij kl ik jl
= Tij ij = Tii = T jj
= Tr ( T )
c.q.d.

Universidad de Castilla- La Mancha Draft Por: Eduardo W. V. Chaves (2012)


Ciudad Real - Espaa
1 TENSORES 29

Ejemplo 1.44
Probar que, si y D son tensores de segundo orden, la siguiente relacin es vlida:
D = Tr ( D )
Solucin: Partimos de la siguiente definicin:
D = ij D ji
= kj D jl ik il = kj D jl lk
= kj D jl lk
123
( D )
kl
= ( D) kl lk = ( D) kk = ( D) ll
= Tr ( D) c.q.d.
Una segunda alternativa para la demostracin sera:
D = ij D ji = ij D jk ik
= ( D ) : 1
= Tr ( D )
c.q.d.

Ejemplo 1.45
Demostrar que:
1
det (S ) S = rjk tpq S rt S jp S kq (1.23)
6
Solucin:
det (S ) = ijk S 1i S 2 j S 3k (1.24)
pqr det(S ) = ijk S pi S qj S rk (1.25)
pqr pqr det (S ) = pqr ijk S pi S qj S rk
1
424
3 (1.26)
6

1
det (S ) = pqr ijk S pi S qj S rk (1.27)
6

Ejemplo 1.46
Demostrar que: A tpq = rjk A rt A jp A kq
Solucin: Sabemos que:
A = rjk A r1 A j 2 A k 3
(1.28)
A tpq = rjk tpq A r1 A j 2 A k 3

La expresin rjk tpq podr ser expresada en funcin de la delta de Kronecker como:

rt rp rq
rjk tpq = jt jp jq
(1.29)
kt kp kq
= rt jp kq + rp jq kt + rq jt kp rq jp kt jq kp rt kq jt rp

Universidad de Castilla- La Mancha Draft Por: Eduardo W. V. Chaves (2012)


Ciudad Real - Espaa
30 PROBLEMAS RESUELTOS DE MECNICA DEL MEDIO CONTINUO

Reemplazando en la expresin anterior (1.29) en la expresin (1.28), y utilizando la propiedad


del operador de sustitucin obtenemos que:
A tpq = A t1 A p 2 A q 3 + A p1 A q 2 A t 3 + A q1 A t 2 A p 3 A q1 A p 2 A t 3 A t1 A q 2 A p 3 A p1 A t 2 A q 3
( ) ( ) (
= A t1 1 jk A pj A qk + A t 2 2 jk A pj A qk + A t 3 3 jk A pj A qk )
= rjk A rt A jp A kq = rjk A tr A pj A qk
c.q.d.

Ejemplo 1.47
1
Demostrar que: A = rjk tpq A rt A jp A kq
6
Solucin:
Partiendo del problema anterior: A tpq = rjk A rt A jp A kq y multiplicando ambos lados por tpq ,
resulta:
A tpq tpq = rjk tpq A rt A jp A kq (1.30)
Utilizando la propiedad tpq tpq = tt pp tp tp = tt pp tt = 6 . Luego, la relacin (1.30)
resulta:
1
A = rjk tpq A rt A jp A kq
6 c.q.d.

Ejemplo 1.48
Demostrar que la siguiente propiedad es vlida:
r r r r r r r r r r r r
(B a) (b c) (B b) (a c) + (B c ) (a b) = Tr (B) a (b c) [ ] (1.31)

Solucin:
El lado derecho de la ecuacin en notacin indicial queda:
r r r
ijk (B a) i b j c k ijk (B b) i a j c k + ijk (B c ) i a j b k =
= ijk [(B i1a1 + B i 2 a 2 + B i 3 a 3 )b j c k (B i1b1 + B i 2 b 2 + B i 3b 3 )a j c k +
+ (B i1 c 1 + B i 2 c 2 + B i 3 c 3 )a j b k ]
= ijk [(B i1a1b j c k + B i 2 a 2 b j c k + B i 3 a 3b j c k ) (B i1b1 a j c k + B i 2 b 2 a j c k + B i 3b 3 a j c k ) +
+ (B i1 c 1 a j b k + B i 2 c 2 a j b k + B i 3 c 3 a j b k )]
= ijk [B i1 (a1b j c k b1a j c k + c 1a j b k ) + B i 2 (a 2 b j c k b 2 a j c k + c 2 a j b k ) +
+ B i 3 (a 3b j c k b 3 a j c k + c 3 a j b k )]

= (1 jk B 11 + 2 jk B 21 + 3 jk B 31 )(a1b j c k b1a j c k + c 1 a j b k ) +
+ (1 jk B 12 + 2 jk B 22 + 3 jk B 32 )(a 2 b j c k b 2 a j c k + c 2 a j b k ) + (1.32)
+ (1 jk B 13 + 2 jk B 23 + 3 jk B 33 )(a 3b j c k b 3 a j c k + c 3 a j b k )

Notar que:

Universidad de Castilla- La Mancha Draft Por: Eduardo W. V. Chaves (2012)


Ciudad Real - Espaa
1 TENSORES 31

a1 a 2 a3
1 jk (a1b j c k b1a j c k + c 1a j b k ) = b1 b 2 b 3 = ijk a i b j c k
c1 c2 c3

2 jk (a1b j c k b1a j c k + c 1a j b k ) = 3 jk (a1b j c k b1a j c k + c 1a j b k ) = 0


Con lo cual la ecuacin (1.32) queda:
r r r
B 11 ijk a i b j c k + B 22 ijk a i b j c k + B 33 ijk a i b j c k = (B 11 + B 22 + B 33 ) ijk a i b j c k = Tr (B) a (b c )[ ]
Notar que tambin se cumple que:
r r r r r r r r r r r r
(B T a) (b c ) (B T b) (a c ) + (B T c ) (a b) = Tr (B) a (b c ) [ ]
ya que Tr (B) = Tr (B T ) . Tambin es vlido que:
r r r r r r r r r r r r
(B a) (b c ) + a ((B b) c) + a (b (B c )) = Tr (B) a (b c ) [ ] (1.33)
r r r r r r r r r r r r
[(B a), b, c ] + [a, (B b), c ] + [a, b, (B c )] = I B [a, b, c ]

Ejemplo 1.49
Demostrar que la siguiente propiedad es vlida:
r
[ r r
]
r r r
( A a) ( A b) ( A c ) = det ( A ) a (b c ) [ ] (1.34)
r r r
donde A es un tensor de segundo orden no-singular, y a , b y c son vectores linealmente
independientes.
Solucin:
A tensor no-singular det( A ) A 0
r r r r r r
a , b , c linealmente independientes a b c 0 . ( )
Escribimos el triple producto escalar en notacin indicial, i.e. a b c = ijk a i b j c k , y
r
(r r )
multiplicamos por ambos lados de la igualdad por el determinante del tensor A , resultando:
r r r
( )
a b c A = ijk a i b j c k A

Fue demostrado en el Ejemplo 1.47 que se cumple que A ijk = pqr A pi A qj A rk , con lo cual:
r r r
( )
a b c A = ijk a i b j c k A
= pqr A pi A qj A rk a i b j c k
= pqr ( A pi a i )( A qj b j )( A rk c k )
r
[
r
= ( A a) ( A b) ( A c )
r
]
Ejemplo 1.50
Demostrar que:

( r r
)
r r
det 1 + a b = 3 + 2 a b (1.35)

Universidad de Castilla- La Mancha Draft Por: Eduardo W. V. Chaves (2012)


Ciudad Real - Espaa
32 PROBLEMAS RESUELTOS DE MECNICA DEL MEDIO CONTINUO

r r
donde y son escalares, a y b son vectores y 1 es el tensor identidad de segundo orden.
Solucin: Si denotamos por A ij ij + a i b j , el determinante de A viene dado por
A = ijk A i1 A j 2 A k 3 , donde A i1 = i1 + a i b1 , A j 2 = j2 + a j b 2 y A k 3 = k 3 + a k b 3 ,
luego podemos decir que:
( r r
)
det 1 + a b = ijk ( i1 + a i b1 ) ( j2 )
+ a j b 2 ( k 3 + a k b 3 ) (1.36)
Desarrollando la expresin (1.36) obtenemos que:
( r r
) [
det 1 + a b = ijk 3 i1 j 2 k 3 + 2 a k b 3 i1 j 2 + 2 a j b 2 i1 k 3 + 2 a i b 1 j 2 k 3 +
+ 2 a j b 2 a k b 3 i1 + 2 a i a k b 1b 3 j2 + 2 a i a j b1b 2 k 3 + 3 a i a j a k b1b 2 b 3 ]
Observemos que:
3 ijk i1 j 2 k 3 = 3 123 = 3
2 ( ijk a k b 3 i1 j 2 + ijk a j b 2 i1 k 3 + ijk a i b1 j 2 k 3 ) =
r r
2 (12 k a k b 3 + 1 j 3 a j b 2 + i 23 a i b1 ) = 2 (a 3b 3 + a 2 b 2 + a1b1 ) = 2 (a k b k ) = 2 (a b)
ijk a i a k b1b 3 j 2 = i 2 k a i a k b1b 3 = a1a 3b1b 3 a 3 a1b1b 3 = 0
ijk a i a j b1b 2 k 3 = ij 3 a i a j b1b 2 = 123 a1 a 2 b1b 2 213 a 2 a1b1b 2 = 0
ijk a i a j a k b1b 2 b 3 = 0
Fijemos que no haca falta expandir los trminos ijk a i a k b1b 3 j 2 , ijk a i a j b1b 2 k 3 ,
ijk a i a j a k b1b 2 b 3 , para saber que son iguales a cero, ya que
r r
ijk a i a k b1b 3 j 2 = (a a) j b1b 3 j 2 = 0 y anlogamente para los otros trminos. Con lo que
hemos demostrado que:
( r r
) r r
det 1 + a b = 3 + 2 a b
c.q.d.
Para = 1 tenemos que:

( r r r r
)
det 1 + a b = 1 + a b

Anlogamente, se puede demostrar que:


(
r r
)
det a b = 3 ijk a i a j a k b1b 2 b 3 = 0

NOTA: Podemos extrapolar la expresin (1.35) de tal forma que:


( )
det I sym + A B = 3 + 2 A : B (1.37)

donde I sym es el tensor identidad de cuarto orden simtrico, A y B son tensores de segundo
orden. Notar que det ( I sym ) = (1) 3 + (1) 2 (0)(0 : 0 ) = 1 y det (1 1) = (0) 3 + (0) 2 (1)(1 : 1) = 0 .

Ejemplo 1.51
r r r r
Dado un tensor A , demostrar que existe un vector no nulo n 0 tal que A n = 0 si y solo si
det ( A ) = 0 , Chadwick (1976).

Solucin: Partimos del hecho que det ( A ) A = 0 y tambin escogemos una base arbitrario
r r r
{f , g, h} (linealmente independiente):

Universidad de Castilla- La Mancha Draft Por: Eduardo W. V. Chaves (2012)


Ciudad Real - Espaa
1 TENSORES 33

r r r
( ) r r
[ r
f g h A = ( A f ) ( A g) ( A h) ] (ver Ejemplo 1.49)
Por el hecho que det ( A ) A = 0 , eso implica que:
r r
[ r
( A f ) ( A g) ( A h) = 0 ]
r r r
Con lo cual concluimos que los vectores ( A f ) , ( A g) , ( A h) son linealmente
dependientes. Esto implica que existen escalares no nulos 0 , 0 , 0 tal que:
r r r r
( A f ) + ( A g) + ( A h) = 0
r
( r
A f + g + h = 0
r r
)
r r
A n = 0
r r r r r
donde n = f + g + h 0 .
r r r
Ahora escogemos dos vectores k , m que no son linealmente dependientes con n y
r r r r r r
reemplazamos esta base {k , m, n} en lugar de los vectores {a, b, c} :
r r r
( ) r
k m h A = ( A k ) [( A m) ( A n)]
r r

r r r r r
( ) r r r
Considerando que A n = 0 y que k m h 0 , ya que la base {k , m, n} est constituida por
vectores linealmente independientes, obtenemos que:
(
r r r
k mh A =0
14243
) A =0
0 c.q.d.

Universidad de Castilla- La Mancha Draft Por: Eduardo W. V. Chaves (2012)


Ciudad Real - Espaa
34 PROBLEMAS RESUELTOS DE MECNICA DEL MEDIO CONTINUO

Ejemplo 1.52
Sea un tensor de segundo orden arbitrario F . Demostrar que los tensores resultantes
C = F T F y b = F F T son tensores simtricos y semi-definidos positivos. Verificar tambin en que
condiciones C y b son tensores definidos positivos.
Solucin: Para demostrar que los tensores son simtricos, tenemos que demostrar que C = C T
y b = bT :
C T = (F T F )T = F T (F T )T = F T F = C
(simetra)
b T = (F F T ) T = (F T )T F T = F F T = b

Con lo cual hemos demostrado que los tensores C = F T F y b = F F T son simtricos.


Para demostrar que los tensores C = F T F y b = F F T son semi-definidos positivos,
partimos de la definicin de un tensor semi-definido positivo, es decir, un tensor A es semi-
r r r r
definido positivo si se cumple que x A x 0 , para todo x 0 . Luego:
r r r r r r r r
x C x = x (F T F ) x x b x = x (F F T ) x
r r r r
= (F x ) (F x ) = (F T x ) (F T x )
r 2 r 2
= F x 0 = FT x 0

En notacin indicial:
x i C ij x j = x i ( Fki Fkj ) x j x i bij x j = x i ( Fik F jk ) x j
= ( Fki x i )( Fkj x j ) = ( Fik x i )( F jk x j )
2 2
= Fki x i 0 = Fik x i 0

Con lo cual demostramos que C = F T F y b = F F T son semi-definidos positivos.


r r r r r r r
Observemos que x C x = F x slo ser igual a cero, con x 0 , si F x = 0 , y por
2

r r r r
definicin F x = 0 con x 0 , si y solo si det ( F ) = 0 , ver Ejemplo 1.51. Luego, los tensores
C = F T F y b = F F T sern tensores definidos positivos si y solo si det ( F ) 0 .

Ejemplo 1.53
r r r r r r
Consideremos los siguientes vectores dX (1) , dX ( 2) , dX (3) , dx (1) , dx ( 2) , dx (3) , en el cual estn
r r r r
relacionados entre si a travs de las transformaciones dx (1) = F dX (1) , dx ( 2) = F dX ( 2) ,
r r
dx (3) = F dX (3) , donde F es un tensor de segundo orden no-singular y F 1 . a.1) Obtener
la relacin entre los escalares dV y dV0 en funcin de F , sabiendo que
dV = dx (1) (dx ( 2 ) dx (3) ) 0 , dV0 = dX (1) (dX ( 2 ) dX (3) ) 0 . a.2) Obtener la relacin entre
r r r r r r
r r r r r r r r
c = dX ( 2 ) dX (3) 0 y c * = dx ( 2 ) dx (3) 0 .
Solucin
a.1) Teniendo en cuenta el enunciado se cumple que:
r r r r r
[ r
dV = dx (1) (dx ( 2 ) dx (3) ) = ( F dX (1) ) ( F dX ( 2 ) ) ( F dX (3) ) ]
Recordar que en el Ejemplo 1.49 hemos demostrado que

Universidad de Castilla- La Mancha Draft Por: Eduardo W. V. Chaves (2012)


Ciudad Real - Espaa
1 TENSORES 35

(
r r r
) r r
[ r
a b c A = ( A a) ( A b) ( A c ) ]
r r
( r
) r r
[ r
dX (1) dX ( 2 ) dX (3) F = ( F dX (1) ) ( F dX ( 2 ) ) ( F dX (3) ) ]
Con eso concluimos que:
r r r r r
[ r
dV = dx (1) (dx ( 2 ) dx (3) ) = ( F dX (1) ) ( F dX ( 2 ) ) ( F dX (3) ) ]
r
[ r
= F dX (1) (dX ( 2) dX (3) )
r
]
Luego:
dV = F dV0
a.2) Teniendo en cuenta la ecuacin anterior, podemos obtener que:
dV = F dV0
r r r r r r
dx (1) (dx ( 2 ) dx (3) ) = F dX (1) dX ( 2 ) dX (3)[ ]
r r r r r r
[
dx (1) (dx ( 2 ) dx (3) ) = F ( F 1 dx (1) ) dX ( 2) dX (3) ]
r r r r
(
r r
dx (1) (dx ( 2 ) dx (3) ) = dx (1) F F T dX ( 2 ) dX (3) [ ])
r r r
[
r
(dx ( 2 ) dx (3) ) = F F T dX ( 2 ) dX (3) ]
r r
c * = F F T c
r r
NOTA 1: Notar que c * F c . Podemos reescribir la ecuacin anterior como
r r r
[ r
dx ( 2 ) dx (3) = F F T dX ( 2) dX (3) ]
r r r r
( F dX ( 2 ) ) ( F dX (3) ) = F F T dX ( 2 ) dX (3) [ ]
El tensor F F T es conocido como tensor cofactor de F , i.e. cof ( F ) = F F T , con eso
definimos la inversa de un tensor como:
cof ( F ) = F F T

[
F F T ]
T
= [cof ( F )]
T

F F 1 = [cof ( F )]
T

1
F 1 = [cof (F )]T = 1 [adj(F )]
F F

F r r
r
dX (1) dx (1) = F dX (1)

r r r
r c * = dx ( 2 ) dx ( 3 )
dX ( 3 ) r r
r r r c* F c r r
c = dX ( 2) dX (3) dx (3) = F dX (3)
r r
c * = [cof ( F )] c

dV = F dV0
r r r
dX ( 2 ) dx ( 2 ) = F dX ( 2 )

r r r
dV0 = dX (1) (dX ( 2 ) dX (3) ) 0 (dx ( 2) dx (3) ) 0
r r r
1 dV = dx (1)
F

Universidad de Castilla- La Mancha Draft Por: Eduardo W. V. Chaves (2012)


Ciudad Real - Espaa
36 PROBLEMAS RESUELTOS DE MECNICA DEL MEDIO CONTINUO

NOTA 2: Vamos considerar un tensor de segundo orden F = A B , y tres vectores tal que
r r r r r r r r r
a (b c ) 0 , y a * = B a , b * = B b , c * = B c , luego, aplicando las definiciones anteriores
podemos decir que:
r r r r
[ r r
F a (b c ) = ( F a ) ( F b ) ( F c ) ] r
[ r r
= ( A B a) ( A B b) ( A B c) ]
r
[ r r
= ( A a*) ( A b*) ( A c*)
r
]
r r
= A a * (b * c * )
r
[ r r
= A (B a ) (B b ) (B c ) ]
r r r
= A B a (b c )

Con lo cual concluimos que: si F = A B se cumple que F = A B = A B .

Ejemplo 1.54
Demostrar que si A y B son tensores ortogonales, el tensor resultante de la operacin
C = A B resulta ser otro tensor ortogonal.
Solucin: Por definicin, un tensor ( C ) es ortogonal cuando se cumple que C 1 = C T :
C 1 = ( A B) 1 = B 1 A 1 = B T A T = ( A B) T = C T c.q.d.

Ejemplo 1.55
Demostrar que adj( A B) = adj(B) adj( A ) y cof( A B) = [cof( A )] [cof(B)] .
Solucin:
Partiendo de la propia definicin de la inversa podemos decir que:

B 1 A 1 =
[adj(B)] [adj(A )]
B A
1
A 1
= [adj(B)] [adj( A )] = [cof(B)] [cof( A)]T
T
A BB

A B (A B ) = [adj(B)] [adj( A )] =
1
( [cof(A)] [cof(B)] ) T
(1.38)

AB
[adj(A B)] = [adj(B)] [adj(A)] = ([cof(A)] [cof(B)])T
A B
adj( A B) = [adj(B)] [adj( A )] = ([cof( A )] [cof(B)])
T

donde hemos utilizado la propiedad que A B = A B . Adems teniendo en cuenta la


definicin de adjunta y cofactor concluimos que:

adj( A B) = ([cof( A B)]) = ([cof( A )] [cof(B)]) [cof( A B)] = [cof( A )] [cof(B)]


T T
(1.39)

Ejemplo 1.56
Demostrar que:
r r r r
( A a) ( A b) = [cof( A )] (a b) (1.40)

Solucin:

Universidad de Castilla- La Mancha Draft Por: Eduardo W. V. Chaves (2012)


Ciudad Real - Espaa
1 TENSORES 37

Partiendo de la relacin A tpq = rjk A rt A jp A kq (ver Ejemplo 1.46), y multiplicando ambos


lados por a t b p , resultando:
A tpq a t b p = rjk A rt A jp A kq a t b p = rjk ( A rt a t )( A jp b p ) A kq

Multiplicamos ambos lados por A qs1 obtenemos que:

A tpq a t b p A qs1 = rjk ( A rt a t )( A jp b p ) A kq A qs1 = rjk ( A rt a t )( A jp b p ) ks = rjs ( A rt a t )( A jp b p )

1
[cof ( A )] sq
Notar que A qs = , con lo cual la ecuacin anterior queda:
A

1
[cof ( A )] sq
A tpq a t b p A qs = A tpq a t b p = [cof ( A )] sq tpq a t b p = rjs ( A rt a t )( A jp b p )
A
r r r r
[cof( A )] (a b) = ( A a) ( A b)

Ejemplo 1.57
Demostrar que:
r
[ r r
] r r
[r r
] r r
[ ]
r r r
a ( A b) ( A c ) + ( A a) b ( A c ) + ( A a) ( A b) c = Tr ([cof ( A )]) a (b c ) [ ]
(1.41)
Solucin:
r r r r
En el ejemplo anterior hemos demostrado que [cof( A )] (a b) = ( A a) ( A b) , luego se
cumplen que:
r r r r r
[ r
a [cof( A )] (b c ) = a ( A b) ( A c ) ]
r r r r
b [cof( A )] (a c ) = b [( A a) ( A c ) ] = ( A a) b ( A c )
r r r r r
[ ]
r r r r r
[ r r
] r
c [cof( A )] (a b) = c ( A a) ( A b) = ( A a) ( A b) c
r
[ ]
Sumando las relaciones anteriores, podemos obtener que:
r r r r r r r r r
a [cof( A )] (b c ) b [cof( A )] (a c ) + c [cof( A )] (a b) =
r
[r r
]
r r r
[ r
] r r
= a ( A b) ( A c ) + ( A a) b ( A c ) + ( A a) ( A b) c [ ]
Segn el Ejemplo 1.48 se cumple que:
([cof( A)] ar ) (b cr ) ([cof( A)] b) (ar cr ) + ([cof( A)] cr ) (ar b) = Tr ([cof( A)])[rcr (ar b)]
T
r T
r T
r r
r r
= II A [c (a b)]
donde II A = Tr [cof( A )] es el segundo invariante principal de A . Con lo cual demostramos
que:
r
[ r r
] r r
[r r
] r r
[ r r r
]
a ( A b) ( A c ) + ( A a) b ( A c ) + ( A a) ( A b) c = II A a (b c ) [ ]
NOTA 1: Podemos resumir que:
r r r r
[r r r r
] [ r r r r
( A a) (b c) + a ( A b) c ) + a b ( A c ) = I A a (b c ) ] [ ] (ver Ejemplo 1.48) (1.42)

Universidad de Castilla- La Mancha Draft Por: Eduardo W. V. Chaves (2012)


Ciudad Real - Espaa
38 PROBLEMAS RESUELTOS DE MECNICA DEL MEDIO CONTINUO

r
[ r r
] r r r
[ r
] r r
[ r r r
]
a ( A b) ( A c) + ( A a) b ( A c ) + ( A a) ( A b) c = II A a (b c ) [ ] (1.43)
r
[ r r
] r r r
( A a) ( A b) ( A c ) = III A a (b c )[ ] (ver Ejemplo 1.49) (1.44)

donde I A = Tr (A ) , II A = Tr ([cof( A )]) , III A = det (A ) .


r r r r r r
NOTA 2: Si consideramos tres vectores linealmente independientes [a (b c )] [a, b, c ] 0 ,
y tres vectores tal que:
r r r r r r
f = 1a + 2 b + 3 c f
r 2 3 ar
r r 1
2 3 b
r r
g = 1 a + 2 b + 3 c g = 1 (1.45)
r r r r r r
2 3 c
h = 1 a + 2 + 3c
b h 1
Segn la regla de Cramer, (ver Ejemplo 1.16), se cumple que:
r r r r r r r r r r r r
f (b c ) [ f , b, c ] [a, f , c ] [a, b, f ]
1 = r r r r r r ; 2 = r r r ; 3 = r r r
a (b c ) [a, b, c ] [a, b, c ] [a, b, c ]
r r r r r r r r r
[g, b, c ] [a, g, c ] [a, b, g]
1 = r r r ; 2 = r r r ; 3 = r r r
[a, b, c ] [a, b, c ] [a, b, c ]
r r r r r r r r r
[h, b, c ] [a, h, c ] [a, b, h]
1 = r r r ; 2 = r r r ; 3 = r r r
[a, b, c ] [a, b, c ] [a, b, c ]
r r r r r r
Desarrollando el triple producto escalar [ f (g h)] [f , g, h] , podemos obtener que:

r r r 1 2 3
r r r
[ f (g h)] = 1 2 3 [a, b, c ]
1 2 3
r r r r r r r r r
[ f , b, c ] [a, f , c ] [a, b, f ]
1 r r r r r r r r r r r r r r r
= r r r [g , b, c ] [a, g, c ] [a, b, g] [a, b, c ] = P [a, b, c ]
[a, b, c ] r r r r r r r r r
[h, b, c ] [a, h, c ] [a, b, h]

donde
r r r r r r r r r
1 2 3 [ f , b, c ] [a, f , c ] [a, b, f ]
1 r r r r r r r r r
P = 1
2 3 = r r r [gr , br , c ] [a, gr , c ] [a, br , gr ] (1.46)
r r r r
1 2 3 [a, b, c ] [h, b, c ] [a, h, c ] [a, b, h]
r r r r r r
Para el caso cuando f = A a , g = A b , h = A c , los invariantes principales de P vienen
dados por:
1
(
r r r r r r r r r
I P = Tr ( P ) = r r r [ A a, b, c] + [a, A b, c] + [a, b, A c ] = I A
[a, b, c]
)

Universidad de Castilla- La Mancha Draft Por: Eduardo W. V. Chaves (2012)


Ciudad Real - Espaa
1 TENSORES 39

r r r r r r r r r r r r r r r r r r
[a , c ] [a, b, A b] [ A a, b, c ] [a, b, A a] [ A a, b, c ] [a, A a, c]
II P = r r r
1 r, A b r r r + r r r r r r + r r r r r
([a, b, c ]) 2 [a, A cr , cr ] [a r
, b, A c ] [ A c, b, c] [a, b, A c ] [ A b, b, c ] [a, A b, c ]

= II A
III P = III A = det (A )
NOTA 3: Consideremos el sistema Cartesiano donde
r r
a = a1e 1 + a 2 e 2 + a 3 e 3 a a a a e 1
r r 1 2 3

b = b
e
1 1 + b 2 2

e + b 3 3

e b =
1 b b 2 b 3 e 2
r cr c c c 3 e 3
c = c 1e 1 + c 2 e 2 + c 3 e 3 1
2
r r r
Consideremos tambin que f = e 1 , g = e 2 , h = e 3 , luego, teniendo en cuenta la ecuacin
anterior y la ecuacin (1.45) podemos concluir que:
r r
f 2 3 ar 1 0 0 e 1 1 2 3 a1 a 2 a 3 e 1
r
1
2 3 b 0 1 0 e =
2 3 b1 b 2 b 3 e 2
g = 1 2 1
r r
2 3 c 0 0 1 e 3 1 2 3 c 1 c 2 c 3 e 3
h 1
luego
1
1 2 3 a 1 a 2 a 3 1 0 0 1 2 3 a1 a 2 a 3
2 3 b1 b 2 b 3 = 0 1 0 2 3 = b1 b 2 b 3
1 1
1 2 3 c 1 c 2 c 3 0 0 1 1 2 3 c 1 c 2 c 3
con lo cual podemos obtener la inversa de un tensor. Consideremos el tensor A cuyas
componentes son:
A 11 A 12 A 13 a1 a2 a3
r r r
A ij = A 21 A 22 A 23 = b1 b2 b 3 A = [a, b, c ]
A 31 A 32 A 33 c 1 c2 c 3

Luego, la inversa P = A 1 , (ver ecuacin (1.46)) resulta:


r r r r r r r r r
[ f , b, c ] [a, f , c ] [a, b, f ]
1 r r r r r r r r r
A 1 = r r r [g, b, c ] [a, g, c ] [a, b, g]
[a, b, c ] r r r r r r r r r
[h, b, c ] [a, h, c ] [a, b, h]

1 0 0 a1 a2 a3 a1 a 3
a2

b1 b2 b3 1 0 0 b1 b 2 b 3
b2 b3 a2 a3 a2 a3
c1 c2 c3 c1 c2 c3 1 0 0
c2 c3 c2 c3 b2 b3
0 1 0 a1 a2 a3 a1 a 2 a 3
1 1 b1 b3 a1 a3 a1 a 3
= b1 b2 b3 0 1 0 b1 b 2 b 3 =
A A c1 c3 c1 c3 b1 b 3
c1 c2 c3 c1 c2 c3 0 1 0
b1 b2 a1 a2 a1 a 2

0 0 1 a1 a2 a3 a1 a 2 a 3 c 1 c2 c1 c2 b1 b 2

b1 b2 b3 0 0 1 b1 b 2 b 3

c c2 c3 c1 c2 c3 0 0 1
1

Universidad de Castilla- La Mancha Draft Por: Eduardo W. V. Chaves (2012)


Ciudad Real - Espaa
40 PROBLEMAS RESUELTOS DE MECNICA DEL MEDIO CONTINUO

1
Teniendo en cuenta que A 1 = [cof( A )]T = 1 [adj( A )] , podemos concluir que:
A A
T
b b3 a2 a3 a2 a3 b b3 b1 b3 b1 b2
2 2
c2 c3 c2 c3 b2 b3 c2 c3 c1 c3 c1 c 2

b b3 a1 a3 a1 a 3 a a3 a1 a3 a1 a 2
[cof( A )]ij = 1 = 2
c c3 c1 c3 b1 b 3 c2 c3 c1 c3 c1 c 2
1
b1 b2 a1 a2 a1 a 2 a2 a3 a1 a3 a1 a 2

c 1 c2 c1 c2 b1 b 2
b 2 b3 b1 b3 b1 b 2

Notar que el coeficiente de la matriz anterior, [cof(A )]ij , es obtenido al resolver el


determinante de la matriz resultante al eliminar la fila i th y la columna j th , y cuyo resultado
multiplicamos por (1) i + j , por ejemplo:
a1 a 2 a3
b b3
[cof(A)]12 = (1) 1+ 2
b1 b 2 b3 = 1
c1 c 3
c1 c2 c3

Ejemplo 1.58
Dados los escalares I C , II C , III C en funcin de los escalares I E , II E , III E :
I C = 2I E + 3

II C = 4 I E + 4 II E + 3 (1.47)

III C = 2 I E + 4 II E + 8 III E + 1
Obtener la forma inversa, i.e. obtener I E , II E , III E en funcin de I C , II C , III C .
Solucin:
Las ecuaciones en (1.47) pueden ser reestructuradas como sigue:
I C 2 0 0 I E 3 2 0 0 I E I C 3
4 4 0 II = II 3
II C = 4 4 0 II E + 3 E C
III 2 4 8 III 1 2 4 8 III E III C 1
C E
1 1
2 0 0 2 0 0 I E 2 0 0 I C 3

4 4 0 4 4 0 II E = 4 4 0 II C 3
2 4 8 2 4 8 III E 2 4 8 III C 1
1
I E 2 0 0 I C 3

II E = 4 4 0 II C 3
III 2 4 8 III 1
E C
donde

Universidad de Castilla- La Mancha Draft Por: Eduardo W. V. Chaves (2012)


Ciudad Real - Espaa
1 TENSORES 41

T
4 0 4 0 4 4
1
1 4 8 2 8 2 4
2 0 0
2 0 0 1
1 1 0 0 2 0 2 0 1
A 1
= 4 4 0 = [cof( A )]T = = 0
A 64 4 8 2 8 2 4 2 4
2 4 8 1 1 1
0 0 2 0 2 0 8
8 8
4 0 4 0 4 4

con lo cual, los escalares I E , II E , III E puede ser obtenidos como:

1 1
0 0 2 ( I C 3)
2
1
I E 2 0 0 C
I 3 IC 3
1 1 1

II E = 4 4 0 II C 3 = 0 II C 3 = ( 2 I C + II C + 3)
III 2 4 8 III 1 2 4 4
E C 1 1 1 III C 1 1
8 8 8 8 ( I C II C + III C 1)

1.1.6 Descomposicin Aditiva de Tensores

Ejemplo 1.59
Encontrar un tensor de cuarto orden P tal que se cumpla que P : A = A dev , donde A es un
tensor de segundo orden.
Solucin: Teniendo en cuenta la descomposicin aditiva de un tensor en una parte esfrica y
otra desviadora, podemos obtener que:
Tr ( A ) Tr ( A )
A = A esf + A dev = 1 + A dev A dev = A 1
3 3
Recurriendo a la definicin de los tensores identidades de cuarto orden definidos por:
I = 11 = ik jl e i e j e k e l = I ijkl e i e j e k e l (1.48)
I = 11 = il jk e i e j e k e l = I ijkl e i e j e k e l (1.49)

I = 1 1 = ij kl e i e j e k e l = I ijkl e i e j e k e l (1.50)

donde se cumple que:


(
I : A = ik jl e i e j e k e l : A pq e p e q)( )
(
= ik jl A pq kp lq e i e j )
(
= ik jl A kl e i e j ) (1.51)
(
= A ij e i e j )
=A

Universidad de Castilla- La Mancha Draft Por: Eduardo W. V. Chaves (2012)


Ciudad Real - Espaa
42 PROBLEMAS RESUELTOS DE MECNICA DEL MEDIO CONTINUO

( )(
I : A = ij kl e i e j e k e l : A pq e p e q )
(
= ij kl A pq kp lq e i e j )
(
= ij kl A kl e i e j ) (1.52)
(
= A kk ij e i e j )
= Tr ( A )1
Entonces, podemos decir que:
Tr ( A ) 1 1 1
A dev = A 1 = I : A I : A = I I : A = I 1 1 : A
3 3 3 3
Con lo cual, concluimos que:
1
P = I 1 1
3
El tensor P es conocido como tensor proyeccin de cuarto orden, Holzapfel (2000).

1.1.7 Ley de Transformacin. Invariantes.

Ejemplo 1.60
Bajo una transformacin de base e i = a ij e j y considerando que las componentes de un tensor
de segundo orden en esta nueva base vienen dadas por:
Tij = a ik a jl Tkl

Demostrar que:
a) Tii = Tkk = Tr (T ) ; b) Tij T ji = Tkl Tlk ; c) det ( T ) = det ( T )
Solucin:
=j
a) Tij = a ik a jl Tkl i Tii = a ik a il Tkl = kl Tkl = Tkk = Tll

b) Tij T ji = (a ik a jl Tkl )(a jp a iq T pq ) = a ik a iq a jl a jp Tkl T pq = kq lp Tkl T pq = Tqp T pq = Tkl Tlk


123 123
= kq = lp

con lo cual hemos demostrado que Tr ( T 2 ) = Tr ( T T ) = Tij T ji


c) det ( Tij ) = det(a ik a jl Tkl ) = det (a ik )det (a jl )det ( Tkl ) = det( Tkl )
1
424
31424
3
=1 =1

Hemos demostrado que Tkk = Tr ( T ) , Tkl Tlk = Tr ( T T ) y det ( T ) son invariantes.

Ejemplo 1.61
Bajo la transformacin de base e i = a ij e j y de las componentes del tensor de segundo orden
simtrico T en esta nueva base Tij = a ik a jl Tkl . Mostrar que I T , II T , III T son invariantes:

I T = Tr ( T ) = Tii ; II T =
1
2
{2
}
I T Tr ( T 2 ) ; III T = det ( T )

Universidad de Castilla- La Mancha Draft Por: Eduardo W. V. Chaves (2012)


Ciudad Real - Espaa
1 TENSORES 43

Solucin:
=j
a) Tij = a ik a jl Tkl i Tii = a ik a il Tkl = kl Tkl = Tkk = Tll

b) Como ya hemos demostrado que I T es un invariante, para demostrar que II T es invariante


es suficiente demostrar que Tr ( T 2 ) = Tr ( T T ) = Tij T ji es un invariante.
Tij T ji = (a ik a jl Tkl )(a jp a iq T pq ) = a ik a iq a jl a jp Tkl T pq = kq lp Tkl T pq = Tqp T pq = Tkl Tlk
123 123
= kq = lp

con lo cual, demostramos que Tr ( T 2 ) = Tr ( T T ) = Tij T ji es un invariante.


c) det ( Tij ) = det(a ik a jl Tkl ) = det (a ik )det (a jl )det ( Tkl ) = det( Tkl )
1
424
31424
3
=1 =1

Acabamos de demostrar que Tkk = Tr (T ) , Tkl Tlk = Tr ( T T ) , det (T ) son invariantes.

Ejemplo 1.62
Demostrar que las siguientes relaciones son invariantes:
C12 + C 22 + C 32 ; C13 + C 23 + C 33 ; C14 + C 24 + C 34
donde C1 , C 2 , C 3 son los autovalores del tensor de segundo orden C .
Solucin:
Cualquier combinacin de los invariantes principales ser un invariante. Intentaremos expresar
las relaciones anteriores en funcin de los invariantes principales.
Consideremos la siguiente relacin:
2
1444424444
(
I C2 = (C1 + C 2 + C 3 ) = C12 + C 22 + C 32 + 2 C1 C 2 + C1 C 3 + C 2 C 3
3
)
II C

C12 + C 22 + C 32 = I C2 2 II C

Comprobando que C12 + C 22 + C 32 es un invariante. Anlogamente podemos obtener las dems


relaciones, con lo cual resumimos a continuacin:
C1 + C 2 + C3 = I C
C12 + C 22 + C32 = I C2 2 II C
C13 + C 23 + C33 = I C3 3 II C I C + 3 III C
C14 + C 24 + C34 = I C4 4 II C I C2 + 4 III C I C + 2 II C2
C15 + C 25 + C35 = I C5 5 II C I C3 + 5 III C I C2 + 5 II C2 I C 5 III C II C

Notar que tambin se cumple que:


( ) ( ) (
C1n +1 + C 2n +1 + C3n +1 = C1n + C 2n + C3n I C C1 C 2n 1 + C3n1 C 2 C1n1 + C3n1 C3 C1n 1 + C 2n1) ( )

Universidad de Castilla- La Mancha Draft Por: Eduardo W. V. Chaves (2012)


Ciudad Real - Espaa
44 PROBLEMAS RESUELTOS DE MECNICA DEL MEDIO CONTINUO

Ejemplo 1.63
Obtener las componentes de la siguiente operacin:
T = A T AT
donde Tij y a ij son las componentes de los tensores T y A , respectivamente.

Solucin: La expresin T = A T A T en notacin simblica queda:


(e a e b ) = a rs (e r e s ) T pq (e p e q ) a kl (e l e k )
Tab
= a rs T pq a kl sp ql (e r e k )
= a rp T pq a kq (e r e k )

Para obtener las componentes de T es suficiente hacer el doble producto escalar por la base
(e i e j ) , resultando:

(e a e b ) : (e i e j ) = a rp T pq a kq (e r e k ) : (e i e j )
Tab
ai bj = a rp T pq a kq ri kj
Tab
Tij = a ip T pq a jq

Observemos que esta operacin viene representada en forma matricial como:


T = A T AT
Si A es la matriz de transformacin entre bases ortonormales se cumple que A 1 = A T
luego, se cumple que T = A T T A , y la representacin de las componentes se muestran en
la Figura abajo:

T = A T AT x3

T33


T23

T22
T32
x3
T33 T13


T31 T12 x2

T13
T21
T23 T11
T32
T31 T22

T12
T21 x2 x1
T11

x1
T = AT T A
Figura 1.1: Ley de transformacin de base para tensor de segundo orden.

Universidad de Castilla- La Mancha Draft Por: Eduardo W. V. Chaves (2012)


Ciudad Real - Espaa
1 TENSORES 45

Ejemplo 1.64
Consideremos que las componentes de un tensor de segundo orden T , en el sistema de
referencia (x1 , x 2 , x3 ) , estn representadas por:
3 1 0
(T )ij = Tij = T = 1 3 0
0 0 1

Sabiendo que la matriz de transformacin de coordenadas del sistema (x1 , x 2 , x3 ) al sistema


(x1 , x 2 , x 3 ) viene dada por:

0 0 1
2 2
A= 0
2 2
2 2
0
2 2
Obtener las componentes del tensor Tij en el nuevo sistema de coordenadas (x1 , x 2 , x 3 ) .
Solucin: La ley de transformacin para un tensor de segundo orden es:
Tij = aik a jl Tkl

Para que la operacin anterior sea posible en forma matricial:


Tij = ai k Tk l ( al j ) T
Luego:
T = A T AT

0 2 2
0 1 0
2 2
2 3 1 0
2 2 2
T = 0 1 3 0 0
2 2 2 2
2 2 0 0 1

0 1 0 0
2 2

Efectuando la operacin de matrices obtenemos que:


1 0 0
T = 0 2 0
0 0 4

Universidad de Castilla- La Mancha Draft Por: Eduardo W. V. Chaves (2012)


Ciudad Real - Espaa
46 PROBLEMAS RESUELTOS DE MECNICA DEL MEDIO CONTINUO

Ejemplo 1.65
Encontrar la matriz de transformacin del sistema ( x, y , z ) al sistema x, y, z , ver Figura 1.2:

z = z
z = z

y

y = y
y

x x


x
x
Figura 1.2: Rotacin.

Solucin: Recordar que se cumple que: si tenemos un espacio inicial y sucesivas


transformaciones hasta el espacio final, la ley de transformacin del espacio inicial al espacio
final viene formada por el producto de las transformaciones en el sentido contrario. Es decir,
situamos en el espacio final y seguimos el sentido contrario de las flechas hasta el espacio
inicial, ver figura abajo.
B

r
a
A r
a

B 1
r
a A 1
C 1 C
espacio
inicial
r espacio
CBA a
final

A 1B 1 C 1 = (CBA ) 1 siortogonale
s A T B T C T = ( CBA ) T

Universidad de Castilla- La Mancha Draft Por: Eduardo W. V. Chaves (2012)


Ciudad Real - Espaa
1 TENSORES 47

Podemos observar que la obtencin del sistema x, y, z es una combinacin de rotaciones


mostradas a continuacin:
Rotacin segn eje z

z = z

del sistema x, y , z al x, y, z
y
cos sin 0

y A = sin cos 0
0 0 1
x x
con 0 360

Rotacin segn eje y


del sistema x, y, z al x, y, z
z = z

z cos 0 sin
B = 0 1 0
sin 0 cos

y = y con 0 180

y z = z z
x x

x
x

Rotacin segn eje z


z = z
z = z

y del sistema x, y, z al x, y, z

y = y
y cos sin 0

x x C = sin cos 0
0 0 1

con 0 360
x
x

Universidad de Castilla- La Mancha Draft Por: Eduardo W. V. Chaves (2012)


Ciudad Real - Espaa
48 PROBLEMAS RESUELTOS DE MECNICA DEL MEDIO CONTINUO

La matriz de transformacin del sistema ( x, y , z ) para el sistema x, y, z ser dada por:


D = CBA
Resultando:
(cos cos cos sin sin ) (sin cos cos + cos sin ) sin cos
D = ( cos cos sin sin cos ) ( sin cos sin + cos cos ) sin sin

cos sin sin sin cos

Los ngulos , , son conocidos como los ngulos de Euler.

Ejemplo 1.66
Si a ij representan las componentes de la matriz de transformacin de base demostrar que las
siguientes ecuaciones se cumplen:
a11
2 2
+ a12 + a132
=1 a11
2 2
+ a 21 2
+ a 31 =1
2 2 2
2 2 2
a 21 + a 22 + a 23 = 1 a12 + a 22 + a 32 = 1
2 2 2 2 2 2
a 31 + a 32 + a 33 = 1 a13 + a 23 + a 33 = 1

a11 a 21 + a12 a 22 + a13 a 23 = 0 a11 a12 + a 21 a 22 + a 31 a 32 = 0

a 21 a31 + a 22 a 32 + a 23 a 33 = 0 a12 a13 + a 22 a 23 + a 32 a 33 = 0
a a + a a + a a = 0 a a + a a + a a = 0
11 31 12 32 13 33 11 13 21 23 31 33

Solucin:
Partimos del principio que la matriz de transformacin de base es una matriz ortogonal, i.e.
a ik a jk = a ki a kj = ij . Con lo cual:

(i = 1, j = 1) 2
a11 2
+ a12 2
+ a13 =1
2 2 2
(i = 2, j = 2) a 21 + a 22 + a 23 =1
2 2 2
(i = 3, j = 3) a31 + a 32 + a 33 =1
a ik a jk = a i1 a j1 + a i 2 a j 2 + a i 3 a j 3 = ij
(i = 1, j = 2) a11 a 21 + a12 a 22 + a13 a 23 = 0

(i = 2, j = 3) a 21 a 31 + a 22 a 32 + a 23 a 33 = 0
(i = 1, j = 3) a11 a 31 + a12 a 32 + a13 a 33 = 0

Solucin Alternativa:
a11 a12 a13 a11 a 21 a 31 1 0 0
AA = 1 T
a a 22 a 23 a12 a 22 a 32 = 0 1 0
21
a 31 a 32 a 33 a13 a 23 a 33 0 0 1

Al efectuar la multiplicacin de matrices obtenemos que:


2
a11 2
+ a12 2
+ a13 a11 a 21 + a12 a 22 + a13 a 23 a11 a 31 + a12 a 32 + a13 a 33 1 0 0

a11 a 21 + a12 a 22 + a13 a 23
2
a 21 2
+ a 22 2
+ a 23 a 21 a 31 + a 22 a 32 + a 23 a 33 = 0 1 0
a a + a a + a a a 21 a 31 + a 22 a 32 + a 23 a 33 2
a 31 2
+ a 32 2
+ a 33 0 0 1
11 31 12 32 13 33

Universidad de Castilla- La Mancha Draft Por: Eduardo W. V. Chaves (2012)


Ciudad Real - Espaa
1 TENSORES 49

1.1.8 Autovalores, Autovectores y Transformaciones


Ortogonales

Ejemplo 1.67
Demostrar que si Q es un tensor de segundo orden ortogonal propio, y E es un tensor de
segundo orden, los autovalores de E no cambian con la transformacin:
E* = Q E QT
Solucin:
Los autovalores ( i ) del tensor E obtenemos a partir del determinante caracterstico:

(
0 = det E * 1 ) (
0 = det E *ij ij )
= det (Q E Q 1 )
T
= det (Q ik E kp Q jp ij )
= det (Q E Q Q 1 Q )
T T
= det (Q Q ik Q jp kp )
ik E kp Q jp
= det [Q (E 1 ) Q ] T
[ (
= det Q ik E kp kp Q jp ) ]
= det
12Q
(3) det (E 1) det
1
(Q3)
424
T
(
= (Q ik )det E kp kp )det (Q ) jp
1 1
= det (E 1 ) (
= det E kp kp )
Con lo que comprobamos que E y E * tienen los mismos autovalores.

Ejemplo 1.68
Sea A un tensor de segundo orden y Q un tensor ortogonal. Si la ley de transformacin
ortogonal aplicada a A viene dada por A * = Q A Q T , demostrar que A 2 = Q A 2 Q T .
*

Solucin:
A 2 = A* A* ( A 2 ) ij = ( A * A * ) ij = A *ik A *kj
* *

= ( Q A Q T ) (Q A Q T ) = (Q ip A pr Q kr )(Q ks A st Q jt )
= QA Q QA Q
T T
= Q ip A pr Q kr Q ks A st Q jt
123 123
=1
= rs
= Q A A QT
= Q ip A pr rs A st Q jt = Q ip A ps A st Q jt
= Q A 2 QT
= Q ip ( A A ) pt Q jt
= (Q A 2 Q T ) ij

Ejemplo 1.69
Dadas las componentes del tensor T :
5 3 3
Tij = 2 6 3
2 2 4

Se pide:

Universidad de Castilla- La Mancha Draft Por: Eduardo W. V. Chaves (2012)


Ciudad Real - Espaa
50 PROBLEMAS RESUELTOS DE MECNICA DEL MEDIO CONTINUO

a) Obtener los invariantes principales de T ;


b) Obtener el polinomio caracterstico asociado a T ;
c) Si 1 , 2 y 3 son los autovalores de T y 1 = 10 . Obtener 2 y 3 > 2 .
Solucin:
a) Los invariantes principales de T son:
I T = Tr ( T ) = 5 + 6 + 4 = 15

6 3 5 3 5 3
II T = + + = 56
2 4 2 4 2 6
III T = det ( T ) = 60
b) El polinomio caracterstico se obtiene al resolver el determinante:
5 3 3
2 6 3 =0 3 2 I T + II T III T = 0 luego:
2 2 4

3 152 + 56 60 = 0
c) En el espacio principal se cumple que:
1 = 10 0 0
Tij = 0 2 0

0 0 3 > 2
donde los invariantes principales son
I T = Tr ( T ) = 1 + 2 + 3 = 15 2 + 3 = 5
III T = det ( T ) = 1 2 3 = 60 23 = 6
Combinando estas dos ecuaciones:
23 = 6 2
(31) = 3
(5 3 ) 3 = 6 3 5 3 + 6 = 0 (2)
2 + 3 = 5 3 = 2

Descartamos la solucin (32) = 2 por la imposicin del problema, luego, 3 = 3 . Resumiendo


as:
10 0 0 I T = 10 + 2 + 3 = 15

Tij = 0 2 0 donde se puede comprobar que: II T = 2 3 + 10 3 + 10 2 = 56
0 0 3 III = 10 2 3 = 60
T

Ejemplo 1.70
Determinar los valores principales y las direcciones principales del tensor cartesiano de
segundo orden T , cuyas componentes se representan matricialmente por:
3 1 0
(T )ij = Tij = T = 1 3 0
0 0 1

Universidad de Castilla- La Mancha Draft Por: Eduardo W. V. Chaves (2012)


Ciudad Real - Espaa
1 TENSORES 51

Solucin: Buscamos soluciones no triviales para (Tij ij ) n j = 0 i , con la restriccin de que


n j n j = 1 . Como ya hemos visto, la solucin no trivial requiere la condicin:

Tij ij = 0

Explcitamente, la expresin anterior queda:


T11 T12 T13 3 1 0
T21 T22 T23 = 1 3 0 =0
T31 T32 T33 0 0 1

Desarrollando el determinante anterior obtenemos la ecuacin cbica:


[
(1 ) (3 ) 2 1 = 0 ]
3 2
7 + 14 8 = 0
Podramos haber obtenido directamente la ecuacin caracterstica anterior a travs de los
invariantes:
I T = Tr ( Tij ) = Tii = T11 + T22 + T33 = 7

T T23 T11 T13 T11 T12


II T =
1
2
( )
Tii T jj Tij Tij = 22
T32 T33
+
T31 T33
+
T21 T22
= 14

III T = Tij = ijk Ti1 T j 2 Tk 3 = 8

utilizando la ecuacin caracterstica ser:


3 2 I T + II T III T = 0 3 72 + 14 8 = 0
Resolviendo la ecuacin cbica podemos obtener las tres races reales, puesto que la matriz T
es simtrica:
1 = 1; 2 = 2; 3 = 4
Podemos adems comprobar si los invariantes estn bien calculados utilizando la expresin de
los invariantes en funcin de los autovalores:
I T = 1 + 2 + 3 = 1 + 2 + 4 = 7
II T = 1 2 + 2 3 + 3 1 = 1 2 + 2 4 + 4 1 = 14
III T = 1 2 3 = 8
Con lo que podemos comprobar que los invariantes son los mismos que los obtenidos
anteriormente.
Clculo de las direcciones principales:
Para obtener las direcciones principales, utilizamos la definicin de autovalor-autovector,
donde cada autovalor i est asociado a un autovector n (i ) .
Para 1 = 1
3 1 1 0 n 1 3 1 1 0 n1 0
1 3 1 0 n 2 = 1 3 1 0 n 2 = 0


0 0 1 1 n 3 0 0 1 1 n 3 0

resultando el siguiente sistema de ecuaciones:

Universidad de Castilla- La Mancha Draft Por: Eduardo W. V. Chaves (2012)


Ciudad Real - Espaa
52 PROBLEMAS RESUELTOS DE MECNICA DEL MEDIO CONTINUO

2n1 n 2 = 0
n1 = n 2 = 0
n1 + 2n 2 = 0
0n = 0
3
n i n i = n12 + n 22 + n 32 = 1

Luego, podemos obtener que: 1 = 1 n (1) = [ 0 0 1] .


NOTA: Esta solucin podra haberse determinado previamente por la situacin particular que
presentan las componentes del tensor. Al ser los trminos T13 = T23 = T31 = T32 = 0 , T33 = 1 ya
es un valor principal, como consecuencia esta direccin ya es una direccin principal.
Para 2 = 2
3 2 1 0 n 1 3 2 1 0 n1 0
1 3 2 0 n = 1 3 2 0 n 2 = 0
2
0 0 1 2 n 3 0 0 1 2 n 3 0

n1 n 2 = 0 n1 = n 2

n1 + n 2 = 0
n = 0
3
Podemos observar que las dos primeras ecuaciones son linealmente dependientes.
Necesitamos entonces de una ecuacin adicional:
1
n i n i = n12 + n 22 + n 32 = 1 2n12 = 1 n1 =
2
Luego:
1 1
2 = 2 n (2) = 0
2 2
Para 3 = 4
3 3 1 0 n 1 3 4 1 0 n1 0
1 3 3
0 n 2 = 1 3 4 0 n 2 = 0

0 0 1 3 n 3 0 0 1 4 n 3 0

n1 n 2 = 0
n 2 = n 2
n1 n 2 = 0
3n = 0
3

1
n i n i = n12 + n 22 + n 32 = 1 2n12 = 1 n1 =
2
Resultando:
1 1
3 = 4 n (3) = m 0
2 2
Podemos entonces resumir que las direcciones principales correspondientes a sus valores
principales son:

Universidad de Castilla- La Mancha Draft Por: Eduardo W. V. Chaves (2012)


Ciudad Real - Espaa
1 TENSORES 53

1 n (1) = [ 0 0 1]

1 1
2 n (2) = 0
2 2
1 1
3 n (3) = m 0
2 2
NOTA: Las componentes utilizadas en este ejemplo son las mismas utilizadas en el Ejemplo
1.64. Adicionamente, podemos verificar que los autovectores forman la matriz de
trnasformacin, A , entre el sistema original, (x1 , x 2 , x3 ) , y el espacio rpincipal, (x1 , x 2 , x 3 ) ,
(ver Ejemplo 1.64).

Ejemplo 1.71
Dado un tensor ortogonal propio Q , a) demostrar que Q tiene un autovalor real e igual a 1 .
b) Demostrar tambin que Q puede ser representado en funcin de un ngulo tal que:
Q = p p + cos (q q + r r ) sin (q r r q )

donde p , q , r , son versores que constituyen una base ortonormal, siendo p la direccin
correspondiente al autovalor = 1 , es decir, p es autovector de Q . c) Obtener los invariantes
r
principales de Q en funcin del ngulo . d) Dado el vector posicin x , determinar el nuevo
r
vector formado por la transformacin ortogonal Q x en el espacio p , q .
Solucin:
a) Teniendo en cuenta la definicin de tensor ortogonal, podemos decir que:
QT Q = 1
QT Q QT = 1 QT
Q T (Q 1) = (Q T 1)
Q T (Q 1) = (Q 1) T
A continuacin obtenemos el determinante de los dos tensores anteriores:
[ ] [
det Q T (Q 1) = det (Q 1) T ] [
= ( 1) 3 det (Q 1) T ]
1424 3
[ ]
det Q det[(Q 1)] = det (Q 1)
T
[ T
] = det[(Q 1)]
= detQ =1

det[(Q 1)] = det[(Q 1)]

donde hemos utilizado las siguientes propiedades del determinante: det[A ] = 3 det[A ] ,
[ ]
det A T = det[A ] , det[A B ] = det[A ]det[B ] . El nico escalar que cumple la expresin
anterior es el cero, luego:
det[(Q 1)] = 0
Teniendo en cuenta la definicin de autovalor, det[(Q 1)] = 0 , concluimos que cuando
= 1 cumple det[(Q 1)] = 0 , luego = 1 es autovalor de Q . Adems, existe una direccin
(autovector) que cumple que Q e 1* = e 1* = e 1* .
b) Vamos considerar que p e 1* , q e *2 , r e *3 constituye una base ortonormal.

Universidad de Castilla- La Mancha Draft Por: Eduardo W. V. Chaves (2012)


Ciudad Real - Espaa
54 PROBLEMAS RESUELTOS DE MECNICA DEL MEDIO CONTINUO

e 3
e 1* p
q e *2
Q e 1* = e 1*

e 2

e1 r e *3

La representacin simblica del tensor en la base e1* , e *2 , e *3 , viene dada por:

Q = Q *ij e *i e *j
* *
= Q11 e1 e 1* + Q12
* *
e1 e *2 + Q13
* *
e1 e *3 +
(1.53)
+ Q *21e *2 e 1* + Q *22 e *2 e *2 + Q *23 e *2 e *3 +
+ Q *31e *3 e 1* + Q *32 e *3 e *2 + Q *33 e *3 e *3

Teniendo en cuenta que e1* es autovector de Q asociado al autovalor = 1 , se cumple que


Q e 1* = e 1* = e 1* , adems haciendo la proyeccin de Q , dado por (1.53), segn direccin e1* ,
obtenemos que:
Q e 1* = e 1*
Q e 1* = [ Q11
* *
e1 e 1* + Q12
* *
e1 e *2 + Q13
* *
e1 e *3 +
+ Q *21e *2 e 1* + Q *22 e *2 e *2 + Q *23 e *2 e *3 +
+ Q *31e *3 e 1* + Q *32 e *3 e *2 + Q *33 e *3 e *3 ] e 1*
* *
= Q11 e1 + Q *21e *2 + Q *31e *3
*
Con lo cual concluimos que Q11 = 1 , Q *21 = 0 , Q *31 = 0 .
Recordar que dos tensores coaxiales tienen las mismas direcciones principales. Un tensor y su
inversa siempre sern tensores coaxiales, luego si Q 1 = Q T , eso implica que Q T y Q son
coaxiales, y e1* tambin ser direccin principal de Q T , luego se cumple que:
Q T e 1* = e 1*
Q T e 1* = [ Q11
* *
e1 e 1* + Q *21e 1* e *2 + Q *31e 1* e *3 +
* *
+ Q12 e 2 e 1* + Q *22 e *2 e *2 + Q *32 e *2 e *3 +
* *
+ Q13 e 3 e 1* + Q *23 e *3 e *2 + Q *33 e *3 e *3 ] e 1*
* * * * * *
= Q11 e1 + Q12 e 2 + Q13 e3
* * *
Con lo cual concluimos que Q11 = 1 , Q12 = 0 , Q13 = 0 . Luego, la expresin (1.53) queda:

Q = e 1* e 1* + Q *22 e *2 e *2 + Q *23 e *2 e *3 + Q *32 e *3 e *2 + Q *33 e *3 e *3 (1.54)

Universidad de Castilla- La Mancha Draft Por: Eduardo W. V. Chaves (2012)


Ciudad Real - Espaa
1 TENSORES 55

En forma de matriz, las componentes de Q en la base e *i vienen dadas por:

x 2*
Q *22
1 0 0
Q ij = 0 Q *22
*
Q *23
*
0 Q *32 Q *33 Q11 =1
Q *32
Q *23 x1*

Q *33

x3*
Recurrimos una vez ms a la condicin de ortogonalidad Q T Q = Q Q T = 1 , o en funcin de
las componentes en el espacio e *i :
1 0 0 1 0 0 1 0 0
Q *ki Q *kj = ij 0 Q * Q *32 0 Q *22 Q *23 = 0 1 0
22
0 Q *23 Q *33 0 Q *32 Q *33 0 0 1
(1.55)
1 0 0 1 0 0
0 [(Q )
* 2
22 + (Q *32 ) 2 ] [Q Q *
22 23
*
+ Q *32 Q *33 ]
= 0 1 0

0 [Q Q
*
22 23
*
+ Q *32 Q *33 ] [(Q ) * 2
33 + (Q *23 ) 2 ] 0 0 1

El determinante de un tensor ortogonal propio es det (Q) = +1 :


1 0 0
0 Q * Q *23 = 1 Q *22 Q *33 Q *23 Q *32 = 1
22 (1.56)
0 Q *32 Q *33

Teniendo en cuenta (1.55) y (1.56) tenemos el siguiente conjunto de ecuaciones:


(Q *22 ) 2 + (Q *32 ) 2 = 1 cos 2 + sin 2 = 1
* * * *
Q 22 Q 23 + Q 32 Q 33 = 0 cos ( sin ) + sin cos = 0
* 2 * 2 2 2
(Q 33 ) + (Q 23 ) = 1 cos + sin = 1
* * * * cos cos ( sin )(sin ) = 1
Q 22 Q 33 Q 23 Q 32 = 1
Con lo cual hemos demostrado la existencia de un ngulo que cumpla con las condiciones
anteriores.
1 0 0 1 0 0
Q *ij = 0 Q *22 *
Q 32 = 0 cos sin

(1.57)
0 Q *23 Q *33 0 sin cos
Retomando la expresin (1.54), y teniendo en cuenta (1.57), concluimos que:
Q = e 1* e 1* + (cos ) e *2 e *2 + ( sin )e *2 e *3 + (sin )e *3 e *2 + (cos ) e *3 e *3
[ ] [
= e 1* e 1* + cos e *2 e *2 + e *3 e *3 sin e *2 e *3 e *3 e *2 ]

Universidad de Castilla- La Mancha Draft Por: Eduardo W. V. Chaves (2012)


Ciudad Real - Espaa
56 PROBLEMAS RESUELTOS DE MECNICA DEL MEDIO CONTINUO

Considerando que p e 1* , q e *2 , r e *3 , demostramos que:


Q = p p + cos (q q + r r ) sin (q r r q )
Es interesante verificar que la descomposicin aditiva de Q en una parte simtrica y otra
antisimtrica, en el espacio e *i , resulta:
1 0 0 0 0 0
Q *ijsym
= 0 cos 0 ; Q *ijanti
= 0 0 sin
0 0 cos 0 sin 0
1444 4244443 1444 44244444 3
[p p +cos (q q +r r )]ij [sin (q r r q ) ]ij
anti
Observemos que el formato de Q *ij tiene el mismo formato que presenta un tensor
antisimtrico ( W ) en el espacio definido por su vector axil:
0 0 0
Wij* = 0 0
0 0
donde es el mdulo del vector axil.
c) Teniendo en cuenta (1.57), queda de fcil demostracin que I Q = II Q = 1 + 2 cos , III Q = 1 .
r
d) Representamos el vector posicin x a travs de sus componentes y la base p , q , r :
r
x = pp + qq + rr .
r r r
Luego, se cumple que: x p = ( pp + qq + rr ) p = p ; x q = q ; x r = r
Luego, ver Figura 1.3, se cumple que:
r
[
~ = Q xr = p p + cos (q q + r r ) sin (q r r q )
x ] [pp + qq + rr ]
= pp + (q cos r sin )q + (r cos + q sin )r

e 1* p r
x
q e *2
O r r
~
x =Q x

r e *3
Figura 1.3

Universidad de Castilla- La Mancha Draft Por: Eduardo W. V. Chaves (2012)


Ciudad Real - Espaa
1 TENSORES 57

Ejemplo 1.72
r r r r
Considrense las transformaciones tensoriales p = U p y p = R p , donde R es un tensor
ortogonal de segundo orden y U es un tensor de segundo orden con U U 1 = 1 , i.e. U 1 .
r r
Obtener las leyes de transformacin entre p y p .
Solucin:
El problema planteado se puede esquematizar a travs de la siguiente figura:

r R
U p

r
r p
p
?

Teniendo en cuenta que R 1 = R T (tensor ortogonal), es decir, existe la inversa de R y


r r
considerando p = R p obtenemos que:
r r
p = R p
r r
R 1 p = R 1 R p
r r r
R 1 p = 1 p = p
r r r r
Reemplazando p = R 1 p en p = U p , obtenemos que:
r r r r
p = U p p = U p
r r r r
R 1 p = U p R 1 p = U p
r r r r
R R 1 p = R U p U 1 R 1 p = U 1 U p (1.58)
r r r r r
1 p = R U p (R U) 1 p = 1 p = p
r r r r
p = (R U) p p = (R U) 1 p

o en Notacin indicial:
p i = U ij p j p i = U ij p j
R ij1p j = U ij p j R ij1p j = U ij p j
R ki R ij1p j = R ki U ij p j U ki1R ij1p j = U ki1U ij p j (1.59)
kj p j = R ki U ij p j (R ki U ij ) p j = kj p j = p k
1

p k = (R ki U ij )p j p k = (R ki U ij ) 1 p j

Universidad de Castilla- La Mancha Draft Por: Eduardo W. V. Chaves (2012)


Ciudad Real - Espaa
58 PROBLEMAS RESUELTOS DE MECNICA DEL MEDIO CONTINUO

r
U p R

U 1
R 1 = R T
r r
p (R U) p

(R U) 1 = U 1 R T

1.1.9 Representacin Espectral

Ejemplo 1.73
Sea w un tensor antisimtrico de segundo orden y V un tensor de segundo orden definido
positivo cuya representacin espectral viene dado por:
3
V= a =1
a n ( a ) n ( a )

Demostrar que el tensor antisimtrico w puede ser representado por:


3
w = w ab n (a ) n (b)
a ,b =1
a b

Demostrar tambin que se cumple la relacin:


3
w V V w = w ab ( b a ) n ( a ) n (b)
a ,b =1
a b

Solucin:
Es cierto que
3
3
w 1 = w n ( a ) n ( a ) = w n ( a) n (a )
a =1 a =1

( )
3
r
= w n ( a ) n ( a )
a =1

w (n )
3
= b
(b )
n ( a ) n ( a )
a ,b =1
r r
donde hemos aplicado la propiedad de un tensor antisimtrico w n = w n , donde w es el
vector axil asociado al tensor w . Expandiendo la expresin anterior obtenemos que

Universidad de Castilla- La Mancha Draft Por: Eduardo W. V. Chaves (2012)


Ciudad Real - Espaa
1 TENSORES 59

w = wb (n (b) n (1) ) n (1) + wb (n (b) n ( 2) ) n ( 2) + wb (n (b) n (3) ) n (3) =


( ) ( )
= w1 n (1) n (1) n (1) + w2 n ( 2) n (1) n (1) + w3 n (3) n (1) n (1) + ( )
+ w (n
1
(1)
) ( )
n ( 2 ) n ( 2 ) + w2 n ( 2 ) n ( 2) n ( 2) + w3 n (3) n ( 2) n ( 2 ) + ( )
+ w (n
1
(1)
n ( 3)
) n ( 3)
+ w2 (n ( 2)
n ( 3)
) n ( 3)
+ w3 (n ( 3)
n ( 3)
) n ( 3)

Simplificando la expresin anterior resulta que:


w = w2 (n (3) ) n (1) + w3 (n ( 2) ) n (1) +
( )
+ w1 n (3) n ( 2 ) w3 n (1) n ( 2 ) + ( )
w1 (n ) n
( 2) ( 3)
+ w2 (n ) n
(1) ( 3)

Adems teniendo en cuanta que w1 = w 23 = w 32 , w2 = w13 = w 31 , w3 = w12 = w 21 , w


an puede ser expresado por:
w = w 31 n (3) n (1) + w 21 n ( 2) n (1) +
+ w 32 n (3) n ( 2 ) + w12 n (1) n ( 2 ) +
+ w 23 n ( 2 ) n (3) + w13 n (1) n (3)
el cual es exactamente igual a
3
w = w ab n (a ) n (b)
a ,b =1
a b

Los trminos wV y V w pueden ser obtenidos como sigue a continuacin:


3
3
a ,b =1

w V = w ab n n b n (b) n (b)
(a) (b )

b =1

ab
3 3
= b w ab n ( a ) n (b ) n (b ) n (b ) =
a ,b =1
w
a ,b =1
b ab n ( a ) n (b )
a b ab

y
3
3 (a )

a =1

V w = a n n
(a)
a ,b =1
w ab n n
(a) (b )



ab
3
= w
a ,b =1
a ab n ( a ) n (b )
a b

Luego,
3 3

a ,b =1
(a)
w V V w = b w ab n n a w ab n n
(b )

a ,b =1
(a) (b )



a b ab
3
= w
a ,b =1
ab ( b a ) n ( a ) n (b )
a b

Anlogamente, es posible demostrar que:

Universidad de Castilla- La Mancha Draft Por: Eduardo W. V. Chaves (2012)


Ciudad Real - Espaa
60 PROBLEMAS RESUELTOS DE MECNICA DEL MEDIO CONTINUO

3
w V 2 V 2 w = w ab (2b 2a ) n ( a ) n (b)
a ,b =1
a b

Ejemplo 1.74
Dado un tensor definido positivo C , cuyas componentes cartesianas de este tensor vienen
dadas por:
2 0 1
C ij = 0 4 0
1 0 2

Obtener los siguientes tensores: a) C 2 ; b) U = C . c) Comprobar si los tensores C y U son


coaxiales.
Solucin:
Observemos que los tensores C 2 y U = C son tensores coaxiales con el tensor C .
Haciendo la representacin espectral del tensor C :
3
C= N
a =1
a
(a) (a )
N

( a ) son los autovectores del tensor C . Luego,


donde a son los autovalores del tensor C , y N
se cumple que:
3 3
C2 =
a =1
(a) N
2a N (a) ; U= C =
a =1
(a) N
aN (a )

Clculo de los autovalores y autovectores del tensor C .


Podemos verificar que segn la estructura de las componentes del tensor C ya conocemos un
auto valor 2 = 4 que est asociado a la direccin N i( 2) = [0 1 0] . Para calcular los restantes
autovalores obtenemos el determinante caracterstico siguiente:
2 1 1 = 2 1 = 1
=0 ( 2 ) 2 = 12 ( 2 ) = 1
1 2 3 = 2 + 1 = 3
Asociado al autovalor 1 = 1 tenemos el siguiente autovector:

2 1 1 N 1(1) 0 1 1 N 1(1) 0
1 = 1 1 (1) = 0 N 1(1) = N 3(1)
2 1 N 3(1) 0 N 3

con la restriccin N i(1) N i(1) = 1 , resultando que:


N 1(1) N 1(1) + N (21) N (21) + N 3(1)N (31) = 1
N (1) N (1) + N (1)N (1) = 1
1 1 1 1
1
N 1(1) =
2
1
N 3(1) = N 1(1) = m
2

Universidad de Castilla- La Mancha Draft Por: Eduardo W. V. Chaves (2012)


Ciudad Real - Espaa
1 TENSORES 61

Asociado al autovalor 3 = 3 tenemos el siguiente autovector:

2 3 1 N 1(3) 0 1 1 N (3) 0
1 = 1 1 (3) = 0
1
N 1(3) = N 3(3)
2 3 N (33) 0 N
3

con la restriccin N i(3) N i(3) = 1 , resultando que:

N 1(3)N 1(3) + N (23) N (23) + N 3(3)N (33) = 1


1
N 1(3)N 1(3) + N 1(3) N 1(3) = 1 N 1(3) =
2
1
N 3(3) = N 1(3) =
2
Resumiendo tenemos que:

(1) = 1 1 1 1
1 = 1 N i 0 m 0
2 2
2 2
2 = 4 ( 2 ) = [0
N 1 0] Matriz
de transformacin
A= 0 1 0
i
1 1 1
3 = 3 ( 3) =
N
1
0 0
i 2 2
2 2

Luego se cumple que:


C = A C AT C = AT C A
En el espacio principal tenemos que:
1 0 0
2

Cij = 0 16 0
1 0 0 0 0 9

Cij = 0 4 0
1 0 0 1 0 0
0 0 3
U = C =0 4 0 = 0 2 0
ij ij

0 0 3 0 0 3

Observemos que el tensor C es un tensor definido positivo, luego sus autovalores son
positivos. En espacio original tenemos las siguientes componentes:
T
1 1 1 1
0 1 0 0 2 0
2 2 2 5 0 4
2
C ij = 0 1 0 0 16 0 0 1 0 = 0 16 0
1 1
0 0 0 9 1 0
1
4 0 5
2 2 2 2

Observemos que este resultado podra haber sido obtenido fcilmente a travs de la operacin
C 2 = C C , o en componentes:
2 0 1 2 0 1 5 0 4
C ij2 = C ik C kj = 0 4 0 0 4 0 = 0 16 0
1 0 2 1 0 2 4 0 5

Anlogamente:

Universidad de Castilla- La Mancha Draft Por: Eduardo W. V. Chaves (2012)


Ciudad Real - Espaa
62 PROBLEMAS RESUELTOS DE MECNICA DEL MEDIO CONTINUO

T
1 1 1 1 3 +1 3 1
0 1 0 0 0 0
2 2 2 2 2 2
U ij = 0 1 0 0 2 0 0 1 0 = 0 2 0
1 1 1 1 3 1
0 0 0 3 0 3 + 1
0

2 2
2 2 2 2

c) Los tensores C y U son coaxiales ya que hemos obtenido los autovalores de U en el


espacio principal de C . Tambin podemos comprobar que son tensores coaxiales porque se
cumple que C U = U C , en componentes.
3 +1 3 1
2 0 1 0 3,098 0 2,098
2 2
C ik U kj = 0 4 0 0 2 0 = 0 8 0
1 0 2 3 1 3 + 1 2,098 0 3,098
2 0
2

3 +1 3 1
0 2 0 1 3,098 0 2,098
2 2
U ik C kj = 0 2 0 0 4 0 = 0 8 0
3 1 0 3 + 1 1 0 2 2,098 0 3,098

2 2

Ejemplo 1.75
Sea C un tensor de segundo orden simtrico y R un tensor ortogonal propio. Las
componentes de estos tensores en el sistema Cartesiano vienen dadas por:

0 0 1
2 0 1
2 2
C ij = 0 4 0 ; R ij = 0
2 2
1 0 2
2 2
0
2 2
a) Obtener los siguientes tensores: a.1) C 8 ; a2) U = C .
b) Obtener tambin los invariantes principales de C .
c) Teniendo en cuenta que los tensores b y C estn relacionados entre si a travs de la
siguiente transformacin ortogonal C = R T b R , obtener el tercer invariante principal de b .
Solucin:
3281 0 3280

a) Anlogo al Ejemplo 1.74. Respuesta: C = 0 655368
0
3280 0 3281
b)
I C = Tr (C ij ) = C ii = C11 + C 22 + C 33 = 8

II C =
1
2
( )
C ii C jj C ij C ij =
4 0 2 1 2 0
+ +
0 2 1 2 0 4
= 19 ; III C = C = ijk C i1C j 2 C k 3 = 12

Universidad de Castilla- La Mancha Draft Por: Eduardo W. V. Chaves (2012)


Ciudad Real - Espaa
1 TENSORES 63

c) Teniendo en cuenta las propiedades de determinante, el tercer variante principal de b puede


ser expresado por:
C det (C ) = det (R T b R ) = det (R T )det (b)det (R ) = det (b) = III b = 12
1424 3 123
= +1 = +1

Ejemplo 1.76
Sea un tensor de segundo orden simtrico S con det (S ) 0 . Considerando que S tiene dos
autovalores iguales, i.e. S 2 = S 3 y S1 S 2 , demostrar que S puede ser representado por:
S = S 1n (1) n (1) + S 2 (1 n (1) n (1) )

donde n (1) es el autovector de S asociado al autovalor S1 , 1 es el tensor identidad de


segundo orden.
Solucin:
Partimos de la representacin espectral de S :
3
S= S n
a =1
a
(a)
n ( a )
(1.60)
= S1n (1) n (1) + S 2 n ( 2 ) n ( 2 ) + S 3 n (3) n (3)
= S1n (1) n (1) + S 2 (n ( 2 ) n ( 2 ) + n (3) n (3) )

Recordar que 1 es un tensor esfrico, con lo cual cualquier direccin es una direccin
principal. Partiendo de este principio adoptamos el espacio principal de S para hacer la
representacin espectral de 1 :
3
1= n
a =1
(a)
n ( a ) = n (1) n (1) + n ( 2 ) n ( 2 ) + n (3) n (3)
(1.61)
n ( 2)
n ( 2)
+ n ( 3)
n ( 3)
= 1 n (1)
n (1)

Reemplazando lo anterior en (1.60), obtenemos que:


S = S 1n (1) n (1) + S 2 (n ( 2) n ( 2) + n (3) n (3) )
= S 1n (1) n (1) + S 2 (1 n (1) n (1) )

1.1.10 Teorema de Cayley-Hamilton

Ejemplo 1.77
Dado un tensor de segundo orden arbitrario T , demostrar el teorema de Cayley-Hamilton,
que afirma que todo tensor cumple su propia ecuacin caracterstica.
Solucin:
Partimos de la ecuacin caracterstica del tensor: 3 2 I T + II T III T = 0 , cuya ecuacin se
cumple para los autovalores de T , 1 , 2 , 3 , luego:

Universidad de Castilla- La Mancha Draft Por: Eduardo W. V. Chaves (2012)


Ciudad Real - Espaa
64 PROBLEMAS RESUELTOS DE MECNICA DEL MEDIO CONTINUO

31 21 I T + 1 II T III T = 0
32 22 I T + 2 II T III T = 0
33 23 I T + 3 II T III T = 0
Reestructurando las ecuaciones anteriores en forma matricial, obtenemos que:
31 0 0 21 0 0 1 0 0 1 0 0 0 0 0

0 32 00 22 0 I T + 0 2 0 II T 0 1 0 III T = 0 0 0

0 0 33 0 0 23 0 0 3 0 0 1 0 0 0 (1.62)

Tij 3 Tij 2 I T + Tij II T III T ij = 0 ij

Notar que en el espacio principal de T se cumple que:


1 0 0

Tij = 0 2 0
0 0 3

1 0 0 1 0 0 21 0 0

Tij = Tik Tkj = 0
2
2 0 0 2 0 =0 22 0
0 0 3 0 0 3 0 0 23

1 0 0 1 0 0 1 0 0 31 0 0

Tij = Tik Tkp
3
T pj = 0 2 0 0 2 0 0 2 0 =0 32 0
0 0 3 0 0 3 0 0 3 0 0 33

Teniendo en cuenta la ley de transformacin entre espacios de un tensor de segundo orden, i.e.
Tij = Fik Tkp F pj1 , donde Fij es la matriz de transformacin del espacio original ( Tij ) al espacio
principal ( Tij ). Notar que tambin se cumple que Tij 2 = Fik Tkp2 F pj1 y Tij 3 = Fik Tkp3 F pj1 , (ver
Ejemplo 1.68). Con lo cual la ecuacin en (1.62) puede ser reescrita como:
Tij 3 Tij 2 I T + Tij II T III T ij = 0 ij
3
Fik Tkp F pj1 Fik Tkp2 F pj1 I T + Fik Tkp F pj1 II T III T Fik kp F pj1 = 0 ij
(
Fik Tkp3 Tkp2 I T + Tkp II T III T kp F pj1 = 0 ij)
Fsi1 Fik Tkp
3
( )
Tkp2 I T + Tkp II T III T kp F pj1 F jt = Fsi1 0 ij F jt = 0 st
( )
sk Tkp3 Tkp2 I T + Tkp II T III T kp pt = Fsi1 0 ij F jt = 0 st
Tst3 Tst2 I T + Tst II T III T st = 0 st
T 3 T 2 I T + T II T III T 1 = 0

Ejemplo 1.78
Partiendo del teorema de Cayley-Hamilton obtener la inversa de un tensor T en funcin de
potencia de tensores.
Solucin:
El teorema de Cayley-Hamilton afirma que:

Universidad de Castilla- La Mancha Draft Por: Eduardo W. V. Chaves (2012)


Ciudad Real - Espaa
1 TENSORES 65

T 3 T 2 I T + T II T III T 1 = 0

Haciendo el producto escalar de la expresin anterior por el tensor T 1 obtenemos que:


T 3 T 1 T 2 T 1 I T + T T 1 II T III T 1 T 1 = 0 T 1
T 2 TI T + 1 II T III T T 1 = 0

T 1 =
1
III T
(T 2 TI T + 1 II T )

Ejemplo 1.79
Dado el tensor T representado por sus componentes en el sistema cartesiano:
5 0 0
T = 0 2 0
0 0 1

Comprobar el teorema de Cayley-Hamilton.


Solucin:
El teorema de Cayley-Hamilton tambin se aplica para las componentes del tensor:
T 3 T 2 I T + T II T III T 1 = 0
donde:
I T = 5 + 2 + 1 = 8 ; II T = 10 + 2 + 5 = 17 ; III T = 10
luego:
5 3 0 0 125 0 0 5 2 0 0 25 0 0

T 3
=0 23 0 = 0 8 0 ; T 2
=0 22 0 = 0 4 0
0 0 1 0 0 1 0 0 1 0 0 1

Aplicando el teorema de Cayley-Hamilton verificamos que:
125 0 0 25 0 0 5 0 0 1 0 0 0 0 0
0 8 0 8 0 4 0 + 17 0 2 0 10 0 1 0 = 0 0 0

0 0 1 0 0 1 0 0 1 0 0 1 0 0 0
1444444444442444444444443

0 0 0 0 0 0
0 0 0 = 0 0 0

0 0 0 0 0 0
c.q.d.

Ejemplo 1.80
Dada una matriz P representada por sus componentes por Pij (i, j = 1,2,3,4) . a) Obtener la
inversa, b) los invariantes, y c) la ecuacin caracterstica. Aplicar los apartados para la matriz
P:

Universidad de Castilla- La Mancha Draft Por: Eduardo W. V. Chaves (2012)


Ciudad Real - Espaa
66 PROBLEMAS RESUELTOS DE MECNICA DEL MEDIO CONTINUO

1 2 3 1 1 0 0 0
2 2 1 2 0 1 0 0
P= . Considerar que 1=
4 1 5 3 0 0 1 0

3 1 2 4 0 0 0 1
Solucin:
Aplicando el Teorema de Cayley-Hamilton se cumple que:
P 4 + P 3 I1 + P 2 I 2 + P I 3 + I 41 = 0
(
P P 3 + P 2 I1 + P I 2 + 1 I 3 + I 41 = 0 )
P (P (P 2
)
+ P I1 + 1 I 2 + 1 I 3 + I 41 = 0 )


(
)
P P P P + 1 I 1 + 1 I 2 + 1 I 3 + I 41 = 0
1442443

C1


4 4 2
(
44 3

)
P P C1 + 1 I 2 + 1 I 3 + I 4 1 = 0
1

C2
(
P C2 + 1 I 3 + I 4 1 = 0 )
C3 + I 4 1 = 0
donde hemos denotado:
C0 = P
(
C1 = P C0 + 1 I 1 )
C2 = P (C 1 +1 I )
2

C3 = P (C 2 +1 I )
3

Podemos sacar la traza de C3 + I 41 = 0 , resultando que:


Tr (C3 + I 4 1) = Tr (0 )
Tr (C3 ) + Tr ( I 4 1) = Tr (C3 ) + I 4 Tr (1) = Tr (C3 ) + 4 I 4 = 0
Tr (C3 )
I4 =
4
Anlogamente podemos definir que:
Tr (C2 ) Tr (C1 ) Tr (C0 )
I3 = ; I2 = ; I1 =
3 2 1
Con eso obtenemos que:
Tr (C0 )
I1 = = (1 + 2 + 5 + 4) = 12
1
con eso podemos definir la matriz C1 = P C0 + 1 I 1 : ( )
1 2 3 1 1 2 3 1 1 0 0 0 8 14 14 6
2
2 1 2 2
2 1 2 0
1 0 0 8 13 5 7
C1 = 12 =
4 1 5 3 4 1 5 3 0 0 1 0 13 6 16 3

3 1 2 4 3 1 2 4 0 0 0 1 11 2 4 21

Universidad de Castilla- La Mancha Draft Por: Eduardo W. V. Chaves (2012)


Ciudad Real - Espaa
1 TENSORES 67

Tr (C1 ) (8 13 16 21) (42)


I2 = = = = 21
2 2 2
A su vez obtenemos C2 = P C1 + 1 I 2 ( )
1 2 3 1 8 14 14 6 1 0 0 0 37 22 15 17
2 0
2 1 2 8 13 5 7 1 0 0 7 2 5 5
C2 = + 21 =
4 1 5 3 13 6 16 3 0 0 1 0 10 12 14 2

3 1 2 4 11 2 4 21 0 0 0 1 9 14 11 5

Tr (C2 ) ( 37 2 14 + 5) ( 48)
I3 = = = = 16
3 3 3
A su vez obtenemos C3 = P C2 + 1 I 3 ( )
1 2 3 1 37 22 15 17 1 0 0 0 32 0 0 0
2 0
2 1 2 7 2 5 5 1 0 0 0 32 0 0
C3 = + 16 =
4 1 5 3 10 12 14 2 0 0 1 0 0 0 32 0

3 1 2 4 9 14 11 5 0 0 0 1 0 0 0 32

Tr (C3 ) 4(32)
I4 = = = 32 = det (P )
4 4
Luego, la ecuacin caracterstica queda:
P 4 + P 3 I1 + P 2 I 2 + P I 3 + I 41 = 0 P 4 12P 3 + 21P 2 + 16P 321 = 0
Los coeficientes de la ecuacin caracterstica podran haber sido obtenidos al resolver:
det ( P 1) P 1 = 0

c) La inversa puede ser obtenida partiendo de una de las relaciones obtenida anteriormente:
P (C2 + 1 I 3 ) + I 4 1 = 0
( )
P 1 P C2 + 1 I 3 + I 4 P 11 = 0 ( )
C2 + 1 I 3 + I 4 P 1 = 0

P 1 =
1
I4
(C2 + 1 I 3 ) P 1 =
1
det (P )
adj[P ] (
adj[P ] = C2 + 1 I 3 )
Luego:
37 22 15 17 1 0 0 0 21 22 15 17
0
1 7 2 5 5 1 0 0 1 7 14 5 5
P 1 = + 16 =
(32) 10 12 14 2 0 0 1 0 32 10 12 2 2

9
14 11 5 0 0 0 1 9 14 11 21

NOTA 1: Este procedimiento que acabamos de realizar, en la literatura, se conoce como


Mtodo de Faddeev-Leverrier.

Universidad de Castilla- La Mancha Draft Por: Eduardo W. V. Chaves (2012)


Ciudad Real - Espaa
68 PROBLEMAS RESUELTOS DE MECNICA DEL MEDIO CONTINUO

Notar que la inversa tambin puede ser obtenida a travs del procedimiento utilizado en la
ecuacin (1.46), i.e.:

1 0 0 0 1 2 3 1 1 2 3 1 1 2 3 1

2 2 1 2 1 0 0 0 2 2 1 2 2 2 1 2

4 1 5 3 4 1 5 3 1 0 0 0 4 1 5 3
3 3 3 1
1 2 4 1 2 4 1 2 4 0 0 0

0 1 0 0 1 2 3 1 1 2 3 1 1 2 3 1

2 2 1 2 0 1 0 0 2 2 1 2 2 2 1 2
4 1 5 3 4 1 5 3 0 1 0 0 4 1 5 3 21 22 15 17
1 3
1 2 4
3
1 2 4
3
1 2 4
0
1 0 0

32 = 1 7 14 5 5
32 10
0 0 1 0 1 2 3 1 1 2 3 1 1 2 3 1 12 2 2
9
2 2 1 2 0 0 1 0 2 2 1 2 2 2 1 2
14 11 21
4 1 5 3 4 1 5 3 0 0 1 0 4 1 5 3
3 3 3 0
1 2 4 1 2 4 1 2 4 0 1 0

0 0 0 1 1 2 3 1 1 2 3 1 1 2 3 1

2 2 1 2 0 0 0 1 2 2 1 2 2 2 1 2
4 1 5 3 4 1 5 3 0 0 0 1 4 1 5 3
3 3 3 0
1 2 4 1 2 4 1 2 4 0 0 1

Universidad de Castilla- La Mancha Draft Por: Eduardo W. V. Chaves (2012)


Ciudad Real - Espaa
1 TENSORES 69

NOTA 2: Tambin podemos obtener los coeficientes del polinomio caracterstico a travs del
siguiente procedimiento. Considerando P 4 P 3 I 1 + P 2 I 2 P I 3 + I 4 1 = 0
El ltimo coeficiente, en el caso I 4 = det(P ) = 32 .
El coeficiente I 3 se obtiene por la suma de los determinantes de las matrices resultantes al
eliminar 1 fila y 1 columna asociados a la diagonal principal, i.e.:
1 2 3 1 1 2 3 1 1 2 3 1 1 2 3 1
2 2 1 2 2 2 1 2 2 2 1 2 2 2 1 2
I3 = + + +
4 1 5 3 4 1 5 3 4 1 5 3 4 1 5 3

3 1 2 4 3 1 2 4 3 1 2 4 3 1 2 4
2 1 2 1 3 1 1 2 1 1 2 3
= 1 5 3 + 4 5 3 + 2 2 2 + 2 2 1 = 16
1 2 4 3 2 4 3 1 4 4 2 5

El coeficiente I 2 se obtiene por la suma de los determinantes de las matrices resultantes al


eliminar 2 filas y 2 columnas asociados a la diagonal principal, i.e.:
1 2 3 1 1 2 3 1 1 2 3 1
2 2 1 2 2 2 1 2 2 2 1 2
I2 = + + +
4 1 5 3 4 1 5 3 4 1 5 3

3 1 2 4 3 1 2 4 3 1 2 4
1 2 3 1 1 2 3 1
2 2 1 2 2 2 1 2
+ + +
4 1 5 3 4 1 5 3

3 1 2 4 3 1 2 4
1 2 3 1
2 2 1 2
+
4 1 5 3

3 1 2 4
5 3 2 2 2 1 1 1 1 3 1 2
= + + + + + = 21
2 4 1 4 1 5 3 4 4 5 2 2
El coeficiente I 1 se obtiene por la suma de los determinantes de las matrices resultantes al
eliminar 3 filas y 3 columnas asociados a la diagonal principal, i.e.:
1 2 3 1 1 2 3 1 1 2 3 1 1 2 3 1
2 2 1 2 2 2 1 2 2 2 1 2 2 2 1 2
I1 = + + +
4 1 5 3 4 1 5 3 4 1 5 3 4 1 5 3

3 1 2 4 3 1 2 4 3 1 2 4 3 1 2 4
= [4] + [1] + [2] + 5 = 12 = Tr (P )

Universidad de Castilla- La Mancha Draft Por: Eduardo W. V. Chaves (2012)


Ciudad Real - Espaa
70 PROBLEMAS RESUELTOS DE MECNICA DEL MEDIO CONTINUO

Ejemplo 1.81
Demostrar que si A es un tensor de segundo orden se cumple que:

a) II A =
1
2
{
[Tr( A )]2 Tr( A 2 ) }
b) det ( A ) =
1
6
{
[Tr (A )]3 + 2 Tr( A 3 ) 3Tr(A ) Tr(A 2 ) }
Solucin:
a) Fue demostrado en el Ejemplo 1.78 que III A A 1 = (A 2 AI A + 1 II A ) , luego, aplicando el
doble producto escalar con el tensor identidad, obtenemos que:
( )
III A A 1 : 1 = A 2 AI A + 1 II A : 1 = A 2 : 1 A : 1 I A + 1 : 1 II A
1
III A Tr ( A ) = Tr ( A ) Tr ( A ) I A + Tr (1) II A = Tr ( A 2 ) ( I A ) 2 + 3 II A
2

Teniendo en cuenta la inversa de un tensor A 1 =


[cof ( A )]T , podemos decir que:
III A


III A Tr ( A 1 ) = Tr ( III A A 1 ) = Tr III A
[cof ( A )]T
( )
= Tr [cof ( A )]T = Tr ([cof ( A ) ]) = II A
III A

Con lo cual, podemos decir que:
III A Tr ( A 1 ) = II A = Tr ( A 2 ) ( I A ) 2 + 3 II A
II A 3 II A = Tr ( A 2 ) ( I A ) 2

II A =
1
2
{
( I A ) 2 Tr ( A 2 ) }
b) Partiendo del teorema de Cayley-Hamilton, que afirma que todo tensor cumple su propia
ecuacin caracterstica:
A 3 A 2 I A + AII A III A 1 = 0 (1.63)

donde I A = [Tr (A )] , II A =
1
2
{ }
[Tr( A )]2 Tr( A 2 ) , III A = det (A ) son los invariantes principales

del tensor A . Haciendo el doble producto escalar con el tensor identidad de segundo orden
( 1 ) en la expresin (1.63) obtenemos que:
A 3 : 1 A 2 : 1 I A + A : 1II A III A 1 : 1 = 0 : 1
Tr ( A 3 ) Tr ( A 2 ) I A + Tr ( A ) II A III A [Tr(1)] = [Tr(0)]
Tr ( A 3 ) Tr ( A 2 ) Tr ( A ) + Tr ( A )
1
2
{
[Tr(A )]2 Tr(A 2 ) III A 3 = 0}
1 1
Tr ( A 3 ) Tr ( A 2 ) Tr ( A ) + [Tr ( A )] Tr ( A ) Tr ( A 2 ) III A 3 = 0
3

2 2
1
2
{ }
2 Tr ( A 3 ) 3 Tr ( A 2 ) Tr ( A ) + [Tr ( A )] III A 3 = 0
3

Con lo cual obtenemos que:

III A = det ( A ) =
1
6
{
[Tr( A )]3 + 2 Tr( A 3 ) 3Tr (A 2 ) Tr( A ) }
o en notacin indicial:

Universidad de Castilla- La Mancha Draft Por: Eduardo W. V. Chaves (2012)


Ciudad Real - Espaa
1 TENSORES 71

III A = det ( A ) =
1
6
{
A ii A jj A kk + 2A ij A jk A ki 3A ij A ji A kk }
NOTA: Es interesante destacar que los invariantes principales del tensor A vienen formados
por la combinacin de los tres invariantes fundamentales de un tensor de segundo orden, a
saber, ( Tr (A ) , Tr ( A 2 ) y Tr ( A 3 ) ), es decir:
I A = Tr ( A )

II A =
1
2
2
{
I A Tr ( A 2 ) }
1
{
III A = [Tr ( A )] + 2 Tr ( A 3 ) 3 Tr ( A 2 ) Tr ( A )
6
3
}

Ejemplo 1.82

Demostrar que II T = III T Tr ( T 1 ) , donde II T =


1
2
{ }
[Tr( T )]2 Tr(T 2 ) es el segundo invariante
principal de T , III T es el tercer invariante principal, es decir, el determinante de T .

Solucin: Fue demostrado en el Ejemplo 1.78 que T 1 =


1
(
III T
)
T 2 TI T + 1 II T , luego,

aplicando el doble producto escalar con el tensor identidad, obtenemos que:

T 1 : 1 =
1
III T
(
T 2 TI T + 1 II T : 1 =
1
)
III T
(
T 2 : 1 T : 1 I T + 1 : 1 II T )
Tr ( T 1 ) =
1
III T
(
Tr ( T 2 ) Tr ( T ) I T + Tr (1) II T )
III T Tr ( T 1 ) = Tr ( T 2 ) I T2 + 3 II T
14243
= 2 II T
1
III T Tr ( T ) = II T

Ejemplo 1.83
Demostrar que:
r r
r r 1 (c b )
(1 + c b) 1 = 1 r r (1.64)
( + c b)
r r
donde c , b son vectores, 1 es el tensor identidad de segundo orden, y y son escalares.
Solucin:
r r
Haciendo que T = (1 + c b) , y teniendo en cuenta la expresin de la inversa obtenida en
el Ejemplo 1.78:

T 1 =
1
III T
(
T 2 TI T + 1 II T ) (1.65)

A continuacin obtenemos T 2 :

Universidad de Castilla- La Mancha Draft Por: Eduardo W. V. Chaves (2012)


Ciudad Real - Espaa
72 PROBLEMAS RESUELTOS DE MECNICA DEL MEDIO CONTINUO

r r r r
T 2 = T T = (1 + c b) (1 + c b)
r r r r r r r r
= 2 1 1 + 1 (c b) + (c b) 1 + 2 (c b) (c b)
r r r r r r r r
donde se cumple que (c b) (c b) = (c b)(c b) , ver Ejemplo 1.20. Luego, la expresin
anterior puede ser rescrita como:
r r r r r r
T 2 = 2 1 + 2 (c b) + 2 (c b)(c b)
y la traza viene dada por:

[ r r r r r r
Tr ( T 2 ) = Tr 2 1 + 2 (c b) + 2 (c b)(c b) ]
r r r r r r
= 2 Tr (1) + 2 Tr (c b) + 2 (c b) Tr (c b)
r r r r r r
= 3 2 + 2 (c b) + 2 (c b)(c b)
r r r r
= 3 2 + 2 (c b) + 2 (c b) 2
A continuacin calculamos los invariantes principales de T
r r r r r r
I T = Tr (1 + c b) = Tr (1) + Tr (c b) = 3 + (c b)

[ r r 2
] r r r r
( I T ) 2 = 3 + (c b) = 9 2 + 6 (c b) + 2 (c b) 2

II T =
1
2
{ 2
} {
1
2
r r r r
[ r r r r
I T Tr ( T 2 ) = 9 2 + 6 (c b) + 2 (c b) 2 3 2 + 2 (c b) + 2 (c b) 2 ]}
r r
= 3 + 2 (c b)
2

r r r r
III T = det (1 + c b) = 3 + 2 c b (ver Ejemplo 1.50)
Luego, la expresin (1.65) queda:
III T T 1 = T 2 I T T + II T 1
r r r r r r
= 2 1 + 2 (c b) + 2 (c b)(c b)
[ ]
r r r r
[ ]
r r
3 + (c b) (1 + c b) + 3 2 + 2 (c b) 1
r r r r r r r r r r
= 2 1 + 2 (c b) + 2 (c b)(c b) 3 2 1 3 (c b) (c b)1
r r r r r r (1.66)
2 (c b)(c b) + 3 2 1 + 2 (c b)1
r r r r
= 1 2 + (c b)1 (c b)
r r r r
= ( 2 + c b)1 (c b)
1 r r r r
= ( 3 + 2 c b)1 (c b) = [adj( T ) ] = [cof ( T ) ]
T


r r r r
Teniendo en cuenta que T = (1 + c b) , III T = 3 + 2 c b , la expresin anterior
queda:
1 III T r r
T 1 = 1 (c b)
III T
r r r r (1.67)
1 III T (c b) 1 (c b)
= 1 = 1 r r
III T III T ( 3 + 2 c b )

o aun:

Universidad de Castilla- La Mancha Draft Por: Eduardo W. V. Chaves (2012)


Ciudad Real - Espaa
1 TENSORES 73

r r 1 r r
(1 + c b) 1 = 1 r r (c b) Notacin Tensorial (1.68)
( + c b)
1
( ij + c i b j ) 1 = ij (c b ) Notacin Indicial (1.69)
( + c k b k ) i j

[[1] + [{c}{b} ] ] T 1
=
1

[ 1]

( + {c} {b})
T
[
{c}{b}T ] Notacin matricial (1.70)

NOTA 1: La expresin anterior tambin es vlida para matrices de n-dimensiones.


Para el caso particular cuando = 1 , = 1 , obtenemos que:
r r
r r 1 (c b)
(1 + c b) = 1 r r (1.71)
1+ c b

NOTA 2: La expresin (1.68) puede ser reescrita como:


r r 1 r r
T 1 = (1 + c b) 1 = 1 r r (c b)
( + c b )
= r
1 r r
[r r
r ( 2 + c b)1 (c b) =
( + c b )
3 2
1
det ( T )
[adj( T )] ]
r r r r r r
con lo cual concluimos que adj(1 + c b) = ( 2 + c b)1 (c b) .
NOTA 3: Podemos extender la ecuacin (1.68) de tal forma que:
1
( I sym + A B ) 1 = I sym ( A B)
( + A : B )
donde ahora tenemos que I sym es el tensor identidad de cuarto orden, A y B son tensores de
segundo orden, y y son escalares. Con eso es de fcil demostracin que ( I sym ) 1 = I sym .

Ejemplo 1.84
r r 1 r r
Teniendo en cuenta que (1 + c b) 1 = 1 r r (c b) , demostrar que:
( + c b )
r r 1
( A + a b) 1 = A 1

r

r
( + b A 1 a)
r
[ r
( A 1 a) (b A 1 ) ] (1.72)

r r
donde a , b son vectores, A es un tensor de segundo orden, con det ( A ) 0 ( A 1 ), y ,
son escalares.
Solucin:
r r
Observemos que el trmino (A + a b) , puede ser reescrito como:
r r r r
(A + a b) = A (1 + A 1 a b)

Utilizando la propiedad de la inversa tal que ( A B ) 1 = B 1 A 1 , podemos decir que:


r r
[ r r 1 r r
]
(A + a b) 1 = A (1 + A 1 a b) = (1 + A 1 a b) 1 A 1

Universidad de Castilla- La Mancha Draft Por: Eduardo W. V. Chaves (2012)


Ciudad Real - Espaa
74 PROBLEMAS RESUELTOS DE MECNICA DEL MEDIO CONTINUO

r
Observemos que el resultado de la operacin A 1 a resulta un vector y lo denotamos por el
r r
vector c = A 1 a , con lo cual podemos reescribir la expresin anterior como:
r r r r
(A + a b) 1 = (1 + A 1 a b) 1 A 1
r r
= (1 + c b) 1 A 1
1 r r
r r (c b) A
1
= 1
( + c b )
1 r r
= 1 A 1 r r (c b) A
1
( + c b )
1 r r
= A 1 r r c bA
1
( + c b )
r
1 1 r
= A 1 r r ( A a) (b A )
1
( + c b )
1 r r
= A 1 r r ( A 1 a) (b A 1 )
( + b A 1 a)
La expresin anterior en notacin indicial queda
1
(A ij + a i b j ) 1 = A ij1 ( A ik1 a k )(b s A sj1 )
( + b p A pq1 a q )
r r r r
Cuidado con la operacin ( A 1 a) b A 1
(a b) , esta ltima no tiene consistencia, ya
14243
Expresin errnea
r r
que no podemos tener un producto escalar (contraccin) con un escalar (a b) . En notacin
r r r
indicial se puede comprobar c b = c i b i = ( A 1 a) i b i = A ik1 a k b i , luego, las expresiones
r r
posibles son ( A 1 a) b = b i A ik1a k = a k A ik1b i = A ik1b i a k = A ik1 a k b i = A ik1a k b i .
1
4
r 24
3 1
424
3r 142r4 3 1 424 3 1 424 3
r r r r r r r
bA 1a aA T b A 1:(ba) A T :( ab ) A 1:( ab )T

Para el caso particular cuando = 1 , = 1 , recaemos en la frmula de Sherman-Morrison:


r r
r r 1 ( A 1 a) (b A 1 ) Frmula de Sherman-Morrison
1
( A + a b) = A r r (1.73)
1 + b A 1 a (notacin tensorial)

En notacin matricial la ecuacin anterior queda:

{ }{
[ A] 1 {a} {b}T [ A] 1 }
T
Frmula de Sherman-Morrison
[[ A] + [{a}{b} ] ]
T 1
= [ A] 1

1 + {b}T [ A] 1 {a} (notacin matricial)
(1.74)

r r r r
NOTA 1: Observar que si (A + a b) = A (1 + A 1 a b) , el determinante viene
definido por:
r r
[ r r
]
r r
det (A + a b) = det A (1 + A 1 a b) = det [A ]det (1 + A 1 a b)
r
[ ]
r
= det [A ]( 3 + 2 b A 1 a)
con lo cual, la expresin (1.72) puede aun ser reescrita como:

Universidad de Castilla- La Mancha Draft Por: Eduardo W. V. Chaves (2012)


Ciudad Real - Espaa
1 TENSORES 75

r r
( A + a b) 1 =
1
{ r r r r
A ( 2 + b A 1 a) A 1 A ( A 1 a) (b A 1 )

[ ]}
r r r r
con = det (A + a b) = A ( 3 + 2 b A 1 a) . (1.75)
con eso concluimos que:
r r
{ r r r r
adj(A + a b) = A ( 2 + b A 1 a) A 1 A ( A 1 a) (b A 1 ) [ ]}
NOTA 2: Podemos extrapolar la expresin (1.72) de tal forma que:

( D + A B ) 1 =
1

D 1

( + B : D 1 : A )
[(D 1
: A ) (B : D 1 ) ] (1.76)

donde ahora tenemos que D es un tensor de cuarto orden, A y B son tensores de segundo
orden, y , son escalares. Aun podemos decir que:

( D + A B ) 1 =
1

{D ( 2
+ B : D 1 : A )D 1 D (D 1 : A ) (B : D 1 )[ ]}
con = det (D + A B ) = D ( 3 + 2 B : D 1 : A ) . (1.77)
donde podemos concluir que:
det (D + A B ) = det (D )( 3 + 2 B : D 1 : A ) (1.78)
{
adj(D + A B ) = D ( 2 + B : D 1 : A )D 1 D (D 1 : A ) (B : D 1 ) [ ]} (1.79)

Ejemplo 1.85
r r r r
a) Dado un tensor de segundo orden C = (1 + a b + c d) , demostrar que:
r r r r r r r r
1 + a b + c d = 3 + 2 (c d) + 2 (a b) + (a b)(c d) (a d)(b c ) [r r r r r r r r
]
(1.80)
r r r r r r r r
donde 1 + a b + c d = det (1 + a b + c d) representa el determinante del
r r r r
tensor C . b) Para el caso particular cuando = 1 , d = a , c = b , demostrar que:
r r r r r r r r 2
det (1 + a b + b a) = 1 + ( + )(a b) a b (1.81)

Solucin:
r r r r
Definimos un tensor auxiliar D = 1 + a b y a su vez tenemos que C = (D + c d) .
r r r r
Segn el Ejemplo 1.84, ecuacin (1.75), se cumple que det (D + c d) = D (1 + d D 1 c ) ,
donde:
r r r r
det (D) D = det (1 + a b ) = 3 + 2 (a b) y
r r 1 r r
(D ) 1 = (1 + a b) 1 = 1 r r (a b)
( + a b )

Universidad de Castilla- La Mancha Draft Por: Eduardo W. V. Chaves (2012)


Ciudad Real - Espaa
76 PROBLEMAS RESUELTOS DE MECNICA DEL MEDIO CONTINUO

Con eso, podemos decir que:


r r r r
det (D + c d) = D (1 + d D 1 c )

[ r r
] r 1
= 3 + 2 (a b ) (1 + d 1


r
r r r
r (a b) c
( + a b )

[ r r
] 1 r r
= 3 + 2 (a b ) 1 + d 1 c


r
r r r r
r d (a b) c
( + a b )

[ r r
]1 r r
= 3 + 2 (a b ) 1 + d c


r
r r r r
r (d a) (b c )
( + a b )

[ r r
]
r r
= 3 + 2 (a b ) 1 + (c d)

r
r r r r
r (a d)(b c )
2 ( + a b )

r r r r r r r r r r r r
Notar que (d a) (b c ) = (a d) (b c ) = (a d)(b c ) .
123 123
escalar escalar

r r
[ r r
] r r
[ r r
det (D + c d) = 3 + 2 (a b) 1 + (c d) 3 + 2 (a b) ]

r
r r r r
r (a d)(b c )
( + a b)
2

[ r r

] r r

r r r r
= 3 + 2 (a b) 1 + (c d) (a d)(b c )

3
[ 2 r r 2 r r r r r r
] r r r r
= + (c d) + (a b) + (a b)(c d) (a d)(b c )
Luego:
r r r r r r r r r r r r
[ r r r r
det (1 + a b + c d) = 3 + 2 (c d) + 2 (a b ) + (a b )( c d) (a d)(b c ) ]
Con eso demostramos la ecuacin (1.80).
r r r r
Para el caso particular cuando d = a , c = b , tenemos que:
r r r r r r r r r r r r
[ r r r r
det (1 + a b + c d) = 3 + 2 (c d) + 2 (a b) (a d)(b c ) (a b)(c d) ]
r r r r r r r r r r r r
[ r r r r
det (1 + a b + b a) = 3 + 2 (b a) + 2 (a b) (a a)(b b) (a b)(b a) ]
r r
[
r r r r r r r r
= 3 + 2 ( + )(a b) (a a)(b b) (a b)(a b) ]
r r 2 r 2 r 2 r r 2
En el Ejemplo 1.1 hemos demostrado que a b = a b a b . Con eso ( )
demostramos que:
r r r r r r r r 2
det (1 + a b + b a) = 3 + 2 ( + )(a b) a b

Para el caso particular cuando = 1 , obtenemos que:


r r r r r r r r 2
det (1 + a b + b a) = 1 + ( + )(a b) a b

Ejemplo 1.86
r r r r
a) Obtener la inversa del tensor de segundo orden C = (1 + a b + c d) .

Universidad de Castilla- La Mancha Draft Por: Eduardo W. V. Chaves (2012)


Ciudad Real - Espaa
1 TENSORES 77

r r r r
p p (B q) (B q)
b.1) Dado el tensor D = B + r r r r donde B = B T y B 1 , demostrar que
pq q B q
se cumple que:
r r r r
( p q + p B 1 p ) r r
D 1 = B 1 + r r 2
( p q) ( p q)
[
[q q ] r 2 r qr ( B 1 pr ) ]sym
(1.82)

b.2) Si B es un tensor definido positivo, determinar las condiciones para que D sea un tensor
no-singular.
Solucin:
r r r r
a) Denotando por A = (1 + a b) quedamos con C = ( A + c d) , y teniendo en cuenta
que
r r 1
( A + a b) 1 = A 1

r

r
( + b A 1 a)
[ r r
( A 1 a) (b A 1 ) ] (1.83)

Ver Ejemplo 1.84, ecuacin (1.72). Luego


r r
( A + c d) 1 = A 1 r

r
(1 + d A 1 c )
[ r r
( A 1 c ) (d A 1 ) ] (1.84)

En el Ejemplo 1.83 hemos demostrado que:


r r
r r 1 (c b )
(1 + c b) 1 = 1 r r (1.85)
( + c b)
Con eso concluimos que:
r r
r r 1 (a b)
A 1 = (1 + a b) 1 = 1 r r
( + a b )
Adems tenemos que
r r r r r r
r 1 ( a b ) r 1 r ( a b ) r 1 r (b c) r
A c = 1
1
r r c = 1 c r r c = c r r a
( + a b ) ( + a b ) ( + a b )

r r r r r r
r r 1 ( a b ) 1 r r ( a b ) 1 r ( d a) r
d A = d 1
1
r r = d 1 d r r = d r r b
( + a b) ( + a b ) ( + a b )

Con eso concluimos que
r r r r r r
[
r r r r
] [ r r r r
(1 + a b + c d) 1 = (1) 1 + ( 2 ) (a b) + ( 3) (1) c + ( 2 ) (b c )a (1) d + ( 2 ) (a d)b ]
(1.86)
donde
1
(1) =


( 2) = r r
( + a b )

Universidad de Castilla- La Mancha Draft Por: Eduardo W. V. Chaves (2012)


Ciudad Real - Espaa
78 PROBLEMAS RESUELTOS DE MECNICA DEL MEDIO CONTINUO


( 3) = r r
(1 + d A 1 c )
r r 1 r r r r r r
d A 1 c = (d c ) r r (d a )(b c )
( + a b )
b.1) Podemos reescribir el tensor D como:
r r r r r r r r
p p (B q) (B q) p p (B q) (B q)
D = B 1 + 1 r r 1 r r = B 1 + (B B ) r r (B B )
1 1
r r
pq q B q pq q Bq
r r r r r r r r
p p 1 ( B q ) ( B q ) ( B 1 p) p ( B 1 B q ) ( B q )
= B 1 + B r r B
1
r r = B 1 + r r r r
pq q B q pq q Bq
r r r r
( B 1 p ) p q ( B q )
= B 1 + r r r r
pq q Bq
Denotando por
r r r r r r r r 1 1
a = ( B 1 p ) ; b=p ; c =q ; d = (B q) ; = r r ; = r r
pq q Bq

[ r r r r
D = B 1 + a b + c d ] = B C D 1 = ( B C ) 1 = C 1 B 1

[ r r r r
]
donde C = 1 + a b + c d . La inversa del tensor C puede ser obtenido a travs del
apartada (a) con = 1 . Adems tenemos que:
(1) = 1 ,
1 1 1
( 2) = r r = r r = r r = r r r r
( + a b) (1 + a b) p q (1 + r r ( B 1 p) p) ( p q + p B 1 p)
1 r r
pq
r r 1 r r r r r r
d A 1 c = (d c ) r r ( a d )(b c )
( + a b )
r r 1 r r r r
= (( B q ) q ) + r r r 1 r
(( B 1 p) ( B q ) )( p q )
( p q + p B p)
r r r r
r r ( p ( B T B ) q ) ( p q )
= q B q + r r r r
( p q + p B 1 p)
1 1
( 3) = r
1 r
= r r r r r r
(1 + d A c ) q B q 1 r r ( p ( B T B ) q ) ( p q )
1 + r
r q B q + r r r r

q Bq ( p q + p B 1 p )
r r r 1 r
( p q + p B p)
= r r r r
( p ( B T B ) q ) ( p q )
r r r r
1 ( p q + p B 1 p ) 1
( 2 ) (3) = r r r 1 r r r r r = r r r r
( p q + p B p) ( p ( B B ) q ) ( p q ) ( p ( B B ) q ) ( p q )
T T

r r 1 1 r r
( 2 ) ( 3) (a d ) = r r r r (( B p) ( B q ))
( p (B T
B) q) ( p q )
1 r r 1
r r r ( p (B B) q ) = r r
T
= r
( p ( B T B) q) ( p q ) ( p q)

Universidad de Castilla- La Mancha Draft Por: Eduardo W. V. Chaves (2012)


Ciudad Real - Espaa
1 TENSORES 79

r r 1 r r 1
( 2 ) ( 3) (b c ) = r r r r ( p q) = r r
( p (B T
B) q ) ( p q ) ( p (B B) q )
T

r r r r 1 1 r r r r
( 2 ) (3) (b c )(a d ) = r r r r (( B p) ( B q ))( p q ) = 1
( p (B T
B) q ) ( p q )
La expresin (1.86) queda:
r r r
[ r r r r
] [
r r r
C 1 = 1 + ( 2 ) (a b ) + ( 3) c + ( 2 ) (b c )a d + ( 2 ) (a d )b ]
r r
[ ] r r
[ ] r r r r
[
r r r r
C 1 = 1 + ( 2 ) a b + ( 3) c d + ( 3) ( 2 ) (a d ) c b + ( 3) ( 2 ) (b c ) a d +] [ ]
r r r r r r
+ ( 3) (22 ) (b c )(a d ) a b [ ]
{ r r r r r r
}[r r
] [
r r r r
]
C 1 = 1 + ( 2 ) + ( 3) (22 ) (b c )(a d ) a b + ( 3) c d + ( 3) ( 2 ) (a d ) c b + [ ]
r r r r
+ ( 3) ( 2 ) (b c ) a d [ ]
notar que: { ( 2 ) + (3) (22 ) (b c )(a d )}= ( 2 ) {1 + (3) ( 2 ) (b c )(a d )}= ( 2 ) {1 1} = 0
r r r r r r r r

r r
[ ]
r r r r
[ r r r r
]
C 1 = 1 + ( 3) c d + ( 3) ( 2 ) (a d ) c b + ( 3) ( 2 ) (b c ) a d [ ]
r r
[ ]r r r r
[ r r r r
]
C 1 = 1 + ( 3) c d + ( 3) ( 2 ) (a d ) c b + ( 3) ( 2 ) (b c ) a d [ ]
r r r r
( p q + p B 1 p) r r 1 r r
=1 + r r r r [q ( B q ) ] + r r [q p ] + L
( p (B B) q ) ( p q )
T
( p q)

L+ r
1
( p (B B) q )
T
1 r
r ( B p) ( B q )
r
[ ]
Con eso, podemos obtener que:
D 1 = C 1 B 1
r r r r
( p q + p B 1 p ) r r 1 r r
= 1 + r r r r [q ( B q )] + r r [q p ] + L
( p (B B) q ) ( p q )
T
( p q)

L+ r
1
( p (B B) q )
T
1 r r 1
r ( B p) ( B q ) B [ ]

r r r r
( p q + p B p)
1
r r 1 r r
= B 1 + r r r r [q ( B q ) ] B + r r [q p ] B + L
1 1

( p (B B) q ) ( p q )
T
( p q)

L+ r
1 1 r r
[
r ( B p) ( B q ) B
( p (B B) q )
T
1
]
notar que:
{[qr ( B qr )] B } = [qr ( B qr )] B = [q ( B qr ) ]B
1
ij ik
1
kj i k
1
kj [ ]
= q i ( B kp q p ) B kj1 = q i ( B kp B kj1 q p )
= [q ( B B ) q ) ]
r r T
ij

[( B pr ) ( B qr )] B = ( B pr ) ( B B ) qr
1 1 1 T

Si ahora consideramos que el tensor B es simtrico, i.e. B = B T , quedamos con:

Universidad de Castilla- La Mancha Draft Por: Eduardo W. V. Chaves (2012)


Ciudad Real - Espaa
80 PROBLEMAS RESUELTOS DE MECNICA DEL MEDIO CONTINUO

r r r r
( p q + p B 1 p ) r r
D 1 = B 1 + r r r r
( p q) ( p q)
[q q ] + r 1r
( p q)
[qr ( pr B )] + ( pr1qr ) [( B
1 1
p) q ]
r r

{[qr ( pr B )] + [( B }
r r r 1 r
( p q + p B p) r r
[q q ] + r 1r p) q ]
r r
= B 1 + r r r r
1 1
( p q) ( p q) ( p q)
r r r 1 r
( p q + p B p) r r
= B 1 + r r 2
( p q)
[q q ] r 2 r
( p q)
[qr ( B 1
p)]
r sym

r r r
Notar que, debido la simetra de B se cumple que p B 1 = B 1 p = t y B T B = 1 .
b.2) Un tensor ser no-singular si det (D ) 0 . Utilizando la expresin obtenida anteriormente:

[ r r r r
D = B 1 + a b + c d ]
( [ r r r r
]) r r r r
det (D) = det B 1 + a b + c d = det ( B )det 1 + a b + c d [ ]
Como B es un tensor definido positivo esto implica que det ( B ) > 0 . Luego, la condicin para
[
que D sea no-singular es que det 1 + a b + c d
r r r r
] 0 . Utilizando el determinante
obtenido en el Ejemplo 1.85:
r r r r r r r r r r
[ r r r r
det (1 + a b + c d ) = 3 + 2 (c d ) (a d )( b c ) (a b )( c d ) ]
r r r r r r r r r r r r r r r r
donde = 1 , a b = ( B 1 p ) p = p B 1 p , a d = ( B 1 p ) ( B q ) = p q , b c = p q
r r r r r r r r 1 r r
c d = q ( B q ) = q B q , (c d ) = r r q B q = 1 ,
q B q

[(a d )(b c ) (a b )(c d ) ] = r r r [ ]


r r r r r r r r 1 1 r r r r r 1 r r r
r ( p q )( p q ) ( p B p )( q B q )
pq q B q
Con eso podemos decir que:

[ r r r r
det 1 + a b + c d = r r r] 1
( p q )(q B q )
r r r r
[ r 1 r r r
r ( p q )( p q ) ( p B p )(q B q ) 0 ]
r r r r r r r r
Luego, las condiciones son: p 0 , q 0 , ( p q ) 0 , i.e. p y q no pueden ser vectores
ortogonales. Otra condicin que hay que cumplir es:
r r r r r r r r
( p q )( p q ) ( p B 1 p )(q B q ) 0
142 4 43 4 144424443
>0 >0
r r
Notar que por el hecho que B sea definido positivo, el trmino (q B q ) > 0 siempre ser
r r r r
positivo cualquier que sea q 0 . Lo mismo ocurre con ( p B 1 p ) > 0 , ya que si un tensor es
definido positivo su inversa tambin lo es. Observar tambin que para que D sea definido
r r r r r r r r
positivo ( det (D ) > 0 ) hay que cumplir ( p q ) 2 > ( p B 1 p )(q B q ) y ( p q ) > 0 . Estas dos
r r r r r r
condiciones pueden ser reemplazadas por ( p q ) > ( p B 1 p )(q B q ) .

Ejemplo 1.87
Dado un tensor de segundo orden A = A () y un escalar , demostrar que se cumple que:

dA dA
= A Tr A 1 (1.87)
d d

Universidad de Castilla- La Mancha Draft Por: Eduardo W. V. Chaves (2012)


Ciudad Real - Espaa
1 TENSORES 81

Solucin:
En el Ejemplo 1.81 y Ejemplo 1.78, hemos demostrado, respectivamente, que se cumplen:

III A = det ( A ) = A =
1
6
{
[Tr( A )]3 + 2 Tr(A 3 ) 3Tr( A 2 ) Tr(A ) } (1.88)

III A A 1 = A 2 AI A + II A 1 (1.89)

donde I A = Tr (A ) , II A =
1
2
{
[Tr ( A )]2 Tr ( A 2 ) . }
Notar tambin que las siguientes derivadas se cumplen:
d [I A ] d [Tr ( A )] d [A kk ] d [A ik ik ] d [A ik ] dA dA
= = = = ik = : 1 = Tr
d d d d d d d
[
d Tr ( A 2 ) ] d (A 2 )
= Tr 2A
dA dA
d
= Tr
d d = 2 Tr A d

[
d Tr ( A 3 ) ]
= 3Tr A 2
dA
d d
Tomando la derivada de (1.88) con respecto a obtenemos que:
d ( III A ) 1 d
d
=
6
d
{
[Tr( A )]3 + 2 Tr(A 3 ) 3Tr( A 2 ) Tr(A ) }
1
= 3[Tr ( A )]
2 d [Tr ( A )]
+2
d Tr ( A 3 ) [
3
]
d Tr ( A 2 ) [ ]
Tr ( A ) 3 Tr ( A 2 )
d [Tr ( A )]

6 dt d d d
1 dA 2 dA dA dA
3[Tr ( A )] Tr + 6 Tr A 6 Tr A
2
= Tr ( A ) 3Tr ( A 2 ) Tr
6 d d d d

= Tr A 2

dA

d
dA
Tr A


d
1
2
2
{ dA
Tr ( A ) + [Tr ( A )] Tr ( A 2 ) Tr
d
}
o aun
d ( III A ) dA dA dA
= Tr A 2 Tr A d I A + II A Tr d (1.90)
d d
dA
Haciendo el producto escalar de la ecuacin (1.89) con , obtenemos que:
d

III A A 1
dA
d
(
= A 2 AI A + II A 1
dA
d
)
= A2
dA
d
A
dA
d
I A + II A
dA
d
y sacando la traza con obtenemos que:
dA 2 dA dA dA
Tr A 1 III A = Tr A A I A + II A
d d d d
(1.91)
dA dA dA
= Tr A 2 Tr A I A + Tr II A
d d d
Si comparamos las ecuaciones (1.90) y (1.91) concluimos que:
dA dA
A 1
d ( III A )
= III A Tr A 1 = III A Tr
d d d

Universidad de Castilla- La Mancha Draft Por: Eduardo W. V. Chaves (2012)


Ciudad Real - Espaa
82 PROBLEMAS RESUELTOS DE MECNICA DEL MEDIO CONTINUO

1.1.11 Tensores Istropos y Anistropos

Ejemplo 1.88
Sea el tensor de cuarto orden C , cuyas componentes vienen dados por:
Cijkl = ij kl + ik jl + il jk (1.92)
donde ij son las componentes del tensor identidad de segundo orden, y , , son
escalares.
a) Qu tipo de simetra presenta el tensor C ? b) Que condiciones hay que cumplir para que
C sea un tensor simtrico?
Solucin:
El tensor presenta simetra mayor si se cumple que C ijkl = C klij . Teniendo en cuenta (1.92),
concluimos que:
C klij = kl ij + ki lj + kj li = Cijkl

Verificamos ahora si el tensor presenta simetra menor, por ejemplo C ijkl = C ijlk
Cijlk = ij lk + il jk + ik jl C ijkl

Se puede comprobar este hecho fcilmente por adoptar i = 2 , j = 1 , k = 1 , l = 2 , con eso:


Cijkl = C 2112 = 21 12 + 21 12 + 22 11 =

Cijlk = C 2121 = 21 21 + 22 11 + 21 12 =

Luego, el tensor C solo ser simtrico (simetra menor y mayor) si = , resultando:


Cijkl = ij kl + ( ik jl + il jk )

Verifiquemos que ij kl presenta simetra mayor y menor, mientras que los tensores ik jl ,
il jk no son simtricos. Fijemos tambin que ( ik jl + il jk ) = 2I ijkl
sym
.

Ejemplo 1.89
Sea el tensor de cuarto orden C , cuyas componentes vienen dadas por:
Cijkl = ij kl + ( ik jl + il jk ) (1.93)
donde y son constantes. Demostrar que C es un tensor istropo.
Solucin: Un tensor de cuarto orden ser istropo si se cumple que C ijkl = C ijkl , donde Cijkl
son las componentes del tensor debido a transformacin de coordenadas. Teniendo en cuenta
que la ley de transformacin de las componentes de un tensor de cuarto orden viene dada por:
C ijkl = a im a jn a kp a lq C mnpq (1.94)

donde aij es la matriz de transformacin de base, luego:

Universidad de Castilla- La Mancha Draft Por: Eduardo W. V. Chaves (2012)


Ciudad Real - Espaa
1 TENSORES 83

[ (
C ijkl = a im a jn a kp a lq mn pq + mp nq + mq np )]
= a im a jn a kp a lq mn pq + (a im a jn a kp a lq mp nq + a im a jn a kp a lq mq np )
(
= a in a jn a kq a lq + a ip a jq a kp a lq + a iq a jn a kn a lq ) (1.95)
(
= ij kl + ik jl + il jk )
= C ijkl

donde hemos utilizado que a ik a jk = ij , o en notacin matricial AA T = 1 , ya que la matriz de


transformacin es una matriz ortogonal. Luego, hemos demostrado que C es istropo, i.e. las
componentes de C no cambian para cualquier cambio de base.

Ejemplo 1.90
Sea C un tensor de cuarto orden simtrico e istropo representado por:
Cijkl = ij kl + ik ( jl + il jk ) (notacin indicial)
C = 1 1 + 2 I (notacin tensorial)
donde , son escalares constantes, 1 es el tensor identidad de segundo orden, I es la
parte simtrica del tensor identidad de cuarto orden, es decir, I I sym .
Se pide:
a) Dado un tensor de segundo orden simtrico , obtener que viene dado por la
siguiente operacin = C : . Expresar el resultado en notacin tensorial e indicial.
b) Demostrar que y presentan los mismos autovectores (mismas direcciones
principales).
c) Si son los autovalores (valores principales) del tensor , obtener tambin los
autovalores del tensor .

Solucin: a)
Notacin tensorial: Notacin indicial
ij = C ijkl kl
= C: [ (
= ij kl + ik jl + il jk )] kl
= (1 1 + 2 I ) : (
= ij kl kl + ik jl kl + il jk kl )
= 1 1
{ : + 2 I{
Tr ( )
:
sym
(
= ij kk + ij + ji )
= Tr ( )1 + 2 = ij kk + 2 ( )
ijsym
= ij kk + 2 ij

donde hemos considerado la simetra del tensor = T .


b) y c) Partiendo de la definicin de autovalor y autovector del tensor :
n = n
Reemplazando el valor de obtenido anteriormente podemos decir que:

Universidad de Castilla- La Mancha Draft Por: Eduardo W. V. Chaves (2012)


Ciudad Real - Espaa
84 PROBLEMAS RESUELTOS DE MECNICA DEL MEDIO CONTINUO

(Tr()1 + 2 ) n = n
Tr ( )1 n + 2 n = n
Tr ( )n + 2 n = n
2 n = n Tr ( )n = ( Tr ( ) )n
Tr ( )
n = n
2
n = n
Con lo cual concluimos que y presentan los mismos autovectores (son coaxiales). Y
adems los autovalores de se pueden obtener como:
Tr ( )
=
2

Si denominamos que (1) = 1 , (2) = 2 , (3) = 3 y (1) = 1 , ( 2) = 2 , (3) = 3 . La forma


explcita de la relacin anterior viene dada por:
1 0 0 1 0 0 1 0 0
0 1 Tr ( )
0 = 0 2 0 0 1 0
2 2
2
0 0 3 0 0 3 0 0 1

1 0 0 1 0 0 1 0 0
donde tambin se cumple que: 0 2
0 = Tr ( ) 0 1 0 + 2 0 2 0
0 0 3 0 0 1 0 0 3

Ejemplo 1.91
a) Obtener la inversa del tensor de cuarto orden C = 2 I + 1 1 donde I I sym es el tensor
identidad simtrico de cuarto orden, 1 es el tensor identidad de segundo orden, y > 0 ,
son escalares.
E
b) Obtener el determinante de C . Adems si consideramos que = ,
(1 + )(1 2 )
E
= , que valores deben asumir E y para que el tensor sea definido positivo.
2(1 + )
c) Obtener la expresin inversa de = C : en funcin de > 0 , , donde y son
tensores simtricos de segundo orden.
Solucin:
a) Recurrimos a la ecuacin obtenida en (1.76):

( D + A B ) 1 =
1

D 1

( + B : D 1 : A )
[(D 1
: A ) (B : D 1 ) ]
Haciendo que D = I , A = B = 1 , = 2 , = , obtenemos que:

C 1 = ( 2 I + 1 1) 1 =
1 1
2
I

2 ( 2 + 1 : I 1 : 1)
[
(I 1 : 1) (1 : I 1 ) ]

Universidad de Castilla- La Mancha Draft Por: Eduardo W. V. Chaves (2012)


Ciudad Real - Espaa
1 TENSORES 85

Recordar que se cumple que I 1 = I , (I 1 : 1) = I : 1 = 1 . A continuacin obtenemos el valor


del escalar 1 : I 1 : 1 = 1 : I : 1 = 1 : 1 = Tr (1) = 3 . Tambin lo expresamos en notacin indicial:
1
1 : I 1 : 1 = 1 : I : 1 = ij I ijkl
sym
kl = ij ( ik jl + il jk ) kl
2
1
= ( ij ik jl kl + ij il jk kl )
2
1
= ( jj + jj ) = 3
2
Resultando que:
1
C 1 = ( 2 I + 1 1) 1 = I (1 1)
2 2 ( 2 + 3 )

Verifiquemos si se cumple que C : C 1 = I sym I :


1
C : C 1 = ( 2 I + 1 1) : I (1 1)
2 2 ( 2 + 3 )
2 2 2
C : C 1 = ( I :I I : (1 1) + (1 1) : I (1 1) : (1 1)
2 2 ( 2 + 3 ) 2 2 ( 2 + 3 )
Segn el Ejemplo 1.27 se cumplen que I : I = I , I : (1 1) = (1 1) : I = 1 1 , y
(1 1) : (1 1) = 3(1 1) . Con eso obtenemos que:

2 3 2
C : C 1 = I + + (1 1) = I

2 (
14444442 + 3 ) 2 2 ( 2 + 3 )
424444444 3
=0

b) Podemos utilizar directamente la expresin (1.37) del Ejemplo 1.51


( )
det I sym + A B = 3 + 2 A : B

Haciendo = 2 , = , A = B = 1 concluimos que:


det (2 I + 1 1) = ( 2 ) 3 + ( 2 ) 2 1 : 1 = (2 ) 3 + (2 ) 2 3 = (2 ) 2 (2 + 3)

Para que C sea positivo definido, los autovalores tienen que ser positivos, i.e.:
E
>0 = >0
2(1 + )
E E E
2 + 3 > 0 2 +3 = >0
2(1 + ) (1 + )(1 2) (1 2)
Denotando por y1 = (1 + ) 0 , y 2 = (1 2 ) 0 , podemos concluir que:

Universidad de Castilla- La Mancha Draft Por: Eduardo W. V. Chaves (2012)


Ciudad Real - Espaa
86 PROBLEMAS RESUELTOS DE MECNICA DEL MEDIO CONTINUO

E > 0

E E y1 > 0
= = >0
2(1 + ) 2 y1 E < 0
y < 0
1
E > 0

E E y 2 > 0
2 + 3 = = >0
(1 2 ) y 2 E < 0
y < 0
2
Las condiciones anteriores tienen que cumplir simultneamente. Luego, a travs de la grfica
abajo, podemos concluir que E > 0 y 1 < < 0,5 .

y ()
y2 = (1 2 ) 0

y1 = (1 + ) 0
zona no factible
zona no factible

1
1

( y 2 > 0 E > 0) E >0 ( y 1 > 0 E > 0)


( y 1 < 0 E < 0) ( y 2 < 0 E < 0)
0,5

c)
= C: C 1 : = C 1 : C : C 1 : = I sym : = sym =
= C 1 :
1 1
= I 1 1 : = I: 1 1 :
2 2 ( 2 + 3 ) 2 2 ( 2 + 3 )
1
= Tr ( )1
2 2 ( 2 + 3 )
Ver Figura 1.4. Es interesante comparar esta ltima expresin con el Ejemplo 1.41.

Universidad de Castilla- La Mancha Draft Por: Eduardo W. V. Chaves (2012)


Ciudad Real - Espaa
1 TENSORES 87

x3 C x3
33 33
= C:

13 13
23 32 23 32
31 22 31 22

12 12
21 x2 21 x2
11 11

x1 x1
= C 1 :

C 1

Figura 1.4

Ejemplo 1.92
) un tensor de segundo orden, denominado de tensor acstico elstico, y definido
Sea Q e (N
como:
Q e (N Ce N
) =N

donde C e es un tensor de cuarto orden simtrico e istropo dado por C e = (1 1) + 2 I ,


cuyas componentes son: Cijkl e
= ij kl + ( ik jl + il jk ) . Obtener las componentes del
tensor acstico en funcin de las constantes , .
Solucin:
Utilizando notacin simblica obtenemos que:
Q e (N Ce N
) =N = N e
i i ( ) (C e
pqrs e p e q e r e s ) (N e )
j j

( )
= N i C epqrs N j ip sj e q e r = N
C e N e e
p pqrs s q (r )
) son:
Luego, las componentes de Q e (N
C e N
Q e qr = N p pqrs s

p [
+ ( + ) N
=N pq rs pr qs ps qr s ]
= pq rsN pN s + (N
N + N N
p pr qs s

p ps qr s )

N + (N
= N N + N N )
q r r q s qr s

Ya que N es un versor se cumple que N s N s = 1 , resultando que:

Q e qr = qr + ( + )N
N
q r ; ) = 1 + ( + )N
Q e (N N

Universidad de Castilla- La Mancha Draft Por: Eduardo W. V. Chaves (2012)


Ciudad Real - Espaa
88 PROBLEMAS RESUELTOS DE MECNICA DEL MEDIO CONTINUO

Ejemplo 1.93
Sea Q un tensor de segundo orden simtrico y dado por:
) = 1 + ( + )N
Q (N N

donde , son constante y N es un versor.


) y determinar las restricciones de e para que exista la
a) Obtener los autovalores de Q (N
) , i.e. Q 1 .
inversa de Q (N
E E
b) Teniendo en cuenta que = , = , determinar los valores posibles de
(1 + )(1 2 ) 2(1 + )
) sea un tensor definido positivo.
( E , ) para que Q (N
).
c) Obtener la inversa de Q (N
Solucin:
r r
a) Fue demostrado en el Ejemplo 1.50 que, dados dos vectores a y b se cumple que:
( r r
)
r r
det 1 + a b = 3 + 2 a b
Los autovalores se obtienen al resolver el determinante caracterstico det (Q 1) = 0 , donde
i son los autovalores de Q . Luego:

(
det 1 + ( + )N N 1 = 0 )
(
det ( )1 + ( + )N N =0 )
Haciendo = ( ) y = ( + ) concluimos que:

(
det ( )1 + ( + )N N =0 )
N
( ) 3 + ( ) 2 ( + )N =0
{
=1

( ) 2 [( ) + ( + )] = 0
( ) 2 [( + 2 ) ] = 0
La ecuacin caracterstica anterior, ecuacin cbica en , tiene las siguientes soluciones:
1 =
( ) = 0
2
( ) [( + 2 ) ] = 0
2 solucin

2 =
[( + 2 ) ] = 0 = ( + 2 )
3

En el espacio principal de Q , las componentes de Q vienen dadas por:


0 0
Qij = 0 0

0 0 ( + 2 )
Para que haya la inversa de Q , el determinante de Q tiene que ser distinto de cero:
0
Q = 2 ( + 2 ) 0
+ 2 0 2

Universidad de Castilla- La Mancha Draft Por: Eduardo W. V. Chaves (2012)


Ciudad Real - Espaa
1 TENSORES 89

b) Un tensor ser definido positivo si sus autovalores son mayores que cero, luego:
E
= 2(1 + ) > 0


+ 2 = E E E (1 )
+2 = >0
(1 + )(1 2) 2(1 + ) (2 2 + 1)

(1 + ) 0 1

Verifiquemos que 2 1
(2 + 1) 0 0,5

Denotando por y1 = (1 + ) 0 , y 2 = (1 ) 0 , y 3 = ( 2 2 + 1) 0 , podemos reescribir


las condiciones como:
E > 0

E y1 > 0
= 2 y > 0
1 E < 0
y < 0
1

y 2 , y3 > 0
E > 0
+ 2 = Ey2 > 0 y 2 , y3 < 0

y3 y2 > 0, y3 < 0
E < 0
y2 < 0, y3 > 0
Resumiendo:
E > 0 ] 1 ; 0,5[ ] 1 ; [

E < 0 ] ; 1[

y 2 = (1 ) 0 y ()

y1 = (1 + ) 0
zona no factible

1 =1
= 1

E<0 E >0 E >0



= 0,5

y 3 = ( 2 2 + 1) 0

Universidad de Castilla- La Mancha Draft Por: Eduardo W. V. Chaves (2012)


Ciudad Real - Espaa
90 PROBLEMAS RESUELTOS DE MECNICA DEL MEDIO CONTINUO

) en el espacio principal de Q (N
c) Las componentes de la inversa de Q (N ) viene dadas por:

1
0 0
0 0
1 1
Qij = 0 0
inversa
Qij 1 = 0 0 Q 1 =
2 ( + 2 )
0 0 ( + 2 ) 1
0 0
( + 2 )

) 1 son Q 1 = Q 1 = 1 1
Los valores principales de Q (N , Q31 = . Recordar que un
1 2
( + 2 )
tensor y su inversa comparten el mismo espacio principal, i.e. son tensores coaxiales. Adems
) 1 como:
podemos hacer la representacin espectral de Q (N
3
Q 1 = Q
a =1
1 ( a )
a N
(a)
N

= Q11N (1) N
(1) + Q 1N (2) N ( 2 ) + Q 1N ( 3) N ( 3)
2 3
(1) N
= Q 1 (N (1) + N
( 2) N ( 2 ) ) + Q 1N ( 3) N ( 3)
1 3

= ( 3)
Q11 (1 NN )+ ( 3) (3)
Q31N ( 3)
N
N
= Q11 (1 N ) + Q 1N
N

3

( 3) = N
donde hemos considerado que N . Es interesante ver el Ejemplo 1.76. Luego:

N
Q 1 = Q11 (1 N ) + Q 1N
N

3

1 N
)+ 1 N

= (1 N N
( + 2 )
1 1 1 N

= 1 NN+ N
( + 2 )
1 1 1
= 1 N N
( + 2 )
1 +
= 1 N N
( + 2 )

Notar que Q 1 = N( 1 N
Ce N )
C e 1 N
, donde Ce
1
=
1
2
I

2 (2 + 3 )
(1 1) .
1
Ce N
Calculamos Qinv = N :

Universidad de Castilla- La Mancha Draft Por: Eduardo W. V. Chaves (2012)


Ciudad Real - Espaa
1 TENSORES 91

C 1 N
Qinv = N

1 1 ( + )
(Qinv ) jk = N i C ijkl
1
Nl = N i ij kl N l
2 2 2 (2 + 3 )
ik jl il jk

1 1
(Qinv ) jk = (Ni ik jl N l + N i il jk N l ) N
N
2 2 2 (2 + 3 )
i ij kl l

1
(Qinv ) jk = (Nk N j + N l N l jk ) N j N

4 2 (2 + 3 )
k

1 1
(Qinv ) jk = + N j Nk
4 4 2 (2 + 3 )
jk

1 2 +
(Qinv ) jk = + N j Nk
4 4 (2 + 3 )
jk

Luego:
1 2 +
Qinv = 1 + N N
4 4 (2 + 3 )
Notar que 0 e (2 + 3 ) 0 y que adems son las mismas condiciones para que C 1
(ver Ejemplo 1.91).

Universidad de Castilla- La Mancha Draft Por: Eduardo W. V. Chaves (2012)


Ciudad Real - Espaa
92 PROBLEMAS RESUELTOS DE MECNICA DEL MEDIO CONTINUO

1.1.12 Descomposicin Polar

Ejemplo 1.94
Considrese un tensor F que tiene inversa ( det ( F ) 0 ), y que puede ser descompuesto
como:
F = Q U = V Q
( a ) , y V tiene los autovalores
Si U tiene los autovalores a asociados a los autovectores N a
(a )
asociados a los autovectores n , probar que:
a = a
( a ) y n ( a ) .
Obtener tambin la relacin que hay entre los autovectores N
Solucin:
Partiendo de la definicin de autovalor, autovector del tensor U :
UN
(a) = N
a
(a) (aqu el ndice no indica suma)
Por la definicin de F podemos obtener las siguientes relaciones:
QT F = QT Q U = QT V Q
QT F = U = QT V Q
Reemplazando en la definicin de autovalor, autovector:
UN (a ) = N
a
(a)

QT V Q N
(a ) = N
a
(a)

Q QT V Q N
(a ) = Q N
a
(a)
123
1

Resultando:
V Q N
(a) = Q N
a
(a)

V n ( a ) = a n ( a )

donde hemos considerado que n ( a ) = Q N


( a ) . Adems comparando las dos definiciones de
autovalor y autovector de los tensores U y V podemos comprobar que tienen los mismos
autovalores y distintos autovectores y que estn relacionados por la transformacin ortogonal
n ( a ) = Q N
(a) .

1.1.13 Tensor Esfrico y Desviador

Ejemplo 1.95
Considrese un tensor de segundo orden simtrico y su parte desviadora s dev .
s
a) Obtener el resultado de la operacin s : .

Universidad de Castilla- La Mancha Draft Por: Eduardo W. V. Chaves (2012)


Ciudad Real - Espaa
1 TENSORES 93

b) Demostrar tambin que los tensores y dev son tensores coaxiales.


Solucin:
a) Teniendo la definicin de un tensor desviador = esf + dev = esf + s . Obtenemos que:
I
s= 1 . Luego:
3
I
1
s
=
3 = [ ] 1 [I ] 1
3
En notacin indicial:
s ij ij 1 [I ] 1
= ij = ik jl kl ij
kl kl 3 kl 3
Con lo cual
s ij 1 1 1
s ij = s ij ik jl kl ij = s ij ik jl s ij kl ij = s kl kl s ii = s kl
kl 3 3 3 {
=0

s
s: =s

b) Para demostrar que dos tensores son coaxiales, hay que cumplir que:
I
dev = dev = ( esf ) = esf = 1
3
I I
= 1 = 1
3 3
I
= 1 = dev
3

Con lo cual demostramos que los tensores y dev son coaxiales, es decir, tienen las mismas
direcciones principales.

1.1.14 Otros

Ejemplo 1.96
1 1
Considere J = [det (b )] 2 = ( III b ) 2 , donde b es un tensor de segundo orden simtrico, b = b T .
Obtener la derivada de J y de Ln(J ) con respecto a b .
Solucin:
1

( III b ) 2
J
=
b b
1 1 III 1 1
= ( III b ) 2 b
= ( III b ) 2 III b b T
2 b 2
1 1 1
= ( III b ) 2 b 1 = J b 1
2 2

Universidad de Castilla- La Mancha Draft Por: Eduardo W. V. Chaves (2012)


Ciudad Real - Espaa
94 PROBLEMAS RESUELTOS DE MECNICA DEL MEDIO CONTINUO

1

Ln III b 2
[Ln(J )] 1 III b 1 1
= = = b
b b 2 III b b 2

1.1.15 Notacin de Voigt

Ejemplo 1.97
a) Escribir la relacin = C : en notacin de Voigt, donde C = 1 1 + 2 I es un tensor
de cuarto orden, y los tensores simtricos y estn estructurados segn notacin de Voigt
como:
11 11

22 22

{} = 33 ; {} = 33
12 2 12
23 2 23

13 2 13

b) Escribir la ecuacin = C 1 : en notacin de Voigt, donde el tensor C 1 (ver Ejemplo


1.90) viene dado por:
1
C 1 = I 1 1
2 2 ( 2 + 3 )
Solucin:
Podemos escribir la relacin = (1 1 + 2 I ) : en notacin indicial:
1
2
(
ij = ij kl + 2 ik jl + il jk ) kl [ (
= ij kl + ik jl + il jk )] kl

El tensor identidad de segundo orden en notacin de Voigt queda:
1
1
1 0 0
1
ij = 0 1 0 Voigt
{} =
0 0 1 0
0

0
Luego el trmino (1 1)ij = ij kl en notacin de Voigt queda:

Universidad de Castilla- La Mancha Draft Por: Eduardo W. V. Chaves (2012)


Ciudad Real - Espaa
1 TENSORES 95

1 1 1 1 0 0 0
1 1 1 1 0 0 0

I ijkl = ij kl Voigt
[]
1 1
I = [1 1 1 0 0 0] =
1 1 0 0 0
= {}{}
T

0 0 0 0 0 0 0
0 0 0 0 0 0 0

0 0 0 0 0 0 0

El tensor identidad de cuarto orden, I ijkl =


1
2
( )
ik jl + il jk , en notacin de Voigt:

I1111 I1122 I1133 I1112 I1123 I1113 1 0 0 0 0 0


I 1 0 0 0 0
2211 I 2222 I 2233 I 2212 I 2223 I 2213 0
I 3311 I 3322 I 3333 I 3312 I 3323 I 3313 0 0 1 0 0 0
I ijkl [I ] =
Voigt =
I1211 I1222 I1233 I1212 I1223 I1213 0 0 0 12 0 0
I 2311 I 2322 I 2333 I 2312 I 2323 I 2313 0 0 0 0 12 0

I1311 I1322 I1333 I1312 I1323 I1313 0 0 0 0 0 12

Con eso podemos decir que C = 1 1 + 2 I y en notacin de Voigt queda:


1 1 1 0 0 0 1 0 0 0 0 0 + 2 0 0 0
0
1
1 1 0 0 0
1 0 0 0 0
+ 2 0 0 0
1 1 1 0 0 0 0 0 1 0 0 0 + 2 0 0 0
[C ] = + 2 =
0 0 0 0 0 0 0
1
0 0 2 0 0 0 0 0 0 0
0 0 0 0 0 0 0 0 0 0 2 0 0
1
0 0 0 0

0 0 0 0 0 0 0 0 0 0 0 12 0 0 0 0 0
Luego:
11 + 2 0 0 0 11
+ 2 0 0 0 22
22
33 + 2 0 0 0 33
= (1 1 + 2 I ) : Voigt
=
12 0 0 0 0 0 212
23 0 0 0 0 0 2 23

13 0 0 0 0 0 213
14444444444 4244444444444 3

{ } = [C ]{ }
b)
= C 1 :
1 1
= I 1 1 : = I: 1 1 :
2 2 ( 2 + 3 ) 2 2 ( 2 + 3 )
1
= Tr ( )1
2 2 ( 2 + 3 )
1
ij = ij kk ij
2 2 ( 2 + 3 )
Notar que:

Universidad de Castilla- La Mancha Draft Por: Eduardo W. V. Chaves (2012)


Ciudad Real - Espaa
96 PROBLEMAS RESUELTOS DE MECNICA DEL MEDIO CONTINUO

1 +
11 = 11 (11 + 22 + 33 ) 11 = 11 ( 22 + 33 )
2 2 ( 2 + 3 ) ( 2 + 3 ) 2 ( 2 + 3 )
1 +
22 = 22 (11 + 22 + 33 ) 22 = 22 (11 + 33 )
2 2 ( 2 + 3 ) ( 2 + 3 ) 2 ( 2 + 3 )
1 +
33 = 33 (11 + 22 + 33 ) 33 = 33 (11 + 22 )
2 2 ( 2 + 3 ) ( 2 + 3 ) 2 ( 2 + 3 )
1 1 1
12 = 12 ( kk ) 12 = 12 212 = 12
2 2 ( 2 + 3 ) {
=0
2
1
2 23 = 23

1
213 =
13
Reestructurando las ecuaciones anteriores en notacin de Voigt, obtenemos:
+
( 2 + 3 ) 0 0 0
2 ( 2 + 3 ) 2 ( 2 + 3 )

+
11
0 0 0 11
2 ( 2 + 3 ) ( 2 + 3 ) 2 ( 2 + 3 )
+ 22
0
22
33 2 ( 2 + 3 ) 0 0
2 ( 2 + 3 ) ( 2 + 3 ) 33
= 1
2 12 0 0 0 0 0 12
2 23 23
1
213 0 0 0 0 0 13

1
0 0 0 0 0

{ } = [C ]1 { }

Ejemplo 1.98
r
Considrese T ( x , t ) un tensor simtrico de segundo orden, el cual es funcin de la posicin
r
( x ) y del tiempo (t ) . Considrese tambin que las componentes del tensor segn la direccin
x3 son iguales a cero, i.e. T13 = T23 = T33 = 0 .
r r
NOTA: Definimos T ( x , t ) como un campo tensorial, i.e. el valor de T ( x , t ) depende de la
posicin y del tiempo. Si el tensor es independiente de una direccin para todo el dominio
r r
( x ) , e.g. si T ( x , t ) es independiente de la direccin x3 , ver Figura 1.5, el problema puede ser
considerado como bidimensional (estado plano) simplificando bastante las ecuaciones.

Universidad de Castilla- La Mancha Draft Por: Eduardo W. V. Chaves (2012)


Ciudad Real - Espaa
1 TENSORES 97

2D
x2 T T12
Tij = 11
T12 T22
x2
T22
T12 T22

T12 T12
T11 T11
T11
x1
T12
x1
T22
x3

Figura 1.5: Problema bidimensional (2D).

a) Obtener las componentes T11 , T22


, T12 , tras un cambio de base en el plano x1 x 2 tal
como se indica en la Figura 1.6;
b) Obtener tambin el valor de correspondiente a las direccin principales de T .
c) Obtener los valores de Tij , (i, j = 1,2) , cuando T11 = 1 , T22 = 2 , T12 = 4 y = 45 .
Obtener tambin los valores principales y direcciones principales de T ;
d) Dibujar una grfica mostrando la relacin entre y las components T11 , T22
y T12 , para
una variacin de 0 hasta 360 .
OBS.: Utilizar notacin de Voigt, y expresar los resultados en funcin de 2 .

a11 a12 0 cos sin 0


x2
a ij = a 21 a 22 0 = sin cos 0
x 2 x1 0 0 1 0 0 1


x1

Figura 1.6: Matriz de transformacin para problema bidimensional (2D).

Solucin:
a) Podemos utilizar directamente la ley de transformacin en la notacin de Voigt
{T } = [M] {T } , donde

Universidad de Castilla- La Mancha Draft Por: Eduardo W. V. Chaves (2012)


Ciudad Real - Espaa
98 PROBLEMAS RESUELTOS DE MECNICA DEL MEDIO CONTINUO

T11 T11
T T
22 22
T T
{T } = 33 ; {T } = 33
T12 T12
T23
T23

T13 T13

a11 2 a12 2 a13 2 2a11 a12 2a12 a13 2a11 a13


2
a 21 a 22 2 a 23 2 2a 21 a 22 2a 22 a 23 2a 21 a 23
2
a 32 2 a 33 2
[M] = a 31 2a 31 a 32 2a 32 a 33 2a 31 a 33

a 21 a11 a 22 a12 a13 a 23 (a11 a 22 + a12 a 21 ) (a13 a 22 + a12 a 23 ) (a13 a 21 + a11 a 23 )
a a a 32 a 22 a 33 a 23 (a 31 a 22 + a 32 a 21 ) (a 33 a 22 + a 32 a 23 ) (a 33 a 21 + a 31 a 23 )
31 21
a 31 a11 a 32 a12 a 33 a13 (a 31 a12 + a 32 a11 ) (a 33 a12 + a 32 a13 ) (a 33 a11 + a 31 a13 )
En este caso particular la matriz de transformacin [M] tras eliminar filas y columnas
asociadas con la direccin x3 queda:

T11 a11 T11


2 2
a12 2a11 a12
T = a 2 2
22 21 a 22 2a 21 a 22 T22 (1.96)
T12 a 21 a11 a 22 a12 a11 a 22 + a12 a 21 T12

T = A T AT
x2

x 2
T22
T22 T12 T11

T12
T11 T11 x1
P
P P
T11

T12 T12 x1
x1
T22
T22

T = AT T A

Figura 1.7: Matriz de transformacin para problema bidimensional (2D).

La matriz de transformacin ( a ij ) en el plano viene dada en funcin de un nico parmetro,


:
a11 a12 a13 cos sin 0
a ij = a 21 a 22 a 23 = sin cos 0 (1.97)
a 31 a 32 a 33 0 0 1

Universidad de Castilla- La Mancha Draft Por: Eduardo W. V. Chaves (2012)


Ciudad Real - Espaa
1 TENSORES 99

Reemplazando los valores de los coeficientes en la ecuacin (1.96) obtenemos que:


T11 cos sin 2 2 cos sin T11
2

T = sin 2 cos 2

2 sin cos T22
22 (1.98)
T12 sin cos cos sin cos 2 sin 2 T12

Tomando partido de las siguientes relaciones trigonomtricas, 2 cos sin = sin 2 ,
1 cos 2 1 + cos 2
cos 2 sin 2 = cos 2 , sin 2 = , cos 2 = , obtenemos que:
2 2
1 + cos 2 1 cos 2
sin 2
T11 2 2 T11
T = 1 cos 2 1 + cos 2
22 sin 2 T22
2 2
T12 T12

sin 2 sin 2
cos 2
2 2
Explcitamente las componentes vienen dadas por:
1 + cos 2 1 cos 2
T11 = T11 + T22 + T12 sin 2
2 2
1 cos 2 1 + cos 2
=
T22 T11 T22 T12 sin 2
2 2
sin 2 sin 2
T12 = T11 + T22 + T12 cos 2
2 2
Reestructurando la expresin anterior an podemos decir que:

T11 + T22 T11 T22


T11 = + cos 2 + T12 sin 2
2 2
T + T T
22 11 T 22
= 11
T22 cos 2 T12 sin 2 (1.99)
2 2
T T22
T12 = 11 sin 2 + T12 cos 2
2

b) Recordemos que las direcciones principales se caracterizan por la ausencia de las


componentes tangenciales, es decir, Tij = 0 para i j . Si queremos encontrar las direcciones
principales en el caso plano hacemos que T12 = 0 , obteniendo as:
T T22 T11 T22
T12 = 11 sin 2 + T12 cos 2 = 0 sin 2 = T12 cos 2
2 2
sin 2 2 T12 2 T12
= tg(2 ) =
cos 2 T11 T22 T11 T22
Con lo cual:

1 2 T12
= arctg (1.100)
2 T11 T22

Universidad de Castilla- La Mancha Draft Por: Eduardo W. V. Chaves (2012)


Ciudad Real - Espaa
100 PROBLEMAS RESUELTOS DE MECNICA DEL MEDIO CONTINUO

Para encontrar los valores principales (autovalores), debemos resolver la siguiente ecuacin
caracterstica:
T11 T T12
T12 T22 T
=0 (
T 2 T ( T11 + T22 ) + T11 T22 T122 = 0 )
cuya solucin viene dada por:

T(1, 2 ) =
[ ( T11 + T22 )] [ (T11 + T22 )]2 (
4(1) T11 T22 T122 )
2(1)

=
T11 + T22

[(T11 + T22 )] 2
(
4 T11 T22 T122 )
2 4
Reestructurando la ecuacin anterior obtenemos que:

2
T + T22 T T22 (1.101)
T(1, 2 ) = 11 11 + T122
2 2

c) Para obtener las componentes Tij , (i, j = 1,2) aplicamos directamente (1.99), donde T11 = 1 ,
T22 = 2 , T12 = 4 y = 45 , i.e.:

1+ 2 1 2
T11 = 2 + 2 cos 90 4 sin 90 = 2,5

1+ 2 1 2
=
T22 cos 90 +4 sin 90 = 5,5
2 2
1 2
T12 = sin 90 4 cos 90 = 0,5
2
El ngulo correspondiente a la direccin principal viene dado por:
1 2 T12 2 (4)

= arctg = ( = 41.4375 )
2 11T T 22 1 2
r
Las valores principales de T ( x , t ) pueden ser obtenidos como sigue:
2
T11 + T22 T T22 T1 = 5.5311
T(1, 2 ) = 11 + T122
2 2 T2 = 2.5311
d) Teniendo en cuenta la ecuacin (1.99) y variando de 0 hasta 360 podemos los valores
de T11 , T22
, T12 , y cuyos valores estn ilustrados en la siguiente grfica:

Universidad de Castilla- La Mancha Draft Por: Eduardo W. V. Chaves (2012)


Ciudad Real - Espaa
1 TENSORES 101

x1
T1
= 41.437
T2

x1
= 131.437
8 2
Components

T1 = 5.5311

T22
6

T22 2
T12
T11
0
0 50 100 150 200 250 300 350

-2
45 x1 T11

T12 -4 T2 = 2.5311
= 86.437

-6

TS max = 4.0311

Ejemplo 1.99
Obtener los valores principales y direcciones principales de la parte simtrica del tensor T ,
cuyas componentes en el sistema cartesiano vienen dadas por:
5 1
Tij = (i, j = 1,2)
3 4
Solucin:
La parte simtrica del tensor viene dada por:
5 2
Tijsym =
1
( )
Tij + T ji =
2 2 4
Valores principales:
5 2
=0 2 9 + 16 = 0
2 4
La solucin de la ecuacin cuadrtica viene dada por:
9 (9) 2 4 (1) (16) 1 T1 = 6,5615
(1, 2 ) =
2 1 2 T2 = 2,4385

Universidad de Castilla- La Mancha Draft Por: Eduardo W. V. Chaves (2012)


Ciudad Real - Espaa
102 PROBLEMAS RESUELTOS DE MECNICA DEL MEDIO CONTINUO

Podemos dibujar el crculo de Mohr del tensor T sym :

TSsym

(T11sym , T12sym )

2
TII = 2,4385 TI = 6,5615 T Nsym

En el caso plano, la direccin principal se puede obtener directamente a travs de la relacin:


2 T12sym 22
tan(2) = = =4 = 37,982
T11sym sym
T22 54

1.1.16 Campo de Tensores.

Ejemplo 1.100
Encuentre el gradiente de la funcin f ( x1 , x2 ) = sin( x1 ) + exp x1x2 en el punto (0,1).
Solucin:
Por definicin el gradiente de una funcin escalar viene definido de la forma:
f f
xr f = e1 + e2
x1 x 2
donde:
f f
= cos( x1 ) + x 2 exp x1x2 ; = x1 exp x1x2
x1 x 2

[ ] [
xr f ( x1 , x 2 ) = cos( x1 ) + x 2 exp x1 x2 e 1 + x1 exp x1x2 e 2 ]
xr f (0,1) = [2] e 1 + [0] e 2 = 2e 1

Ejemplo 1.101
r
Supongamos que v y son respectivamente vector y escalar, y dos veces diferenciables
continuamente. Usando notacin indicial, demostrar que:
r r
a) xr ( xr v ) = 0
b) xr ( xr ) = xr 2

Universidad de Castilla- La Mancha Draft Por: Eduardo W. V. Chaves (2012)


Ciudad Real - Espaa
1 TENSORES 103

c) xr ( ) = xr + xr
r r r
d) xr (v ) = xr v + xr v
e) xr ( A B) = xr A : B + A ( xr B) (donde A y B son tensores de segundo orden)
Solucin:
a) Considerando que
r r
xr v = ijk v k , j e i (1.102)
()
xr () = e l (1.103)
xl

luego
r r
xr ( xr v ) = ( ijk v k , j e i ) e l = ( ijk v k , j il )
xl x l
(1.104)
= ( ljk v k , j )
xl
= ljk v k , jl

Observemos que ljk es un tensor antisimtrico en lj y vk , jl es simtrico con lj , luego:

ljk v k , jl = 0 (1.105)

b)

xr ( xr ) = ( ,i e i ) e j = ( ,i ij )
x j x j
, j
= = , jj (1.106)
x j

2
= = = xr
2

x j x j x 2
j

c)
[ xr ( )]i = ( ) ,i = ,i + ,i
(1.107)
= [ xr ]i + [ xr ]i
r
d) El resultado de la operacin xr (v ) resulta un escalar, luego:
r
xr (v ) = (vi ) ,i = ,i v i + v i , i
r r
= xr v + xr v
e) Considerando que ( A B) ij = A ik B kj , [ xr ( A B)]i = ( A B) ij , j = ( A ik B kj ) , j , luego
( A ik B kj ) , j = A ik , j B kj + A ik B kj , j = [ xr A : B ]i + [A ( xr B)]i

Universidad de Castilla- La Mancha Draft Por: Eduardo W. V. Chaves (2012)


Ciudad Real - Espaa
104 PROBLEMAS RESUELTOS DE MECNICA DEL MEDIO CONTINUO

Ejemplo 1.102
Probar la identidad:
r r r r
xr (a + b) = xr a + xr b
Solucin:
r r r r
Considerando que a = a j e j y b = b k e k y xr = e i podemos expresar xr (a + b) como:
x i

(a j e j + b k e k ) a j b k a b r r
e i = e j e i + e k e i = i + i = xr a + xr b
x i x i x i x i x i
c.q.d.
Desarrollando directamente en notacin indicial:
r r r r
xr (a + b) = (a i + b i ), i = a i , i +b i , i = xr a + xr b

Ejemplo 1.103
r r
Obtener las componentes de ( xr a) b .
Solucin:
r r
Considerando: a = a j e j ; b = b k e k y xr = e i ( i = 1,2,3 ) podemos decir que:
x i

r r (a j e j ) a j a j a j
( xr a) b = e i (b k e k ) = e j e i (b k e k ) = b k ik e j = b k e j
x i
x i x i x k

Expandiendo el ndice mudo k :


a j a j a j a j
bk = b1 + b2 + b3
x k x1 x 2 x 3
luego:
a1 a a
j =1 b1 + b2 1 + b3 1
x1 x 2 x 3
a 2 a a
j = 2 b1 + b2 2 + b3 2
x1 x 2 x 3
a 3 a a
j = 3 b1 + b 2 3 + b3 3
x1 x 2 x 3

Ejemplo 1.104
Probar que la siguiente relacin es vlida:
r
q 1 r r 1 r
= x q 2 q xr T
r
x
T T T
r r r
donde q( x , t ) es un campo vectorial arbitrario y T ( x , t ) un campo escalar.
Solucin:

Universidad de Castilla- La Mancha Draft Por: Eduardo W. V. Chaves (2012)


Ciudad Real - Espaa
1 TENSORES 105

r
q qi q i 1 1
xr = = qi ,i 2 qiT,i
T xi T T ,i T T
1 r 1 r
= xr q 2 q xr T (escalar)
T T

Ejemplo 1.105
Demostrar que:
r r r r r r
a) rot (a) = xr (a) = ( xr a) + ( xr a) (1.108)
r r r r r r r r r r r
b) xr (a b) = ( xr b)a ( xr a)b + ( xr a) b ( xr b) a (1.109)
r r r r r
c) xr ( xr a) = xr ( xr a) xr a
2
(1.110)

d) xr ( xr ) = xr + ( xr ) ( xr )
2
(1.111)

Solucin:
r r
a) El resultado de la operacin xr (a) ser un vector, cuyas componentes vienen dadas
por:
[r r
x
r
(a) i ] = ijk (a k ) , j
= ijk ( , j a k + a k , j )
= ijk a k , j ijk , j a k (1.112)
r r
= ( xr a) i ijk ( xr ) j a k
r r r
= ( xr a) i ( xr a) i
r r r r r r
con lo que comprobamos la identidad: rot (a) = xr (a) = ( xr a) + ( xr a) .
r r r r
Las componentes del producto vectorial (a b) vienen dadas por (a b) k = kij a i b j . Luego:

[r r
x
r r
]
(a b) l = lpk ( kij a i b j ) , p
(1.113)
= kij lpk (a i , p b j + a i b j , p )

b) Considerando que kij = ijk , el resultado de ijk lpk = il jp ip jl y reemplazando en la


expresin anterior obtenemos que:

[r r
x
r r
]
(a b) l = kij lpk (a i , p b j + a i b j , p )
= ( il jp ip jl )(a i , p b j + a i b j , p )
(1.114)
= il jp a i , p b j ip jl a i , p b j + il jp a i b j , p ip jl a i b j , p
= al , p b p a p, p b l + al b p, p a p b l , p

Podemos observar que [( ar ) br ] = a


r
x l l, pb p , [( r
x a)b]l
r r
= a p, p b l , [( r
x b)a]l
r r
= al b p, p ,

[ r r
]
( xr b) a l = a p b l , p .
r r r r
c) Las componentes del producto vectorial ( xr a) vienen dadas por ( xr a) i = ijk a k , j .
123
ci
Luego:

Universidad de Castilla- La Mancha Draft Por: Eduardo W. V. Chaves (2012)


Ciudad Real - Espaa
106 PROBLEMAS RESUELTOS DE MECNICA DEL MEDIO CONTINUO

[r r
x
r r
]
( xr a) q = qli c i ,l
= qli ( ijk a k , j ) ,l (1.115)
= qli ijk a k , jl

Considerando que qli ijk = qli jki = qj lk qk lj , la expresin anterior queda:

[r r
x
r r
]
( xr a) q = qli ijk a k , jl
= ( qj lk qk lj )a k , jl
(1.116)
= qj lk a k , jl qk lj a k , jl
= a k ,kq a q ,ll

Podemos observar que [ xr ( xr a)]q = a k , kq y xr 2 a q = a q ,ll .


r
[ r
]
d)
xr ( xr ) = ( ,i ) ,i
= ,ii + ,i ,i (1.117)
= xr + ( xr ) ( xr )
2

donde y son funciones escalares.


Otra identidad interesante que origina de la anterior es:
xr ( xr ) = xr + ( xr ) ( xr )
2

(1.118)
xr ( xr ) = xr + ( xr ) ( xr )
2

Restando las dos identidades anteriores obtenemos que:


xr ( xr ) xr ( xr ) = xr xr
2 2

(1.119)
xr ( xr xr ) = xr xr
2 2

Ejemplo 1.106
a) Probar que xr ( xr v ) = 0 y que xr ( xr ) = 0 , donde es un campo escalar, y v es
r r r r r

un campo vectorial;
[r r r
]
b) Demostrar que xr ( xr v ) v = ( xr v )( xr v ) + xr ( xr v ) v ( xr v ) ( xr v ) ;
r r r r
[ r r
] r r r r

r r r r r r r r
c) Teniendo en cuenta que = xr v , demostrar que xr ( xr 2 v ) = xr 2 ( xr v ) = xr 2 .
Solucin:
r r
Considerando: xr v = ijk v k , j e i

( xr )
r
xr
r
v =
x l
(
ijk v k , j e i ) e l


= ijk
x l
( )
v k , j il = ijk
x i
vk , j ( )
= ijk v k , ji

Universidad de Castilla- La Mancha Draft Por: Eduardo W. V. Chaves (2012)


Ciudad Real - Espaa
1 TENSORES 107

r
La segunda derivada de v es simtrica en ij , i.e. v k , ji = v k ,ij , mientras que ijk es antisimtrico
en ij , i.e., ijk = jik , luego:
ijk v k , ji = ij1v1, ji + ij 2 v 2, ji + ij 3 v3, ji = 0
Observar que ij1v1, ji es el doble producto escalar de un tensor simtrico con un antisimtrico,
cuyo resultado es cero.
Anlogamente demostramos que:
r r
xr ( xr ) = ijk , kj e i = 0 i e i = 0

NOTA: El rotacional del gradiente de un escalar resulta ser igual al vector nulo, y la
divergencia del rotacional de un vector resulta ser igual a cero.
r r r
b) Denominamos por = xr v , con eso, quedamos con
r r
[( r r
xr xr v v ) ]
r r r
= xr ( v )
Recurrimos a la identidad (1.109), luego, se cumple que:
r r r r r r r r r r r
xr ( v ) = ( xr v ) ( xr )v + ( xr ) v ( xr v )
r r r
Fijemos que el trmino xr = xr ( xr v ) = 0 , que fue demostrado en el apartado a).
Luego, concluimos que:
r r r r r r r r r
xr ( v ) = ( xr v ) + ( xr ) v ( xr v )
r r r r
[
r r r
]
r r
= ( xr v )( xr v ) + xr ( xr v ) v ( xr v ) ( xr v )
c) Recurriendo a la identidad (1.110) podemos decir que:
2r r r r r
xr v = xr ( xr v ) xr ( xr v )
r r r
= xr ( xr v ) xr
Aplicando el rotacional a la expresin anterior obtenemos que:
r 2r
r r r r r
xr ( xr v ) = xr [ xr ( xr v )] xr ( xr )
14442r 444 3
=0
donde hemos tenido en cuenta que el rotacional del gradiente de un escalar resulta el vector
r r r
nulo. Recurrimos una vez ms la identidad (1.110) para expresar el trmino xr ( xr ) ,
resultando:
r 2r
r r r r 2 r
xr ( xr v ) = xr ( xr ) = xr ( xr ) + xr
r
[ r 2 r
= xr xr ( xr v ) + xr
1442443
]
=0
2
r r
= x ( x v )
r r

Ejemplo 1.107
Demostrar que:
r r r r r r r r r r r r
a) xr (a b) = ( xr a) b + a ( xr b) rot (a) b + a rot (b) (1.120)

Solucin:

Universidad de Castilla- La Mancha Draft Por: Eduardo W. V. Chaves (2012)


Ciudad Real - Espaa
108 PROBLEMAS RESUELTOS DE MECNICA DEL MEDIO CONTINUO

r r
La operacin xr (a b) resulta un escalar:
r r
xr (a b) = ( ijk a j b k ) ,i
= ijk a j ,i b k + ijk b k ,i a j
1r2r3 1r r
23
( a)k ( b) j
r r r r r r
= ( xr a) b + a ( xr b)

Ejemplo 1.108
a) Si T es un tensor de segundo orden, obtener la representacin simblica en la base
r r r r
Cartesiana de: a.1) ( xr T ) , a.2) ( xr T )T , a.3) ( xr T T ) , y a.4) ( xr T T )T . a.5)
r
Considerando c un vector constante, demostrar que:
r r r r r r
xr ( T c) = ( xr T ) c = c xr T
T
[ ]
b) Obtener la notacin simblica de xr ( xr )T .
r
u r r
c.1) Consideremos el tensor de segundo orden F = r , demostrar que xr ( xr F )T = 0 ,
x
r T
r
c.2) x F = 0 . c.3) Obtener la expresin explcita de las components de xr F .
r

Solucin:
r Tqj
a.1) ( xr T ) = e p Tqj (e q e j ) = e p e q e j = Tqj , p ipq e i e j = ipq Tqj , p e i e j
x p x p
r
a.2) ( xr T )T = ipq Tqj , p e j e i = jpq Tqi , p e i e j
r T jq
a.3) ( xr T T ) = e p T jq (e q e j ) = e p e q e j = ipq T jq , p e i e j
x p x p
r
a.4) ( xr T T )T = jpq Tiq , p e i e j

donde hemos tenido en cuenta la definicin e j e k = ijk e i .


r r
a.5) Haciendo que a = T c = ( Tqj c j )e q = aq e q , podemos obtener que:
r r a q a
xr ( T c ) = xr a = e p a q e q = ipq e i = ipq q e i = ipq aq , p e i
x p x p x p
ipq a q , p e i = ipq ( Tqj c j ), p e i = ipq Tqj , p c j e i + ipq Tqj c j , p e i = ipq Tqj , p c j e i
{
=0

r c j
donde hemos tenido en cuenta que c es constante, i.e. c j , p = = 0 jp .
x p
r
Notar que ipq Tqj , p son las componentes de ( xr T )ij (vase (a.1)), luego
r r r r
[ r
] r r
[
xr ( T c ) = ipq Tqj , p c j e i = ( xr T )ij c j e i = ( xr T ) c i e i = c ( xr T )T i e i ]
r r
( xr T ) c = ipq Tqj , p e i e j c k e k = ipq Tqj , p c k e i jk = ipq Tqj , p c j e i

Universidad de Castilla- La Mancha Draft Por: Eduardo W. V. Chaves (2012)


Ciudad Real - Espaa
1 TENSORES 109

b) En el apartado hemos demostrado que ( xr ) = ipq qj , p e i e j se cumple, luego


r r qj , p
xr ( xr )T =
xs
(
e s ipq qj , p e j e i = ipq ) xs
( )
e s e j e i = ipq tsj qj , ps e t e i

= ( iqp )( tjs ) qj , ps e t e i = iqp tjs qj , ps e t e i = qpi jst qj , ps e t e i


Notar que:
r r qj , p
xr ( xr ) =

xs
(
e s ipq qj , p e i e j = ipq ) xs
( )
e s e i e j = ipq qj , ps tsi e t e j

= its ipq qj , ps e t e j = ( tp sq tq sp ) qj , ps e t e j
= ( tp sq qj , ps tq sp qj , ps )e t e j = ( sj , ts tj , ss )e t e j

uq r r
c.1) Si consideramos que qj = Fqj = = u q , j , y reemplazamos en xr ( xr )T , podemos
x j
obtener que:
r r
xr ( xr F )T = iqp tjs Fqj , ps e t e i = iqp tjsuq , jps e t e i

Notar que uq , jps = uq , pjs = uq , psj , i.e. es simtrico en js , y el tensor tjs = tsj es antisimtrico
en js , luego tjsuq , jps = 0tqp , y xr ( xr F )T = 0ti e t e i = 0 .
Solucin alternativa:
Teniendo en cuenta que
it ij is
iqp tjs = qt qj qs = it qj ps + ij qs pt + is pj qt is qj pt qs pj it ps qt ij
pt pj ps
concluimos que
iqp tjs Fqj , ps = ( it qj ps + ij qs pt + is pj qt is qj pt )
qs pj it ps qt ij u q , jps
= it qj psu q , jps + ij qs pt uq , jps + is pj qt u q , jps is qj pt u q , jps qs pj it u q , jps ps qt iju q , jps
= it u j , jss + u s ,its + ut , ppi u j , jti it u s , pps ut ,ipp = 0ti

Notar que it u j , jss = it u p , pss = it u p ,ssp = it us , pps , u s ,its = u j ,itj = u j , jti , ut , ppi = ut ,ipp .
r
c.2) Expresando xr F T en notacin indicial podemos obtener que:
r Fqj
xr F T = e p Fqj (e j e q ) = e p e j e q = Fqj , p ipj e i e q
x p x p
= ipj Fqj , p e i e q = ipj uq , jp e i e q = 0 ip e i e q

Notar que uq , jp = uq , pj es simtrico en jp mientras que ipj = ijp es antisimtrico en jp .


r
c.3) Expresando xr F en notacin indicial obtenemos (vase (a.1)):
r
xr F = ipq Fqj , p e i e j = ipquq , jp e i e j

Expandiendo el trmino ipquq , jp podemos obtener que:

Universidad de Castilla- La Mancha Draft Por: Eduardo W. V. Chaves (2012)


Ciudad Real - Espaa
110 PROBLEMAS RESUELTOS DE MECNICA DEL MEDIO CONTINUO

ipquq, jp = ip1u1, jp + ip 2u2, jp + ip 3u3, jp


1
424
3 1424 3 1424 3
i11u1, j1 + i12u2, j1 + i13u3, j1
+ + +
i 21u1, j 2 + i 22u2, j 2 + i 23u3, j 2
+ + +
i 31u1, j 3 + i 32u2, j 3 + ip 3u3, j 3

luego
u3,12 u2,13 u3, 22 u 2, 23 u3,32 u 2,33 F31, 2 F21,3 F32, 2 F22,3 F33, 2 F23,3
r
( xr F )ij = u1,13 u3,11 u1, 23 u3, 21 u1,33 u3,31 = F11,3 F31,1 F12,3 F32,1 F13,3 F33,1
u2,11 u1,12 u2, 21 u1, 22 u2,31 u1,32 F21,1 F11, 2 F22,1 F12, 2 F23,1 F13, 2

Notar que
F13, 2 F12,3 F23, 2 F22,3 F33, 2 F32,3 u1,32 u1, 23 (u2,32 u2, 23 u3,32 u3, 23
r T
( xr F )ij = F11,3 F13,1 F21,3 F23,1 F31,3 F33,1 = u1,13 u1,31 (u2,13 u 2,31 u3,13 u3,31
F12,1 F11, 2 F22,1 F21, 2 F32,1 F31, 2 u1, 21 u1,12 (u 2, 21 u 2,12 u3, 21 u3,12

= 0ij

Ejemplo 1.109
r r r r r
Sea v un campo vectorial que es funcin de x , i.e. v = v ( x ) , donde sus componentes vienen
dadas por:
v1 = x1 5 x 2 + 2 x3

v 2 = 5 x1 + x 2 3 x3
v = 2 x + 3 x + x
3 1 2 3
r r
a) Obtener el gradiente de v ; b) Obtener xr v : 1 ; c) Hacer la descomposicin aditiva del
r
tensor xr v a travs de su parte simtrica y otra antisimtrica; d) Obtener el vector axil
r
asociado al tensor antisimtrico ( xr v ) anti .
Solucin: a)
v1 v1 v1

r x1 x 2 x3 1 5 2
v v v v 2 v 2
1 3
r r
xr v = r componente
s ( xr v ) ij = i = 2 = 5
x x j x x 2 x3
1 1

v3 v 3 v3 2 3
x1 x 2 x3
r r
b) xr v : 1 = Tr ( xr v ) = 1 + 1 + 1 = 3 . Observar que la traza del gradiente es igual a la divergencia,
r v1 v 2 v3
i.e.: xr v = vi ,i = v1,1 + v 2, 2 + v 3,3 = + + = 3.
x1 x 2 x3
r r r
c) xr v = ( xr v ) sym + ( xr v ) anti =
1
[ r r 1
] [r r
( xr v ) + ( xr v ) T + ( xr v ) ( xr v ) T
2 44424443 1 2 44424443
]
1 r r
=( xr v ) sym =( xr v ) anti

Universidad de Castilla- La Mancha Draft Por: Eduardo W. V. Chaves (2012)


Ciudad Real - Espaa
1 TENSORES 111

r r
Luego, las componentes de ( xr v ) sym y ( xr v ) anti vienen dadas respectivamente por:
1 0 0 0 5 2
1 vi v j 1 v i v j
0 3
r r
( xr v ) ijsym = + = 0 1 0 ; ( xr v ) ijanti = = 5
2 x j x i 2 x j xi
0 0 1 2 3 0

d) Recordar que
1 v1 v3
1 v1 v 2
0


2 x 2 x1
2 x 3 x1
1 v 1 v 2 v 3
r
[ ]
(W) ij ( xr v ) anti ij vianti
,j
v
= 2 1 0
2 x3 x 2
2 x1 x 2
1 v
1 v3 v 2
v (1.121)

3 1 0
2 x 2 x 3
2 x1 x 3
0 W12 W13 0 W12 W13 0 w3 w2

= W21 0
W23 = W12 0
W23 = w3 0 w1
W31 W32 0 W13 W23 0 w2 w1 0
r
donde w1 , w2 , w3 son las componentes del vector axil w correspondiente al tensor
r
antisimtrico W ( xr v ) anti , luego para el problema propuesto:
0 w2 0 5 2
w3 w1 = 3
w
0 3 w1 = 5 0 3 w2 = 2
w1 w20 2 3 0 w = 5
3
r
El vector axil en la base Cartesiana queda: w = 3e 1 + 2e 2 + 5e 3 .
1 r r
Solucin Alternativa d) Recurriendo al Ejemplo 1.36 donde hemos demostrado que (a x )
2
r r
es el vector axil asociado al tensor antisimtrico ( x a ) anti . Luego, el vector axil asociado al
r
[r
tensor antisimtrico ( xr v ) anti = (v ) ( xr )
r
]
anti
es el vector w =
r
2
(
1 rr r
x v . )
e 1 e 2 e 3
r 1 1 v v v v v v
w= = 3 2 e 1 3 1 e 2 + 2 1 e 3
2 x1 x 2 x3 2 x 2 x3 x1 x 3 x1 x 2
v1 v2 v3

=
1
2
[ ]
(3 (3) )e 1 ((2) (2) )e 2 + (5 (5))e 3 = 3e 1 + 2e 2 + 5e 3

Ejemplo 1.110
r r
Sea un tensor de segundo orden definido por l = xr v . Teniendo en cuenta que D = ( xr v ) sym
r
y W = ( xr v ) anti demostrar que
r r
W D + D W = 2(D W) anti = ( xr v xr v ) anti = ( l l ) anti
Solucin:

Universidad de Castilla- La Mancha Draft Por: Eduardo W. V. Chaves (2012)


Ciudad Real - Espaa
112 PROBLEMAS RESUELTOS DE MECNICA DEL MEDIO CONTINUO

En el Ejemplo 1.34 hemos demostrado que, dado un tensor de segundo orden arbitrario l
se cumple que
l
anti
l sym
+l sym
l anti
= 2( l anti
l sym anti
)

Luego, se cumple que W D + D W = 2(D W ) anti . Teniendo en cuenta la definicin de


simetra y antisimetra, D =
1
2
[
l +l
T
] , W = 12 [l l ] , podemos concluir que:
T

W D + D W = 2(D W ) anti
2
[
= (l + l T ) (l l T )
4
anti
]
1
[
= l l + l l T l T l l T l T
2
anti
]
1
= 1
2
l l l l
4
T
4 4 2 [
T
4 44
anti
3
1
+ l l l T l
2
] [ ]
T anti

=0
1
[
= l l (l l ) T
2
anti
]
1
[
= 2( l l ) anti
2
anti
]
r r
= ( l l ) anti = ( xr v xr v ) anti

OBS.: Fijemos que el tensor resultante l l T l T


l es simtrico, ya que:
(l l T
l T
l )T = l l T
l T
l .

Ejemplo 1.111
Consideremos un escalar J = F det(F ) y un tensor de segundo orden arbitrario definido
r dF
por l = xr v = F& F 1 , donde F& representa la derivada temporal de F . Demostrar que
dt
se cumple que:

d(J ) & r
J = J ( xr v ) (1.122)
dt

Solucin:
En el Ejemplo 1.87 hemos demostrado que, dado un tensor de segundo orden arbitrario
dA dA
A = A () , se cumple que = A Tr A 1 . Haciendo A = F y = t podemos obtener
d d
que:
dF
dt
=
dJ
dt
dF
= F Tr ( ) (
F 1 = J Tr F& F 1 = J Tr F& F 1 = J Tr l = J Tr( xr v ) = J ( xr v )
r r
) ( )
dt
Solucin alternativa:
En el Ejemplo 1.46 hemos demostrado que dado un tensor de segundo orden F la relacin
F tpq = rjk Frt F jp Fkq se cumple, y si aplicamos la derivada material podemos obtener que:

Universidad de Castilla- La Mancha Draft Por: Eduardo W. V. Chaves (2012)


Ciudad Real - Espaa
1 TENSORES 113

DF D
tpq = ( rjk Frt F jp Fkq ) = rjk F&rt F jp Fkq + rjk Frt F& jp Fkq + rjk Frt F jp F&kq (1.123)
Dt Dt

Segn el enunciado del problema tenemos que l = F& F 1 F& = l F , con lo cual las
siguientes relaciones se cumplen F&rt = l rs Fst , F& jp = l js Fsp y F&kq = l ks Fsq , y la ecuacin en
(1.123) puede ser reescrita como:
DF
tpq = rjk F&rt F jp Fkq + rjk Frt F& jp Fkq + rjk Frt F jp F&kq
Dt
= rjk l rs Fst F jp Fkq + rjk Frt l js Fsp Fkq + rjk Frt F jp l ks Fsq

Multiplicamos ambos lados de la igualdad por ut v p w q , resultando:


DF
tpq ut v p w q = rjk l rs Fst F jp Fkq ut v p w q + rjk Frt l js Fsp Fkq ut v p w q + rjk Frt F jp l ks Fsq u t v p w q
Dt
= rjk ( l rs Fst ut )( F jp v p )( Fkq w q ) + rjk ( Frt ut )( l js Fsp v p )( Fkq w q ) + rjk ( Frt ut )( F jp v p )( l ks Fsq w q )
= rjk ( l rs a s )(b j )(c k ) + rjk (a r )( l js b s )(c k ) + rjk (a r )(b j )( l ks c s )

donde hemos denotado por a s = Fst ut , b j = F jp v p , c s = Fsq w q . La ecuacin anterior en


notacin tensorial queda:
DF r r r
Dt
r r r r r
[
r r r r
] [
u ( v w ) = ( l a) (b c ) + a ( l b) c + a b ( l c ) ]
r r r
[
= Tr ( l ) a (b c ) ]
donde hemos usado la propiedad de traza (ver Ejemplo 1.48). La ecuacin anterior aun puede
ser reescrita como:
DF r r r
Dt
r r r
[
u ( v w ) = Tr ( l ) a (b c ) ]
r r r
= Tr ( l ){( F u) [( F v ) ( F w ) ]}
r r r
= Tr ( l ) F u ( v w )
donde hemos aplicado la propiedad de determinante (ver Ejemplo 1.49), con lo cual
DF
concluimos que = Tr ( l ) F .
Dt

Ejemplo 1.112
r
Considrese un campo vectorial representado por su campo vector unitario b ( x ) , ver Figura
r r
1.8. Obtener un tensor proyeccin de segundo orden P tal que se cumpla que p = P u ,
r r r
donde u es un vector arbitrario y p es ortogonal al campo definido por el versor b ( x ) .

Universidad de Castilla- La Mancha Draft Por: Eduardo W. V. Chaves (2012)


Ciudad Real - Espaa
114 PROBLEMAS RESUELTOS DE MECNICA DEL MEDIO CONTINUO

r
b ( x )

Figura 1.8: Campo vectorial

Solucin:
El problema planteado se puede apreciar en la figura abajo:

r r r
p = P u u
r
a

r
b ( x )

r r r r
Luego, a travs de suma de vectores se cumple que: u = a + p . Adems el vector a puede ser
r r r r
obtenido a travs de la proyeccin de u segn la direccin b : a = a b = (u b ) b . Con eso
podemos decir que:
r r r
p=ua p i = ui ai
r r r r
= u (u b ) b = u (u b ) b = u i (u k b k )b i
r r
= 1 u (b b ) u = u u b b
k ik k k i

[ ]
= 1 (b b ) u
r
r
= ( ik b k b i )u k
= P u = Pik u k
Con lo cual concluimos que el tensor proyeccin de segundo orden viene dado por:
P = 1 b b
Este mismo resultado podra haber sido obtenido a travs del producto vectorial. Dibujando
el problema planteado en otra perspectiva para mejor visualizacin, tenemos que:
r
u b
r
b ( x )
r
u

r
b (u b )

Universidad de Castilla- La Mancha Draft Por: Eduardo W. V. Chaves (2012)


Ciudad Real - Espaa
1 TENSORES 115

r r
Teniendo en cuenta que a (b a) = [(a a)1 a a] b , ver Ejemplo 1.16, podemos decir
r r r r r r

r
[ r
] [ r r
que: b (u b ) = (b b )1 b b u = 1 b b u = p ]
Ejemplo 1.113
r r
Dado un campo vectorial v ( x ) , demostrar que se cumple la siguiente relacin:
r r 1 r r r
xr v v = xr (v 2 ) v ( xr v )
2
r
donde v es el mdulo de v .
Solucin:

Fijemos que
1 r 2
2
[ ] 1
2
r r 1
2
1
2
r r
x (v ) i = [ xr (v v )]i = (v k v k ) ,i = (v k ,i v k + v k v k ,i ) = v k v k ,i = (v xr v ) i .
r r
En un punto del campo vectorial v vamos considerar un plano normal a v y recordar que la
r
proyeccin de un tensor de segundo orden segn una direccin ( v ) resulta un vector el cual
r
no necesariamente tiene la misma direccin que ( v ), con eso vamos representar los siguientes
r r r r
vectores xr v v y v xr v :

r r
xr v v

r r r
r cv
( xr v ) r
r r r
v r r c ( xr v )
v xr v

r r r r
c = v ( xr v )

Verifiquemos que a travs de suma de vectores se cumple que:


r r r r r
xr v v + c = v xr v
r r r r r
c = v xr v xr v v
r r r r r
c = v xr v v xr v T
r r r r r r
c = v ( xr v xr v T ) = v 2( xr v ) anti
r r
Si consideramos que w es el vector axil asociado al tensor antisimtrico ( xr v ) anti se cumple
r r r r r r r r
que: ( xr v ) anti v = w v v ( xr v ) anti = v w . Adems se cumple tambin que
r r r r
rot (v ) xr v = 2 w . Luego,
r r r r r r r r
c = v 2( xr v ) anti = v 2w = v ( xr v ) (1.124)
con eso concluimos que:

Universidad de Castilla- La Mancha Draft Por: Eduardo W. V. Chaves (2012)


Ciudad Real - Espaa
116 PROBLEMAS RESUELTOS DE MECNICA DEL MEDIO CONTINUO

r r r r r
xr v v + c = v xr v
r r r r r
xr v v = v xr v c
r r 1 r r r
xr v v = xr (v 2 ) v ( xr v )
2
r r r
Es interesante observar que cuando ( xr v ) resulta ser un tensor simtrico, ( xr v ) = ( xr v ) sym ,
r r r r r r r r r r
se cumple que ( xr v ) anti = 0 , c = 0 , ( xr v ) = 0 , xr v v = v xr v y tiene la misma
r
direccin que v .
r r r r r r r
En el caso que se cumpla ( xr v ) = ( xr v ) anti tenemos que: c = v 2( xr v ) anti = 2v ( xr v ) , ver
r r r r r
expresin (1.124). Con eso se cumple tambin que v xr v = xr v v , y adems, v es
r r
perpendicular al vector ( xr v ) , ver figura abajo:

r r
( xr v ) = ( xr v ) anti r r
xr v v r r
cv
r r r
r c ( xr v )
v

r r
( xr v )

r r
v xr v

r r r r r r
c = v ( xr v ) = 2v ( xr v )

Figura 1.9
Solucin Alternativa:
r r r r r r r r r
xr v v = (( xr v ) sym + ( xr v ) anti ) v = ( xr v ) sym v + ( xr v ) anti v
r r r r r r r r
= ( xr v ) sym v + ( xr v ) anti v + (( xr v ) anti v ( xr v ) anti v )
r r r r r r
= (( xr v ) sym v ( xr v ) anti v ) + 2( xr v ) anti v
1
[
r r r r
] r
= (( xr v ) + ( xr v )T ) (( xr v ) ( xr v )T ) v + 2( xr v ) anti v
2
r r

1 r r r r r r r r
= (2( xr v )T ) v + 2( xr v ) anti v = v ( xr v ) + 2( xr v ) anti v
2
1 r 2 r r r
= x (v ) v ( xr v )
2
r r r r r r r r r
Recordar que ( xr v anti )T = ( xr v ) anti , luego 2( xr v ) anti v = v 2( xr v ) anti = v ( xr v ) .
Ejemplo 1.114
r r
Considrese un campo vectorial estacionario u( x ) . Obtener las componentes del diferencial
r r r
total du . Considerando que u( x ) representa el campo de desplazamientos y es independiente

Universidad de Castilla- La Mancha Draft Por: Eduardo W. V. Chaves (2012)


Ciudad Real - Espaa
1 TENSORES 117

de la componente x3 , hacer la representacin grfica del campo de desplazamiento en un


elemento diferencial de rea dx1 dx 2 .
Solucin: Segn la definicin de diferencial total y de gradiente se cumple que:

r r r r r r
u( x ) dx u( x + dx )

r r r r r r
x2
r
x r r du u( x + dx ) u( x )
x + dx r r r
du = xr u dx

x1
x3

Luego, las componentes vienen dadas por:

u1 u1 u1 u1 u u
du1 = dx1 + 1 dx 2 + 1 dx3
x1 x 2 x3
du1 x1 x 2 x3 dx
u i du = u 2 u 2 u 2
1

u u u
du i = dx j 2 x dx 2 du 2 = 2 dx1 + 2 dx 2 + 2 dx3
x j x 2 x3 x1 x 2 x3
du 3 u 3
1

u 3 u 3 dx3 u u u
x1 x 2 x3 du 3 = 3 dx1 + 3 dx 2 + 3 dx3
x1 x 2 x3
con
du1 = u1 ( x1 + dx1 , x 2 + dx 2 , x3 + dx3 ) u1 ( x1 , x 2 , x3 )

du 2 = u 2 ( x1 + dx1 , x 2 + dx 2 , x3 + dx3 ) u 2 ( x1 , x 2 , x3 )
du = u ( x + dx , x + dx , x + dx ) u ( x , x , x )
3 3 1 1 2 2 3 3 3 1 2 3

Para el caso plano, es decir, cuando el campo es independiente de x3 , el campo de


desplazamientos en el elemento diferencial de rea viene definido por:
u1 u1
du1 = u1 ( x1 + dx1 , x 2 + dx 2 ) u1 ( x1 , x 2 ) = x dx1 + x dx 2
1 2

du = u ( x + dx , x + dx ) u ( x , x ) = 2 dx + 2 dx u u
2 2 1 1 2 2 2 1 2
x1
1
x 2
2

o an:
u1 u1
u1 ( x1 + dx1 , x 2 + dx 2 ) = u1 ( x1 , x 2 ) + x dx1 + x dx 2
1 2

u ( x + dx , x + dx ) = u ( x , x ) + u 2 dx + u 2 dx
2 1 1 2 2 2 1 2
x1
1
x 2
2

Observemos que la expresin anterior es equivalente a la expansin en serie de Taylor


teniendo en cuenta solo hasta trminos lineales. La representacin del campo de
desplazamiento en el elemento diferencial de rea se muestra en la Figura 1.10.

Universidad de Castilla- La Mancha Draft Por: Eduardo W. V. Chaves (2012)


Ciudad Real - Espaa
118 PROBLEMAS RESUELTOS DE MECNICA DEL MEDIO CONTINUO

u 2 u 2 u
u2 + dx 2 u2 + dx1 + 2 dx 2
x 2 x1 x 2

( x1 , x 2 + dx 2 ) ( x1 + dx1 , x 2 + dx 2 )

u1 u1 u
u1 + dx 2 u1 + dx1 + 1 dx 2
x 2 x1 x 2
r
du
dx 2

u1
u1 + dx1
(u1 ) x1

( x1 , x 2 ) ( x1 + dx1 , x 2 )
x2
(u 2 ) u 2
u2 + dx1
x1
dx1
x1

144444444444444444424444444444444444443

=
644444444444444444474444444444444444448

x 2 ,u 2
u1
dx2
x2
u 2
u2 + dx2
x2 B

B B B

dx 2 dx 2
+
O A A
u 2
u2 A
dx1
O A x1
dx1 O
dx1
u1

u1
u1 + dx1
x1
x1 ,u1

Figura 1.10

Universidad de Castilla- La Mancha Draft Por: Eduardo W. V. Chaves (2012)


Ciudad Real - Espaa
1 TENSORES 119

Ejemplo 1.115
r
Dado un campo tensorial de segundo orden, T ( x ) . Demostrar que si no hay una fuente de
r r r
T ( x ) se cumple que la divergencia de T ( x ) es igual a cero, i.e. xr T = 0 . Para la
demostracin considerar el campo tensorial en un elemento diferencial de volumen
dV = dx1 dx 2 dx 3 en el sistema Cartesiano.
Solucin:
r
Primero vamos establecer el campo T ( x ) en el diferencial de volumen. Para ello, partimos de
r
la definicin del diferencial de T ( x ) que viene definido a travs del gradiente como:
r r r
dT T ( x + dx ) T ( x ) r r r r r r r r
r T ( x + dx ) T ( x ) = xr T dx T ( x + dx ) = T ( x ) + xr T dx
dT = xr T dx
En componente la expresin anterior queda:
r r r
Tij ( x + dx ) = Tij ( x ) + Tij ,k dx k
r
= Tij ( x ) + Tij ,1 dx1 + Tij , 2 dx 2 + Tij ,3 dx3
r Tij Tij Tij
= Tij ( x ) + dx1 + dx 2 + dx3
x1 x 2 x 3
r r
La representacin de las componentes del campo Tij ( x + dx ) se pueden apreciar en la Figura
r Ti1
1.11. Observar que en la cara normal a x1 + dx1 actan las componentes Ti1 ( x ) + dx1 , ya
x1
que segn nuestra convencin el primer ndice indica la direccin haca donde apunta y el
segundo ndice indica el plano normal.
r r
Una vez establecido el campo de Tij ( x + dx ) en el elemento diferencial de volumen,
r r
aplicamos el balance total de las componentes del campo Tij ( x + dx ) segn las direcciones x1 ,
x 2 , x3 .
r r
Balance total de Tij ( x + dx ) en dV segn direccin x1 es igual a cero (no hay fuente):

T T T
T11 + 11 dx1 dx 2 dx3 + T13 + 13 dx 3 dx1 dx 2 + T12 + 12 dx 2 dx1 dx3 T11 dx 2 dx3
x1 x3 x 2
T13 dx1 dx 2 T12 dx1 dx3 = 0
Simplificando la expresin anterior obtenemos que:
T11 T T
dx1 dx 2 dx3 + 13 dx3 dx1 dx 2 + 12 dx 2 dx1 dx3 = 0
x1 x3 x 2
T11 T12 T13
+ + =0
x1 x 2 x3
Anlogamente segn las direcciones x 2 y x3 vamos obtener, respectivamente, que:
T21 T22 T23 T31 T32 T33
+ + =0 y + + =0
x1 x 2 x3 x1 x 2 x3

Universidad de Castilla- La Mancha Draft Por: Eduardo W. V. Chaves (2012)


Ciudad Real - Espaa
120 PROBLEMAS RESUELTOS DE MECNICA DEL MEDIO CONTINUO

x3 Cara oculta
T11
T33 T21
T33 + dx3
Cara oculta x3
T23
T23 + dx3
x3 T31
T13
T13 + dx3
x3 T32 dx 3
T32 + dx2
x2
T12
T22 T31
T31 + dx1 T22
T22 + dx2
x1 T12 x2 x2
T12 + dx2
x2
T32
T21
T21 + dx1 dx1
T11 x1
T11 + dx1
x1

T13
x1 T23
Cara oculta

T33

dx 2

Figura 1.11: Componentes del campo tensorial en un elemento diferencial de volumen.

Luego, tenemos el siguiente conjunto de ecuaciones que hay que cumplir simultneamente:
T11 T12 T13
+ + =0
x x x T1 j , j = 0
1 2 3 T11,1 + T12, 2 + T13,3 = 0
T21 T22 T23
+ + =0 T21,1 + T22, 2 + T23,3 = 0 T2 j , j = 0 Tij , j = 0 i
x1 x 2 x 3
T31 T32 T33 T31,1 + T32, 2 + T33,3 = 0 T3 j , j = 0
+ + =0
x1 x 2 x3
Demostrando as que en la ausencia de fuente la divergencia es igual a cero:
r
Tij , j = 0 i ( xr T ) i = 0 i tensorial
xr T = 0

NOTA 1: Si tenemos un campo tensorial, el orden de la fuente (o sumidero) es de un orden


menor que el campo tensorial, e.g. si el campo tensorial es un vector, la fuente de este campo
tensorial ser un escalar.
NOTA 2: Si la divergencia de un campo tensorial es positiva indica que tenemos una fuente
del campo tensorial, caso contrario, si la divergencia es negativa tenemos un sumidero.

Universidad de Castilla- La Mancha Draft Por: Eduardo W. V. Chaves (2012)


Ciudad Real - Espaa
1 TENSORES 121

Ejemplo 1.116
Demostrar que:
[( xr T ) ur ] ar = [ xr ( T ar )] ur (1.125)
r r r r
donde T = T (x ) es un campo tensorial de segundo orden, u = u( x ) es un campo vectorial, y
r r
a es un vector arbitrario (independiente de ( x ) ).
Solucin:
r r
Observar que la operacin [( xr T ) u] a resulta un vector, que en notacin indicial queda:
{[( xr T ) ur ] ar}i = [( xr T ) ur ]ik (ar ) k [ ] [ ]
= ( xr T ) ikp u p a k = Tik , p u p a k = Tik , p u p a k (1.126)
Observar tambin que:
r r r
( T a ) i = Tik a k [ xr ( T a )]ij = ( T a ) i , j = ( Tik a k ), j
gradiente

r
[ xr ( T a )]ij = ( Tik a k ), j = Tik , j a k + Tik a k , j = Tik , j a k
{
=0
r r
donde hemos considerado que a es independiente de (x ) . Con eso, podemos concluir que:
{ }
{[ xr ( T ar )] ur}i = [ xr ( T ar )]ij u j = Tik , j a k u j = Tik , p u p a k (1.127)
Si comparamos (1.126) con (1.127) demostramos (1.125).

Ejemplo 1.117
r r
Demostrar que si el mdulo de un vector, = (t ) , es constante en el tiempo eso implica que
r
r d
es ortogonal a para todo tiempo t .
dt
Solucin:
r r r
Partimos de la definicin del mdulo de un vector: = , con lo cual:
2

r
d ( ) = d (r r ) = d (r ) r + r d (r ) = 2r d (r ) = 0
2


r d

r

dt dt dt dt dt dt
NOTA: Un caso particular de este ejemplo es el movimiento circular donde:

r
v r
x = constante r
r r dx
r r dx x
x v= dt
dt

Universidad de Castilla- La Mancha Draft Por: Eduardo W. V. Chaves (2012)


Ciudad Real - Espaa
122 PROBLEMAS RESUELTOS DE MECNICA DEL MEDIO CONTINUO

1.1.17 Teoremas con Integrales

Ejemplo 1.118
r
Comprobar el Teorema de la divergencia (Teorema de Gauss) para el campo vectorial F
cuyas componentes Cartesianas viene dadas por Fi = xi + ( x32 x 3 ) i 3 . Considerar la frontera
definida por el cilindro x12 + x 22 1 , 0 x3 1 .
Solucin:
El Teorema de la divergencia afirma que:
r r
xr F dV = F n dS

V S

donde n es la normal a la superficie y apunta hacia fuera.


S (2)
n ( 2 )
x12 + x 22 1 x3

r r
r =1 r S (1)

h =1 r n (1)
x

x2

x1 n (3) S ( 3)

Figura 1.12.
r
Clculo de xr F dV :
V
r
[
xr F = Fi ,i = xi + ( x 32 x3 ) i 3 ],i
= xi ,i + ( x32 x3 ) ,i i 3 = ii + ( x32 x3 ) ,3
= 3 + (2 x3 1) = 2 x3 + 2
Luego:
x3 =1
r
xr F dV = (2 x3 + 2) dV =
A x =(02 x 3
+ 2)dx3 dA = 3 dA = 3(r 2 ) = 3
V V 3 A

donde A viene definido por el crculo x12 + x 22 1 .


r
Clculo de F n dS
S

Universidad de Castilla- La Mancha Draft Por: Eduardo W. V. Chaves (2012)


Ciudad Real - Espaa
1 TENSORES 123

Separamos la frontera en tres superficies: S (1) , S ( 2) , S (3) , ver Figura 1.12. Luego,
r r r r
F n dS = F n F n F n
(1)
dS (1) + ( 2)
dS ( 2 ) + ( 3)
dS (3)
S S ( 1) S ( 2) S ( 3)
r
Las componentes explcitas de son: F1 = x1 + ( x32 x3 ) 13 = x1 , F2 = x 2 , F
r
F3 = x3 + ( x3 x 3 ) 33 = x3 . La representacin de F en la base Cartesiana viene dada por:
2 2
r
F = x1e 1 + x 2 e 2 + x32 e 3 . Las normales correspondientes a cada superficie vienen definidas a
continuacin:
r 1
n (1) // r n (1) = ( x1 e 1 + x 2 e 2 ) ; n ( 2 ) = e 3 ; n (3) = e 3
x12 + x 22

En la superficie S (1) se cumple que:


r 1
F n (1) dS (1) = ( x e 1 1 + x 2 e 2 + x 32 e 3 ) ( x1 e 1 + x 2 e 2 )dS (1)
S ( 1) S ( 1) x12 + x 22
x12 + x 22
dS (1) = 1dS
(1)
= = 2rh = 2
S ( 1) x12 + x 22 S (1)

donde hemos considerado el rea del cilindro ( 2rh = 2 ).


En la superficie S ( 2) se cumple que x3 = 1 :
r
F n ( x e + x 2 e 2 + 1e 3 ) (e 3 ) dS ( 2 ) = 1dS
( 2)
dS ( 2 ) = 1 1
(2)
= r 2 =
S (2) S (2) S (2)

donde hemos considerado el rea del crculo ( r 2 = ).


En la superficie S (3) se cumple que x3 = 0 :
r
F n ( x e + x 2 e 2 + 0e 3 ) (e 3 )dS (3) = 0dS
( 3)
dS (3) = 1 1
( 3)
=0
( 3) ( 3) (3)
S S S
r r r r
Con lo cual: F n dS = F n (1) dS (1) + F n ( 2 ) dS ( 2 ) + F n
( 3)
dS (3) = 3
S S (1 ) S (2) S ( 3)
r r
Luego, comprobando as el Teorema de la divergencia: xr F dV = F n dS = 3 .
V S

Ejemplo 1.119
Sea un dominio de rea delimitado por el contorno como muestra en la Figura 1.13.
Considrese tambin que m es un campo tensorial de segundo orden y un campo escalar.
Demostrar que se cumple la siguiente relacin:

[m : x ( x
r r )]d = [( xr ) m] n d [( xr m) xr xr ]d


[m ij , ij ] d = ( , i m ij )n j d [m ij , j , i ] d

Universidad de Castilla- La Mancha Draft Por: Eduardo W. V. Chaves (2012)


Ciudad Real - Espaa
124 PROBLEMAS RESUELTOS DE MECNICA DEL MEDIO CONTINUO

n
x2

x1
Figura 1.13.

Solucin: Se puede aplicar directamente la definicin de integracin por partes para la


demostracin. Pero partiremos de la definicin del teorema de la divergencia. Luego dado un
r
tensor v se cumple que:
r r
r
x v d = v n d indicial
v j , j d = v j n j d

r r
Pero si consideramos que el tensor v es el resultante de la operacin v = xr m y lo
equivalente en notacin indicial v j = , i m ij y reemplazndolo en la expresin anterior
obtenemos que:

v

j, j
d = v j n j d

[,

i m ij ]
,j


dV = , i m ij n j d

[, ij ]
m ij + , i m ij , j d = , i m ij n j d

[,

ij ]


m ij d = , i m ij n j d

[ , i ]
m ij , j d

Lo equivalente en notacin tensorial:

[m :

x ( x
r r )]d = [( xr ) m] n d [ xr ( xr m)]d
c.q.d.
NOTA: Si consideramos ahora un dominio de volumen V delimitado por una superficie S
r
con normal n y sea N un vector y T un escalar tambin se cumple que:

N T,
V
i ij
S V

dV = N i T , i n j dS N i , j T , i dV

r r r
N xr ( xr T )dV = ( xr T N ) ndS xr T xr NdV

V S V

donde hemos aplicado directamente la definicin de integracin por partes.

Universidad de Castilla- La Mancha Draft Por: Eduardo W. V. Chaves (2012)


Ciudad Real - Espaa
1 TENSORES 125

Ejemplo 1.120
r r r
Si un campo vectorial se define como: b = xr v , probar que:

b n
S
i i
d S = , i ijk v k , j dV = , i b i dV
V

V
r r
donde es una funcin nicamente de x , i.e., = ( x ) .
r r r
Solucin1: Si b = xr v , luego b i = ijk v k , j . Reemplazando en la integral de superficie anterior
resulta:

b n
S
i i
dS = ijk v k , j n i dS
S

Aplicando el teorema de la divergencia de Gauss, resulta:

b n
S
i i
dS = ijk v k , j n i dS
S

= ( ijk v k , j ), i dV
V


= ( ijk , i v k , j + ijk v k , ji ) dV
V


V
3 1424 3
= (, i ijk v k , j + ijk v k , ji ) dV = , i b i dV
1
424
V
bi 0 c.q.d.
Solucin 2:

b n
S
i i
V

dS = (b i ), i dV = (, i b i + b i , i ) dV
V

como b i = ijk v k , j b i ,i = ijk v k , ji = ijk v k ,ij = 0

b n
S
i i
dS = , i b i dV = , i ijk v k , j dV
V

V

Ejemplo 1.121
Sea un dominio de volumen V delimitado por la superficie S . a) Demostrar que:
r r
( x n + n x) dS = 2V 1
S

donde n es el versor normal exterior a la superficie S . b) Demostrar tambin que:


r r
( r
x ) x dV = ( n ) x
dS dV
V S V


V
ik , k
x j dV = ik n k x j dS ij dV
S

V

Universidad de Castilla- La Mancha Draft Por: Eduardo W. V. Chaves (2012)


Ciudad Real - Espaa
126 PROBLEMAS RESUELTOS DE MECNICA DEL MEDIO CONTINUO

r r
x ( r
x ) dV = x ( n )
dS T dV
V S V

x
V
i jk , k
dV = xi jk n k dS ji dV
S

V

donde es un tensor de segundo orden arbitrario.

S r
dS = ndS
x2 n
V dS
B

r
x

x1
x3

Solucin:
a) Teniendo en cuenta solo el primer trmino del integrando, podemos decir que:
r r r
( x n ) dS = ( x 1 n ) dS = ( x 1) n dS
S S S

Aplicando el teorema de la divergencia obtenemos que:


r r r
( x n ) dS = ( x 1) n dS =
S S V
r
x ( x 1) dV

Seguiremos el desarrollo en notacin indicial:

x n
S
i j
dS = x i jk n k dS = ( jk x i ) ,k dV
S V


= ( jk ,k xi + jk x i ,k ) dV
V

Teniendo en cuenta que jk ,k = 0 j , xi ,k = ik , concluimos que:

x n
S
i j
dS = ji dV = ji dV = jiV
V

V

r
( x n ) dS = V 1
T
= V1
S

Universidad de Castilla- La Mancha Draft Por: Eduardo W. V. Chaves (2012)


Ciudad Real - Espaa
1 TENSORES 127

r
Anlogamente, concluimos que (n x ) dS = V 1 . Con lo cual es cierto que:
S

r r
( x n + n x) dS = 2V 1
S

b) Verifiquemos que se cumple que


( x j ik ) ,k = x j ,k ik + x j ik ,k
{
= jk

x j ik ,k = ( x j ik ) ,k ij
r r
( xr ) x = xr ( x )
Con eso podemos decir que:

( r
x
r r
) x dV = xr ( x ) dV dV x
V
j ik , k
V

dV = ( x j ik ) ,k dV ij dV
V
V V V

( r
x
r r
) x dV = ( x ) n dS dV x
V
j ik , k
S

dV = x j ik n k dS ij dV
V
V S V

r
= ( n ) x dS dV

S

= ( ik n k ) x j dS ij dV
V
S V

donde hemos aplicado el teorema de la divergencia a la primera integral del lado derecho de la
igualdad.
Teniendo en cuenta que
[( xr ) xr ]T = [ xr ( xr ) ]T
r r T
x ( xr ) = [ xr ( x )] T
En indicial
xi jk ,k = ( xi jk ) ,k ji

Con eso podemos decir que:

r r
x ( r
x ) dV = [ xr ( x )]T
dV T dV x i jk , k
dV = ( xi jk ) , k dV ji dV
V V V V V V

r r
x ( r
x ) dV = ( x ) n dS T dV x i jk , k
dV = ( xi jk )n k dS ji dV
V S V V S V

r
= x ( n ) dS T dV

= xi ( jk n k ) dS ji dV
S V S V

Ejemplo 1.122
Sea un escalar que viene dado por:

Universidad de Castilla- La Mancha Draft Por: Eduardo W. V. Chaves (2012)


Ciudad Real - Espaa
128 PROBLEMAS RESUELTOS DE MECNICA DEL MEDIO CONTINUO

GM
= r
a
r r r
donde G y M son escalares y constantes, y a es el mdulo del vector a 0 . a) Obtener el
r r
gradiente de . b) Obtener el gradiente de para el caso particular cuando a = x y dibujar el
campo xr en el espacio Cartesiano.
Solucin:
GM
( xr ),i r ,i = r
a
= GM 1 ( ar ) ,i
ar 2
(1.128)
x ,i ,i
Fijemos que:
r r r 1 1
1 r r 2 r r 1 r r 2
1
( a ) ,i = ( a a ) = (a a ) ( a a ) ,i = ( a a ) ( a k a k ) ,i
2

,i 2 2
1 1
1 r r r r 1
= (a a ) 2 ( a k ,i a k + a k a k ,i ) = (a a ) 2 ( a k ,i a k ) = r ( a k , i a k )
2 a
o en notacin tensorial:
r 1 r r
xr ( a ) = r (a xr a ) (1.129)
a

Luego, la expresin (1.128) queda:


1 r 1 1
( xr ),i r
GM
,i = GM r 2 ( a ) ,i = GM r 2 r (ak ,i ak ) = r 3 (ak ,i ak )
x ,i a a
a a
(1.130)
GM r r
= r 3 (a xr a ) i
a
r
r a
Adems, teniendo en cuenta que el versor segn la direccin de a viene dado por a = r , an
a
podemos decir que:

( xr ),i = GM r r
r GM r
r
r 3 (a x a ) i = r 2 (a x a ) i (1.131)
a a
r r
b) Para el caso particular cuando a = x tenemos que:
r 1 1 1 r
( x ) ,i = r ( x k ,i x k ) = r ( ki x k ) = r ( xi ) donde r = x = x12 + x 22 + x32
x x x
o en notacin tensorial:
r 1 r r 1 r 1 r
xr ( x ) = r ( x xr x ) = r ( x 1) = r ( x ) = x
x x x

Con lo cual
GM r
( xr )i r ,i = r
x
= GM 1
xr 2
( x ) = GM ( xr )
,i r3 i (1.132)
x i ,i x

Universidad de Castilla- La Mancha Draft Por: Eduardo W. V. Chaves (2012)


Ciudad Real - Espaa
1 TENSORES 129

o en notacin tensorial:
GM GM r GM
xr = xr r = x = r 2 x (1.133)
x xr 3 x

Observar que el campo vectorial xr es radial, i.e. es normal a las superficies de las esferas
r r 2
definidas por x y disminuye con x = r 2 . La ecuacin (1.133) tambin puede ser reescrita
como:
r GM GM GM (r )
= b = = 2 r = r = r = (r )r (1.134)
r r r r r

x3

Esferas

x xr
x = 1

r
x

r x2
b
xr

x1
xr

Figura 1.14
GM
NOTA: Este ejemplo = r representa el potencial gravitacional que tiene la siguiente
x
r m3
propiedad b = xr , ver Figura 1.14, donde G = 6,67384 10 11 es la constante
kg s 2
gravitacional, M es la masa total del planeta. Verificamos las unidades:
m 3 kg kg m m N m J
[] = GM
r = = 2 = = ( Unidad de energa por unidad de masa )
x kg s 2 m s kg kg kg (energa especfica)

[br ]= [ ] = xr = mJkg = mN kgm = skgmkg = sm (Unidad(unidad


r
x 2 2
de fuerza por unidad de masa)
de aceleracin)

Universidad de Castilla- La Mancha Draft Por: Eduardo W. V. Chaves (2012)


Ciudad Real - Espaa
130 PROBLEMAS RESUELTOS DE MECNICA DEL MEDIO CONTINUO

r r r r
Es interesante comprobar tambin que xr b = xr [ xr ] = 0 , ver Ejemplo 1.106.
r
Podemos obtener b en la superficie de la Tierra a travs de
r GM
b = xr = r 2 x
x

donde la masa total de la Tierra es M 5,98 10 24 kg y el radio aproximado R 6,37 10 6 m ,


resultando
r GM GM
b = r 2 x = 2 x 9,82 x
x R
r m
su mdulo denotamos por g = b 9,82 .
s2
r
Adoptando por x el sistema que tiene su origen en el centro del cuerpo de masa M , e
r r
invocando la ley de Newton ( F = ma ), podemos obtener la fuerza que est sometido un
r
cuerpo de masa ( m ) que se encuentra bajo la influencia del campo gravitacional b = xr :
r r r GMm
F = ma = mb = r 2 x (1.135)
x
Podemos expresar la relacin anterior en un sistema genrico tal y como se indica abajo:

x 2

M x1
r
r x r
x3 F ( Mm ) F ( mM )
m

r
x (M ) r
x (m )
x2 r r r
x ( M ) + x = x ( m)
r r r
x = x (m) x (M )

x1
x3

r
Luego, para el sistema x la fuerza viene dada por:
r r r
GMm ( x ( m) x ( M ) ) Ley de gravitacin universal de
F ( mM ) = r r (1.136)
r ( m) r ( M ) 2
x ( m) x ( M ) Newton
x x

Universidad de Castilla- La Mancha Draft Por: Eduardo W. V. Chaves (2012)


Ciudad Real - Espaa
1 TENSORES 131

r
donde utilizamos la nomenclatura F (mM ) para indicar que es la fuerza en m debido a la
influencia de M . Observar tambin que en M tenemos la misma fuerza en mdulo y
r
direccin, pero de sentido contrario F (Mm ) .

Ejemplo 1.123
1 r
Considerando que = donde r = x = x12 + x 22 + x32 , se pide:
r
a) Demostrar que:

[ r r
] 2 2 2
xr xr ( x 0) 2 2 + 2 + 2 = 0 Ecuacin de Laplace
x1 x 2 x3
(1.137)

r r
[ ]
para r 0 . Utilizamos la nomenclatura xr ( x 0) para indicar que el origen no est
incluido.
b) Dada una superficie cerrada S que contiene el origen, demostrar que:

( ) n dS = 4
S
r
x (1.138)

donde n es el versor normal a la superficie.


Solucin:
Fue obtenido en el Ejemplo 1.122 que
GM GM r GM
xr = xr r = x = r 2 x (1.139)
x xr 3 x

Haciendo que GM = 1 obtenemos que:
1 1 r 1
xr = xr r = r 3 x = r 2 x (1.140)
x x x

o en notacin indicial:
1 r 1
( xr ) i = r 3 x = r 3 xi (1.141)
x x
i
Calculando la divergencia de la relacin anterior quedamos con:
x 1
( xr ) = ,ii
x i ,i
xr = r 3i = r 3 xi r 3
x x x
,i ,i
(1.142)
x i ,i 3 r
= r 3 x i r 4 ( x ) ,i
x x

r 1 r
En el Ejemplo 1.122 hemos demostrado que xr ( x ) = r ( x ) y adems teniendo en cuenta
x
que xi ,i = ii = 3 , podemos decir que:

Universidad de Castilla- La Mancha Draft Por: Eduardo W. V. Chaves (2012)


Ciudad Real - Espaa
132 PROBLEMAS RESUELTOS DE MECNICA DEL MEDIO CONTINUO

3 r
xr ( xr ) = 3
r3 x i
r 4 ( x ) ,i
x x

3 3 x
= r 3 xi r 4 ri
x x x
(1.143)
3 3 x x
= r 3 + ri 5i
x x
r 2
3 3x
= r 3 + r 5 =0
x x

c) Adoptamos una esfera arbitraria de radio r cuya rea de la superficie es 4r 2 . Luego:



( ) n dS = r
1 n dS = 1 x n dS = 1
r
x
S x
2

x
r 2
x S
r 2
x
dS
S S (1.144)
1 1
= 2 ( rea) = 2 (4r 2 ) = 4
r r
Observar que x n = 1 ya que para la esfera se cumple que x // n .
Es interesante verificar que a travs del teorema de la divergencia hay que cumplir que:

[ ]dV = ( ) n dS
V
r
x
r
x
S
r
x
V
,ii dV
= ,i ni dS
S
(1.145)

[ r r
]
Hemos demostrado anteriormente que xr xr ( x 0) = 0 , pero eso solo es vlido para
r r
todo x 0 (no est incluido el origen). Es decir, teniendo en cuenta el resultado (1.144), y
r r
para que (1.145) tenga consistencia, en x = 0 tenemos una fuente (manantial o sumidero) e
igual a ( 4 ). Con eso es muy intuitivo concluir que cualquier superficie cerrada que no
(
contenga el origen se cumple que xr n dS = 0 .
S
)

Ejemplo 1.124
a) Demostrar que:

( ) n dS = 4GM (r )
S
(1.146)

GM
donde = es el potencial gravitacional, y M (r ) es la masa total contenida en la
r
esfera de radio r , donde la superficie de contorno de la esfera denotamos por S .
b) Considerando un planeta que tiene forma de esfera de radio r = a , obtener la masa total del
planeta en funcin de la densidad de masa, donde la densidad de masa es funcin del radio, i.e.
= (r ) .
c) Obtener el potencial gravitacional para r < a y r a . En este apartado considerar la
densidad de masa constante en el planeta = 0
Solucin:
a) En el Ejemplo 1.123 hemos demostrado que:

Universidad de Castilla- La Mancha Draft Por: Eduardo W. V. Chaves (2012)


Ciudad Real - Espaa
1 TENSORES 133

( ) n dS = r n dS = 4
1
(1.147)
S S

Multiplicamos ambos lados de la igualdad por GM (r ) obtenemos que:


1
GM (r ) n dS = 4GM (r )
r
S

GM (r )

n dS = 4GM (r ) (1.148)
S r
[] n dS = 4GM (r )
S

b)

Planeta esfrico

r=a

(r )
r

La masa total viene definida por:


M = (r )dV
V
(1.149)

Notar que V = 43 r 3 dV = 43 3r 2 dr = 4r 2 dr . Con lo cual:


r =a


M = (r )dV = (r )4r
2
dr (1.150)
V r =0

c) Recordar que en el Ejemplo 1.122 (ver ecuacin (1.134)) hemos obtenido que
r GM GM GM (r )
= b = = 2 r = r = r = (r )r (1.151)
r r r r r
Utilizando la ecuacin (1.148) podemos decir que:

Universidad de Castilla- La Mancha Draft Por: Eduardo W. V. Chaves (2012)


Ciudad Real - Espaa
134 PROBLEMAS RESUELTOS DE MECNICA DEL MEDIO CONTINUO

[] n dS = 4GM (r )
S
r
b n dS = (r ) r n dS = (r ) dS = (r )(4r 2 ) = 4GM (r )

123
S S =1 S (1.152)
(r )r = GM (r )
2

GM (r )
(r ) =
r2
donde M (r ) = V 0 = 43 r 3 0 . Luego:
GM (r ) 4G 0
(r ) = = r
r2 3
d(r ) 4G 0
= r (1.153)
dr 3
4G 0
d ( r ) = rdr
3
Integrando la ecuacin anterior obtenemos que:
4G 0
d = 3
rdr =

4G 0 r 2
(r ) = +C (1.154)
3 2
2G 0 2
(1) (r ) = r +C
3
donde hemos denotado que (r ) = (1) (r ) para r < a . Para valores de r a el potencial
gravitacional viene dado por
GM 4Ga 3 0
= = = (2) ; ra (1.155)
r 3r
donde M es la masa total del planeta cuyo valor es M = V 0 = 43 a 3 0 . Notar que el potencial
tiene que ser continuo en r = a , (ver Parker (2003)), con lo cual:
(1) (r = a) = ( 2) (r = a)
2G 0 2 4Ga 3 0
a +C =
3 3a (1.156)
2Ga 0 2Ga 3 0 4 3 2GM 3 3MG
3
C = = = =
a a 34 a 4 2a
Con lo cual la ecuacin (1.154) queda
2G 0 2 2G 0 2 3MG MG 2 3MG MG r 2 3
(1) (r ) = r +C = r = 3r = 2 2 (1.157)
3 3 2a 2a 2a 2a 2a 2
resumimos:
MG r 2 3
(r ) = para r<a
2
2a 2a 2
2
(1.158)
MG
(r ) = r
para ra

Universidad de Castilla- La Mancha Draft Por: Eduardo W. V. Chaves (2012)


Ciudad Real - Espaa
1 TENSORES 135

(r )

superficie del planeta

MG
a
punto de inflexin

3MG
2a

Figura 1.15: Potencial gravitacional vs. radio.

Figura 1.16: Potencial gravitacional (Ref. Wikipedia: Gravitational potential).

Ejemplo 1.125
a) Demostrar que la rbita de un planeta se realiza en un plano. b) Demostrar las Leyes de
Kepler, (Johannes Kepler (1571-1630):
b.1) Primera Ley de Kepler (1609): La rbita de cada planeta es una elipse, teniendo el Sol en unos
de los focos de la elipse;
b.2) Segunda Ley de Kepler (1609): El vector posicin Sol=>Planeta en movimiento describe una
rea a una tasa constante;
b.3) Tercera Ley de Kepler (1618): Si T (periodo orbital) representa el tiempo necesario para que
un planeta realice una vez su rbita elptica, cuyo eje mayor es 2a , se cumple que T 2 = a 3 ,
donde es una constante.

Universidad de Castilla- La Mancha Draft Por: Eduardo W. V. Chaves (2012)


Ciudad Real - Espaa
136 PROBLEMAS RESUELTOS DE MECNICA DEL MEDIO CONTINUO

Recordatorio: Expresiones relacionados con la elipse

x2

r
x
b
f2

f1 x1
b

a a

r p
Ecuacin de la elipse: x = r =
1 + e cos

a 2 b2 p2
Excentricidad: e = ; 0 < e < 1 , donde se cumple que a 2 = .
a2 (1 e 2 ) 2
rea de la elipse: A = ab
Solucin:

M - masa del Sol


m - masa del planeta
r
x
x = r
x
x2 r r r
r dx r d 2 x dv
v= , a= 2 =
x3 r r r dt dt dt
r c = xv
dx r r
=v a // x
dt
x
r r r
x F // a
t=0
Sol r
h x1

Figura 1.17: rbita del planeta.

a) Para demostrar que la rbita se realiza en un plano tenemos que demostrar que la normal
r r
formada por el plano x y v no cambia con el tiempo, es decir, si el vector c no vara con el
tiempo, i.e. es una constante.
En el Ejemplo 1.122 en la ecuacin (1.135) hemos demostrado que:

Universidad de Castilla- La Mancha Draft Por: Eduardo W. V. Chaves (2012)


Ciudad Real - Espaa
1 TENSORES 137

r r r GMm r GM
F = ma = mb = r 2 x ; a = r 2 x (1.159)
x x
r r r
Obtenemos la tasa del vector c = x v :
r
dc d r r d r r r d r r r r r r
= ( x v ) = ( x ) v + x (v ) = v12
3v + 13
x2 a =0
dt dt dt dt r
=0
r
=0
r r r
Con lo cual hemos demostrado que el vector c = x v no vara con el tiempo, implicando
que la rbita se realiza en un plano.
b.1) Primera Ley de Kepler
Ya que la rbita del planeta se realiza en un plano, adoptamos como dicho plano el x1 x 2 ,
r
luego el vector c tiene misma direccin que x3 , ver Figura 1.17.
r
Expresamos c en funcin de x .
r r
r dx d r d( x ) r dx
v= = ( x x) =
x + x
dt dt dt dt
y
r
r r r r d( x ) r dx
c = x v = ( x x ) x + x =
dt dt

r
r d( x ) r 2 dx
= x 1 3x + x x
x 2
dt r
=0
dt
r 2 dx
= x x
dt
r GM r r
Teniendo en cuenta que a = r 2
x , calculamos el vector a c que tiene misma direccin
x
r r r r
que v , i.e. (a c ) // v :

r r GM r 2 dx dx dx dx
a c = r 2 x x x = GM x x = GM ( x ) x ( x x )
x dt dt dt dt

dx
= GM
dt
r r r
( )
r r r r r r
donde hemos utilizando la propiedad a b c = (a c )b (a b)c , ver Ejemplo 1.16. Notar
dx dx
tambin que se cumple que x = 0 ya que x siempre se cumple, y x x = x = 1 .
2

dt dt
Teniendo en cuenta que GM es una constante, tambin se cumple que:
r r dx d (GM x )
a c = GM =
dt dt
r
Ya que el vector c no vara con el tiempo, tambin se cumple que:
r r r
r r dv r d ( v c )
ac = c =
dt dt
Luego:

Universidad de Castilla- La Mancha Draft Por: Eduardo W. V. Chaves (2012)


Ciudad Real - Espaa
138 PROBLEMAS RESUELTOS DE MECNICA DEL MEDIO CONTINUO

r r
d (v c ) d (GM x )
=
dt dt
Integrando en el tiempo la ecuacin anterior obtenemos que:
r r r
v c = GM x + h
r r
donde h es el vector constante de integracin y no depende del tiempo. Notar que h se
r r
encuentra en el plano x1 x 2 , ya que (v c ) y x tambin se encuentran en el plano x1 x 2 ,
ver Figura 1.17. Calculamos entonces:
r r
x h = x h cos = h cos
r
donde hemos denotado por h = h . Con lo cual:
r r r r r r r r r
= c c = ( x v ) c = (v c ) x
2
c2 = c
( r
) r r r r r r
= GM x + h ( x x ) = x GM x x + x h x = x GM + x h cos
r
= x (GM + h cos )
= r (GM + h cos )
r
donde hemos considerado que r = x . Luego, obtenemos la siguiente ecuacin de la elipse:
c2
c2 GM p
r= = =
(GM + h cos ) (GM + h cos ) 1 + e cos
GM
donde hemos considerado que:
c2 h
p= y e= (1.160)
GM GM
b.2) Segunda Ley de Kepler

r 1 r r 1 r r
A = x S
S 0
dA = x ds
2 2
x2
S

A
r
x

x1

r
La tasa de dA queda:

Universidad de Castilla- La Mancha Draft Por: Eduardo W. V. Chaves (2012)


Ciudad Real - Espaa
1 TENSORES 139

r r r r r
D (dA) 1 D( x ds ) 1 D( x ) r 1 r D ( ds )
= = ds + x
Dt 2 Dt 2 Dt 2 Dt
r
1 D( x ) r 1 r r
= ds + xv
2 1Dt
4243 2
r
=0

1r
= c (constante)
2
y su mdulo:
r
D (dA) D(dA) 1 r 1
= = c = c
Dt Dt 2 2

NOTA: Como consecuencia de la segunda ley tenemos que si las reas de dos sectores son
iguales el tiempo necesario para recoger sus trayectos sern iguales. Luego, segn la Figura
1.18 si las reas de los sectores OCD y EFO son iguales los tiempos recogido de C D y
E F tambin sern iguales. Como consecuencia, el planeta cuando est ms cerca del Sol
tendr mayor velocidad que cuando est ms alejado.

sector EFO sector OCD


E
D
O
A A

C
F

Figura 1.18: rbita del planeta.

b.3) Tercera Ley de Kepler


Si T es el tiempo total para una rbita completa (periodo orbital), obtenemos que:
T T
D (dA) 1 1
A=
0
Dt
dt =
0
2
c dt = cT
2

1
Teniendo en cuenta el rea de la elipse: A = ab , concluimos que cT = ab , con lo cual:
2
2ab 4 2 a 2 b 2
T= T2 = (1.161)
c c2
A travs de las relaciones de la elipse se cumple que:
a 2 b2
e= b2 = a 2 a 2e2 b 2 = a 2 (1 e 2 )
a2

Universidad de Castilla- La Mancha Draft Por: Eduardo W. V. Chaves (2012)


Ciudad Real - Espaa
140 PROBLEMAS RESUELTOS DE MECNICA DEL MEDIO CONTINUO

p2 p
y teniendo en cuenta que a 2 = 2 2
a= 2
(1 e 2 )a = p en la relacin anterior,
(1 e ) (1 e )
podemos decir que:
b2
b 2 = a 2 (1 e 2 ) b 2 = ap p=
a
Con lo cual la ecuacin (1.161) puede ser reescrita como:
4 2 a 2 b 2 4 2 a 2 ab 2 4 2 a 3 p 4 2 3
T2 = = = = a = a3 (1.162)
c2 c 2a c2 GM
p 1
donde hemos tenido en cuenta que 2
= , ver ecuacin (1.160).
c GM

Universidad de Castilla- La Mancha Draft Por: Eduardo W. V. Chaves (2012)


Ciudad Real - Espaa
1 TENSORES 141

1.2 Ejercicios Propuestos

Problema 1.1
Obtener el resultado de las siguientes expresiones, si dichas expresiones SON CORRECTAS:
ii kl
ij ij
ij ik ij

Problema 1.2
Demostrar que:
a) ii = 3 ;
b) ij ij = 3 ;
c) ijk jki = 6 ;
d) ijk A j Ak = 0 ;
e) ij jk = ik ;
f) ij ijk = 0 .

Problema 1.3
Utilizando las propiedades del operador de permutacin ( ijk ) y de la delta de Kronecker ( ij ),
obtener el resultado de las siguientes expresiones:
a) 3 jk a j a k
b) ijk kj
c) ijk a 2Tkj
d) 1 jk 3 j v k

Problema 1.4
Utilizando notacin indicial probar que:
r r r r r r r r r r r r
(s t ) (u v ) = (s u)(t v ) (s v )(t u) (1.163)

Universidad de Castilla- La Mancha Draft Por: Eduardo W. V. Chaves (2012)


Ciudad Real - Espaa
142 PROBLEMAS RESUELTOS DE MECNICA DEL MEDIO CONTINUO

Problema 1.5
Dadas las componentes del tensor A :
1 1 0
A ij = 1 2 2 (1.164)
0 2 3

Obtener los siguientes valores:


a) A ii ;
b) A ij A ij ;
c) A ij A jk para i = 1 , k = 1 y para i = 1 , k = 2 .

Problema 1.6
Dadas las componentes del tensor R :
12 9 4
25
25 5
3 4
R ij = 0 (1.165)
5 5
16 12 3
25 25 5

El tensor R es un tensor ortogonal propio? Demostrar.

Problema 1.7
Encontrar la matriz de rotacin A que describe primero una rotacin sobre el eje x1 de 90 ,
y a continuacin efecta una rotacin de 45 sobre el eje rotado x3 .

Problema 1.8
1 1
Dadas dos bases cartesianas (e i ) y (e *i ) , donde e 1* = (2e 1 + 2e 2 + e 3 ) y e *2 = (e1 e 2 ) .
3 2
Se pide:
a) Expresar e*3 en funcin de e i ;
b) Expresar e i en funcin de e *i ;
r r
c) Si v = 6e 1 6e 2 + 12e 3 encontrar v i* .

Problema 1.9
La siguiente tabla muestra los ngulos entre los ejes originales xi y los ejes transformados xi* :

Universidad de Castilla- La Mancha Draft Por: Eduardo W. V. Chaves (2012)


Ciudad Real - Espaa
1 TENSORES 143

x1 x2 x3

x1* 135 60 120

x 2* 90 45 45

x3* 45 60 120

a) Encontrar la matriz de transformacin A ;


b) Si B es un tensor de segundo orden dado por sus componentes en la base xi :

3 4 2
B ij = 4 0 1 (1.166)
2 1 3

Encontrar las componentes de B *ij con respecto a los ejes rotados xi* .

Problema 1.10
Utilizando notacin indicial probar que:
r
a) = 0 ;
r
b) v = 0 .

Problema 1.11
Si ( x1 , x 2 , x3 ) = a ij x i x j , con aij constante. Demostrar que:

, i = (aij + a ji ) x j (1.167)
, ij = aij + a ji (1.168)

Problema 1.12
Demostrar que:
2 ( ) = 2 + 2() ( ) + 2 (1.169)

Problema 1.13

Probar que si y D son tensores de segundo orden la siguiente relacin es vlida:


: D = Tr ( D T ) (1.170)

Problema 1.14

Universidad de Castilla- La Mancha Draft Por: Eduardo W. V. Chaves (2012)


Ciudad Real - Espaa
144 PROBLEMAS RESUELTOS DE MECNICA DEL MEDIO CONTINUO

Si T y D son tensores de segundo orden La relacin


T :D = D : T (1.171)
es vlida siempre?
En caso positivo, Cundo la relacin anterior no es vlida?
En caso negativo, Cundo la relacin anterior es vlida?

Problema 1.15
Si T , D y S son tensores de segundo orden y n un vector, se puede afirmar que las
relaciones
1) T : D = D : T
2) T n = n T
3) Tr ( T D S) = Tij D jk S kl
son siempre vlidas?. Razona la respuesta.

Problema 1.16
a) Demostrar que
r r
( r r
det 1 + a b = 1 + a b )
r r
donde 1 es el tensor identidad de segundo orden, y a , b son vectores.
b) Verificar si se cumple la relacin (demostrar):

[(ar br ) (cr dr ): 1] = (br cr )(ar dr )


Problema 1.17
Demostrar que tras una transformacin ortogonal aplicadas a vectores, estos preservan su
mdulo y preserva los ngulos entre vectores.

Problema 1.18
Considrense los siguientes tensores:
1
K ijkl = ij kl ; J ijkl = I ijkl K ijkl
3
donde ij son las componentes del tensor identidad de segundo orden 1 , y

I ijkl =
1
2
( )
ik jl + il jk es la parte simtrica del tensor identidad de cuarto orden.

Demuestre que el resultado del doble producto escalar de cualquier tensor de segundo orden
simtrico A con:
a) el tensor J resulta ser la parte desviadora de A ;
b) el tensor K resulta ser la parte esfrica de A .

Universidad de Castilla- La Mancha Draft Por: Eduardo W. V. Chaves (2012)


Ciudad Real - Espaa
1 TENSORES 145

Problema 1.19
Demostrar que tras una transformacin ortogonal aplicadas a vectores, estos preservan sus
mdulos y preservan los ngulos entre vectores

Problema 1.20
Obtener las componentes de T , cuyo tensor se obtiene por la siguiente operacin:
T = B T BT
Siendo Tij y B ij las componentes de los tensores T y B , respectivamente.
NOTA: Utilizar notacin simblica y notacin indicial.

Problema 1.21
Demostrar que
(
r r
) ( r r
det Q + a b = det (Q) 1 + Q 1 a b )
r r
donde Q es un tensor de segundo orden, y a , b son vectores.

Problema 1.22
Dado un tensor de segundo orden T demostrar que las siguientes expresiones son invariantes:
Tr ( T )
Tr ( T T )
det ( T )

Problema 1.23
Dadas las componentes de un tensor de segundo orden:
1 1 0
A ij = 1 1 0
0 0 2

y sus valores principales:


2 0 0
A ij = 0 2 0
0 0 0

Obtener los invariantes del tensor desviador del tensor A .

Problema 1.24
Cuantas componentes independientes tiene el tensor cuarto orden A si:
a) A es no simtrico;

Universidad de Castilla- La Mancha Draft Por: Eduardo W. V. Chaves (2012)


Ciudad Real - Espaa
146 PROBLEMAS RESUELTOS DE MECNICA DEL MEDIO CONTINUO

b) A presenta slo simetra menor;


c) A presenta simetra mayor y menor.

Problema 1.25
Considrese un tensor de cuarto orden C , teniendo su representacin tensorial e indicial
como se sigue:
C = 1 1 + 2 I
(
C ijkl = ij kl + ik jl + il jk )
y su inversa viene dada por:
1
C 1 = 1 1 + I
2 (3 + 2 ) 2

1
C ijkl =
2 (3 + 2 )
ij kl +
1
4
(
ik jl + il jk )
Demostrar que C : C 1 = I sym I , donde I es el tensor identidad de cuarto orden simtrico.

Problema 1.26
Considrese un tensor de cuarto orden B , cuyas componentes vienen dadas por la siguiente
expresin:
B ijkl = ij kl + ik jl + il jk

donde , y son constantes. Demostrar que el tensor B es istropo.

Problema 1.27
El campo del tensor de tensiones de Cauchy de un medio continuo viene representado por:
3 x1 5 x 22 0
r 2
ij ( x ) = 5 x 2 0 2 x3
0 2 x3 0

Obtener las fuerzas msicas (por unidad de volumen) para que el medio continuo est en
equilibrio.
10 x 2 + 3
Respuesta: b i = 2
0

Problema 1.28

Universidad de Castilla- La Mancha Draft Por: Eduardo W. V. Chaves (2012)


Ciudad Real - Espaa
2 Cinemtica del Continuo
2.1 Ejercicios Resueltos

2.1.1 Descripcin del Movimiento, Derivada Material,


Velocidad, Aceleracin

Ejemplo 2.1
Un continuo viene definido por un cuadrado de lado b , y est sometido a movimiento de
slido rgido definido por una rotacin antihoraria por un ngulo de 30 . Encontrar las
ecuaciones de movimiento. Obtener tambin la nueva posicin de la partcula D .
r r
Nota: Considerar los sistemas x y X superpuestos.

X 2 , x2
x2
C

D C x1
D
30
B
b
30 B
A = A b X 1 , x1

r r r r
Solucin: Aplicamos las ecuaciones de movimiento de un slido rgido x = c + Q X = Q X ,
r r
con c = 0 . Las componentes de Q son las mismas que las componentes de la matriz de
r r
transformacin del sistema x para el sistema x , i.e.:
cos sin 0
Q ij = sin cos 0
0 0 1
Luego, las partculas vienen gobernadas por las ecuaciones de movimiento:
148 PROBLEMAS RESUELTOS DE MECNICA DEL MEDIO CONTINUO

x1 cos 30 sin 30 0 X 1

x 2 = sin 30 cos 30 0 X 2
x 0 0 1 X 3
3
La partcula que inicialmente estaba en D ( X 1 = 0 , X 2 = b , X 3 = 0 ) mueve para la siguiente
posicin:
x1D cos 30 sin 30 0 0 b sin 30
D
x 2 = sin 30 cos 30 0 b = b cos 30
x D 0 0 1 0 0
3

Ejemplo 2.2
Un movimiento del medio continuo en la descripcin material viene dado por:
x1 = exp t X 1 exp t X 2
t t
x 2 = exp X 1 + exp X 2 (2.1)
x = X
3 3

para t > 0 . Encontrar las componentes de la velocidad y aceleracin en coordenadas


espaciales y materiales.
Solucin:
Velocidad:

r r V1 = exp t X 1 + exp t X 2
r r Dx ( X , t ) componentes
V ( X , t) = V2 = exp t X 1 exp t X 2 (2.2)
Dt V = 0
3

Aceleracin:
A1 = exp t X 1 exp t X 2
t t
A2 = exp X 1 + exp X 2 (2.3)
A = 0
3

Para encontrar las componentes de la velocidad y la aceleracin reemplazamos las


ecuaciones del movimiento:
Velocidad (descripcin espacial)
v1 = x2

v2 = x1 (2.4)
v = 0
3
Aceleracin (descripcin espacial)
a1 = x1 = v 2

a 2 = x 2 = v1 (2.5)
a = 0
3

Universidad de Castilla- La Mancha Draft Por: Eduardo W. V. Chaves (2012)


Ciudad Real - Espaa
2 CINEMTICA DEL CONTINUO 149

Ejemplo 2.3
El campo de velocidad de un fluido viene dado por:
r
v = x1e 1 + x2 e 2 + x3e 3 (2.6)
y el campo de temperatura es:
r
T ( x , t ) = 3 x 2 + x3 t (2.7)
Encontrar la tasa de cambio en el tiempo de la temperatura.
Solucin:
r
La tasa de cambio de una propiedad viene dada por la derivada material T ( x , t ) :
r r
DT T ( x , t ) T ( x , t ) T T T T
= + vj = + v1 + v2 + v3 (2.8)
Dt t x j t x1 x 2 x3
DT
= x3 + (0 x1 + 3 x2 + tx3 )
Dt (2.9)
= x3 + (3x2 + tx3 )

Ejemplo 2.4
Dado el movimiento:
xi = X i + 0,2tX 2 1i (2.10)
y el campo de temperatura (estacionario):
r
T ( x ) = 2 x1 + x 22 (2.11)
a) Encontrar el campo de temperatura en la descripcin material;
b) Encontrar la tasa de cambio de la temperatura para una partcula que en la configuracin
de referencia ocupaba la posicin (0,1,0) .
Solucin:
Segn las ecuaciones del movimiento tenemos que:
x1 = X 1 + 0,2tX 2 11 = X 1 + 0,2tX 2
x 2 = X 2 + 0,2tX 2 12 = X 2
x3 = X 3 + 0,2tX 2 13 = X 3
Luego:
r r r
[ r 2
]
T ( x ( X , t )) = 2 x1 ( X , t ) + x 2 ( X , t )
= 2( X 1 + 0,2tX 2 ) + ( X 2 )
2

r
= 2 X 1 + ( X 2 + 0,4t )X 2 = T ( X , t )
b) La derivada material de la temperatura viene dada por:
r
DT ( X , t ) & r
T ( X , t ) = 0,4 X 2
Dt

Universidad Castilla- La Mancha Draft Por: Eduardo W. V. Chaves (2012)


Ciudad Real - Espaa
150 PROBLEMAS RESUELTOS DE MECNICA DEL MEDIO CONTINUO

Para la partcula ( X 1 = 0; X 2 = 1; X 3 = 0) tenemos que:


T& (( X 1 = 0; X 2 = 1; X 3 = 0), t ) = 0,4 X 2 = 0,4

Ejemplo 2.5
r r
Determinar el campo de velocidad V ( X , t ) en la descripcin material y el campo de
r r
aceleracin A( X , t ) de una partcula en el tiempo t en funcin de la tasa de cambio de los
r r
desplazamientos U ( X , t )
Solucin:
r r D r r r&
V ( X , t) = U ( X , t) = U (2.12)
Dt
r r D r r r&
A( X , t ) = V ( X , t) = V =
Dt
(2.13)
D2 r r &r&
= 2 U ( X , t) = U
Dt
r& &r&
A =V = U (2.14)

Ejemplo 2.6
Considrense las siguientes ecuaciones del movimiento en la descripcin Lagrangiana:
r
x1 ( X , t ) = X 2 t 2 + X 1 x1 1 t
2
0 X 1
r Forma Matricial
x 2 ( X , t ) = X 3 t + X 2 x 2 = 0 1 t X 2 (2.15)
r x 0 0
x 3 ( X , t ) = X 3 3 1 X 3

Es este un movimiento posible? Si as es, encontrar los campos de desplazamiento,


velocidad y aceleracin en la descripcin Lagrangiana y Euleriana. Considrese un partcula
P , que en el tiempo t = 0 ocupaba la posicin X 1 = 2, X 2 = 1, X 3 = 3 , encontrar la
velocidad de P en los tiempos t = 1s y t = 2 s .
Solucin:
El movimiento es posible si J 0 . Verificamos que el movimiento es posible:
x1 x1 x1
X 1 X 2 X 3 1 t 2 0
xi x 2 x 2 x 2
J= = =0 1 t =1 0
X j X 1 X 2 X 3
x3 x3 x3 0 0 1
X 1 X 2 X 3
r r r
El campo vectorial de desplazamiento viene dado por la definicin u = x X . Utilizando
las ecuaciones del movimiento (2.15) obtenemos que:

Universidad de Castilla- La Mancha Draft Por: Eduardo W. V. Chaves (2012)


Ciudad Real - Espaa
2 CINEMTICA DEL CONTINUO 151

u1

(Xrr , t ) = x ( Xrr, t ) X
1 1 = X 2t 2
u 2 (Xr , t ) = x ( Xr , t ) X
2 2 = X 3t (2.16)

u 3 (X , t ) = x ( X , t ) X
3 3 =0

que son las componentes del desplazamiento en la descripcin Lagrangiana. La velocidad y


la aceleracin vienen dadas por:

V1 =
r
( )
du 1 X , t
=
d
(
X 2t 2 = 2 X 2t ) dV1
dt dt A1 = dt = 2 X 2


V 2 =
r
( )
du 2 X , t d
= ( X 3t ) = X 3 ;

A2 =
dV 2
=0 (2.17)
dt dt dt
( )
r
du 3 X , t d dV 3
V3 = = (X 2t ) = 0 A3 = dt = 0
dt dt

La forma inversa de (2.83) nos proporcionan las ecuaciones del movimiento en la


descripcin Euleriana:
r
X 1 1 t 2 t 3 x1 X 1 ( x , t ) = x1 t 2 x 2 + t 3 x 3
r
X 2 = 0 1 t x 2 X 2 ( x , t ) = x 2 tx 3 (2.18)
X 0 r
3 0 1 x 3 X 3 ( x , t ) = x 3

Luego, los campos de desplazamiento, velocidad y aceleracin en la descripcin Euleriana


se obtienen al reemplazar las ecuaciones (2.18) en las expresiones (2.16) y (2.17), i.e.:

(Xr ( xr , t ), t ) = X ( xr , t )t = ( x tx )t = u ( xr , t )
r
u1 2
2
2 3
2
1

u 2 (Xr ( xr , t ), t ) = X ( xr , t )t = x t = u ( xr , t )
3 3 2 (2.19)

u 3 (X ( xr , t ), t ) = u ( xr , t ) = 0
3

(Xr ( xr , t ), t ) = 2 X ( xr , t )t = 2( x tx )t = v ( xr , t )
r
V1 2 2 3 1

V 2 (Xr ( xr , t ), t ) = X ( xr , t ) = x = v ( xr , t )
3 3 2 (2.20)

V3 (X ( xr , t ), t ) = v ( xr , t ) = 0
3

(Xr ( xr , t ), t ) = 2 X ( xr , t ) = 2( x
r r
A1 2 2 tx 3 ) = a1 ( x , t )

A2 (Xr ( xr , t ), t ) = a ( xr , t ) = 0
2 (2.21)

A3 (X ( xr , t ), t ) = a ( xr , t ) = 0
3

Teniendo en cuenta la descripcin Lagrangiana de la velocidad dada por (2.17), la velocidad


de la partcula P ( X 1 = 2, X 2 = 1, X 3 = 3 ) en el tiempo t = 1s viene dada por:
r r r
V1 ( X , t ) = 2 X 2 t = 2 m / s ; V2 ( X , t ) = X 3 = 3m / s ; V3 ( X , t ) = 0
Observemos que en el tiempo t = 1s la partcula P ocupa una nueva posicin definida por:
x1 = X 2 t 2 + X 1 = 3 ; x 2 = X 3t + X 2 = 4 ; x3 = X 3 = 3
Luego, la velocidad de la partcula P tambin puede ser obtenida por (2.20), i.e.:

Universidad Castilla- La Mancha Draft Por: Eduardo W. V. Chaves (2012)


Ciudad Real - Espaa
152 PROBLEMAS RESUELTOS DE MECNICA DEL MEDIO CONTINUO

r
v1 ( x , t ) = 2( x 2 tx 3 )t = 2( 4 1 3) 1 = 2m / s
r
v 2 ( x , t ) = x 3 = 3m / s
v ( xr , t ) = 0
3
Observemos que, la velocidad de la partcula es la misma sea utilizando la descripcin
Lagrangiana o la Euleriana, ya que la velocidad es una propiedad intrnseca de la partcula.
La velocidad de la partcula P en el tiempo t = 2 s viene dada por:
V1 (Xrr , t ) = 2 X t = 2 2 1 = 4m / s
2

V 2 (Xr , t ) = X = 3m / s
3

V3 (X , t ) = 0
En el tiempo t = 2 s la nueva posicin de la partcula P queda definida por:
r
x1 ( X , t ) = X 2 t 2 + X 1 = 6
r
x2 ( X , t ) = X 3t + X 2 = 7
r
x 3 ( X , t ) = X 3 = 3
Como podemos verificar en la figura abajo, la descripcin Lagrangiana del movimiento
r r
x ( X , t ) describe la trayectoria de la partcula P .

Trayectoria de la partcula P
r
viP ( x , t = 1s) = [2;3;0]
r
Vi P ( X P , t = 1s) = [2;3;0]
t = 1s
r Partcula P
t0 Vi ( X P , t = 2s ) = [4;3;0]
P

X iP = [2;1;3]
P
xiP = [3;4;3]

Partcula P

t = 2s
xiP = [6;7;3]
r
viP ( x , t = 2s ) = [4;3;0]

NOTA: Notar que la velocidad espacial no se puede obtener a travs de


r r
DX ( x , t ) r r r
= 0 v ( x , t ) . Podemos verificar este hecho con el ejemplo propuesto:
Dt
r r r r r
DX i ( x , t ) X i ( x , t ) X i ( x , t ) r X i ( x , t ) r X i ( x , t ) r
= + v1 ( x , t ) + v 2 ( x, t ) + v3 ( x , t )
Dt t x1 x 2 x 3
Luego:

Universidad de Castilla- La Mancha Draft Por: Eduardo W. V. Chaves (2012)


Ciudad Real - Espaa
2 CINEMTICA DEL CONTINUO 153

r r r r r
DX 1 ( x , t ) X 1 ( x , t ) X 1 ( x , t ) r X 1 ( x , t ) r X 1 ( x , t ) r
= + v1 ( x , t ) + v 2 ( x, t ) + v3 ( x , t )
Dt t x1 x 2 x 3
( ) [
= 2tx 2 + 3t 2 x3 + 1 2( x 2 tx3 )t t 2 x3 + t 3 0 = 0 ]
r r r r r
DX 2 ( x , t ) X 2 ( x, t ) X 2 ( x , t ) r X 2 ( x , t ) r X 2 ( x, t ) r
= + v1 ( x , t ) + v 2 ( x, t ) + v 3 ( x, t )
Dt t x1 x 2 x3
= ( x3 ) + [0 2( x 2 tx3 )t + 1 x3 t 0] = 0
r r r r r
DX 3 ( x , t ) X 3 ( x , t ) X 3 ( x, t ) r X 3 ( x , t ) r X 3 ( x , t ) r
= + v1 ( x , t ) + v 2 ( x, t ) + v3 ( x, t )
Dt t x1 x 2 x3
= (0 ) + [0 2( x 2 tx3 )t + 0 x3 + 1 0] = 0
r r r
Adems, teniendo en cuenta que u = x X , se cumple que:
r r r r
r r
v ( X , t) =
Dx ( X , t ) D r r
Dt
=
Dt
( r r
u( X , t ) X ( x , t ) = )
Du( X , t ) r& r
Dt
u( X , t )

Adems, tambin se cumple que:


r r r r r r
r r r& r Du( x , t ) u( x , t ) u( x , t ) r r
v ( x , t ) = u( x , t ) = + r v ( x, t )
Dt t x

Ejemplo 2.7
El campo de velocidad de un medio continuo, expresado en forma Euleriana es el
siguiente:
x1 2 x2 3 x3
v1 = ; v2 = ; v3 = (2.22)
1+ t 1+ t 1+ t
Se pide:
r
a) Determinar la relacin entre las coordenadas espaciales y materiales xi = xi ( X , t ) ;
b) Obtener las componentes de la aceleracin cuando se utiliza la descripcin espacial
del movimiento.
c) Obtener las componentes de la aceleracin cuando se utiliza la descripcin
Lagrangiana del movimiento.
Solucin:
dxi
a) Considerando que vi =
dt
dx1 x dx dt
v1 = = 1 1 = (2.23)
dt 1 + t x1 1 + t
1 1
x 1
dx1 = 1 + t dt Lnx 1 = Ln(1 + t ) + Ln(C1 )
(2.24)
x1 = C1 (1 + t )

La condicin inicial t = 0 x1 = X 1 implica que C1 = X 1

x1 = X 1 (1 + t ) (2.25)

Universidad Castilla- La Mancha Draft Por: Eduardo W. V. Chaves (2012)


Ciudad Real - Espaa
154 PROBLEMAS RESUELTOS DE MECNICA DEL MEDIO CONTINUO

dx 2 2 x 2 dx 2dt
v2 = = 2 = (2.26)
dt 1 + t x2 1 + t
1 2
x 2
dx 2 = 1 + t dt Lnx 2 = 2Ln(1 + t ) + LnC 2
(2.27)
2
x2 = C 2 (1 + t )

para t = 0 x 2 = X 2 C 2 = X 2

x2 = X 2 (1 + t ) 2 (2.28)

dx3 3 x3 dx 3dt
v3 = = 3 = (2.29)
dt 1 + t x3 1 + t
1 3
x 3
dx3 = 1 + t dt Lnx 3 = 3Ln(1 + t ) + LnC 3
(2.30)
3
x3 = C3 (1 + t )

y para t = 0 x3 = X 3 C3 = X 3

x3 = X 3 (1 + t ) 3 (2.31)
Las ecuaciones del movimiento:
x1 = X 1 (1 + t )
2
x2 = X 2 (1 + t ) (2.32)
3
x3 = X 3 (1 + t )
r r
b) Conocido v ( x , t ) en la descripcin espacial (Euleriana), podemos aplicar la
derivada material:
r r
r r v ( x , t ) r r r r
a ( x, t ) = + v ( x , t ) v ( x , t ) (2.33)
t
vi
ai = + (v i , k )v k
t
(2.34)
v
a i = i + (vi ,1 v1 + vi , 2 v 2 + vi ,3 v3 )
t
luego,
x1 x 1
a1 = + 1 + 0 + 0 = 0
1 + t 1 + t
2
(1 + t )
2x2 2x 2 2 x2
a2 = + 0 + 2 + 0 = (2.35)
(1 + t ) 2
1+ t 1+ t (1 + t )
2

3 x3 3x 3 6 x3
a3 = + 0 + 0 + 3 =
(1 + t )
2
1 + t 1 + t (1 + t ) 2

c) La velocidad en la descripcin Lagrangiana viene dada por:

Universidad de Castilla- La Mancha Draft Por: Eduardo W. V. Chaves (2012)


Ciudad Real - Espaa
2 CINEMTICA DEL CONTINUO 155

V1 = X 1

V2 = 2 X 2 (1 + t ) (2.36)
2
V3 = 3 X 3 (1 + t )
luego,
dV1
a1 = =0
dt
dV
a2 = 2 = 2 X 2 (2.37)
dt
dV
a3 = 3 = 6 X 3 (1 + t )
dt

Ejemplo 2.8
Respecto a un conjunto de ejes materiales X i y espaciales xi superpuestos, el campo de
desplazamientos de un cuerpo continuo viene dado por:
x1 = X 1

x2 = X 2 + AX 3 (2.38)
x = X + AX
3 3 2

en las que A es constante. Hallar las componentes del vector desplazamiento en las formas
material y espacial.
Solucin:
Vector desplazamiento:
r r r
u= xX (2.39)
u1 = x1 X 1 = 0

u 2 = x 2 X 2 = X 2 + AX 3 X 2 = AX 3 (2.40)
u = x X = X + AX X = AX
3 3 3 3 2 3 2

Las ecuaciones del movimiento inverso son obtenidas a continuacin:


x1 1 0 0 X1
x = 0 1 A X 2
2 (2.41)
x3 0 A 1 X 3

1 0 0
det 0 1 A = 1 A 2 (2.42)
0 A 1

la inversa:
1 A 2 0 0
1
0 1 A (2.43)
1 A2
0 A 1

luego,

Universidad Castilla- La Mancha Draft Por: Eduardo W. V. Chaves (2012)


Ciudad Real - Espaa
156 PROBLEMAS RESUELTOS DE MECNICA DEL MEDIO CONTINUO

X1 1 A 2 0 0 x1
X = 1
2 1 A2 0 1 A x2 (2.44)
X 3 0 A 1 x3


X 1 = x1

1
X 2 = ( x 2 Ax3 ) (2.45)
1 A2
1
X 3 = ( x3 Ax 2 )
1 A2
Componentes del vector desplazamientos en coordenadas espaciales:

u1 = x1 X 1 = 0

1 A( x3 Ax 2 )
u 2 = x 2 X 2 = x 2 2
( x 2 Ax3 ) = (2.46)
1 A 1 A2
1 A( x 2 Ax3 )
u1 = x3 X 3 = x3 2
( x3 Ax 2 ) =
1 A 1 A2

Ejemplo 2.9
Considrese las ecuaciones del movimiento:
x1 = X 1

x2 = X 2 + X 3t (2.47)
x = X + X t
3 3 3

Determinar las velocidades de las partculas que pasan por el punto (0,1,2) en los tiempos
t1 = 0 s y t 2 = 1 s
Solucin:
El campo de velocidad viene dado por:
r r r r
Dx ( X , t )
V ( X ,t) = (2.48)
Dt
en componentes:
V1 = 0

V2 = X 3 (2.49)
V = X
3 3

r r
Para t = 0 s tenemos que x = X , luego, ( X 1 = 0, X 2 = 1, X 3 = 2)

V1 = 0

V2 = 2 (unidades de velocidad) (2.50)
V = 2
3

Universidad de Castilla- La Mancha Draft Por: Eduardo W. V. Chaves (2012)


Ciudad Real - Espaa
2 CINEMTICA DEL CONTINUO 157

Para t = 1 s , la partcula que est pasando por ( x1 = 0, x 2 = 1, x3 = 2) en la configuracin de


referencia ocupaba la posicin:
x1 = 0 = X 1

x 2 = 1 = X 2 + X 3 ( X 1 = 0; X 2 = 0; X 3 = 1) (2.51)
x3 = 2 = X 3 + X 3

luego,
V1 = 0

V2 = 1 (unidades de velocidad) (2.52)
V = 1
3

Ejemplo 2.10
Dado un sistema de referencia e i , el movimiento de una partcula del medio continuo est
definido por las siguientes ecuaciones:
ct ct
x1 = X 1 sin 2 + X 2 cos
2 2

2
X1 + X 2 X1 + X 2
ct ct (2.53)
x 2 = X 1 cos 2 + X 2 sin
2 2 2


X1 + X 2 X1 + X 2
x3 = X 3

donde c es una constante.


Determinar las componentes de la velocidad en coordenadas materiales y espaciales.
Solucin:
Las componentes de la velocidad en la descripcin material (Lagrangiana) son:
r c ct ct
V1 ( X , t ) = X cos X 2 sin
2 2
X 1 + X 2
1 2 2 X2 +X2
X1 + X 2 1 2
r c ct ct (2.54)
V2 ( X , t ) = X sin + X 2 cos
2 2
X 1 + X 2
1 2 2 X2 +X2
X1 + X 2 1 2
r
V3 ( X , t ) = 0

Teniendo en consideracin (2.53), podemos notar que se cumple la siguiente relacin:


x12 + x22 = X 12 + X 22 (2.55)
Luego, las componentes de la velocidad en la descripcin espacial (Euleriana) son:
r cx
v1 ( x , t ) = 2 2 2
x1 + x 2
r c x1 (2.56)
v 2 ( x, t ) =
x12 + x 22
r
v3 ( x , t ) = 0

Las ecuaciones inversas del movimiento son:

Universidad Castilla- La Mancha Draft Por: Eduardo W. V. Chaves (2012)


Ciudad Real - Espaa
158 PROBLEMAS RESUELTOS DE MECNICA DEL MEDIO CONTINUO

ct ct
sin 2 cos
2

x 2 + x 2 0
x1 + x 2 1 2
X1 x1
X = cos c t sin c t 0 x
2 x2 + x2 (2.57)
x2 + x2
2
X 3 1 2 1 2 x
0 0 1 3


Ejemplo 2.11
El campo de velocidad tiene las siguientes componentes:
v1 = x1

x2
v2 = (2.58)
2t + 3
v3 = 0

en la descripcin Euleriana. Encontrar las ecuaciones paramtricas de la trayectoria de la


partcula que en la configuracin de referencia estaba en ( X 1 , X 2 , X 3 ).
Solucin:
Para encontrar la trayectoria debemos resolver el sistema:
dx1
dt = x1

dx 2 x
= 2 (2.59)
dt 2t + 3
dx3
=0
dt
con las condiciones iniciales
x1 (t = 0) = X 1

x2 (t = 0) = X 2 (2.60)
x (t = 0) = X
3 3

x1 t
dx1 x

X1
x1 0
= dt Ln 1 = t
X1
x1 = X 1 exp t

x2
(2.61)
( ) ( )
t
dx 2 dt x 2

X2
x2
=0
2t + 3
Ln 2
X2
= Ln 2t + 3 Ln 3

x2 = X 2
3
t +1

x3 = X 3

Luego, las ecuaciones del movimiento vienen dadas por:


2
x1 = X 1exp t ; x2 = X 2 t + 1 ; x3 = X 3 (2.62)
3

Universidad de Castilla- La Mancha Draft Por: Eduardo W. V. Chaves (2012)


Ciudad Real - Espaa
2 CINEMTICA DEL CONTINUO 159

Ejemplo 2.12
Considrense las siguientes ecuaciones del movimiento:
x1 = X 1

x2 = 2 t X 3 + X 2 (2.63)
x = X
3 3

r
y una cantidad fsica representada por el campo escalar q ( x , t ) en la descripcin Euleriana:
r
q ( x , t ) = 2 x1 + x 2 x3 + 1 (2.64)
Se pide:
a) Obtener la descripcin Lagrangiana de esta cantidad fsica;
b) Obtener la velocidad en las descripciones Lagrangiana y Euleriana;
c) Obtener la tasa de cambio de la cantidad fsica en cuestin.
d) Obtener la tasa de cambio local de q en el punto espacial (1,3,2) .
Solucin:
r r r r
a) La descripcin Lagrangiana es inmediato, q ( x , t ) = q ( x ( X , t ), t ) = Q( X , t ) , es decir,
r
reemplazamos las ecuaciones del movimiento (2.63) en la expresin de la variable q ( x , t )
dada por (2.64):
r
Q( X , t ) = 2 X 1 + X 2 + ( 2t 1) X 3 + 1 (2.65)
b) La velocidad
r r r r
Dx ( X , t )
V ( X , t) = (2.66)
Dt
Descripcin Lagrangiana
V1 = 0

V2 = 2 X 3 (2.67)
V = 0
3
Las ecuaciones del movimiento inversa:
x1 = X 1 X 1 = x1

x2 = 2 t X 3 + X 2 X 2 = x 2 2 t x3
x = X X = x
3 3 3 3

Luego, la descripcin Euleriana de la velocidad viene dada por:


v1 = 0

v 2 = 2 x3 (2.68)
v = 0
3
c) La tasa de cambio de la variable viene dada por la derivada material
D r
Q& = Q( X , t ) = 2 X 3 (2.69)
Dt

Universidad Castilla- La Mancha Draft Por: Eduardo W. V. Chaves (2012)


Ciudad Real - Espaa
160 PROBLEMAS RESUELTOS DE MECNICA DEL MEDIO CONTINUO

o
r
q( x , t ) r
q& = + xr q v
1
42t4 3 (2.70)
= 0 ( estacionario )

q& = 0 + q, i v i
q q q
=0+ v1 + v2 + v3 = [(2)(0) + (1)(2 x 3 ) + ( 1)(0)] (2.71)
x1 x 2 x 3
= 2 x3

Podramos haber obtenido este resultado partiendo de que Q& = 2X 3 y reemplazando


X 3 = x3 , obteniendo:
r r r
q& ( x , t ) = Q& ( X ( x , t ), t )
r (2.72)
q& ( x , t ) = 2 x3
r
d) Observemos que el campo de la cantidad fsica en cuestin es estacionario, i.e. q = q ( x ) ,
r
q ( x )
luego la tasa local = 0 para cualquier punto espacial.
t

Ejemplo 2.13
Dado el campo de desplazamientos (descripcin Lagrangiana):
u1 = ktX 2 ; u2 = 0 ; u3 = 0

y el campo de la temperatura (descripcin Euleriana):


r
T ( x , t ) = ( x1 + x 2 ) t

a) Encontrar la tasa de cambio de la temperatura para una partcula que en el tiempo t = 1s


est pasando por el punto (1,1,1) .
Solucin:
r r
r T T x r dT ( X , t )
&
Podemos aplicar las dos definiciones: T ( x , t ) = + r &
T ( X , t) =
t x t t
A travs de la relacin u i = xi X i podemos obtener las ecuaciones del movimiento:
u1 = x1 X 1 x1 = X 1 + ktX 2
u 2 = x2 X 2 x2 = X 2
u 3 = x3 X 3 x3 = X 3
El campo de temperatura en la descripcin material queda:
r r r
T ( x ( X , t ), t ) = ( x1 + x 2 ) t = (( X 1 + ktX 2 ) + ( X 2 ) ) t = X 1t + kX 2 t 2 + X 2 t = T ( X , t )
Luego, la derivada material viene dada por:
r
T& ( X , t ) = X 1 + 2kX 2 t + X 2
Si queremos encontrar la tasa de la temperatura para una partcula que est pasando por el
punto x1 = 1, x 2 = 1, x3 = 1 en t = 1s , tenemos dos posibilidades. 1) encontrar la posicin de
la partcula en la configuracin de referencia y reemplazar en la ecuacin anterior. 2)

Universidad de Castilla- La Mancha Draft Por: Eduardo W. V. Chaves (2012)


Ciudad Real - Espaa
2 CINEMTICA DEL CONTINUO 161

obtener la expresin de la tasa de la temperatura en la descripcin espacial, para esto


r r
necesitamos de las ecuaciones de movimiento X ( x , t ) :
x1 = X 1 + ktX 2 X 1 = x1 ktx 2

x2 = X 2 X 2 = x2
x = X X = x
3 3 3 3
r r r
T& ( X ( x , t ), t ) = X 1 + 2kX 2 t + X 2 = ( x1 ktx 2 ) + 2kt ( x 2 ) + ( x 2 ) = T& ( x , t )
r
Simplificando tenemos que T& ( x , t ) = x1 + ktx 2 + x 2 . Luego:
T& ( x1 = 1, x 2 = 1, x3 = 1, t = 1) = (1 k ) + 2k + 1 = k + 2
Solucin Alternativa:
r
r T T x T T x i
T& ( x , t ) = + r = + (escalar )
t x t t x i t
T x1 T x 2 T x3
= ( x1 + x 2 ) + + +
x1 t x 2 t x3 t
= ( x1 + x 2 ) + (tkX 2 + t (0) + (0)(0) )
= x1 + x 2 + tkX 2
Observemos que x 2 = X 2 , luego:
r
T& ( x , t ) = x1 + x 2 + tkx 2

Ejemplo 2.14
Considrese las siguientes ecuaciones del movimiento:

x1 = X 1

t
x2 = X 2 + X 3 (2.73)
2
t
x3 = X 3 + 2 X 2

Se pide:
a) Para que valores de t > 0 (tiempo) este movimiento es posible y que tenga sentido
fsico?
b) Determinar las componentes de la velocidad en la descripcin Lagrangiana y
Euleriana;
c) Obtener la ecuacin de la trayectoria.
Solucin:
a) Obteniendo el determinante del Jacobiano:
1 0 0
xi t2
J=F = = 0 1 2t = 1 (2.74)
X j 4
0 2t 1

con lo que el movimiento es posible para t < 2 s , ya que:

Universidad Castilla- La Mancha Draft Por: Eduardo W. V. Chaves (2012)


Ciudad Real - Espaa
162 PROBLEMAS RESUELTOS DE MECNICA DEL MEDIO CONTINUO

t2
J =1 >0t <2 s (2.75)
4
b) Velocidad en la descripcin Lagrangiana:

V = 0
1
D t X3
V2 = X2 + X3 = (2.76)
Dt 2 2
D t X2
V3 = X3 + X2 =
Dt 2 2

La inversa del tensor F viene dada por:


T
1 t
0 t 0 1
t
2
2
t
2 1 0 1 0 2

1 1 1 0 0 1 0 1 0
F 1 = adj( F ) = [cof (F )]T Fij1 = t t

F F J 2 1 0 1 0 2
0 0 1 0 1 0
(2.77)
t

1 2
0 2t 0 1
J 0 0
1
Fij1 = 0 1 2t
J
0 2t 1

r r
Las ecuaciones inversas del movimiento X = F 1 x :
x1 1 0 0 X 1 X1 J 0 0 x1
x = 0 1 t X X = 1 0 1

2t x 2
2 2 2 2 J (2.78)
x3 0 2 1 X 3
t X 3 0
2t 1 x3

Reemplazando X i en la expresin de la velocidad, obtenemos la velocidad en la


descripcin espacial:
t t
x2x3 x2
x3
2 2 x tx 2 = 2 x 2 tx 3
v1 = 0 ; v 2 = 2
= 3 22 ; v3 = (2.79)
t 4t t2 4 t2
2 2
2 2
c) La trayectoria se obtiene eliminando t de las ecuaciones del movimiento (2.73):
x1 = X 1

X2 X2 (2.80)
( x3 X 3 ) X 3 = ( x 2 X 2 ) X 2 x3 =
X3
x2 2 + X 3
X3

Universidad de Castilla- La Mancha Draft Por: Eduardo W. V. Chaves (2012)


Ciudad Real - Espaa
2 CINEMTICA DEL CONTINUO 163

Ejemplo 2.15
r
La velocidad en un punto x de un fluido estacionario viene dada por:
r b 2 ( x12 x 22 ) b 2 x1 x 2
v =U 1 + 2U
e e 2 + Ve 3 (2.81)
( x12 + x 22 ) 2 ( x12 + x 22 ) 2

donde U y V son constantes.


r r
Mostrar que xr v = 0 y encontrar la aceleracin de la partcula en x .
Solucin:
r v v v x ( x 2 3x 2 ) x ( x 2 3x 2 )
xr v = vi ,i = 1 + 2 + 3 = 2Ub 2 1 21 2 23 + 2Ub 2 1 21 2 23 = 0
x1 x2 x3 ( x1 + x2 ) ( x1 + x2 )
La aceleracin:
r
r v r r r r
a= + xr v v = xr v v
t
Las componentes del gradiente espacial de la velocidad vienen dadas por:
x1 (3 x 22 x12 ) x 2 (3 x12 x 22 ) 0
r 2Ub 2

( xr v ) ij = 2 2 3
x 2 (3 x12 x 22 ) x1 (3 x 22 x12 ) 0
( x1 + x 2 )
0 0 0
r r
Las componentes de la aceleracin vienen dadas por a i = ( xr v ) ij (v ) j :

2 x1U 2 b 4
2 2 3
( x1 + x 2 )

a i = 2 x 2U 2 b 4
2 2 3
( x1 + x 2 )

0
Ejemplo 2.16
r
Dada una funcin escalar en la configuracin de referencia ( X , t ) . Obtener la relacin
r r
entre el gradiente material de ( X , t ) , es decir, Xr ( X , t ) , y el gradiente espacial de
r r
( x , t ) , es decir, xr ( x , t ) .
Solucin:
Recordemos que una variable que est en la configuracin de referencia la podemos
expresar en la configuracin actual a travs de las ecuaciones del movimiento, es decir,
r r r r
( X , t ) = ( X ( x , t ), t ) = ( x , t ) . Luego, partiendo de la definicin del gradiente de un
escalar podemos obtener que:
r r r r r
r ( X , t ) ( X ( x , t ), t ) x ( x , t ) r
Xr ( X , t ) = r = r r= r F = xr ( x , t ) F
X x X x
Y la forma inversa:
r r r r r
r ( x , t ) ( x ( X , t ), t ) X ( X , t ) r
xr ( x, t ) = r = r r= r F 1 = Xr ( X , t ) F 1
x X x X

Universidad Castilla- La Mancha Draft Por: Eduardo W. V. Chaves (2012)


Ciudad Real - Espaa
164 PROBLEMAS RESUELTOS DE MECNICA DEL MEDIO CONTINUO

Ejemplo 2.17
Para el siguiente campo de velocidades:

v1 = 0 ; v2 = 0 ; v3 = f ( x1 , x 2 ) x3

Se pide:
a) Hallar las trayectorias de las partculas;
b) Determinar la densidad de masa ( ), sabiendo que en t = 0 , = f ( x1 , x 2 ) .
Solucin:
dx1
= v1 = 0 x1 (t ) = C1 para t = 0 x1 = X 1 x1 (t = 0) = C1 = X 1 ;
dt
dx 2
= v 2 = 0 x 2 (t ) = C 2 para t = 0 x 2 = X 2 x 2 (t = 0) = C 2 = X 2
dt
dx3 dx3
dt
= v3 = f ( x1 , x 2 ) x3 = f (C1 , C 2 ) x3
x3
= f (C , C 1 2 ) dt Ln( x 3 ) = f (C1 , C 2 )t + k

haciendo k = Ln(C 3 ) obtenemos que:


Ln( x3 ) Ln(C 3 ) = f ( X 1 , X 2 )t
x x
ln 3 = f ( X 1 , X 2 )t 3 = exp f ( X 1 , X 2 )t x3 = C 3 exp f ( X 1 , X 2 )t
C3 C3
para t = 0 x3 = X 3 x3 (t = 0) = C 3 = X 3
Resumiendo:
x1 = X 1
x2 = X 2 (2.82)
f ( X 1 , X 2 )t
x3 = X 3 exp

Densidad de masa:
0 xi
= con Fij =
F X j

1 0 0
Fij = 0 1 0 = exp f ( X 1 , X 2 )t
? ? exp f ( X 1 , X 2 )t
Los valores ( ? ) no son necesarios para obtener el determinante. Luego:
0 f (X1, X 2 )
= =
F exp f ( X 1 , X 2 )t
Observemos que segn el enunciado, t = 0 , = f ( x1 , x 2 ) , y segn expresiones (2.82)
concluimos que 0 = f ( X 1 , X 2 ) .

Universidad de Castilla- La Mancha Draft Por: Eduardo W. V. Chaves (2012)


Ciudad Real - Espaa
2 CINEMTICA DEL CONTINUO 165

Ejemplo 2.18
D
Calcular la derivada material para la propiedad cuando esta est descrita en las
Dt
siguientes coordenadas:
r
Materiales: ( X , t ) = X 1t 2 ;
r x1t 2
Espaciales: ( x , t ) = .
(1 + t )
Solucin:
r
a) Derivada material de ( X , t ) = X 1t 2 :
D r r
( X , t ) & ( X , t ) = 2 X 1t
Dt
r x1t 2
b) Derivada material de ( x , t ) = :
(1 + t )
r r r
D r ( x , t ) r ( x , t ) ( x , t )
( x , t ) = + x v =
r + vi
Dt t t xi
r r r r
( x , t ) ( x , t ) ( x , t ) ( x , t )
= + v1 + v2 + v3
t x1 x 2 x3
r
x t 2 ( x , t )
= 1 + v1 + 0 + 0
t (1 + t ) x1
Necesitamos conocer la velocidad v1 . Partimos del principio de que una propiedad es algo
intrnsico a la partcula luego:
r
( X , t ) = X 1t 2
r r r x t2 x1
( X ( x, t ), t ) = ( x , t ) = 1 X1 =
(1 + t ) (1 + t )
La velocidad queda:

( )
r D r x
v( X , t) = X 1t 2 = 2 X 1t e 1 v ( x , t ) = 2 1 t e 1
Dt (1 + t )
Retomando a la derivada material tenemos que:
r
D r x t 2 ( x , t )
( x , t ) = 1 + v1
Dt t (1 + t ) x1
2 x1t x t2 t2
= 1 2 +
X1
(1 + t ) (1 + t ) (1 + t )
2 x1t x t2 t2 x
= 1 2 +
2 1 t
(1 + t ) (1 + t ) (1 + t ) (1 + t )
2 x1t
=
(1 + t )

Universidad Castilla- La Mancha Draft Por: Eduardo W. V. Chaves (2012)


Ciudad Real - Espaa
166 PROBLEMAS RESUELTOS DE MECNICA DEL MEDIO CONTINUO

Tambin podramos haber obtenido este mismo resultado partiendo que se cumple que
D r r
( X , t ) & ( X , t ) = 2 X 1t y adems conociendo las ecuaciones del movimiento
Dt
x1
X1 = , obtenemos que:
(1 + t )
D r r
( X , t ) & ( X , t ) = 2 X 1t
Dt
D r r r r r x
( X ( x, t ), t ) & ( X ( x, t ), t ) & ( x, t ) = 2 1 t
Dt (1 + t )

Ejemplo 2.19
Sean las siguientes ecuaciones del movimiento en la descripcin Lagrangiana:

x1 = X 1t 2 + 2 X 2 t + X 1 x1 t + 1 2t
2
0 X 1
2
forma
Matricial
x 2 = 2 X 1 t + X 2 t + X 2 x 2 = 2t 2 t +1 0 X 2 (2.83)
x = 1 X t + X x 0 0 1
t + 1 X 3
3 2 3 3 3 2

Encontrar las componentes del vector desplazamiento en la descripcin Lagrangiana y


Euleriana.
Solucin:
r r r
El vector desplazamiento viene dado por u = x X . Reemplazando las ecuaciones del
movimiento (2.83) obtenemos:
u1 = x1 X 1 = X 1t 2 + 2 X 2 t
2
u 2 = x 2 X 2 = 2 X 1t + X 2 t

u 3 = x 3 X 3 = 2 X 3 t
1

que son las componentes del vector desplazamiento en la descripcin Lagrangiana


(material).
Para obtener la descripcin Euleriana debemos obtener las ecuaciones inversas de
(2.83), resultando:
2tx 2 x1 (1 + t )
X1 = 3
(1 + t ) x 3t 1 t t 2
1
X 2t 0
1 2
1
2 x1t 2 x 2 (1 + t 2 )
X 2 = 3 2
2t (1 + t 2 ) 0 x2 X 2 =
X 3t 1 t t 3t 3 1 t t 2 x 3 3t 3 1 t t 2
3 0 0 2 x3

1
2
(t + 2) X 3 =
(t + 2)
r r r
Podemos utilizar, una vez ms, la definicin u = x X , pero ahora reemplazamos las
coordenadas materiales, obteniendo as las componentes del vector desplazamiento en
coordenadas espaciales (descripcin Euleriana):

Universidad de Castilla- La Mancha Draft Por: Eduardo W. V. Chaves (2012)


Ciudad Real - Espaa
2 CINEMTICA DEL CONTINUO 167

2tx 2 x1 (1 + t )
u1 = x1 X 1 = x1 3
3t 1 t t 2
2 x1t 2 x 2 (1 + t 2 )
u 2 = x 2 X 2 = x 2
3t 3 1 t t 2
2 x3
u 3 = x 3 X 3 = x 3
(t + 2)

Ejemplo 2.20
Las siguientes ecuaciones X 2 , x2
describen el movimiento de las Configuracin de referencia
partculas de un cuerpo (medio
continuo): 1 B
C
1
x1 = X 1 + 0,2 X 2 t
E
x2 = X 2 1
x = X
3 3 A
O
X 1 , x1
D
G

X 3 , x3
Figura 2.1: Configuracin de referencia t = 0 .
En t = 0 , este cuerpo tiene forma de cubo de lado unitario con un vrtice en el origen,
punto O, como se indica en la Figura 2.1. Determinar la configuracin del cuerpo en el
instante t = 2 s .
Solucin:
Para obtener la configuracin actual del cuerpo para el instante t = 2 s analizaremos
independientemente el movimiento de las partculas. La partcula que ocupa posicin O
(origen) en t = 0 tiene coordenadas materiales:
X1 = 0

X 2 = 0
X = 0
3
Sustituyendo en la expresin del movimiento:
x1 = 0

x i ( X 1 = 0, X 2 = 0, X 3 = 0, t ) x 2 = 0
x = 0
3
Concluyendo que la partcula del origen no cambia de posicin durante el movimiento.
Las partculas que ocupan la lnea OA en la configuracin inicial tienen como coordenadas
de referencia: ( X 1 , X 2 = 0, X 3 = 0) . En coordenadas espaciales:
x1 = X 1 + 0,2 X 2 t = X 1

x2 = X 2 = 0
x = X = 0
3 3

Universidad Castilla- La Mancha Draft Por: Eduardo W. V. Chaves (2012)


Ciudad Real - Espaa
168 PROBLEMAS RESUELTOS DE MECNICA DEL MEDIO CONTINUO

Es decir, todas las partculas que estn en la lnea OA no se mueven durante el


movimiento. Anlogamente, podemos verificar que la recta ( X 1 , X 2 = 0, X 3 = 1) en la
configuracin de referencia ( X 1 , X 2 = 0, X 3 = 1) no se mueve:
x1 = X 1 + 0,2 0 2 = X 1

x2 = X 2 = 0
x = X = 0
3 3

Las partculas que estn en la lnea CB ( X 1 , X 2 = 1, X 3 = 0) en el tiempo t = 2 s ocuparn


las posiciones:
x1 = X 1 + 0,2 1 2 = X 1 + 0,4

x2 = X 2 = 1
x = X = 0
3 3

Luego, todas las partculas que estn en la lnea CB se desplazarn 0,4 segn la direccin
x1 .

Las partculas que estn en la lnea OC en t = 0 , tras el movimiento ocuparn las


posiciones:
x1 = X 1 + 0,2 X 2 t = 0 + 0,2 2 X 2 = 0,4 X 2

x2 = X 2
x = X = 0
3 3

Siguiendo el mismo procedimiento para las partculas restantes, se obtiene la configuracin


final ( t = 2 s ) del cuerpo representada por la Figura 2.2.

x2

0,4 0,4

C C 1 B B Configuracin del cuerpo


en t = 2 s
E E 1
A=A
O
1 x1
D
G=G

x3

Figura 2.2: Configuracin actual t .

Ejemplo 2.21
Considrense las ecuaciones del movimiento dadas por:
x1 = X 1 + t 2 X 2 x1 1 t2 0 X 1
2
Forma
Matricial
x 2 = t X 1 + X 2 x 2 = t 2 1 0 X 2
x = X x 0 0 1 X 3
3 3 3

Universidad de Castilla- La Mancha Draft Por: Eduardo W. V. Chaves (2012)


Ciudad Real - Espaa
2 CINEMTICA DEL CONTINUO 169

Se pide:
1) Determinar la trayectoria de una partcula Q que originalmente, en t 0 , estaba en
X i = (1,2,1) ;
2) Considerando la configuracin actual t = 0,5s . Determinar las componentes de la
velocidad y de la aceleracin de una partcula P que originalmente estaba en
X i = (16 ; 4 ;1) ;
15 15

3) Obtener las ecuaciones del movimiento en la descripcin Euleriana;


4) Obtener las componentes de la velocidad y la aceleracin de una partcula que en el
tiempo ( t = 0,5s ) pasa por el punto xi = (1,0,1) .
NOTA: Considerar el sistema internacional de unidades.
Solucin:
1)Utilizando las ecuaciones del movimiento y reemplazando las coordenadas materiales del
punto X i = (1,2,1) , obtenemos:
x1 = 1 + 2t 2
2
x2 = 2 + t
x = 1
3

Las ecuaciones anteriores representan la trayectoria de la partcula. Para obtener la ecuacin


de la partcula, eliminamos el tiempo de las ecuaciones de movimiento, obteniendo as:
x1 2 x 2 = 3

x3 = 1
Lo cual indica que la partcula se mueve en lnea recta segn la ecuacin ( x1 2 x 2 = 3) en
el plano x3 = 1 , ver figura siguiente.

X 3 , x3 trayectoria de la partcula

( x1 2 x 2 = 3)
x3 = 1

X 2 , x2

X 1 , x1

2) Las componentes de la velocidad y de la aceleracin de la partcula P vienen dadas


respectivamente por:
r V1 = 2tX 2
r r Dx componente s
V ( X , t) = V 2 = 2tX 1
Dt V = 0
3

Universidad Castilla- La Mancha Draft Por: Eduardo W. V. Chaves (2012)


Ciudad Real - Espaa
170 PROBLEMAS RESUELTOS DE MECNICA DEL MEDIO CONTINUO

r A1 = 2 X 2
r r Dv componente s
A( X , t ) = A2 = 2 X 1
Dt A = 0
3
Luego, para la partcula situada originalmente en X i = (16 ; 4 ;1) , en t = 0,5s tiene:
15 15

V1 = 2 0,5 15
4
= 15
4
m/s A1 = 2 15
4 8
= 15 m / s2

V 2 = 2 0,5 (15 ) = 15 m / s A2 = 2 (15 ) = 15 m / s
16 16 16 32 2
y
V = 0 A = 0
3 3

3) Invirtiendo las ecuaciones del movimiento resulta:


x1 t 2 x 2
1 X =
x1 = X 1 + t 2 X 2 X 1 = x1 t 2 X 2 1 t4
2 2
x t 2 x1
x2 = t X 1 + X 2 X 2 = x2 t X 1 X 2 = 2
x = X X = x 1 t4
3 3 3 3
X 3 = x3


4) Podemos obtener la velocidad y la aceleracin de la partcula que en el tiempo ( t = 0,5s )
pasa por el punto xi = (1,0,1) simplemente obteniendo la expresin de la velocidad y
aceleracin en coordenadas espaciales:
Velocidad:
x 2 t 2 x1 4
1 v = 2t
1 t4 v1 = 15 m / s
V1 = 2tX 2
sustituyen do
x1 t 2 x 2 = 0,5 s 16
V
2 = 2 X 1 t 2v = 2 t 4
t v 2 = m / s
V = 0 X1 , X 2
1 t x (1, 01)
15
3 v 3 = 0 v 3 = 0

Aceleracin:
x 2 t 2 x1 8
1 a = 2 2

1 t4 a1 = 15 m / s
A1 = 2 X 2
sustituyen do
x1 t 2 x 2 =0,5 s 32
A
2 = 2 X 1 2a = 2 4
t a 2 = m / s2
A = 0 X1 , X 2
1 t x (1, 01)
15
3 a 3 = 0 a 3 = 0

Podemos obtener la posicin inicial de esta partcula utilizando las ecuaciones inversas del
movimiento obtenidas en el apartado 3, xi (1,0,1) , resultando:

Universidad de Castilla- La Mancha Draft Por: Eduardo W. V. Chaves (2012)


Ciudad Real - Espaa
2 CINEMTICA DEL CONTINUO 171

x1 t 2 x 2 1 (0,5 2 )(0) 16
1X = = =
1 t4 1 (0,5) 4 15
x 2 t 2 x1 0 (0,5 2 )(1)
4
X 2 = 4
= 4
=
1 t 1 (0,5) 15
X = x = 1
3 3

Podemos verificar que es la misma partcula P referida en el apartado 2. Es lgico que


hayamos encontrado las mismas velocidades y aceleraciones utilizando las coordenadas
materiales o espaciales, ya que la velocidad y aceleracin son propiedades intrnsecas de una
partcula.

Ejemplo 2.22
La aceleracin de una partcula en un medio continuo est descrita por:
r r
r r D v v r r
a ( x , t) = = + xr v v
Dt t
Demostrar que la aceleracin tambin se puede escribir como:
r r r
Dv v v2 r r r v v2 r r
= + xr v ( xr v ) =
+ r
x
v rot v

Dt t 2 t 2
Solucin:
Para demostrar la relacin anterior es suficiente demostrar por identificacin de trminos:
r r v2 r r r
xr v v = xr v ( xr v )

2
En notacin simblica:
v2 r r r 1
xr v ( xr v ) = e i
2 x i
( )
v j v j (v i e i )
x r
e r (v s e s )
2
Utilizado la definicin del operador de permutacin (Captulo 1) podemos expresar el
producto vectorial como:
v2 r r 1 v
xr
r
v ( xr v ) = e i
2 x i
( )
v j v j (vi e i ) rst s e t
x r
2
1 v j v s
= e i 2v j rst itk v i e k
2 x i x r

En el captulo 1 del libro se demostr que rst itk = rst kit = rk si ri sk , luego:
v2 r r r v j v s
xr v ( xr v ) = v j
e i ( rk si ri sk )v i e k
2 x i x r
v j v v
=vj e i rk si v i s ri sk v i s e k
x i x r x r
v j v v
=vj e i v s s v i k e k
x i x k x i

Universidad Castilla- La Mancha Draft Por: Eduardo W. V. Chaves (2012)


Ciudad Real - Espaa
172 PROBLEMAS RESUELTOS DE MECNICA DEL MEDIO CONTINUO

v2 r r r v j v v
xr v ( xr v ) = v j
e i v s s e k + v i k e k
2 x i x k x i
v j v v
= sj v s e i v s s ik e i + v i k e k
x i x k x i
v s v s v k
= vs e i v s e i + v i e k
x i x i x i
v e
= k k vi
x i
r r
= xr v v
NOTA: Es interesante que el lector compare ste desarrollo con el Ejemplo 1.113
(captulo 1).

Ejemplo 2.23
r r r
Consideremos las ecuaciones de movimiento x ( X , t ) y el campo de temperatura T ( x , t )
dado por:
x1 = X 1 (1 + t )
r
x 2 = X 2 (1 + t ) ; T ( x ) = x12 + x 22
x = X
3 3

Encontrar la tasa de cambio de la temperatura de la partcula P en el tiempo t = 1s , dicha


partcula P estaba en el punto ( X 1 = 3, X 2 = 1, X 3 = 0) en el tiempo t = 0 .
Solucin 1:
En esta primera solucin obtenemos la derivada material de la temperatura Lagrangiana,
r
para ello tenemos que obtener la temperatura en la descripcin Lagrangiana T ( X , t )
(Temperatura Lagrangiana):
r
T ( x ) = x12 + x 22

sustituyen do
las ecuaciones del movimiento

r
T ( X , t ) = X 12 (1 + t ) 2 + X 22 (1 + t ) 2
La derivada material de la temperatura Lagrangiana viene dada por:
r
r DT dT ( X , t )
&
T ( X , t) = = 2 X 12 (1 + t ) + 2 X 22 (1 + t )
Dt dt
Sustituyendo t = 1s , ( X 1 = 3, X 2 = 1, X 3 = 0) , en la ecuacin anterior obtenemos que:
r
T& ( X , t ) = 2 X 12 (1 + t ) + 2 X 22 (1 + t ) = 2(3) 2 (1 + 1) + 2(1) 2 (1 + 1) = 40
Solucin 2:
En esta solucin alternativa, utilizamos directamente la definicin de derivada material de
r r
r DT T ( x ) T ( x ) r
una propiedad Euleriana, i.e. T& ( x , t ) = = + vk ( x, t ) .
Dt t x k
De las ecuaciones del movimiento obtenemos:

Universidad de Castilla- La Mancha Draft Por: Eduardo W. V. Chaves (2012)


Ciudad Real - Espaa
2 CINEMTICA DEL CONTINUO 173

r
x1 = X 1 (1 + t ) v1 ( X , t ) = X 1
velocidad
r
x 2 = X 2 (1 + t ) v 2 ( X , t ) = X 2
x = X r
3 3
v 3 ( X , t ) = 0
Las ecuaciones del movimiento en la descripcin Euleriana vienen dadas por:
x1
X 1 = (1 + t )
x1 = X 1 (1 + t )
movimiento inverso x2
x2 = X 2 (1 + t ) X 2 =
x = X (1 + t )
3 3 X 3 = x3


Luego, es posible obtener la velocidad Euleriana como:
r r r x1 r
V1 ( X ( x , t ), t ) = X 1 ( x , t ) = (1 + t ) = v1 ( x , t )

r r r x2 r
V 2 = ( X ( x , t ), t ) = X 2 ( x , t ) = = v 2 ( x, t )
(1 + t )
V3 = v 3 ( xr , t ) = 0


r
Con eso, la derivada material de la temperatura Euleriana, T ( x , t ) , viene dada por:
r r
DT ( x , t ) & r T ( x ) T T T
T ( x, t ) = + v1 + v2 + v3
Dt 12 t3 x1 x 2 x 3
= 0 (Campo estacionar io)

r x x r 2x 2 2x 2 2
T& ( x , t ) = 2 x1 1 + 2 x 2 2 + 0 T& ( x , t ) = 1 + 2 = ( x12 + x 22 )
1+ t 1+ t 1+ t 1+ t 1+ t
La posicin de la partcula P en el tiempo t = 1s viene dada por:
x1 = X 1 (1 + t ) = 3(1 + 1) = 6

x 2 = X 2 (1 + t ) = 1(1 + 1) = 2
x = X = 0
3 3

Luego, sustituyendo las coordenadas espaciales en la expresin de la derivada material de la


temperatura obtenemos que:
r 2 2
T& ( x , t ) = T& ( x1 = 6, x 2 = 2, x 3 = 0, t = 1) = ( x12 + x 22 ) = (6 2 + 2 2 ) = 40
1+ t 1+1
r
Alternativamente, la expresin T& ( x , t ) puede tambin ser obtenido como sigue:
r
T& ( X , t ) = 2 X 12 (1 + t ) + 2 X 22 (1 + t )
2 2
r r r 2 r 2 x x
T& ( X ( x , t ), t ) = 2[X 1 ( x , t )] (1 + t ) + 2[X 2 ( x , t )] (1 + t ) = 2 1 (1 + t ) + 2 2 (1 + t )
(1 + t ) (1 + t )
2 r
= ( x12 + x 22 ) = T& ( x , t )
(1 + t )

Universidad Castilla- La Mancha Draft Por: Eduardo W. V. Chaves (2012)


Ciudad Real - Espaa
174 PROBLEMAS RESUELTOS DE MECNICA DEL MEDIO CONTINUO

Ejemplo 2.24
Sea el movimiento:
x i = X i (1 + t )

t > 0
Determinar el campo de velocidad en la descripcin espacial.
Solucin:
La velocidad ser obtenida a travs de la derivada temporal de las ecuaciones del
movimiento:
d
Vi = x& i = [X i (1 + t )] = X i (2.84)
dt
Para hallar la velocidad en la descripcin espacial tenemos que obtener la inversa de las
ecuaciones del movimiento y reemplazar en la ecuacin (2.84):
xi
x i = X i (1 + t ) X i = (1 + t )

v = X ( xr , t ) = x i
i i
1+ t

Ejemplo 2.25
r
Considrese el campo de temperatura T ( x ) en la descripcin espacial y las ecuaciones del
movimiento siguientes:
T = 2( x12 + x 22 )

xi = X i (1 + t ) i {1,2}

Encuentre en t = 1s la tasa de cambio de temperatura de una partcula que ocupaba la


posicin (1,1) en la configuracin de referencia.
NOTA: Podemos observar que el campo de temperatura es un campo estacionario, es
r
decir T = T ( x ) .
Solucin 1:
En esta primera solucin obtendremos la ecuacin de la temperatura en la descripcin
material:
r
T ( x ) = 2( x12 + x 22 )

reemplazan do las ecuaciones
del movimiento

r
[
T ( X , t ) = 2 X 2 (1 + t ) 2 + X 2 (1 + t ) 2
1 2 ]
La derivada material viene dada por:
r
&
r
T ( X , t) =
DT dT ( X , t )
Dt
=
dt
[
= 2 2 X 12 (1 + t ) + 2 X 22 (1 + t ) ]
Reemplazando t = 1s y las coordenadas materiales ( X 1 = 1; X 2 = 1) obtenemos:
T& ( X 1 = 1; X 2 = 1; t = 1) = 16

Universidad de Castilla- La Mancha Draft Por: Eduardo W. V. Chaves (2012)


Ciudad Real - Espaa
2 CINEMTICA DEL CONTINUO 175

Solucin 2:
En esta segunda solucin usaremos directamente la definicin de derivada material para
propiedades descritas en coordenadas espaciales:
r
T ( x ) = 2( x12 + x12 )x i = (1 + t ) X i
; i {1,2}
r r
& r DT T ( x ) T ( x ) x k
T ( x, t ) = = + i {1,2}
Dt t x k t
r
r T ( x )
dado que T ( x ) no es funcin del tiempo =0:
t
r T x1 T x 2
T& ( x , t ) = 0 + +
x1 {
t x 2 {
t
V1 = X 1 V2 = X 2

r x x
T& ( x , t ) = 0 + 4 x1 1 + 4 x 2 2
1+ t 1+ t
2 2
r 4x 4x
T& ( x , t ) = 1 + 2
1+ t 1+ t
La partcula que en la configuracin de referencia ocupaba la posicin (1,1) , en el tiempo
t = 1s estar en la posicin x i = (1 + t ) X i = 2 X i , es decir, ( x1 = 2; x 2 = 2 ):

4( 2) 2 4( 2) 2
T& ( x1 = 2; x 2 = 2; t = 1) = + = 16
1+1 1+1

Ejemplo 2.26
Dadas las siguientes ecuaciones del movimiento:
x1 = X 1 exp t + X 3 (exp t 1)
t t
x 2 = X 2 + X 3 (exp exp )
x = X
3 3

Se pide:
Determinar las componentes de la velocidad y de la aceleracin en coordenadas materiales
y espaciales.
Solucin:
Primero obtenemos la inversa de la ecuacin del movimiento:
x1 = X 1 exp t + X 3 (exp t 1) x1 X 1 exp t = x 3 (exp t 1)
t

t
x 2 = X 2 + X 3 (exp exp ) x 2 X 2 = x 3 (exp t exp t )

x = X X = x x = X X = x
3 3 3 3
3 3 3 3

resultando:
X 1 = x1 exp t exp t (exp t 1)
2t t
X 2 = x 2 x 3 (exp 1)exp (2.85)
X = x
3 3

Universidad Castilla- La Mancha Draft Por: Eduardo W. V. Chaves (2012)


Ciudad Real - Espaa
176 PROBLEMAS RESUELTOS DE MECNICA DEL MEDIO CONTINUO

x1 exp X 1 X 1 exp exp t (exp t 1) x1


t t
0 (exp t 1) 0

x2 = 0 1 (exp t exp t ) X 2 inversa
X 2 = 0 1 (exp 2 t 1)exp t x 2
x 0 0 1 X X 0 0 1 x
3 3 3 3

a) La velocidad en la descripcin material viene dada a travs de sus componentes:


V1 = X 1 exp t + X 3 exp t
D r
V i= V 2 = X 3 exp t + X 3 exp t = X 3 (exp t + exp t )
x j ( X , t) (2.86)
Dt V = 0
3

b) La aceleracin en la descripcin material viene dada por:


r A1 = X 1exp t + X 3 exp t
r DV i ( X , t )
Ai ( X , t ) = A2 = X 3 (exp t exp t )
(2.87)
Dt A = 0
3

Para obtener la velocidad y la aceleracin en la descripcin espacial es suficiente sustituir en


las ecuaciones (2.86) y (2.87) los valores de X 1 , X 2 , X 3 , dados por la ecuacin (2.85),
resultando:
v1 = x1 + x 3 a1 = x1 + x 3
t t t t
v 2 = x 3 (exp + exp ) ; a 2 = x 3 (exp exp )
v = 0 a = 0
3 3
Velocidad en la Aceleraci n en la
descripci n espacia l descripci n espacial

Ejemplo 2.27
El movimiento de un medio continuo viene definido por las siguientes ecuaciones:
x1 = 12 ( X 1 + X 2 )exp t + 12 ( X 1 X 2 )exp t
t t
x 2 = 2 ( X 1 + X 2 )exp 2 ( X 1 X 2 )exp
1 1

x = X
3 3

0 t constante
Expresar las componentes de la velocidad en la descripcin material y espacial.
Solucin:
Las componentes de la velocidad utilizando la descripcin material son:
r
Dx1 ( X , t ) 1 1
V1 = = ( X 1 + X 2 )exp t ( X 1 X 2 )exp t
Dt 2 2
r
Dx 2 ( X , t ) 1 1
V 2 = = ( X 1 + X 2 )exp t + ( X 1 X 2 )exp t (2.88)
Dt 2 2
V3 = 0

Para expresar las componentes de la velocidad en la descripcin espacial tenemos que


obtener la inversa de las ecuaciones de movimiento, xi = x i ( X 1 , X 2 , X 3 ) resultando:

Universidad de Castilla- La Mancha Draft Por: Eduardo W. V. Chaves (2012)


Ciudad Real - Espaa
2 CINEMTICA DEL CONTINUO 177

(exp t + exp t ) (exp t exp t )


0
x1 2 2 X 1
(exp
t
exp t ) (exp t + exp t )
x2 = 0 X 2
2 2
x
3 0 0 1 X 3


X1 (exp 2t + 1)exp t (exp 2t 1)exp t 0 x1
1
X 2 = (exp 2t 1)exp t
inversa (exp 2 t + 1)exp t 0 x2
X 2 0 0 2 x 3
3
Para obtener la velocidad en la descripcin espacial es suficiente reemplazar las ecuaciones
anteriores en las expresiones de la velocidad (2.88), resultando:
v1 = x 2

v 2 = x1
v = 0
3
Ejemplo 2.28
Dado el movimiento:
x i = ( X 1 + ktX 2 ) i1 + X 2 i 2 + X 3 i 3 i {1,2,3}
y la temperatura
T = x1 + x 2
Encontrar la tasa de cambio de T para la partcula que en la configuracin actual est
situada en el punto (1,1,1) .
Solucin:
Explcitamente las ecuaciones de movimiento son:
x1 = X 1 + ktX 2

x2 = X 2
x = X
3 3

Reemplazando xi en la expresin de la temperatura, se obtiene la temperatura en la


configuracin material:
r r
T ( x ) = x1 + x 2 T ( X , t ) = X 1 + ktX 2 + X 2
La derivada material de la temperatura viene dada por:
r DT D ( X 1 + ktX 2 + X 2 )
T& ( X , t ) = = = kX 2 = k x 2 ( T& = k
1,1,1)
Dt Dt
Solucin alternativa:
La derivada material para una propiedad expresada en la descripcin espacial viene dada
por:
DT T T x k
T& ( x1 , x 2 , x 3 , t ) = = +
Dt t x k t
Considerando T = x1 + x 2 , obtenemos:

Universidad Castilla- La Mancha Draft Por: Eduardo W. V. Chaves (2012)


Ciudad Real - Espaa
178 PROBLEMAS RESUELTOS DE MECNICA DEL MEDIO CONTINUO

T T x1 T x 2 T x 3
T& ( x1 , x 2 , x 3 , t ) = + + +
t x1 t
{ x 2 {
t x 3 {
t
=0
{ =0 =0
=0

T& ( x1 , x 2 , x 3 , t ) = kX 2
Hallando la inversa de las ecuaciones del movimiento:
x1 = X 1 + ktX 2 X 1 = x1 ktx 2
inversa
x2 = X 2 X 2 = x 2
x = X X = x
3 3 3 3

T& ( x1 , x 2 , x 3 , t ) = kX 2 = kx 2
Para la partcula que en la configuracin actual pasa por el punto (1,1,1) :
T& ( x1 = 1, x 2 = 1, x 3 = 1, t ) = k

Ejemplo 2.29
Dado un campo de velocidad estacionario. Se pregunta: Las velocidades de las partculas
son constantes? Justificar la respuesta. En caso negativo, en situacin se cumple.
Solucin:
r
Un campo ( x , t ) se dice que es estacionario si la tasa local no vara con el tiempo:
r
( x , t ) r
=0 = ( x ) Campo estacionario (2.89)
t
Un campo de velocidad estacionario (movimiento estacionario) se indica en la Figura 2.3.
Luego, como se verifica en la Figura 2.3, la representacin del campo para los tiempos t1 y
t 2 no cambia. Pero, eso no implica que las velocidades de las partculas no estn
r
cambiando a lo largo del tiempo. Fijemos nuestra atencin en un punto espacial fijo x * . En
r r
el tiempo t1 la partcula Q est pasando por x * y tiene velocidad v * , consideremos
tambin una partcula P que est pasando por otro punto y que tiene velocidad tal que
r r r
v P (t1 ) v * . Para un tiempo t 2 la partcula P est pasando ahora por el punto x * y si el
r r
campo es estacionario, la velocidad de la partcula P en x * tiene que ser v * , i.e.
r r
v P (t 2 ) = v * . Esto se puede verificar fcilmente a travs de la derivada material de la
velocidad (que est asociada siempre con la misma partcula):
r r r r
Dv ( x , t ) r r v ( x , t ) r r r r r r r r
a ( x, t ) = + xr v v ( x , t ) = xr v v ( x ) = a ( x )
Dt 142t4 3 (2.90)
r
= 0 (Estaciona rio)

Para que la aceleracin de la partcula sea igual a cero, adems de campo de velocidad
r
estacionario, el campo de velocidad tambin tiene que ser homogneo, i.e. xr v = 0 .
Verifiquemos tambin que, aunque la velocidad espacial sea independiente del tiempo la
material no necesariamente lo ser, ya que:
r r r r r r r
v ( x ) = v ( x ( X , t )) = v ( X , t ) (2.91)

Universidad de Castilla- La Mancha Draft Por: Eduardo W. V. Chaves (2012)


Ciudad Real - Espaa
2 CINEMTICA DEL CONTINUO 179

t1 r r
v ( x)

r r r r
v ( x * , t1 ) = v * = v Q

Partcula - P Partcula - Q
r r
v P v*
r
x*

t2 r r
v ( x)

r r r r
v ( x * , t2 ) = v * = v P

Partcula - P

r
x*

Figura 2.3: Campo estacionario.

2.1.2 Tensores de Deformacin Finita, Deformacin


Homognea

Ejemplo 2.30
Una barra (considerada como un slido de una dimensin) sufre un estiramiento uniforme
de todos sus puntos dado por:
= exp at (2.92)
donde a = ctte .
Se pide:
r r r
a) Obtener las ecuaciones del movimiento x = x ( X , t ) ;
b) Obtener las componentes del tensor velocidad de deformacin D .

Universidad Castilla- La Mancha Draft Por: Eduardo W. V. Chaves (2012)


Ciudad Real - Espaa
180 PROBLEMAS RESUELTOS DE MECNICA DEL MEDIO CONTINUO

x1

= exp at

Figura 2.4.

Solucin:
Haciendo el planteamiento en 1D :
ds dx
= = = exp at dx = exp at dX (2.93)
dS dX

dx = exp
at
dX (2.94)

x1 = exp at X 1 + C (2.95)
para t = 0 x = X , luego
x = exp 0 X 1 + C X = X + C C = 0 (2.96)
Obtenemos as las ecuaciones del movimiento:
x1 = exp at X 1

x2 = X 2 (2.97)
x = X
3 3

El campo de velocidad:
dx1 at
v1 = dt = a X 1 exp = a x1

v 2 = 0 (2.98)
v = 0
3

Tensor velocidad de deformacin:
a 0 0
1 v v j
D ij = i + Dij = 0 0 0 (2.99)
2 x j xi
0 0 0

Ejemplo 2.31
Considrese la ley del movimiento dada por las siguientes expresiones:
x1 = X 1 + 2 X 3

x2 = X 2 2 X 3
x = X 2 X + 2 X
3 3 1 2

Universidad de Castilla- La Mancha Draft Por: Eduardo W. V. Chaves (2012)


Ciudad Real - Espaa
2 CINEMTICA DEL CONTINUO 181

Determinar las componentes del tensor de deformacin de Green-Lagrange ( E ).


Solucin 1:
Campo de desplazamiento
u1 = x1 X 1 = 2 X 3

u 2 = x 2 X 2 = 2 X 3
u = x X = 2 X + 2 X
3 3 3 1 2

Partiendo de las componentes del tensor de deformacin de Green-Lagrange en funcin


del gradiente material de los desplazamientos:
1 u i u j u k u k

E ij = + +
2 X j X i X i X j

1 u i u j 1 u k u k
+


= +
2 X j X i 2 X i X j



sym
u 1 u k u k
= i +
X j 2 X i X j

donde el gradiente material de los desplazamientos viene dados por:
u1 u1 u1

X 1 X 2 X 3 0 0 2
u i u 2 u 2 u 2
= = 0 0 2
X j X 1 X 2 X 3

u 3 u 3 u 3 2 2 0
X 1 X 2 X 3
Verifiquemos que el gradiente material de los desplazamientos es un tensor antisimtrico.
Es decir, la parte simtrica es el tensor nulo. Slo queda el trmino:
0 0 2 T 0 0 2 2 2 0
1 u u k 1
E ij = k = 0 0 2 0 0 2 = 2 2 0
2 X i X j 2
2 2 0 2 2 0 0 0 4

Solucin 2:
Podemos aplicar directamente la definicin:

E ij =
1
2
( 1
) (
C ij ij = Fki Fkj ij
2
)
donde:
x1 x1 x1

X 1 X 2 X 3 1 0 2
xi x 2 x 2 x 2
Fij = = = 0 1 2
X j X 1 X 2 X 3

x3 x3 x3 2 2 1
X 1 X 2 X 3
Luego:

Universidad Castilla- La Mancha Draft Por: Eduardo W. V. Chaves (2012)


Ciudad Real - Espaa
182 PROBLEMAS RESUELTOS DE MECNICA DEL MEDIO CONTINUO

1 0 2 T 1 0 2 1 0 0 2 2 0
1
E ij = 0 1 2 0 1 2 0 1 0 = 2 2 0
2
2 2 1 2 2 1 0 0 1 0 0 4

Ejemplo 2.32
Consideremos una transformacin homognea definida por las siguientes ecuaciones:
x1 = X 1 + 2 X 2 + X 3

x2 = 2 X 2 (2.100)
x = X + 2 X
3 1 3

Demostrar que para una transformacin homognea, vectores paralelos en la configuracin


de referencia, siguen paralelos tras la deformacin.
Para la demostracin considere dos partculas A y B cuyos vectores posicin en la
configuracin de referencia son:
r
X A = e 1 + e 2
r (2.101)
X B = 2e 1 + 2e 2 + e 3

Solucin:
El vector que une las dos partculas en la configuracin de referencia viene dado por:
r r r
V = B A = e 1 + e 2 + e 3 (2.102)
El gradiente de deformacin:
1 2 1
xi
Fij = = 0 2 0 (2.103)
X j
1 0 2

Podemos obtener los vectores posicin de las partculas en la configuracin actual:


r r r r
dx = F dX Transforma
cin homognea
x =FX (2.104)
luego,
1 2 1 1 3 1 2 1 2 7
xiA = 0 2 0 1 = 2 ; xiB = 0 2 0 2 = 4 (2.105)
1 0 2 0 1 1 0 2 1 4

y el vector que une estos dos puntos es:


r r r
v = x B x A = 4e 1 + 2e 2 + 3e 3 (2.106)
r
luego cualquier vector paralelo a V , por ejemplo 2e 1 + 2e 2 + 2e 3 , despus de la
r
transformacin: 8e 1 + 4e 2 + 6e 3 , es paralelo a v .

Universidad de Castilla- La Mancha Draft Por: Eduardo W. V. Chaves (2012)


Ciudad Real - Espaa
2 CINEMTICA DEL CONTINUO 183

Ejemplo 2.33
Considere una base ortonormal cartesiana e i y considere una deformacin de corte puro
representada por la deformacin homognea:
r r
x = X + k t X 2e1 (2.107)
o explcitamente:
x1 = X 1 + k t X 2

x2 = X 2 (2.108)
x = X
3 3

Obtener la forma geomtrica en la configuracin actual de la Figura 2.5 representada por


un rectngulo en la configuracin de referencia.
X2

B C

O A X1
Figura 2.5

Solucin:
El gradiente de deformacin:
1 k t 0
xi
Fij = = 0 1 0 (2.109)
X j
0 0 1
r r r r r
Verificamos que se trata de un caso de deformacin homognea, x = F X + c con c = 0 .
Determinante del Jacobiano:
J = F =1 (2.110)
Verificamos que para este caso no hay dilatancia.
Para la lnea BC , que tiene como coordenadas en la configuracin de referencia ( X 1 , X 2 ,0)
tenemos que:
x1 = X 1 + k t X 2

x2 = X 2 (2.111)
x = 0
3
Para la lnea OA , de coordenadas ( X 1 ,0,0) :
x1 = X 1

x2 = 0 (2.112)
x = 0
3

Universidad Castilla- La Mancha Draft Por: Eduardo W. V. Chaves (2012)


Ciudad Real - Espaa
184 PROBLEMAS RESUELTOS DE MECNICA DEL MEDIO CONTINUO

luego no se mueve. La deformacin final se puede apreciar en la Figura 2.6.

x2

B B C C

O A x1
Figura 2.6

Ejemplo 2.34
Sean las ecuaciones del movimiento:
2 2
x1 = X 1 + X2 ; x2 = X1 + X 2 ; x3 = X 3 (2.113)
2 2
Se pide:
a) Probar que esta deformacin es un ejemplo de transformacin homognea;
r
b) Determinar las componentes del campo de desplazamientos u en coordenadas
materiales y espaciales;
c) Determinar en la configuracin actual la figura geomtrica formada por las
partculas que en la configuracin de referencia formaban un crculo:
X 12 + X 22 = 2 X3 = 0
d) Obtener las componentes del tensor derecho de deformacin de Cauchy-Green
( C ) y del tensor de deformacin de Green-Lagrange ( E ).
e) Obtener los valores principales de los tensores C y E .
Solucin:
a) Una transformacin homognea si es el del tipo:
xi = Fij X j (2.114)
donde
2
1 0
2
xi 2
Fij = = 1 0 (2.115)
X j 2
0 0 1

Universidad de Castilla- La Mancha Draft Por: Eduardo W. V. Chaves (2012)


Ciudad Real - Espaa
2 CINEMTICA DEL CONTINUO 185

2
1 0
x1 2 X1
x = 2
2 1 0 X 2 (2.116)
2
x3 0 0 1 X 3

Comprobando que es una transformacin homognea. Su forma inversa se obtiene las


ecuaciones del movimiento en coordenadas espaciales:

X1 2 2 0 x1 X 1 = 2 x1 2 x 2
X = 2
2 2 0 x 2 X 2 = 2 x1 + 2 x 2 (2.117)
X 3 0 0 1 x3 X = x
3 3

b) El campo de desplazamientos viene dado por:


2 2
u1 = x1 X 1 = X 1 + X 2 X1 = X2
2 2
r r r 2 2
u= x X componente
s u 2 = x 2 X 2 = X1 + X 2 X 2 = X1 (2.118)
2 2
u 3 = x 3 X 3 = 0

En coordenadas espaciales queda:

(
u1 = x1 X 1 = x1 2 x1 2 x 2 = x1 + 2 x 2 )
(
u 2 = x 2 X 2 = x 2 2 x1 + 2 x 2 = 2 x1 x 2 ) (2.119)
u 3 = x 3 X 3 = x3 x3 = 0

c) Dada la ecuacin de las partculas que en la configuracin de referencia formaba un


crculo:
X 12 + X 22 = 2 ; X3 = 0 (2.120)
En la configuracin actual queda:

(2 x
1 2 x2 ) + (
2
2 x1 + 2 x 2 ) 2
=2 (2.121)

Desarrollando obtenemos que:


3x12 + 3 x 22 4 2 x1 x 2 = 1 (ecuacin de una elipse) (2.122)
Ver Figura 2.7. Es interesante que el lector verifique las posiciones de las partculas
P ( X 1 = 0; X 2 = 2 ; X 3 = 0) y Q( X 1 = 2 ; X 2 = 2 ; X 3 = 0) en la configuracin deformada.

Universidad Castilla- La Mancha Draft Por: Eduardo W. V. Chaves (2012)


Ciudad Real - Espaa
186 PROBLEMAS RESUELTOS DE MECNICA DEL MEDIO CONTINUO

1,5
Conf. de Referencia
Conf. Deformada
1

0,5

0
x2

-2 -1 0 1 2

-0,5

-1

-1,5

-2
x1

Figura 2.7: Curva material

d) El tensor derecho de deformacin de Cauchy-Green y el tensor de deformacin de


Green-Lagrange vienen dados, respectivamente, por:
1
C = FT F ; E= (C 1) (2.123)
2
Luego las componentes de C son:
2 2
1 0 1 0 3
2

0
2 2
2

2 2 3
C ij = 1 0 1 0 = 2 0 (2.124)
2 2 2
0 0 1 0 0 1 0 0 1

Obteniendo los autovalores de (2.124). Ya conocemos un autovalor C 3 = 1 . Para obtener


los otros autovalores es suficiente resolver:

3 3+ 2 2
C 2 C1 = 2,91421
2 2 1 2
=0 C 3C + = 0 (2.125)
3 4 32 2
2 E
2 C 2 = 2
0,08579

Resumiendo:
3 3
C1 = + 2 ; C 2 = 2 ; C3 = 1 (2.126)
2 2

Universidad de Castilla- La Mancha Draft Por: Eduardo W. V. Chaves (2012)


Ciudad Real - Espaa
2 CINEMTICA DEL CONTINUO 187

Las componentes del tensor E son:


3
2 0
2 1 0 0 1 2 2 0
1 1
1
(
E ij = C ij ij ) =
2
2
3
0 0 1 0 = 2 2 1 0 (2.127)
2 2 4
0 0 1 0 0 1 0 0 0



Los valores principales de E son obtenidos por:

1 2 1+ 2 2
E E1 = 0,95711
4 2 2 E 7 4
=0 E =0 (2.128)
2 1 2 16 1 2 2
E
2 4 E 2 = 4
0,45711

Luego, los valores principales de E son:


1+ 2 2 1 2 2
E1 = ; E2 = ; E3 = 0 (2.129)
4 4
Solucin Alternativa:
Recordar que los tensores C y E son coaxiales, es decir, podemos trabajar en el espacio
principal para obtener los autovalores de E :
3 + 2 2 1 + 2 2
0 0 0 0
2 1 0 0 4
1 32 2 1 2 2
E ij =
1
2
(
C ij ij ) = 0
2 2

0 0 1 0 = 0
4

0 (2.130)

0 0 1 0 0 1 0 0 0


Ejemplo 2.35
Consideremos las siguientes ecuaciones del movimiento:
11
x1 = X 1 + X1 + X 2
X2 ; ; x2 =
x3 = X 3 (2.131)
22
r
a) Obtener el campo de desplazamiento ( u ) en las descripciones Lagrangiana y Euleriana;
b) Determinar la curva material en la configuracin actual de una circunferencia material
definido en la configuracin de referencia como:
X 12 + X 22 = 2 X3 = 0
c) Obtener las componentes del tensor derecho de deformacin de Cauchy-Green y el
tensor de deformacin de Green-Lagrange;
d) Obtener los estiramientos principales.
Solucin:
El gradiente de deformacin viene dado por:

Universidad Castilla- La Mancha Draft Por: Eduardo W. V. Chaves (2012)


Ciudad Real - Espaa
188 PROBLEMAS RESUELTOS DE MECNICA DEL MEDIO CONTINUO

2 1 0
xi 1
Fij = = 1 2 0 ; J = F = 0.75
X j 2
0 0 2

y combinando con las ecuaciones del movimiento dada en (2.131), obtenemos que:
x1 2 1 0 X 1
x = 1 1 2 0 X ; x i = Fij X j
2 2 2
x3 0 0 2 X 3

Con eso, podemos verificar que el ejemplo propuesto es un caso de deformacin


r r
homognea con c = 0 . La forma inversa de la ecuacin anterior es:
4 2
X 1 = 3 x1 3 x 2
X1 4 2 0 x1
X = 1 2 4 0 x 2 4
2 3 2 X 2 = x1 + x 2 (2.132)
3 3
X 3 0 0 3 x3
X 3 = x3


r r r
El campo de desplazamiento viene definido por u = x X , con lo cual las componentes
del desplazamiento Lagrangiano resulta ser:
r 1 1
u1 ( X , t ) = x1 X 1 = X 1 + 2 X 2 X 1 = 2 X 2
r
1 1
u i = xi X i u 2 ( X , t ) = x 2 X 2 = X 1 + X 2 X 2 = X 1 (2.133)
2 2
r
u 3 ( X , t ) = x3 X 3 = 0


Las componentes del desplazamiento Euleriana pueden ser obtenidas por reemplazar las
ecuaciones del movimiento Euleriano (2.132) en (2.133), luego:
r r 1 r 1 2 4 r
u1 ( X ( x, t ), t ) = X 2 ( x , t ) = x1 + x 2 = u1 ( x , t )
2 2 3 3
r r 1 r 1 2 4 r
u 2 ( X ( x , t ), t ) = X 1 ( x , t ) = x1 + x 2 = u 2 ( x , t ) (2.134)
2 2 3 3
r r
u ( X ( x , t ), t ) = u ( xr , t ) = 0
3 3

Las partculas que pertenecen a la circunferencia X 12 + X 22 = 2 en la configuracin de


referencia, formarn una nueva curva en la configuracin actual y que viene definida por:
2 2
4 2 2 4
X 12 + X 22 = 2 x1 x 2 + x1 + x 2 = 2 20 x12 32 x1 x 2 + 20 x 22 = 18
3 3 3 3
el cual es la ecuacin de una elipse (Figura 2.8 muestra la curva material en las diferentes
configuraciones).
Las componentes de C y E pueden ser obtenidas a travs de las definiciones C = F T F
1
y E= (C 1) :
2

Universidad de Castilla- La Mancha Draft Por: Eduardo W. V. Chaves (2012)


Ciudad Real - Espaa
2 CINEMTICA DEL CONTINUO 189

2 1 0 2 1 0 1.25 1 0
1
C ij = Fki Fkj C ij = 1 2 0 1 2 0 = 1 1.25 0
4
0 0 2 0 0 2 0 0 1

1.25 1 0 1 0 0 0.125 0.5 0


1
1
(
E ij = C ij ij
2
) E ij = 1 1.25 0 0 1 0 = 0.5 0.125 0
2
0 0 1 0 0 1 0 0 0

En el espacio principal de C sus componentes vienen dadas por:


21 0 0 1 0 0

C ij = 0 22 0 C ij = 0 2 0
0 0 23 0 0 3

donde i son los estiramientos principales. A continuacin, calculamos los autovalores de
C:
1.25 C 1 C1 = 2.25
= 0 C 2 2.5C + 0.5625 = 0
1 1.25 C C 2 = 0.25
21 0 0 2.25 0 0 1 0 0 1.5 0 0

C ij = 0 22 0= 0 0.25 0 0 2 0 = 0 0.5 0
0 0 23 0 0 1 0 0 3 0 0 1

2.0
curva material
1.5
Reference Conf.
Current Conf.
1.0

0.5

0.0
x2

-2 -1 0 1 2
-0.5

-1.0

-1.5

-2.0
x1

Figura 2.8: Curva material.

Universidad Castilla- La Mancha Draft Por: Eduardo W. V. Chaves (2012)


Ciudad Real - Espaa
190 PROBLEMAS RESUELTOS DE MECNICA DEL MEDIO CONTINUO

Ejemplo 2.36
Probar que

[(detF ) F T ] = 0
r
Xr (2.135)
r r r
Datos: Relacin de Nanson da = J F T dA , o da = da n = J F T N
dA .

Solucin:
Considerando la relacin de Nanson en notacin indicial da n i = J Fki1N k dA , donde
J = detF . Podemos integrar en toda la superficie:
1
n da = J F
S
i
S0
ki N k dA (2.136)

Fijemos que dado una funcin escalar f , se cumple que:


f
n
S
i f da =
V
f ,i dV =
V
xi
dV

Haciendo f = 1 , obtenemos que:

n
S
i da = 0 i

Retomando la ecuacin (2.136), y aplicando el teorema de la divergencia de Gauss para la


segunda integral, obtenemos que:

(J F ), ( )

n da = 0 = J F
1

1
i i ki N k dA = ki k dV0 = J Fki1 dV0 = 0 i
S S0 V0 V0
X k
(2.137)
[(detF ) F T ] dV0 = 0
r

V0
Xr

Luego si es vlido para todo el volumen, tiene que ser vlido localmente:

[(detF ) F T ] = 0
r
Xr (2.138)

Ejemplo 2.37

[
Demostrar que a) E& = F T Xr u& ( X , t)
r r
]sym
[
y b) D = xr u& ( x , t) , donde E es el tensor de
r r
]sym

deformacin de Green-Lagrange y D es el tensor velocidad de deformacin.


Solucin:
a)

E&
D
Dt
E=
D 1 T

Dt 2
( 1
) 1
(
F F 1 = F& T F + F T F& = ( F T F& ) T + ( F T F& ) = F T F& ) [ ] [ ]sym

2 2
Notar que:

F&ij =
D xi ( X , t )
Dt X j X j
=
Dxi ( X , t )
Dt
=
&
X j
[ u& ( X , t )
u i ( X , t )] = i
X j
r r
= Xr u& ( X , t ) ij ( )
con lo cual, demostramos que:

Universidad de Castilla- La Mancha Draft Por: Eduardo W. V. Chaves (2012)


Ciudad Real - Espaa
2 CINEMTICA DEL CONTINUO 191

E&
D
Dt
[
E = F T F& ]
sym
[ r r sym
]
= F T Xr u& ( X , t )

b)

D=l
1
2
= [T 1
2
] [
r r T r r
l + ( l ) = xr v + ( xr v ) = ( xr v ( x , t ))
sym
]
sym r r sym
[
= xr u& ( x , t ) ]
r r r r
donde hemos considerado que v ( x , t ) = u& ( x , t ) .

Ejemplo 2.38
Considrese el siguiente campo de velocidad:
v1 = 5 x 2 + 2 x3

v 2 = 5 x1 3 x3
v = 2 x + 3 x
3 1 2

Demostrar que dicho movimiento corresponde a un movimiento de slido rgido.


Solucin:
En primero vamos obtener el gradiente espacial de la velocidad (l ), cuyas componentes
vienen dadas por:
v1 v1 v1

r x1 x 2 x3 0 5 2
vi ( x, t ) v 2 v 2 v 2
l ij = = = 5 0 3 (2.139)
x j x x 2 x3
1 0

v3 v3 v3 2 3
x1 x 2 x3

Recordar que ( l ) podemos descomponer en una parte simtrica ( D ) y otra antisimtrica


( W ). Tambin podemos verificar que l = D + W = W . Ya que D = 0 , el movimiento es de
slido rgido.
Ejemplo 2.39
Consideremos el siguiente campo de velocidad:
v1 = 3 x 2 + 1x 3

v 2 = 3 x1 5 x3
v = 1x + 5 x
3 1 2

Demostrar que el movimiento correspondiente se trata de un movimiento de slido rgido.


Solucin: Calculamos las componentes del gradiente espacial de velocidad (l ):
v1 v1 v1

r x1 x 2 x3 0 3 1
vi ( x , t ) v 2 v 2 v 2
l ij = = = 3 0 5 = l ijskew
x j x x 2 x3
1 0
v3 v 3 v3 1 5
x1 x 2 x3

Universidad Castilla- La Mancha Draft Por: Eduardo W. V. Chaves (2012)


Ciudad Real - Espaa
192 PROBLEMAS RESUELTOS DE MECNICA DEL MEDIO CONTINUO

Teniendo en cuenta que l puede ser descompuesto de forma aditiva en una parte
simtrica ( l sym D ) y una antisimtrica ( l skew W ), i.e. l = D + W , podemos que
concluir que D = 0 , el cual caracteriza un movimiento de slido rgido.

Ejemplo 2.40
El campo de desplazamientos de un cuerpo viene descrito por las siguientes ecuaciones:
u1 = 3 X 12 + X 2
2
u 2 = 2 X 2 + X 3
2
u 3 = 4 X 3 + X 1
r
Determinar el vector dx (configuracin actual) cuyo vector en la configuracin de
r
referencia estaba representado por dX y pasaba por el punto P(1,1,1) .
X 3 , x3
dX 1
dX k = dX 2
Q dX 3
r
dX
P
X 2 , x2

X 1 , x1

Solucin:
r
Para determinar el vector dx necesitamos obtener el gradiente de deformacin F . Las
componentes del gradiente de deformacin material pueden obtenerse utilizando
directamente la ecuacin:
u i
Fij = ij +
X j

1 + 6 X 1 1 0

Fij = 0 1 + 4X 2 1

1 0 1 + 8 X 3

Las componentes del gradiente de deformacin en el punto P(1,1,1) son:


7 1 0
Fij = 0 5 1
P
1 0 9
r
Una vez obtenido el gradiente de deformacin F , las componentes del vector dx vienen
dadas por:
dx i = Fij dX j

dx1 7 1 0 dX 1 7 dX 1 + dX 2
dx = 0 5 1 dX = 5dX + dX
2 2 2 3
dx 3 1 0 9 dX 3 dX 1 + 9 dX 3

Universidad de Castilla- La Mancha Draft Por: Eduardo W. V. Chaves (2012)


Ciudad Real - Espaa
2 CINEMTICA DEL CONTINUO 193

Ejemplo 2.41
Dadas las componentes del campo de desplazamientos siguientes:
u1 = 2 X 12 + X 1 X 2
2
u 2 = X 2 para X 1 0; X 2 0
u = 0
3

Se pide:
a) Encontrar el vector en la configuracin de referencia cuyo vector en la configuracin

actual es dx i = (OP ) i = (1,0,0) ;
b) Encontrar el estiramiento de un elemento de lnea que en la configuracin actual es el

vector dx i = (OP ) i = (1,0,0) y que pasa por el punto P(1,0,0) .
Solucin:
a) Dadas las componentes del desplazamiento podemos obtener las componentes del
movimiento segn la ecuacin:
u i = xi X i

x1 = u1 + X 1 x1 = X 1 + 2 X 12 + X 1 X 2

x2 = u 2 + X 2 sustituyen
do
x 2 = X 2 + X 22
valores de u1 ,u 2 ,u 3
x = u + X x = X
3 3 3 3 3

Podemos verificar que no se trata de una deformacin homognea, ya que una recta en la
configuracin de referencia no sigue siendo una recta en la configuracin deformada.
Como ejemplo consideremos que unas partculas que ocupan una recta en la configuracin
de referencia, tras la deformada estas partcula ya no formarn una recta en la configuracin
actual, ver Figura 2.9.

2,5

2
P

1,5
x2

P
1

Q Conf. Actual
Q
0,5 Conf. Referencia

0
0 0,5 1 1,5 2 2,5 3 3,5 4 4,5
x1

Figura 2.9: Deformacin de la recta (1,1).

Universidad Castilla- La Mancha Draft Por: Eduardo W. V. Chaves (2012)


Ciudad Real - Espaa
194 PROBLEMAS RESUELTOS DE MECNICA DEL MEDIO CONTINUO

Reemplazando el punto P ( x1 = 1, x 2 = 0, x3 = 0) en las ecuaciones del movimiento anterior,


resulta:
1 = X 1 + 2 X 12 + X 1 X 2
2
0 = X 2 + X 2
0 = X
3

Podemos decir de inmediato que X 3 = 0 y X 22 = X 2 , debido a la restriccin del problema


X 2 0 , luego la nica solucin posible es X 2 = 0 . Sustituyendo los valores de X 3 = 0 y
X 2 = 0 en la primera ecuacin resulta:

resolviend o
X 1 = 1
1 = X 1 + 2 X 12
X 1 = 12

Debido a la restriccin X 1 0 , la nica solucin posible es X 1 = 12 . As:


( X 1 = 12 ; X 2 = 0; X 3 = 0)
Es decir, la partcula que en la configuracin actual ocupa ( x1 = 1, x 2 = 0, x 3 = 0) , en la
configuracin de referencia ocupaba el lugar ( X 1 = 12 ; X 2 = 0; X 3 = 0;) .
r
Calculemos el incremento en esta direccin dX , segn la ecuacin dx j = F jk dX k y su
forma inversa:
dX 1 dx1
dX = F 1 dx
2 jk 2
dX 3 dx 3

Para lo cual debemos calcular antes las componentes del gradiente de deformacin material
F jk . Podemos hacerlo directamente partiendo de la definicin:

x1 x1 x1

X 1 X 2 X 3 (1 + 4 X + X )
1 2 X1 0
x x 2 x 2
F jk = 2 = 0 1 + 2X 2 0
X X 2 X 3
1 0 0 1
x 3 x 3 x 3
X 1 X 2 X 3

Para el punto P( X 1 = 12 ; X 2 = 0; X 3 = 0;) obtenemos las componentes del gradiente de


deformacin:
3 12 0

F jk = 0 1 0
P
0 0 1

Calculamos la inversa ( F jk ) 1 F jk1 . Por definicin:
T
1 0 0 1 0,5 0
1 1 1
F jk1 = adj( F jk ) = 0,5 3 0 = 0 3 0
F jk 3 3
0 0 3 0 0 3

Luego:

Universidad de Castilla- La Mancha Draft Por: Eduardo W. V. Chaves (2012)


Ciudad Real - Espaa
2 CINEMTICA DEL CONTINUO 195

dX 1 dx1 1 0,5 0 1 13
dX = F 1 dx = 1 0
2 jk 2 3 0 0 = 0
3
dX 3 dx 3 0 0 3 0 0

b) El estiramiento viene dado por:


r
dx 12 + 0 + 0
= r = =3
dX (13 )2 + 0 + 0

0,1
0,08
0,06 Conf. Actual
x2

0,04 Conf. Referencia


0,02
0
0 0,5 1 1,5 2 2,5 3 3,5
x1

Figura 2.10: Deformacin de la recta OT .

Ejemplo 2.42
DF
Utilizar la definicin
D
Dt
[det (F )] = ij cof Fij
Dt
( ) para obtener la expresin
D
[det (F )] = Jv i ,i .
Dt
x i
Solucin: Considerando que Fij = , luego:
X j


D
[det (F )] = D xi cof Fij ( )
Dt Dt X j
D x i
=
X j Dt
cof Fij ( )
=
D
X j
(vi )cof Fij( )
r
o an considerando que v i ( x ( X , t ), t ) , podemos decir que:
D
[det (F )] = vi x k cof Fij ( )
Dt x k X j

Y considerando la definicin del cofactor: [cof (Fij )]T = (Fij )1 det (Fij ) , obtenemos que:
D
[det (F )] = vi xk Fij ( ) T
( )
det Fij =
vi
( )
Fkj F ji
1
( )
det Fij =
vi
( ) v
( )
ki det Fij = i det Fij
Dt xk X j x k x k xi
= Jvi ,i

Solucin alternativa: ver Ejemplo 1.111 en el captulo 1.

Universidad Castilla- La Mancha Draft Por: Eduardo W. V. Chaves (2012)


Ciudad Real - Espaa
196 PROBLEMAS RESUELTOS DE MECNICA DEL MEDIO CONTINUO

Ejemplo 2.43
r
Dado el diferencial dx , hallar su derivada material.
Solucin:
D r D r D r D r r
dx = ( F dX ) = ( F ) dX + F ( dX ) = l 1 d3
F2 X
Dt Dt Dt Dt
1
424 3 r
dx
r
0
r r r
= l dx xr v dx
Las componentes vienen dadas por:
D r v
dx = v i , k dx k = i dx k
Dt i x k

Ejemplo 2.44
Considerando las ecuaciones del movimiento:
x1 = X 1 + 4 X 1 X 2
2
x2 = X 2 + X 2
2
x3 = X 3 + X 3
Encontrar el tensor de deformacin de Green-Lagrange E .
Solucin:
El tensor de deformacin de Green-Lagrange viene dado por:
1 T 1
E= ( F F 1) Eij = ( Fki Fkj ij ) (2.140)
2 2

Considerando las ecuaciones del movimiento podemos obtener las componentes del
gradiente de deformacin material F :
x1 x1 x1

X 1 X 2 X 3 (1 + 4 X )
2 4X1 0
x k x 2 x 2 x 2
Fkj = = = 0 1 + 2X 2 0
X j X 1 X 2 X 3
0 0 1 + 2 X 3
x 3 x 3 x 3
X 1 X 2 X 3

(1 + 4 X 2 ) 0 0 (1 + 4 X 2 ) 4X1 0
Fki Fkj
= 4X1 1 + 2X 2 0 0 1 + 2X 2 0
0 0
1 + 2X 3 0 0 1 + 2 X 3
(1 + 4 X 2 ) 2 (1 + 4 X 2 ) 4 X 1 0

= (1 + 4 X 2 ) 4 X 1 ( 4 X 1 ) 2 + (1 + 2 X 2 ) 2 0
0 0 (1 + 2 X ) 2
3
Reemplazando la relacin anterior en la ecuacin (2.140) obtenemos que las componentes
del tensor de deformacin de Green-Lagrange vienen dadas por:

Universidad de Castilla- La Mancha Draft Por: Eduardo W. V. Chaves (2012)


Ciudad Real - Espaa
2 CINEMTICA DEL CONTINUO 197

(1 + 4 X 2 ) 2 1 (1 + 4 X 2 ) 4 X 1 0
1 2 2
E ij = (1 + 4 X 2 ) 4 X 1 ( 4 X 1 ) + (1 + 2 X 2 ) 1 0
2
0 0 (1 + 2 X 3 ) 1
2

Ejemplo 2.45
Obtener los invariantes principales de E en funcin de los invariantes principales de C y
b.
Solucin:
Los invariantes principales de E son:

I E = Tr ( E ) ; II E =
1 2
2
[
I E Tr ( E 2 ) ] ; III E = det ( E )

1
Considerando que E = (C 1) , resulta que:
2
Primer Invariante:
1 1 1 1
I E = Tr ( E ) = (C 1) = Tr (C 1) = [Tr (C ) Tr (1) ] = (I C 3)
2 2 2 2
Segundo Invariante:

II E =
1 2
2
[
I E Tr ( E 2 ) ]
donde
2
1 1
(
I E2 = (I C 3) = I C2 6 I C + 9 )
2 4
2
1 1
[ 1
Tr ( E 2 ) = Tr (C 1) = Tr (C 1) 2 = Tr C 2 2C + 1 ] ( )
2 4 4

[ ( )
1
= Tr C 2 2 Tr (C ) + Tr (1 )
4
]
[ ( )
1
= Tr C 2 2 I C + 3
4
]
Para obtener Tr (C 2 ) , adoptaremos el espacio de las direcciones principales donde se
cumple que:
C12 0 0

C C = C 2 = 0 C 22

( )
0 Tr C 2 = C12 + C 22 + C 32
0 0 C 32

Pero considerando la siguiente relacin:
2
1444424444
(
I C2 = (C1 + C 2 + C 3 ) = C12 + C 22 + C 32 + 2 C1 C 2 + C1 C 3 + C 2 C 3
3
)
II C

C12 + C 22 + C 32 = I C2 2 II C
Luego:

Universidad Castilla- La Mancha Draft Por: Eduardo W. V. Chaves (2012)


Ciudad Real - Espaa
198 PROBLEMAS RESUELTOS DE MECNICA DEL MEDIO CONTINUO

Tr ( E 2 ) =
4
(
1 2
I C 2 II C 2 I C + 3 )
Con lo cual concluimos que el segundo invariante viene dado por:
1 1 2
II E =
2 4
( 1
) (
I C 6 I C + 9 I C2 2 II C 2 I C + 3
4
)

1
= ( 2 I C + II C + 3)
4
Tercer Invariante:
3
1 1
III E = det ( E ) = det (C 1) = det [(C 1)]
2 2
Trabajando en las direcciones principales tenemos que:
C1 1 0 0
det (C 1) = 0 C2 1 0 = (C1 1)(C 2 1)(C 3 1)
0 0 C3 1
= C1 C 2 C 3 C 1 C 2 C1 C 3 C 2 C 3 + C 1 + C 2 + C 3 1
= III C II C + I C 1
luego:
1
III E = ( III C II C + I C 1)
8
Resumiendo:
1
IE = (I C 3 ) I C = 2I E + 3
2
1
II E = ( 2 I C + II C + 3) INVERSA
II C = 4 II E + 4 I E + 3
4
1
III E = ( III C II C + I C 1) III C = 8 III E + 4 II E + 2 I E + 1
8

Ejemplo 2.46
Sea = (I C , II C , III C ) una funcin de valor-escalar, donde I C , II C , III C son los
invariantes principales del tensor derecho de deformacin de Cauchy-Green C . Obtener la
derivada de con respecto a C y con respecto a b . Comprobar que la siguiente igualdad
es vlida F ,C F T = ,b b .
Solucin:
Utilizando la regla de la cadena podemos obtener que:
(I C , II C , III C ) I C II C III C
,C = = + + (2.141)
C I C C II C C III C C

Considerando las derivadas parciales de los invariantes vistas en el captulo 1, podemos


decir que:
I C II C III C
=1 , = IC 1 C T = IC 1 C , = III C C T = III C C 1 , luego:
C C C

Universidad de Castilla- La Mancha Draft Por: Eduardo W. V. Chaves (2012)


Ciudad Real - Espaa
2 CINEMTICA DEL CONTINUO 199


,C = 1+ (I C 1 C ) + III C C 1
I C II C III C
(2.142)

,C = + I C 1 C+ III C C 1

C I II C II C III C

Tambin es vlido que:


,b = + I b 1 b+ III b b 1 (2.143)

b I II b II b III b

Haciendo una contraccin por la izquierda con F y por la derecha por F T en la relacin
(2.142) obtenemos que:

F ,C F T = + I C F 1 F T F C F T + III C F C 1 F T

CI II C II C III C

(2.144)
Y considerando las siguientes relaciones:
F 1 F T = F F T = b
C = F T F F C F T = F F T F F T = b b = b2
Y considerando la relacin C 1 = F 1 b 1 F concluimos que:
C 1 = F 1 b 1 F F C 1 F T = F F 1 b 1 F F T = b 1 b
Luego la expresin (2.144) puede ser rescrita como:
2
F ,C F T = + I C b b + III C b 1 b
I C II C II C III C


F ,C F T = + I C 1 b+ III C b 1 b
I C II C II C III C
Tambin es vlido que:

F ,C F T = + I b 1 B+ III b b 1 b
I b II b II b III b

F ,C F T = ,b b c.q.d.

Verificando la expresin (2.143) podemos concluir que la relacin ,b b = b ,b es vlida,


indicando que los tensores ,b y b son coaxiales.

Ejemplo 2.47
Demostrar que el tensor de deformacin de Green-Lagrange ( E ) y el tensor derecho de
deformacin de Cauchy-Green ( C ) son tensores coaxiales.
Solucin:
Dos tensores son coaxiales cuando presentan las mismas direcciones principales. Tambin
se puede demostrar que son coaxiales cuando se cumpla la relacin:

Universidad Castilla- La Mancha Draft Por: Eduardo W. V. Chaves (2012)


Ciudad Real - Espaa
200 PROBLEMAS RESUELTOS DE MECNICA DEL MEDIO CONTINUO

C E = E C
Partiendo de la definicin C = 1 + 2 E concluimos que:
C E = (1 + 2 E ) E = 1 E + 2 E E = E 1 + 2 E E = E (1 + 2 E ) = E C
Con lo cual se demuestra que los tensores E y C son tensores coaxiales.

Ejemplo 2.48
r r
Obtener la relacin E& = F T D F , partiendo de la definicin (ds ) 2 (dS ) 2 = dX 2 E dX .
Obtener tambin la relacin entre
D
Dt
[
(ds ) 2 y D . ]
Solucin:
r r
Tomando la derivada material de la relacin (ds ) 2 (dS ) 2 = dX 2 E dX :
D
Dt
[
(ds ) 2 (dS ) 2 ] =
D
Dt
[
(ds ) 2 ] =
D r
Dt
[ r
dX 2 E dX ]
D r r r& r r r r r&
= [dx dx ] = 2d{ X E dX + 2dX E& dX + 2dX E d{X
Dt =0 =0
r D r r r
= 2 dx [dx ] = 2dX E& dX
Dt
D r
El trmino [dx ] , puede expresarse de la siguiente forma:
Dt
D D x k
[dx k ] = dX i
D r D
[
Dt [dx ] = Dt F dX
r
] Dt

Dt X i
r D x k D x k
= F& dX Indicial

= dX i = dX i
r Dt X i DX i t
= l F dX
v
= k dX i
X i

Con lo cual podemos decir que:


r r r D r
2dX E& dX = 2dx [dx ]
r Dt r
= 2dx l F dX
r r
= 2 F dX l F dX
r r
= 2dX F T l F dX

Podemos descomponer de forma aditiva el tensor gradiente espacial de velocidad ( l ) en


una parte simtrica ( D ) y otra antisimtrica ( W ):
r r r r
2dX E& dX
= 2 dX F T l F dX
r r
= 2dX F T (D + W) F dX
r r r r
= 2 dX F T D F dX + 2 dX F T W F dX
r r
= 2 dX F T D F dX
r r r r r r
Observemos que dX F T W F dX = dx W dx = W : (dx dx ) = 0 , ya que el tensor W
r r
es antisimtrico y (dx dx ) un tensor simtrico. Con lo que concluimos que:
E& = F T D F

Universidad de Castilla- La Mancha Draft Por: Eduardo W. V. Chaves (2012)


Ciudad Real - Espaa
2 CINEMTICA DEL CONTINUO 201

Con lo cual la relacin entre


D
Dt
[
(ds ) 2 y D queda: ]
D
Dt
[ ] r r r r
(ds ) 2 = 2dX F T D F dX = 2dx D dx

Ejemplo 2.49
Obtener la tasa del determinante del Jacobiano ( J& ) en funcin de la tasa del tensor de
deformacin de Green-Lagrange ( E& ) y tambin en funcin de la tasa del tensor derecho
de deformacin de Cauchy-Green ( C& ).
Solucin:
Considerando que J& = J Tr (D ) , donde D es el tensor tasa de deformacin, y est
relacionado con E& por D = F T E& F 1 , luego:
( ) (
J& = J Tr (D) = J Tr F T E& F 1 = J F T E& F 1 : 1 )
En notacin indicial queda:
J& = J Fki1 E& kp F pj1 ij = J Fki1 F pi1 E& kp
= J F 1 F T : E& = J C 1 : E&
J
= C 1 : C&
2
An podemos expresar J& en funcin de F& , para ello consideremos la siguiente relacin
1
( )
E& kp = F&sk Fsp + Fsk F&sp . Luego J& an puede se expresado por:
2
1
(
J& = J Fki1 F pi1 E& kp = J Fki1 F pi1 F&sk Fsp + Fsk F&sp =
2
)
J 1 1 &
2
(
Fki F pi Fsk Fsp + Fki1 F pi1 Fsk F&sp )
J
(
= si Fki1 F&sk + si F pi1 F&sp =
2
) J
2
( )
Fks1 F&sk + F ps1 F&sp = JFts1 F&st = JF T : F&

Resumiendo, podemos expresar la tasa del determinante del Jacobiano como:


J 1 &
J& = J Tr (D) = J C 1 : E& C :C = = JF T : F&
2
J
= J Tr (C 1 E& ) = Tr (C 1 C& ) = J Tr ( F& F 1 )
2

Ejemplo 2.50
Las componentes del campo de desplazamiento de un medio continuo son:
u1 = 0,1 X 22

u 2 = 0
u = 0
3
Se pide:
a) Es una deformacin posible en un cuerpo continuamente deformable? Justifique su
respuesta;
b) Determinar el tensor derecho de deformacin de Cauchy-Green;

Universidad Castilla- La Mancha Draft Por: Eduardo W. V. Chaves (2012)


Ciudad Real - Espaa
202 PROBLEMAS RESUELTOS DE MECNICA DEL MEDIO CONTINUO

r
c) Encontrar los vectores deformados relativos a los vectores materiales b = 0,01e 1 y
r
c = 0,015 e 2 , los cuales pasaban por el punto P (1,1,0) en la configuracin de referencia;
r r
d) Determinar los estiramientos de estos vectores, b y c , en el punto P (1,1,0) ;
e) Determinar el cambio sufrido por el ngulo comprendido entre los dos vectores.
Solucin:
a) Para que un movimiento sea posible, el determinante del Jacobiano tiene que ser
positivo. El gradiente de deformacin material viene dado por:
1 0 0 0 0,2 X 2 0 1 0,2 X 2 0
u i
Fij = ij + = 0 1 0 + 0 0 0 = 0 1 0
X j
0 0 1 0 0 0 0 0 1

Calculando el determinante: Fij = J = 1 > 0 . Luego es un movimiento posible.

b) El tensor derecho de deformacin de Cauchy-Green viene definido por C = F T F ,


luego las componentes viene dadas por:
1 0 0 1 0,2 X 2 0 1 0,2 X 2 0
C ij = 0,2 X 2 1 0 0 1 0 = 0,2 X 2 0,2 X 2 + 1 0
2 2

0 0 1 0 0 1 0 0 1
r
c) El vector b = 0,01e 1 en el punto P(1,1,0) se deforma segn al criterio:
r r
b = F P
b
b1 1 0,2 1 0 0,01 0,01
b = 0 1 0 0 = 0
2
b 3 0 0 1 0 0
r
y el vector c = 0,015e 2 en la configuracin actual queda:
c 1 1 0,2 1 0 0 0,003
c = 0 1 0 0,015 = 0,015
2
c 3 0 0 1 0 0

d) Para obtener el estiramiento, utilizamos directamente la ecuacin:


r
b 0,01 2
br = r = =1
b 0,01
r
El estiramiento del vector c viene dado por:
r
c 0,003 2 + 0,015 2
cr = r = = 1,0198 1,02
c 0,015

Solucin Alternativa: Teniendo en cuenta que M = M C M y evaluando C en el punto


P obtenemos que:

Universidad de Castilla- La Mancha Draft Por: Eduardo W. V. Chaves (2012)


Ciudad Real - Espaa
2 CINEMTICA DEL CONTINUO 203

1 0,2 X 20 1 0,2 0
C ij ( X 1 = 1, X 2 = 1, X 3 = 0) = 0,2 X 2 0,2 2
+ 1 0
X 22 = 0,2 1,04 0
0 0 1 0 0 1
P

Luego aplicando b = b C b y c = c C c , podemos obtener que:

1 0, 2 0 1
= [1 0 0] 0,2 1,04 0 0 = 1

2
b b = 1
0 0 1 0

1 0, 2 0 0
= [0 1 0] 0,2 1,04 0 1 = 1,04

2
c c = 1,0198
0 0 1 0
r r
e) En la configuracin actual el ngulo que forman los vectores, b y c , puede obtenerse
segn la relacin:
r r
b c
cos = r r
b c

(0,01e 1 + 0e 2 + 0e 3 ) (0,003e 1 + 0,015 e 2 + 0e 3 ) 0,00003


cos = = = 0,196116135
0,01 2 2
0,003 + 0,015 2 0,01 0,000234

= arccos(0,196116135) 78,69
Estos dos vectores, en la configuracin de referencia, formaban un ngulo de 90 , luego el
cambio de ngulo ser:
= 90 78,69 = 11,3
Solucin Alternativa: Dadas dos direcciones en la configuracin de referencia representadas
por sus versores M y N , el ngulo formado por estos versores en la configuracin actual
(tras la deformada) viene dado por:
M C N M C N
cos = =
M C M N C N M N

Haciendo M = b , N = c se cumple que:


1 0, 2 0 0
b C c = [1 0 0] 0,2 1,04 0 1 = 0,2

0 0 1 0

Luego,
b C c b C c 0,2
cos = = = = 0,196116135
b C b c C c b c 1 1,04

Ejemplo 2.51
Obtener una expresin de la densidad de masa en funcin del tercer invariante del tensor
de deformacin de Green 0 = 0 ( III C ) .

Universidad Castilla- La Mancha Draft Por: Eduardo W. V. Chaves (2012)


Ciudad Real - Espaa
204 PROBLEMAS RESUELTOS DE MECNICA DEL MEDIO CONTINUO

Solucin:
Partiendo de la ecuacin:
r r r
0 ( X ) = ( x, t ) J ( x, t )
y considerando que el tercer invariante viene dado por III C = det (C ) = J 2 , obtenemos que:
J = III C , luego:

0 = III C (2.145)

Ejemplo 2.52
En un cierto instante, el campo de desplazamientos de un medio continuo es:
u1 = (a1 1) X 1 ; u 2 = (a 2 1) X 2 + a1X 1 ; u 3 = (a 3 1) X 3
donde es una constante. Determinar a1 , a 2 y a 3 sabiendo que el slido es
incompresible, que un segmento paralelo al eje X 3 no se alarga y que el rea de un
elemento situado en el plano X 1 X 3 no se ha modificado.
Solucin:
r r r
Partiendo de la definicin del campo de desplazamientos ( u = x X ):
u1 = x1 X 1 = (a1 1) X 1 x1 = a1 X 1
u 2 = x 2 X 2 = (a 2 1) X 2 + a1X 1 x 2 = a 2 X 2 + a1X 1
u 3 = x 3 X 3 = (a 3 1) X 3 x3 = a3 X 3
Luego las ecuaciones del movimiento son:
x1 = a1 X 1 x1 a1 0 0 X 1

x 2 = a 2 X 2 + a1X 1 x 2 = a1 a 2 0 X 2 (deformacin homognea)
x = a X x 0 0 a 3 X 3
3 3 3 3
Pudiendo sacar la informacin del determinante F = a1 a 2 a 3 > 0 .
Con la condicin de incompresibilidad dV = F dV0 F J = 1 , con lo cual obtenemos la
siguiente relacin:
a1 a 2 a 3 = 1

Que un segmento paralelo al eje X 3 ( M = [0 0 1] ) no se alarga conlleva a que el


i
estiramiento segn esta direccin es unitario M = 1 :

E M
M = 1 + 2M = 1 + 2E = 1 E 33 = 0
33

Las componentes del tensor de deformacin de Green-Lagrange ( E =


1 T
2
(
F F 1 ): )

Universidad de Castilla- La Mancha Draft Por: Eduardo W. V. Chaves (2012)


Ciudad Real - Espaa
2 CINEMTICA DEL CONTINUO 205

a1 a1 0 a1 0 0 1 0 0
1
E ij = 0 a2 0 a1 a 2 0 0 1 0
2

0 0 a 3 0 0 a 3 0 0 1
a12 + a12 2 1 a1 a 2 0
1
= a1 a 2 2
a2 1 0
2
0 0 a 32 1

Luego:
E 33 = a 32 1 = 0 a 3 = 1
rea en el plano X 1 X 3 no se ha modificado
x1 a1 0 0 X 1

x 2 = a1 a 2 0 X 2
x 0 0 a 3 X 3
3

con N i(1) = [1 0 0] y N i(3) = [0 0 1] obtenemos que:


a1 0 0 1 a1 a1 0 0 0 0

n i(1) = a1 a 2 0 0 = a1 ; n i(3)
= a1 a 2 0 0 = 0
0 0 a 3 0 0 0 0 a 3 1 a 3

Luego el rea en la configuracin actual:


e 1 e 2 e 3
r (1) r (3)
n n = a1 a1 0 = a1e 1 a1 a 3 e 2 + 0e 3
0 0 a3
r r
y su mdulo no se modifica N (1) N (3) = n (1) n (3) = 1 :
r r
n (1) n (3) = 1 = (a1 ) 2 + (a1 a 3 ) 2 a12 a 32 2 + a12 a 32 = 1

Hemos obtenido anteriormente que a 32 = 1 , con lo cual obtenemos que:


1 1
a12 a 32 2 + a12 a 32 = 1 a12 2 + a12 = 1 a12 = a1 =
(1 + )
2
(1 + 2 )
Con lo cual concluimos que:
1
a1 = ; a 2 = (1 + 2 ) ; a3 = 1
(1 + )2

Ejemplo 2.53
El slido de la Figura 2.11 sufre una deformacin uniforme (homognea).
Se pide:
a) Obtener la expresin general de la descripcin material del campo de
r r
desplazamientos U ( X , t ) en funcin del tensor gradiente material de los
desplazamientos J .

Universidad Castilla- La Mancha Draft Por: Eduardo W. V. Chaves (2012)


Ciudad Real - Espaa
206 PROBLEMAS RESUELTOS DE MECNICA DEL MEDIO CONTINUO

b) Obtener dicha expresin sabiendo que, adems, se cumple las siguientes


condiciones de contorno:
r r
u 2 ( X , t ) = u3 ( X , t ) = 0 X 1 , X 2 , X 3
u1 ( X 1 = 0, X 2 , X 3 , t ) = 0
u1 ( X 1 = L, X 2 , X 3 , t ) =
c) Justificar los valores posibles (positivos y negativos) que puede tomar .
d) Calcular los tensores material y espacial de deformacin y el de deformacin
infinitesimales;

x3

x1

x2
Figura 2.11:
Solucin:
r
Una deformacin uniforme viene caracterizada por F ( X , t ) = F (t ) . Adems sabemos que:
r r
F ( X , t ) = 1 + J ( X , t ) Deformaci
n F (t ) = 1 + J (t )
uniforme

donde J es el gradiente material de los desplazamientos y para una deformacin uniforme


no es dependiente de la posicin, con lo cual podemos decir que:
r r
u( X , t ) r r r r r r r
J (t ) = r J (t ) dX = du( X , t )
u( X , t ) = J (t ) X + c (t )
X
r
donde c (t ) es una constante de integracin. Luego:
r r r r
u( X , t ) = J (t ) X + c (t )
En componentes:
u1 J 11 X 1 + J 12 X 2 + J 13 X 3 c1

u 2 = J 21 X 1 + J 22 X 2 + J 23 X 3 + c 2
u J X + J X + J X c
3 31 1 32 2 33 3 3
b) De las condiciones del apartado b) podemos decir que:
r r
condicin 1) u 2 ( X , t ) = u 3 ( X , t ) = 0 X 1 , X 2 , X 3 :

u1 J 11 X 1 + J 12 X 2 + J 13 X 3 c1
J 21 = 0; J 22 = 0; J 23 = 0, c 2 = 0
u 2 = 0 = J 21 X 1 + J 22 X 2 + J 23 X 3 + c 2
u = 0 J X + J X + J X c J 31 = 0; J 32 = 0; J 33 = 0, c3 = 0
3 31 1 32 2 33 3 3

Universidad de Castilla- La Mancha Draft Por: Eduardo W. V. Chaves (2012)


Ciudad Real - Espaa
2 CINEMTICA DEL CONTINUO 207

condicin 2) u1 ( X 1 = 0, X 2 , X 3 , t ) = 0 :
u1 = 0 J 11 X 1 + J 12 X 2 + J 13 X 3 c1

u2 = 0 + 0 {J 12 = 0; J 13 = 0, c1 = 0
u 0 0
3
condicin 3) u1 ( X 1 = L, X 2 , X 3 , t ) =
u1 J 11 L 0

u 2 = 0 + 0 J 11 =
u = 0 0 L
3
Con lo cual podemos decir que las componentes del gradiente material de los
desplazamientos son:

L 0 0
J ij = 0 0 0

0 0 0

Y adems el campo de desplazamientos:

X1
r r r r r L
s u i ( X , t ) = 0
u( X , t ) = J (t ) X + c (t ) componente


0

c) Para que el movimiento sea posible y tenga significado fsico hay que cumplir que
F >0:


1 + L 0 0

F (t ) = 1 + J (t ) Fij = 0
componentes
1 0 F = 1 + > 0 > L
L
0 0 1

d) Tensor material de deformacin (Tensor de deformacin de Green-Lagrange):
1 2
+ 2
0 0
L 2 L
E=
1 T
2
(
F F 1 ) componente
s E ij = 0

0 0
0 0 0


Tensor espacial de deformacin (Tensor de deformacin de Almansi):
1 2
+ 2
1 0 0
e=
1
2
(
1 F FT ) componente
s eij =
L 2 L
2

0 0 0

1 + 0 0 0
L
Tensor de deformacin infinitesimal:

Universidad Castilla- La Mancha Draft Por: Eduardo W. V. Chaves (2012)


Ciudad Real - Espaa
208 PROBLEMAS RESUELTOS DE MECNICA DEL MEDIO CONTINUO


L 0 0
ij = 0 0 0

0 0 0

Ejemplo 2.54
Sobre el tetraedro de la Figura 2.12 se produce una deformacin uniforme ( F = ctte ) con
las siguientes consecuencias:
1. Los puntos O , A y B no se mueven;
2. El volumen del slido pasa a ser " p" veces el volumen inicial;
p
3. La longitud del segmento AC pasa a ser veces la inicial;
2
4. El ngulo AOC pasa a ser de 45 .
Se pide:
a) Justificar por qu no puede utilizar la teora de deformacin infinitesimal;
b) Obtener el tensor gradiente de deformacin, los posibles valores de " p" y el campo
de desplazamiento en su forma material y espacial;
c) Dibujar el slido deformado
x3

O a
B x2
a

x1

Figura 2.12.

Solucin:
a) el ngulo AOC pasa de 90 a 45 por lo que, evidentemente, no se trata de una
pequea deformacin, ya que en el caso de pequeas deformaciones << 1 , y en este

problema tenemos que << 0,7854
4

Universidad de Castilla- La Mancha Draft Por: Eduardo W. V. Chaves (2012)


Ciudad Real - Espaa
2 CINEMTICA DEL CONTINUO 209

b) Estamos en un caso de deformacin homognea. Luego, las ecuaciones del movimiento


viene dadas por:
r r r
x = F (t ) X + c (t )

x1 F11 F12 F13 X 1 c1



x 2 = F21 F22 F23 X 2 + c 2
x F F32 F33 X 3 c3
3 31
El punto O( X 1 = 0, X 2 = 0, X 3 = 0) no se mueve:
0 F11 F12 F13 0 c1 c1 0

0 = F21 F22 F23 0 + c 2 c 2 = 0
0 F F32 F33 0 c3 c 0
31 3
El punto A( X 1 = a, X 2 = 0, X 3 = 0) no se mueve:
a F11 F12 F13 a a aF11 F11 = 1

0 = F21 F22 F23 0 0 = aF21 F21 = 0
0 F F32 F33 0 0 aF F = 0
31 31 31
El punto B( X 1 = 0, X 2 = a, X 3 = 0) no se mueve:
0 1 F12 F13 0 0 aF12 F12 = 0

a = 0 F22 F23 a a = aF22 F22 = 1
0 0 F F33 0 0 aF F = 0
32 32 32
Agrupando las informaciones anteriores:
1 0 F13
Fij = 0 1 F23 ; F = F33
0 0 F33

El volumen del slido pasa a ser " p" veces el volumen inicial. La relacin del diferencial de volumen
en la configuracin de referencia y actual viene dada por:
dV = F dV0 dV = F dV 0 V final = F Vinicial = F33Vinicial

donde hemos tenido en cuenta que la deformacin es homognea. Con lo cual, concluimos
que F33 = p
p
(La longitud del segmento AC pasa a ser veces la inicial). Como estamos con deformacin
2
homognea, una recta en la configuracin de referencia seguir siendo una recta en la
configuracin deformada.
El punto C ( X 1 = 0, X 2 = 0, X 3 = a ) se desplaza como:
x1C 1 0 F13 0 x1C aF13
C C
x 2 = 0 1 F23 0 x 2 = aF23
x C 0 0 p a x C ap
3 3

Universidad Castilla- La Mancha Draft Por: Eduardo W. V. Chaves (2012)


Ciudad Real - Espaa
210 PROBLEMAS RESUELTOS DE MECNICA DEL MEDIO CONTINUO

La longitud del segmento AC en la configuracin de referencia es L AC = a 2 . El vector


que une los puntos A A( x1 = a, x 2 = 0, x3 = 0) , C ( x1 = aF13 , x 2 = aF23 , x3 = ap) en la
configuracin deformada viene dado por:
AC = (aF13 a )e 1 + (aF23 )e 2 + (ap)e 3
Su mdulo:
AC = l AC = (a ( F13 1)) 2 + (aF23 ) 2 + (ap) 2 = a ( F13 1) 2 + ( F23 ) 2 + ( p) 2

p
Utilizando la informacin proporcionada por el problema: l AC = L AC . Con lo cual:
2
p
l AC = L AC
2
p
a ( F13 1) 2 + ( F23 ) 2 + ( p ) 2 = a 2
2
( F13 1) 2 + ( F23 ) 2 + ( p ) 2 = p

Resultando:
( F13 1) 2 + ( F23 ) 2 + p 2 = p 2
( F13 1) 2 + ( F23 ) 2 = 0
F13 = 1

F23 = 0
Resultando:
1 0 1
Fij = 0 1 0
0 0 p
El ngulo AOC pasa a ser de 45 .
dx1(1) 1 0 1 1 1
(1)
dX i(1) = [1 0 0] dxi(1) = Fij dX (j1) dx 2 = 0 1 0 0 = 0
dx (1) 0 0 p 0 0
3

dx1( 2) 1 0 1 0 1
( 2)
dX i( 2 ) = [0 0 1] dxi( 2 ) = Fij dX (j 2 ) dx 2 = 0 1 0 0 = 0
dx ( 2) 0 0 p 1 p
3
r r
dx (1) dx ( 2 ) 2
cos( AOC ) = cos(45 ) = r (1) r ( 2 ) =
dx dx 2

r r r r
donde dx (1) = 1 , dx ( 2) = 1 + p 2 , dx (1) dx ( 2) = 1 . Luego:

1 2
= p = 1
1+ p2 2

Universidad de Castilla- La Mancha Draft Por: Eduardo W. V. Chaves (2012)


Ciudad Real - Espaa
2 CINEMTICA DEL CONTINUO 211

Como el determinante del Jacobiano tiene que ser mayor que cero F = p > 0 , eso implica
que p = 1 :
1 0 1
Fij = 0 1 0
0 0 1

Las ecuaciones del movimiento quedan:


x1 1 0 1 X 1 X 1 + X 3

x 2 = 0 1 0 X 2 = X 2
x 0 0 1 X X
3 3 3
El campo de desplazamientos material queda:
u1 X 1 + X 3 X 1 X 3
r r r r
u( X , t ) = x X u 2 = X 2 X 2 = 0
u X X 0
3 3 3
El campo de desplazamientos espacial:
u1 x3

u 2 = 0
u 0
3
c)

x3

C
x1C aF13 a
C
x 2 = aF23 = 0 C
x C ap a
3

a a
O
B = B x2
a

A = A

x1

Universidad Castilla- La Mancha Draft Por: Eduardo W. V. Chaves (2012)


Ciudad Real - Espaa
212 PROBLEMAS RESUELTOS DE MECNICA DEL MEDIO CONTINUO

Ejemplo 2.55
Considrense las siguientes ecuaciones del movimiento:
x1 = X 1
x 2 = X 2 X 3
x 3 = X 3 + X 2
Se pide:
a) El gradiente de deformacin, el tensor derecho de deformacin de Cauchy-Green, el
tensor izquierdo de deformacin de Cauchy-Green, el tensor de deformacin de Green-
Lagrange y el tensor de deformacin de Almansi. Verificar si se trata de un caso de
deformacin homognea.
b) El tensor derecho de estiramiento, el tensor de rotacin de la descomposicin polar y la
base principal del tensor izquierdo de deformacin de Cauchy-Green de la descomposicin
polar.
c) La longitud final de un elemento de longitud inicial 2 que se encuentra en la direccin
X 3 , y la distorsin angular de un ngulo que inicialmente es de 30 y est en el plano
X1 X 2 .
d) Obtener el tensor de deformacin considerando el caso de pequeas deformaciones.

Solucin:
a) Gradiente de deformacin ( F )
x1 x1 x1

X 1 X 2 X 3 1 0 0
xi x 2 x 2 x 2
Fij = = = 0 1
X j X 1 X 2 X 3

x3 0 1

x3 x3
X 1 X 2 X 3
r r
En general tenemos que dx = F dX , y si estamos en un caso de deformacin homognea
r r r
(caso particular del movimiento) se cumple que x = F X + c , hecho que se puede
r r
comprobar a travs de las ecuaciones del movimiento en forma matricial donde c = 0 :
x1 1 0 0 X1
x = 0 1 X
2 2
x3 0 1 X 3

Tensor derecho de deformacin de Cauchy-Green ( C = F T F ):


1 0 0 1 0 0 1 0 0

C ij = Fki Fkj = 0 1 0 1 = 0 1 + 2
0
0 1 0 1 0 0 1 + 2

Tensor izquierdo de deformacin de Cauchy-Green ( b = F F T )


1 0 0 1 0 0 1 0 0

bij = Fik F jk = 0 1 0 1 = 0 1 + 2
0
0 1 0 1 0 0 1 + 2

Universidad de Castilla- La Mancha Draft Por: Eduardo W. V. Chaves (2012)


Ciudad Real - Espaa
2 CINEMTICA DEL CONTINUO 213

1
El tensor de deformacin de Green-Lagrange ( E = (C 1) ) y el tensor de deformacin
2
1
de Almansi ( e = (1 b 1 ) ) vienen definidos a travs de sus componentes por:
2
1 0 0 1 0 0 0 0 0
1 1 1
E ij = (C ij ij ) = 0 1 + 2
0 0 1 0 = 0 2 0
2 2 2
0
0 1 + 2 0 0 1 0 0 2


0
1 0 0 1 0 0 0 0

1 1 1 1 2
1
eij = ( ij bij ) = 0 1 0 0 0 = 0 0
2 2 1+ 2 2 1 + 2

0 0 1 1 2
0 0
2 0 0
1 + 1+ 2
Se puede verificar el resultado a travs de la relacin E = F T e F :

0 1 0
1 0 0 0 0
0 0 0 0
1 2
1
E ij = 0 1 0 0
0 1 = 0 2 0
2 1+ 2 2
0 1
2 0 1 0 0 2
0 0
1+ 2
b) Segn el formato de las componentes cartesianas de C , podemos verificar que ele
espacio original ya es el espacio principal de C , es decir, las direcciones principales son
N i(1) = [1 0 0] , N i(1) = [0 1 0] , N i(1) = [0 0 1] . Por definicin, el tensor derecho de
estiramiento viene dado por U = C , y sus componentes:


1 0 0 1 0 0
1
U ij = 0 1+ 2
U ij1
0 inversa = 0 0
0 0 1+ 2 1+ 2
1
0 0
2
1+

A travs de la descomposicin polar por la derecha ( F = R U R = F U 1 ), luego:




1 0 0 1 0 0 1+ 2 0 0
1 1
R ij = Fik U kj1 = 0 1 0 0 = 0 1
0 1 1+ 2 1+ 2 0 1
1
0 0
1+ 2
Observemos que segn el formato de las componentes cartesianas de b , tenemos como
direcciones principales [1 0 0] , [0 1 0] , [0 0 1] , pero esta no es la base principal del
tensor b de la descomposicin polar. Como hay dos autovalores iguales correspondientes a
las direcciones [0 1 0] , [0 0 1] , cualquier direccin en el plano x 2 x 3 ser una
direccin principal.

Universidad Castilla- La Mancha Draft Por: Eduardo W. V. Chaves (2012)


Ciudad Real - Espaa
214 PROBLEMAS RESUELTOS DE MECNICA DEL MEDIO CONTINUO

X 2 , x2

Cualquier direccin en el plano


x 2 x 3 es una direccin
principal de b n (1) -direccin principal nica,
asociada al autovalor b1 = 1 .

X 1 , x1

X 3 , x3

Figura 2.13: Espacio principal de b .


Recordar que la descomposicin polar es nica, es decir, slo habr una nica base
principal de b para la descomposicin polar. A travs de la relacin n ( a ) = R N ( a )
podemos encontrar la base de b de la descomposicin polar:
1+ 2 0 0 0 0
n (i 2 ) (
= R N
(2) )
i =
1
0 1

1 =
1 1

1+ 2 0 1 0 1 +
2


1+ 2 0 0 0 0
n (i 3) (
= R N
( 3) )i =
1
0 1

0 =
1

1+ 2 0 1 1 1 +
2
1

3
Verifiquemos adems que se cumple la relacin R = n ( a ) N
(a ) :
a =1

R ij = n i(1)N (j1) + n i( 2) N (j2 ) + n i(3) N (j3)


1 0 0
1 1
= 0[1 0 0] + 1 [0
1 0] + [0 0 1]

0 1+ 2
1+ 2 1
1 0 0 0 0 0 0 0 0
1 0 1 1
= 0 0 0 + 0 + 0 0
2
0 0 0 1 + 0 0 1+ 2 0 0 1

1+ 2 0 0
1
= 0 1
1+ 2 0 1

r
dx ds
c) A travs de la relacin de estiramiento segn una direccin M , M = r = , y
dX dS
teniendo en cuenta que el estiramiento no depende de la integral de lnea (deformacin
homognea), se cumple que:

Universidad de Castilla- La Mancha Draft Por: Eduardo W. V. Chaves (2012)


Ciudad Real - Espaa
2 CINEMTICA DEL CONTINUO 215


L final = ds = M dS = M dS = M Linicial

El estiramiento segn la direccin X 3 viene definido por:

X = C 33 = 1 + 2 E 33 = 1 + 2
3

Luego:
2


L final = M dX 2 = 1 + 2 ( Linicial ) = 2 1 + 2
0

Como se trata de un caso de deformacin homognea, una recta en la configuracin de


referencia sigue sendo una recta en la configuracin actual, ver Figura 2.14.

X 3 , x3

x1A 1 0 0 X 1A
A A A A
x 2 = 0 1 X 2
x3A 0 1 X 3A
Linicial = 2
L final
1 0 0 0 0
= 0 1 0 = 2
2 O X 2 , x2 0 1 2 2

X 1 , x1
Figura 2.14.
Segn la Figura 2.14, podemos comprobar que:
Linicial = 2 2 + (2 ) 2 = 4(1 + 2 )
2

Linicial = 2 1 + 2
Para obtener el ngulo en la configuracin actual que forman dos versores, podemos
utilizar la expresin:
cos + 2 M E N
cos = (2.146)
M N

donde es el ngulo que forman los versores M y N en la configuracin de referencia, y


es el ngulo que estos versores forman en la configuracin actual.
Teniendo en cuenta que el tensor de deformacin de Green-Lagrange es independiente de
r
X , adoptaremos dos versores en el plano X 1 X 2 que forman un ngulo de = 30 :
N i = [1 0 0] , M i = [cos 30 sin 30 0] . Con estos datos tenemos que:

0 0 0 cos 30
1
M E N = [1 0 0]0 2
0 sin 30 = 0
2
0 0 2 0

Universidad Castilla- La Mancha Draft Por: Eduardo W. V. Chaves (2012)


Ciudad Real - Espaa
216 PROBLEMAS RESUELTOS DE MECNICA DEL MEDIO CONTINUO

Los estiramientos:
1 0 0 1
2M
= M C M = [1 0 0] 0 1 +
2
0 0 = 1 M = 1
0 0 1 + 2 0

1 0 0 cos 30
2N
= N C N = [cos 30 sin 30 0] 0 1 + 2
0 sin 30
0 0 1 + 2 0
= cos 2 30 + (1 + 2 ) sin 2 30
= 1 + 2 sin 2 30

Luego, N = 1 + 2 sin 2 30 . Resultando que:

cos + 2 M E N cos 30
cos = =
M N 1 + 2 sin 2 30
Como se trata de un caso de deformacin homognea, podemos adoptar dos rectas en la
configuracin de referencia y obtener estas dos rectas en la configuracin actual y obtener
el ngulo que forman. Por ejemplo, adoptando las rectas OB = [cos 30 0 0] , y
OC = [cos 30 sin 30 0] . Segn las ecuaciones del movimiento, el punto O no se mueve.
A continuacin obtenemos las nuevas posiciones de los puntos B y C , ver Figura 2.15:
x1B 1 0 0 X 1B 1 0 0 cos 30 cos 30
B B
x 2 = 0 1 X 2 = 0 1 0 = 0
x B 0 1 X B 0 1 0 0
3 3

x1C 1 0 0 X 1C 1 0 0 cos 30 cos 30


C C
x 2 = 0 1 X 2 = 0 1 sin 30 = sin 30
x C 0 1 X C 0 1 0 sin 30
3 3

X 3 , x3

A sin 30

O sin 30
X 2 , x2
cos 30 30
B = B
C

X 1 , x1

Figura 2.15.

Universidad de Castilla- La Mancha Draft Por: Eduardo W. V. Chaves (2012)


Ciudad Real - Espaa
2 CINEMTICA DEL CONTINUO 217

A continuacin obtenemos el ngulo que forman los nuevos vectores O B y O C :


O B O C = O B O C cos

cos 2 30 = cos 2 30 cos 2 30 + sin 2 30 + 2 sin 2 30 cos


cos 30
cos =
1 + 2 sin 2 30
d)
0 0 0
ij = 0 0 0
0 0 0

Ejemplo 2.56
Un movimiento de cuerpo rgido est caracterizado por presentar la siguiente ecuacin de
movimiento:
r r r
x = c(t ) + Q(t ) X (2.147)
r
Encontrar la velocidad y aceleracin en funcin de que es el vector asociado con el
& QT .
tensor antisimtrico = Q
Solucin:
r D r r& r& & r
v= x = x = c + Q X
Dt
& QT Q
Considerando que = Q & = Q , la relacin anterior puede an ser escrita
como:
r r r
v = c& + Q X
r r r r
v = c& + ( x c )
Utilizando la propiedad que para un tensor antisimtrico el producto escalar de ste con
r r r r r
un vector a podr ser representado por a = a , donde es el vector asociado al
tensor antisimtrico . Luego:
r r r r
v = c& + ( x c )
r r r r (2.148)
= c& + ( x c )

La aceleracin vendr dada por:


r r &r& &r& && r
a = v& = x =c + Q X
&& = & Q + Q
Considerando que Q & la expresin anterior podr ser representada por:
r &r& r
a =c + (& Q + Q & ) X
r r r
= &c& + & Q X + Q & X
r r r
= &c& + & Q X + Q X
r r r r r
= &c& + & ( x c ) + ( x c )

Universidad Castilla- La Mancha Draft Por: Eduardo W. V. Chaves (2012)


Ciudad Real - Espaa
218 PROBLEMAS RESUELTOS DE MECNICA DEL MEDIO CONTINUO

r r r
Una vez ms utilizando la propiedad a = a , podemos decir que:

+ ( x c) + [ ( x c)]
r &r& r& r r r r r r
a =c (2.149)
r r r r r
Para un movimiento de slido rgido con c = 0 , la velocidad viene dada por v = x , y el
tensor tasa de deformacin D viene dado por:
1 vi v j 1 ( ipq p x q ) ( jpq p x q ) 1
=
x x
= ipq p q + jpq p q
D ij = + +
2 x j xi 2
x j xi 2
x j xi

=
1
2
(
ipq p qj ) ( 1
2
1
) (
+ jpq p qi = ipj p + jpi p = ipj p ipj p = 0 ij
2
)
Una vez ms hemos demostrado que D = 0 para un movimiento de slido rgido.

Ejemplo 2.57
r
a) Un slido gira con una velocidad angular constante = 3 e 3 . Se pide:
a.1) Obtener las componentes de la velocidad en la descripcin espacial y material;
a.2) Obtener la aceleracin en la descripcin espacial (Euleriana);
a.3) Si 3 = 3rad / s encontrar la posicin, velocidad y aceleracin en el tiempo t = 2,5s de
una partcula que en la configuracin de referencia ocupaba la posicin (1,1,0) .
b) Teniendo en cuenta el resultado del Ejemplo 1.122 donde hemos obtenido el vector
r GM r
fuerzas msicas b = r x donde g = b es la aceleracin de la gravedad debido al
x
campo gravitacional. Si consideramos la Tierra como una esfera que gira alrededor de su eje
r
con velocidad angular = 3 e 3 , obtener la aceleracin de la gravedad ( g ) a nivel del mar
en funcin de la latitud .
Solucin:

X 3 , x3
r r
r r = r e r = re r
= 3 e 3
3

r e3
r
e
r e r
x

X 2 , x2

X 1 , x1
Figura 2.16.

Universidad de Castilla- La Mancha Draft Por: Eduardo W. V. Chaves (2012)


Ciudad Real - Espaa
2 CINEMTICA DEL CONTINUO 219

r r r r
a.1) A travs del ejercicio anterior podemos decir que v ( x , t ) = x , o en notacin
indicial:
vi = ijk j x k = i1k
{1 x k + i 2 k
{2 x k + i 3k 3 x k = i 3k 3 x k
=0 =0

= i 313 x1 + i 32 3 x 2 + i 33 3 x3 = i 31 3 x1 + i 32 3 x 2
{
=0

Luego:
v1 = 132 3 x 2 = 3 x 2

v 2 = 2313 x1 = 3 x1 (2.150)
v = 0
3
r
Solamente por verificacin, recordar que el vector vorticidad ( ) es igual al rotacional del
r r r r
campo de velocidad, xr v = rot v = y es igual a 2 veces el vector velocidad angular
r r r
(vector axil) = 2 = 2 w :
e 1 e 2 e 3
r r v v v v v v
xr v = = 3 2 e 1 3 1 e 2 + 2 1 e 3
x1 x 2 x3 x 2 x3 x1 x3 x1 x 2
v1 v2 v3
r
= ( 3 ( 3 ) )e 3 = 2 3 e 3 =
r 1 1 rr 1 r r 1
La velocidad angular viene definida por = xr v = rot v = = ( 2 3 e 3 ) = 3 e 3 .
2 2 2 2
r r r r r
Observar que el campo v ( x , t ) es estacionario, es decir, v = v ( x ) .
Para un movimiento de slido rgido las ecuaciones del movimiento vienen gobernadas
por:
r r
x = Q(t ) X
donde las componentes del tensor ortogonal viene dada por las componentes de la matriz
r r
de transformacin del sistema x al sistema x , luego:
x1 cos (t ) sin (t ) 0 X 1 cos (t ) X 1 sin (t ) X 2

x 2 = sin (t ) cos (t ) 0 X 2 = sin (t ) X 1 + cos (t ) X 2
x 0 0 1 X 3 X3
3
d(t )
Teniendo en cuenta que = y al integrar obtenemos que:
dt

d(t ) = dt (t ) = t

Pudiendo as reescribir las ecuaciones del movimiento como:


x1 cos (t ) sin (t ) 0 X 1 X 1 cos(t ) X 2 sin(t )

x 2 = sin (t ) cos (t ) 0 X 2 = X 1 sin(t ) + X 2 cos(t ) (2.151)
x 0 0 1 X 3 X3
3
Para obtener la expresin de la velocidad en la descripcin material (Lagrangiana),
reemplazamos las ecuaciones del movimiento (2.151) en las expresiones (2.150):

Universidad Castilla- La Mancha Draft Por: Eduardo W. V. Chaves (2012)


Ciudad Real - Espaa
220 PROBLEMAS RESUELTOS DE MECNICA DEL MEDIO CONTINUO

r
v1 ( X , t ) = 3 ( X 1 sin(t ) + X 2 cos(t ))
r
v 2 ( X , t ) = 3 ( X 1 cos(t ) X 2 sin(t )) (2.152)
r
v3 ( X , t ) = 0

a.2) La aceleracin Euleriana obtenemos a travs de la definicin de derivada material de la


r r
velocidad v ( x , t ) :
r r r r r r
r r v ( x , t ) v ( x , t ) x ( X , t ) r r r
a ( x, t ) = + r = xr v v ( x , t )
142t4 3 x t
r
0

donde las componentes del gradiente espacial de la velocidad vienen dadas por:
v1 v1 v1

r r x1 x 2 x 3 0 3 0
v ( x , t ) v v 2 v 2
0 (antisimtrico)
r
r = ( xr v )ij = 2 = 3 0
x ij x x 2 x 3
1 0
v 3 v 3 v 3 0 0
x1 x 2 x 3
Comprobando que realmente se trata de un movimiento de slido rgido. Luego, las
componentes de la aceleracin Euleriana vienen dadas por:
0 3 0 3 x2 32 x1

ai ( x , t ) = [ xr v v ( x , t ) ]i = 3 0 3 x1 = 32 x2
r r r r
0
0 0 0 0 0
r r
Podemos expresar la aceleracin a ( x , t ) = 32 x1 e 1 32 x 2 e 2 en el sistema cilndrico, ver
Figura 2.16. Recordar del captulo 1 de los apuntes que se cumplen que:
x1 = r cos , x1 = r cos , e 1 = e r cos e sin , e 2 = e r sin + e cos . Luego, la
aceleracin en el sistema cilndrico queda:
r
a = 32 x1e 1 32 x 2 e 2
= 32 ( r cos )(e r cos e sin ) 32 ( r sin )(e r sin + e cos )
= 32 r (cos 2 + sin 2 )e r
= 32 re r
r v
= 32 r = a ctpe
v
Esta ltima conocida como la aceleracin centrpeta ( actpe ).
a.3) La partcula que en la configuracin de referencia ocupaba la posicin (1,1,0) describe
una trayectoria circular de radio r = 2 en el plano x1 x 2 , ver Figura 2.17.

Universidad de Castilla- La Mancha Draft Por: Eduardo W. V. Chaves (2012)


Ciudad Real - Espaa
2 CINEMTICA DEL CONTINUO 221

Partcula P en t = 2,5s X 2 , x2
r r
v ( X , t = 0)
Partcula P
r r
v P ( x , t = 2,5) 1
r
r X
x

1 X 1 , x1

Trayectoria de la partcula P

Figura 2.17.
r r
En la configuracin de referencia ( t = 0 ) se cumple que X = x . Para la partcula P
tenemos que:
r
v1P ( x , t = 0) = 3 x 2 = 3 X 2 = (3)(1) = 3
P r
v 2 ( x , t = 0) = 3 x1 = 3 X 1 = (3)(1) = 3
P
v3 = 0

32 X 1 9
r
a iP ( x , t = 0) = 32 X 2 = 9
0 0

Para el tiempo t = 2,5s la posicin, velocidad, y aceleracin de la partcula P vienen dadas
por:
x1P X 1 cos(t ) X 2 sin(t ) cos(3 2,5) sin(3 2,5) 0,59136
P
x 2 = X 1 sin(t ) + X 2 cos(t ) = sin(3 2,5) + cos(3 2,5) = 1,28464
x P X3 0 0
3
r
v1P ( x , t = 2,5) = 3 x 2 = (3)(0,59136) = 3,85391
P r
v 2 ( x , t = 2,5) = 3 x1 = (3)(1,28464) = 1,77409
P
v3 = 0

32 x1 5,322
r
a iP ( x , t = 2,5) = 32 x 2 = 11,562
0 0

b) Para una partcula situada en la superficie de la Tierra, debido a la rotacin se sentir
como si estuviera siendo proyecta hacia fuera segn direccin de r , ver figura abajo. Hay
que tener en cuenta que la fuerza real es la Centrpeta debido a la aceleracin centrpeta.
Por conveniencia, adoptamos una fuerza ficticia, fuerza centrfuga, que sera la aparente
causa de esta proyeccin hacia afuera. Asociada a esta fuerza tenemos la aceleracin
v v
centrfuga ( a ctfu ) que es igual pero de sentido contrario a la aceleracin centrpeta ( a ctpe ).

Universidad Castilla- La Mancha Draft Por: Eduardo W. V. Chaves (2012)


Ciudad Real - Espaa
222 PROBLEMAS RESUELTOS DE MECNICA DEL MEDIO CONTINUO

x3 , z
x3 , z
3 3

r v
x3 r a ctfu
r
R b


x2 , y

x1 , x

La aceleracin de la gravedad para una latitud definida por viene dada por:
v r
g = a ctfu + b

Recordar que dados dos vectores se cumple que


v r v 2 v r r 2
a ctfu +b = a ctfu + 2 a ctfu b cos + b , ver Ejemplo 1.02. Para este caso en
r v v r
particular tenemos que, b =g, a ctpe = a ctfu = 32 r = 32 r . Verificar tambin que
r = R cos y que cos = cos( ) = cos . Con eso, obtenemos que:

v r v 2 v r r 2
g = a ctfu + b = a ctpe 2 a ctpe b cos + b

= ( 32 r ) 2 2( 32 r ) g cos + g 2

= ( 32 R cos ) 2 2( 32 R cos ) g cos + g 2


Resultando
g = g 2 2 g 32 R cos 2 + 34 R 2 cos 2

Observar que en los polos ( = 90 ) se cumple que g Pol = g y en la lnea del ecuador se
cumple que g Ecu = g 2 2 g 32 R + 34 R 2 = ( g 32 R ) 2 = g 32 R .

Universidad de Castilla- La Mancha Draft Por: Eduardo W. V. Chaves (2012)


Ciudad Real - Espaa
2 CINEMTICA DEL CONTINUO 223

Ejemplo 2.58
Considrese una barra sometida a sucesivos desplazamientos como se indica la Figura abajo

B0 B B

L0 L0
L(1) L( 2 ) L(f1) L L( 2 )

L(1)
L
( 2)
L

Demostrar que la deformacin Ingenieril (deformacin de Cauchy) no es aditiva para


incremento sucesivos de deformacin, es decir, (1) + ( 2) .
Solucin:
La deformacin de Cauchy fue definida como:
L L L 0
C = = = 1
L0 L0

Luego, la deformacin total sufrida por el cuerpo, es decir, de la configuracin B0 hasta la


configuracin B es
L( 2 ) L0 L( 2 )
C = = 1
L0 L0

En la configuracin B la deformacin Ingenieril queda:


L(1) L0 L(1)
C(1) = = 1
L0 L0

En la configuracin B teniendo en cuenta solo el incremento de desplazamiento u ( 2) ,


tenemos que:
L( 2) L(1) L( 2)
C( 2) = = (1) 1
L(1) L
Luego
L(1) L( 2 ) L( 2)
C(1) + C( 2 ) = 1 + (1) 1 1 = C
L0 L L0
Un requerimiento esencial de toda deformacin es que pueda caracterizar los
desplazamientos reales, en el caso la longitud final:

Universidad Castilla- La Mancha Draft Por: Eduardo W. V. Chaves (2012)


Ciudad Real - Espaa
224 PROBLEMAS RESUELTOS DE MECNICA DEL MEDIO CONTINUO

L0 L0
L(1)
0
C(1) dx
0

=
L0
1dx = L(1) L0 = L(1)
(1) ( 2)
L1 L1 L + L = L
L( 2 )

0 0

(C2 ) dx = (1) 1 dx = L( 2 ) L(1) = L( 2 )
L
L0 L
0
L

0
C dx =
0 0
L
1 dx = L L0 = L

Deformacin de Green-Lagrange
Observemos que la deformacin de Green-Lagrange en la configuracin B viene dado
por:

G =
L2 L20
= (
1 2
1 )
2 L20 2
Podramos haber obtenido esta misma expresin utilizando la relacin
E = E (1) + F (1)T
E ( 2) F (1) , donde para el caso uniaxial tenemos que E G , E (1) G(1) ,
L(1)
E ( 2 ) G( 2) , F (1) (1) = . Luego:
L0

E ( 2) F (1)
T
E = E (1) + F (1)
G = G(1) + (1) G( 2) (1)

1 L(1) L(1) 1 L( 2) L(1)


2 2

= 1 + 1
2 L0 L0 2 L(1) L0

L( 2) 2 L2
=
0 2
L L0
=
( 2
)
2 L20 2 L20

2.1.3 Descomposicin Polar del Gradiente de Deformacin

Ejemplo 2.59
Considrense las componentes cartesianas del gradiente de deformacin dadas por:
5 3 3
Fij = 2 6 3
2 2 4
Obtener los tensores derecho ( U ) e izquierdo ( V ) de estiramiento y el tensor de rotacin
( R ) de la descomposicin polar.
Solucin:
Antes de obtener los tensores U , V , R , vamos analizar el tensor F .
Para que tenga sentido fsico el determinante de F tiene que ser mayor que cero,
det ( F ) = 60 > 0 . Los autovalores y autovectores F se pueden obtener como:
F11 = 10 asociado al autovector m i(1) = [0,6396021491; 0,6396021491; 0,4264014327]
F22 = 3 asociado al autovector m i( 2 ) = [ 0,5570860145; 0,7427813527; 0,3713906764]

Universidad de Castilla- La Mancha Draft Por: Eduardo W. V. Chaves (2012)


Ciudad Real - Espaa
2 CINEMTICA DEL CONTINUO 225

F33 = 2 asociado al autovector m i(3) = [ 0,4082482905; 0,4082482905; 0,8164965809]


Se puede verificar fcilmente que la base constituida por estos autovectores no forma una
ortonormal, es decir, m i(1) m i( 2) 0 , m i(1) m i(3) 0 , m i( 2) m i(3) 0 . Verificamos tambin que si
B es la matriz que contiene los autovectores de F :
m i(1) 0,6396021491; 0,6396021491; 0,4264014327
( 2)
B = m i = 0,5570860145; 0,7427813527; 0,3713906764
m (3) 0,4082482905; 0,4082482905; 0,8164965809
i
podemos hallar que det (B ) = 0,905 1 , y que B 1 B T . Pero se cumple que:
10 0 0 5 2 2 5 2 2 10 0 0
1
B 0 3 0 B = 3 6 2 = FijT y
B 3 6 2B = 0 3 0
1

0 0 2 3 3 4 3 3 4 0 0 2
Tensor derecho de deformacin de Cauchy-Green, C = F T F (tensor definido positivo):
33 31 29
C ij = Fki Fkj = 31 49 35
29 35 34
Los autovalores y autovectores del tensor C son:
= 9,274739
C11 (1) = [0,6861511933; 0,7023576528 0,1894472683]
autovector
N i

= 3,770098
C 22 autovector
( 2 ) = [0,5105143234; 0,2793856273; 0,8132215099]
N i

= 102,955163
C 33 autovector
(3) = [ 0,518239; 0,65470405; 0,550264423]
N i

Dichos autovectores constituye una base ortonormal luego, se cumple que AC1 = ACT , y
det (AC ) = 1 , donde:
N i(1) 0,6861511933 0,7023576528 0,1894472683

AC = N i( 2) = 0,5105143234 0,2793856273 0,8132215099
N (3) 0,518239 0,65470405 0,550264423
i
cumpliendo que:
C11 0 0 33 31 29 33 31 29 C11 0 0
T
A 0
C 22 T
0 AC = 31 49 35 = C ij ; AC 31 49 35 AC = 0 C 22 0
C
0
0 C 33 29 35 34 29 35 34 0
0 C 33
En el espacio principal de C obtenemos las componentes del tensor derecho de
estiramiento U como:
1 0 0 C11 0 0 3,0454455 0 0

U = Uij = 0 2 0 = 0
C 22 0 = 0 1,9416741 0
0 0 3 0 0
C 33 0 0 10,1466824

y su inversa:
1 1
0 0 0 0
1 3,0454455
1 1
U = Uij
1 1
= 0 0 = 0 0

2 1,9416741

0 1 1
0 0 0
3 10,1466824
Pudiendo as obtener las componentes del tensor U en el espacio original a travs de la ley
de transformacin:

Universidad Castilla- La Mancha Draft Por: Eduardo W. V. Chaves (2012)


Ciudad Real - Espaa
226 PROBLEMAS RESUELTOS DE MECNICA DEL MEDIO CONTINUO

4,66496626 2,25196988 2,48328843


A U AC = 2,25196988 6,00314487 2,80907159 = U ij
T
C
2,48328843 2,80907159 4,46569091
y
0,31528844 0,05134777 0,14302659
A U AC = 2,25196988 0,24442627 0,12519889 = U ij1
T
C
1

0,14302659 0,12519889 0,38221833


Luego, el tensor de rotacin de la descomposicin polar viene dado por la expresin
R = F U 1 , que resulta en un tensor ortogonal propio det (R ) = 1 .
0,9933191 0,10094326 0,05592536
R ij =
= 0,10658955 0,98826538 0,10940847
Fik U kj1
0,04422505 0,11463858 0,9924224
Tensor izquierdo de deformacin de Cauchy-Green, b = F F T (tensor definido positivo):
43 37 28
bij = Fik F jk = 37 49 28
28 28 24
Los autovalores y autovectores del tensor b son:
= 9,274739
b11 n i(1) = [0,6212637156 0,7465251613 0,238183919]
autovector

= 3,770098
b22 autovector
n i( 2 ) = [0,4898263742 0,1327190337 0,8616587383]
= 102,955163
b33 autovector
n (i 3) = [ 0,611638389 0,6519860747 0,448121233]
Observemos que son los mismos autovalores del tensor C , pero con distintos
autovectores. Los autovectores del tensor b tambin constituye una base ortonormal
luego, se cumple que Ab1 = AbT , y det (Ab ) = 1 , donde:
n (i1) 0,6212637156 0,7465251613 0,238183919
( 2)
Ab = n i = 0,4898263742 0,1327190337 0,8616587383
n (3) 0,611638389 0,6519860747 0,448121233
i
cumpliendo que:

b11 0 0 43 37 28 43 37 28
b11 0 0
T
A 0
b22 T
0 Ab = 37 49 28 = bij ; Ab 37 49 28 Ab = 0 b22 0
b
0 0 b33 28 28 24 28 28 24 0 0 b33
Ya que los tensores C y b tienen los mismos autovalores se cumple que Uij = Vij , es decir,
que tienen las mismas componentes en sus respectivos espacios principales. Y como
consecuencia Uij1 = Vij 1 . Luego se cumple que:
Pudiendo as obtener las componentes del tensor U en el espacio original a travs de la ley
de transformacin:
5,3720129 2,76007379 2,41222612
A V Ab = A U Ab = 2,76007379 6,04463857 2,20098553 = Vij
T
b
T
b
2,41222612 2,20098553 3,6519622
y
0,28717424 0,07950684 0,14176921
A V Ab = A U Ab = 0,07950684 0,23396031 0,08848799 = Vij1
T
b
1 T
b
1

0,14176921 0,08848799 0,42079849

Universidad de Castilla- La Mancha Draft Por: Eduardo W. V. Chaves (2012)


Ciudad Real - Espaa
2 CINEMTICA DEL CONTINUO 227

El tensor de rotacin de la descomposicin polar ya obtenido anteriormente tiene ser el


mismo si utilizamos la expresin R = V 1 F .
Tambin podramos haber obtenido los tensores U , V , R a travs de su representacin
espectral. Es decir, si conocemos los estiramientos principales i y los autovectores de C ,
(i ) , y los autovectores de b , n (i ) , es de fcil demostracin que:
N
3
U=
a =1
a
(a ) N
N (a) = N
1
(1) N
(1) + N
2
(2) N
(2) + N
3
( 3) N
( 3)

3
V=
a =1
a n ( a ) n ( a ) = 1 n (1) n (1) + 2 n ( 2 ) n ( 2 ) + 3 n (3) n (3)

3
R= n
a =1
(a) ( a ) = n (1) N
N (1) + n ( 2) N
( 2 ) + n (3) N
( 3)

3
F=
a =1
a
( a ) = n (1) N
n ( a ) N 1
(1) + n ( 2 ) N
2
( 2 ) + n (3) N
3
( 3)

3 3
=
a =1
a R N
(a) N
(a) =

a =1
a n ( a ) n ( a ) R

3
( a ) = n ( a ) n ( a ) R
3


= R a N
a =1
(a ) N

a =1
a

= R U = V R
Como podemos verificar la representacin espectral de los tensores R y F no viene
presentada en el sentido estricto de la representacin espectral, es decir, autovalor y
autovector del tensor.

Ejemplo 2.60
El tensor gradiente de deformacin en un punto del cuerpo viene dado a travs de la
siguiente combinacin lineal de didicas:
F = 0,2e 1 e 1 0,1e 1 e 2 + 0,3e 2 e 1 + 0,4e 2 e 2 + 0,1e 3 e 3
donde e i (i = 1,2,3) representa la base cartesiana. Se pide:
a) Determinar los tensores de deformacin b y C ;
b) Determinar los autovalores y autovectores de b y C ;
c) Escribir F en su representacin espectral en funcin de los autovalores de C ( C a )
3
y verificar si se cumple que F = a n ( a ) N
( a ) , siendo los estiramientos
a
a =1

principales, n los autovectores de b , y N los autovectores de C ;


d) Obtener la representacin espectral y las componentes: del tensor de rotacin ( R )
de la descomposicin polar; y de los tensores de estiramientos U y V ;
Solucin
Las componentes del gradiente de deformacin en forma de matriz vienen dadas por:
F = Fij e i e j = 0,2e 1 e 1 0,1e 1 e 2 + 0,3e 2 e 1 + 0,4e 2 e 2 + 0,1e 3 e 3

0,2 0,1 0
Fij = 0,3 0,4 0
0 0 0,1

Universidad Castilla- La Mancha Draft Por: Eduardo W. V. Chaves (2012)


Ciudad Real - Espaa
228 PROBLEMAS RESUELTOS DE MECNICA DEL MEDIO CONTINUO

a) Las componentes del tensor izquierdo de deformacin de Cauchy-Green, b = F F T ,


vienen dadas por:
T
0,2 0,1 0 0,2 0,1 0 0,05 0,02 0

bij = Fik F jk = 0,3 0,4 0 0,3 0,4 0 = 0,02 0,25 0 (2.153)
0 0 0,1 0 0 0,1 0 0 0,01

Las componentes del tensor derecho de deformacin de Cauchy-Green, C = F T F ,


vienen dadas por:
T
0,2 0,1 0 0,2 0,1 0 0,13 0,1 0
C ij = Fki Fkj = 0,3 0,4 0 0,3 0,4 0 = 0,1 0,17
0 (2.154)
0 0 0,1 0 0 0,1 0 0 0,01
b) Determinar los autovalores y autovectores de b y C ;
C N
=C N
(a)
(a) C C1 = 0

donde el ndice (a ) no indica suma. Observemos que ya conocemos un autovalor de C ,


C (3) = 0,01 , ver componentes de C (2.154). Luego, el determinante caracterstico queda:

0,13 C 0,1
=0
0,1 0,17 C
(0,13 C )(0,17 C ) 0,01 = 0
La solucin de la ecuacin cuadrtica es:
C (1) = 0,25198 ; C ( 2) = 0,04802

Luego:
0,633399 0,77334
C (1) = 0,25198 (1)
N i = 0,77334 ; C ( 2 ) = 0,04802 (2)
N i = 0,63399
0 0
0
C (3) = 0,01 N i(3) = 0
1

Anlogamente para obtener los autovalores y autovectores del tensor b :


b n = b( a ) n ( a )

donde el ndice (a ) no indica suma.. Luego


0,098538 0,995133
b(1) = 0,25198 n i(1)= 0,995133 ; b( 2) = 0,04802 n i( 2) = 0,098538
0 0
0
b(3) = 0,01 n i(3) = 0
1

Observemos que era de esperar que los tensores C y b presentan los mismos autovalores,
pero distintos autovectores.

Universidad de Castilla- La Mancha Draft Por: Eduardo W. V. Chaves (2012)


Ciudad Real - Espaa
2 CINEMTICA DEL CONTINUO 229

0,252 0 0 0,252 0 0

C ij = 0 0,048 0 ;
bij = 0 0,048 0
0 0 0,01 0 0 0,01
Adems la representacin espectral de los tensores C y b viene dadas respectivamente
por:
3 3
C=
a =1
(a ) N
2a N (a) ; b= n
a =1
2
a
(a)
n ( a )

donde a son los estiramientos principales. Considerando que 2a = C a son los autovalores
de C y de b , los estiramientos principales son:
(1) = 0,25198 0,501976 ; ( 2) = 0,04802 0,219134 ; (3) = 0,01 = 0,1
3
c) Para verificar si se cumple F = a n ( a ) N
( a ) . Calculemos las componentes de
a =1

3
( a ) , con los resultados obtenidos anteriormente. Resultando:

a n ( a ) N

a =1

ij

3 ( a ) = n (1) N

a n ( a ) N

a =1

ij
1 i
(1) + n ( 2 ) N
j 2 i
( 2 ) + n (3) N (3)
j 3 i j

0,76958 0,6309 0 0,06247 0,0762 0


= 0,50197 0,0762 0,06247 0 + 0,219134 0,6309 0,76958 0 +

0 0 0 0 0 0
0 0 0
+ 0,10 0 0
0 0 1
0,2 0,1 0
= 0,3 0,4 0
0 0 0,1
Luego, resulta ser cierto.
d)

3
0,832 0,554 0
R= n (a)
N (a )
componentes (R )ij = 0,554 0,832 0
a =1
0 0 1
Que puede ser verificado con:
0,76958 0,6309 0 0,06247 0,0762 0 0 0 0 0,832 0,5547 0
R ij = 0,0762 0,06247 0 + 0,6309 0,76958 0 + 0 0 0 = 0,5547 0,832 0
0 0 0 0 0 0 0 0 1 0 0 1

3
0,333 0,139 0
U= N a
(a)
N (a)
componentes (U)ij 0,139 0,388 0
a =1
0 0 0,1

Universidad Castilla- La Mancha Draft Por: Eduardo W. V. Chaves (2012)


Ciudad Real - Espaa
230 PROBLEMAS RESUELTOS DE MECNICA DEL MEDIO CONTINUO

3
0,222 0,028 0
V= n a
(a)
n (a)
componentes (V )ij 0,028 0,5 0
a =1
0 0 0,1

Ejemplo 2.61
Para un movimiento dado (deformacin de corte):
x1 = X 1 + kX 2 k constante

x2 = X 2
x = X
3 3

Encontrar los tensores F (Gradiente de deformacin), C (Tensor derecho de deformacin


de Cauchy-Green), b (Tensor izquierdo de deformacin de Cauchy-Green), E (Tensor de
deformacin de Green-Lagrange), U (Tensor derecho de estiramiento), V (Tensor
izquierdo de estiramiento) y R (Tensor de rotacin de la descomposicin polar).
Solucin:
Tensor gradiente de deformacin:
x1 x1 x1

X 1 X 2 X 3 1 k 0
x x x 2 x 2
Fij = i = 2 = 0 1 0
X j X X 2 X 3
1
x3 x3 x3 0 0 1
X 1 X 2 X 3

Tensor derecho de deformacin de Cauchy-Green ( C = F T F ), cuyas componentes son:


1 0 0 1 k 0 1 k 0

C ij = Fki Fkj = k 1 0 0 1 0 = k 1 + k 2 0
0 0 1 0 0 1 0 0 1

Tensor izquierdo de deformacin de Cauchy-Green ( b = F F T ), cuyas componentes son:


1 k 0 1 0 0 1 + k 0
2
k

bij = Fik F jk = 0 1 0 k 1 0 = k 1 0
0 0 1 0 0 1 0 0 1

1
Tensor material de deformacin de Green-Lagrange, E = (C 1) , cuyas componentes
2
son:
1 k 0 1 0 0 0 k 0
1 1
E ij = k 1 + k 2 0 0 1 0 = k k 2 0
2 2
0 0 1 0 0 1 0 0 0

Verifiquemos que slo hay deformacin en el plano x1 x 2 .


Teniendo en cuenta la descomposicin polar F = R U = V R , y que:

Universidad de Castilla- La Mancha Draft Por: Eduardo W. V. Chaves (2012)


Ciudad Real - Espaa
2 CINEMTICA DEL CONTINUO 231

C = (V R)T (V R)
= RT VT V R
= RT V V R
= RT V 2 R
= RT b R
Por simplicidad vamos trabajar slo en el plano x1 x 2 , con lo cual representaremos las
componentes del tensor de rotacin como:
cos sin c s
R ij = = (i, j = 1,2)
sin cos s c
donde, se cumple que cos 2 + sin 2 = c 2 + s 2 = 1 . La relacin C = R T b R queda:
1 k c s 1 + k 2 k c s
k 1 + k 2 = s c
k 1 s c
(c 2 + c 2 k 2 + 2 sck + s 2 ) ( sck 2 s 2 k + c 2 k )
= 2 2 2
( sck s k + c k ) (c 2 + s 2 k 2 2 sck + s 2 )

k
De la relacin (c 2 + c 2 k 2 + 2 sck + s 2 ) = 1 (c 2 k 2 + 2 sck + 1) = 1 obtenemos que s =
c.
2
k
Ahora partiendo de la relacin ( sck 2 s 2 k + c 2 k ) = k y considerando que s = c
2
obtenemos que:
k
1 2 2 k
c= = ; s= =
2 2 2
k k +4 k k2 + 4
+1 +1
4 4
Luego:
2 k
2 0
2
k +4 k +4
k 2
R ij = 0
2
k +4 k2 + 4
0 0 1

De la descomposicin polar F = R U = V R , podemos obtener que U = R T F , y que


V = F R T , cuyas componentes son:

2 k 2 k
0 2 0
2
k +4 k2 + 4 1 k 0 k + 4 k2 + 4
k 2 k 2+ k2
U ij = R ki Fkj = 0 0 1 0 = 0
2
k +4 k2 + 4 2
0 0 1 k + 4 k2 + 4
0 0 1 0 0 1

Universidad Castilla- La Mancha Draft Por: Eduardo W. V. Chaves (2012)


Ciudad Real - Espaa
232 PROBLEMAS RESUELTOS DE MECNICA DEL MEDIO CONTINUO

2+k
2
2 k k
2 0 0
1 k 0 k + 4 k2 + 4 2
k +4 k2 + 4
k 2 k 2
Vij = Fik R jk = 0 1 0 0 = 0
2
0 0 1 k + 4 k2 + 4 2
k +4 k2 + 4
0 0 1 0 0 1

Ejemplo 2.62
Un paraleleppedo deformable de dimensiones 2 2 1 se encuentra en su configuracin
de referencia en la posicin que indica la Figura 2.18. Este cuerpo se somete a una
deformacin:
r r
x ( X , t ) = exp X 2t e 1 + tX 12 e 2 + X 3 e 3 (2.155)
siendo ( X 1 , X 2 , X 3 ) las coordenadas materiales y t el tiempo. Para este cuerpo se pide:
r
a) Obtener las componentes del gradiente de deformacin F , en todo punto X e
instante t .
b) Lo mismo para el tensor derecho de deformacin de Cauchy-Green C . Cules
son los estiramientos principales?
c) Obtener tambin las componentes correspondientes al tensor derecho de
estiramiento U y al tensor de rotacin R . Comprobar que este ltimo es un tensor
ortogonal propio.
d) Cul es el volumen del paraleleppedo deformado en el instante t = 1s ?
X2

X3 X1
1 2
Figura 2.18.
Solucin:
a) Segn (2.155), las componentes del vector posicin son x1 = exp X 2t , x 2 = tX 12 ,
x3 = X 3 , luego las componentes del gradiente de deformacin F vienen dadas por:

0 t exp X 2t 0
xi
Fij = = 2tX 1 0 0
X j
0 0 1

b) El tensor derecho de deformacin de Cauchy-Green C viene definido por C = F T F ,


con componentes C ij = Fki Fkj :

Universidad de Castilla- La Mancha Draft Por: Eduardo W. V. Chaves (2012)


Ciudad Real - Espaa
2 CINEMTICA DEL CONTINUO 233

0 2tX 1 0 0 t exp X 2t 0 4t 2 X 12 0 0

C ij = t exp X 2t 0 0 2tX 1 0 0 = 0 2
t exp 2 X 2t
0
0 0 1 0 0 1 0 0 1

Observemos que este espacio es el espacio de las direcciones principales de C . Si i son


los estiramientos principales se cumple la siguiente relacin:
3 3
C = U2 =
a =1
(a ) N
2a N (a) U=
a =1
a
(a) N
N (a)

Como estamos en el espacio principales, podemos obtener los estiramientos principales:


1 = + 4t 2 X 12 ; 2 = + t 2 exp 2 X 2t ; 3 = + 1
3
Ya que el tensor U = a N
(a ) N
( a ) es por definicin un tensor definido positivo,
a =1
implicando que sus valores principales son positivos, luego:
1 = 2tX 1 ; 2 = t exp X 2t ; 3 = 1
c)
1
2tX 0 0
2tX 1 0 0 1
1
U ij = 0 t exp X 2t
0 U ij1
= 0 0
t exp X 2t
0 1 0 1
0
0

Segn la descomposicin polar F = R U R = F U 1 , con eso podemos obtener las


componentes del tensor ortogonal propio R :
1
0 0
0 t exp X 2t
0 2tX 1 0 1 0
1
R ij = 2tX 1 0 0 0 0 = 1 0 0
t exp X 2t
0 1 0 0 1
1
0
0 0

Verificamos que debe cumplir la ortogonalidad R R 1 = R R T = 1 :


0 1 0 0 1 0 1 0 0
R ik R jk = 1 0 0 1 0 0 = 0 1 0
0 0 1 0 0 1 0 0 1

y propio ya que det (R ) = 1 .


d) Para calcular el volumen final utilizaremos la relacin dV = JdV0 , donde J = F es el
determinante del Jacobiano y viene dado por:
0 t exp X 2t 0
J = 2tX 1 0 0 = 2t 2 X 1exp X 2t
0 0 1

Universidad Castilla- La Mancha Draft Por: Eduardo W. V. Chaves (2012)


Ciudad Real - Espaa
234 PROBLEMAS RESUELTOS DE MECNICA DEL MEDIO CONTINUO

Para el tiempo t = 1s tenemos que J = 2 X 1exp X 2 . Luego el volumen en el tiempo t = 1s


vendr dado por:

dV = JdV
V0
0

2 2 1

(2 X exp )dX
X2
= 1 3 dX 2 dX 1
X 1 = 0 X 2 = 0 X 3 =0

= 4(exp 1) 25,556
2

Obs.: No se puede utilizar la expresin V = JV0 porque no se trata de un caso de


deformacin homognea.

Ejemplo 2.63
Un cuerpo continuo experimenta la deformacin:
x1 = X 1 ; x 2 = X 2 + kX 3 ; x3 = X 3 + kX 2
donde k es una constante.
a) Determinar el gradiente de deformacin ( F ); el tensor derecho de deformacin de
Cauchy-Green ( C ); el tensor de deformacin de Green-Lagrange ( E ).
b) Calcular el campo de desplazamiento, la longitud al cuadrado (dx) 2 de los lados
OA y OB , y de la diagonal OC , despus de la deformacin en el pequeo
rectngulo indicado en la figura abajo.

X3

dX 2
B C

dX 3

O A X2
X1

c) Considrese ahora un cuadrado como la figura abajo

x3 C

A
A C

23 B

O B
x2

Universidad de Castilla- La Mancha Draft Por: Eduardo W. V. Chaves (2012)


Ciudad Real - Espaa
2 CINEMTICA DEL CONTINUO 235

c.1) Determinar los estiramientos segn las direcciones OC y BA ; c.2) el ngulo 23 en la


configuracin actual en funcin de k .
c.3) Aplicar la descomposicin polar del tensor F , es decir, determinar U y R
Solucin:
a) C = F T F . Las componentes del gradiente de deformacin son:
1 0 0
xi
Fij = = 0 1 k
X j
0 k 1

1 0 0 1 0 0 1 0 0

C ij = Fki Fkj = 0 1 k 0 1 k = 0 1 + k 2
2k
0 k 1 0 k 1 0 2k 1 + k 2

1
Tensor material de deformacin de Green-Lagrange, E = (C 1) , cuyas componentes
2
son :
1 0 0 1 0 0 0 0 0
1 1
E ij = 0 1 + k 2
2 k 0 1 0 = 0 k 2
2k
2 2
0 2k 1 + k 2 0 0 1 0 2k k 2
r r r
b.1) Campo de desplazamientos, u = x X , cuyas componentes son:
u1 = x1 X 1 = 0 ; u 2 = x 2 X 2 = kX 3 ; u 3 = x 3 X 3 = kX 2
r
b.2) Clculo de (dx ) 2 = dx 2
(dxr )2 = dxr dxr
r r
= F dX F dX
r r
= dX F T F dX
r r
= dX C dX
Explcitamente:
1 0 0 dX 1
(dx ) 2
= [dX 1 dX 2
dX 3 ] 0 1 + k 2
2k dX 2
0 2k 1 + k 2 dX 3
= (dX 1 ) 2 + (dX 2 ) 2 (1 + k 2 ) + (dX 3 ) 2 (1 + k 2 ) + 4k (dX 2 )(dX 3 )

Luego, para la diagonal OC tenemos que [0 dX 2 dX 3 ] , resultando que:


(dx )2 = (dX 2 ) 2 (1 + k 2 ) + (dX 3 ) 2 (1 + k 2 ) + 4k (dX 2 )(dX 3 )
Para el lado OA tenemos que [0 dX 2 0] , resultando:
(dx )2 = (dX 2 ) 2 (1 + k 2 )
Para el lado OB tenemos que [0 0 dX 3 ] , resultando que:
(dx )2 = (dX 3 ) 2 (1 + k 2 )

Universidad Castilla- La Mancha Draft Por: Eduardo W. V. Chaves (2012)


Ciudad Real - Espaa
236 PROBLEMAS RESUELTOS DE MECNICA DEL MEDIO CONTINUO

c) El estiramiento segn una direccin N (configuracin de referencia) viene dado por el la


expresin ( N )2 = N
C N
.

1 1
c.1) Estiramiento segn direccin OC : N i = 0 , con lo cual:
2 2


0
1 0 0
1
( )
OC
2
= 0
1
0 1 + k
2
2k

1

= (1 + k ) 2
2 2
0 1 + k 2
2
2k 1

2

1 1
Estiramiento segn direccin BA : N i = 0 , con lo cual:
2 2


0
1 0 0
1
( ) 2
= 0
1
0 1+ k
2
2k 1
= (1 k ) 2
2
BA
2
0 2k
2
1 + k 2 1

2
c.2) Variacin del ngulo. Podemos utilizar directamente la expresin:
C N
M C N
M
cos = =
C M
M N C N
M N

= [0 0 1] , y segn direccin OA es
donde el versor segn direccin OB es M i

N i = [0 1 0] . Con eso obtenemos que:

1 0 0 0
( )
OB
2
= [0 0 1] 0 1 + k 2
2k 0 = 1 + k 2
0 2k 1 + k 2 1

1 0 0 0
( )
OA
2
= [0 1 0] 0 1 + k 2
2k 1 = 1 + k 2
0 2k 1 + k 2 0

1 0 0 0

Mi C ij N j = [0 0 1] 0 1 + k 2
2k 1 = 2k
0 2k 1 + k 2 0
Resultando que:
C N
M 2k
cos 23 = =
M N 1+ k2

c.3) Descomposicin Polar F = R U = V R , donde:

Universidad de Castilla- La Mancha Draft Por: Eduardo W. V. Chaves (2012)


Ciudad Real - Espaa
2 CINEMTICA DEL CONTINUO 237

3 3
C = U2 = a =1
(a ) N
aN (a ) U= C =
a =1
(a) N
aN (a)

Clculo de los valores principales de C . Verifiquemos que por el formato de las


componentes del tensor C , slo hay deformacin segn el plano x 2 x 3 . Adems ya
conocemos un autovalor 1 = 1 asociado a la direccin Ni(1) = [1 0 0] . Simplificando as el
determinando caracterstico como:
(1 + k 2 ) 2k
=0
2k (1 + k 2 )
( ) (
2 2 1 + k 2 + 1 2k 2 + k 4 = 0 )
2 2(1 + k ) + (1 k )
2 2 2
=0

Las races son: 2 = 1 + k 2 + 2k = (1 + k ) 2 ; 3 = 1 + k 2 2k = (1 k ) 2


Luego, en el espacio principal de C , tenemos que:
1 0 0

C ij = 0 (1 + k ) 2 0

0 0 (1 k ) 2

1 1 1 1
Las direcciones principales son 2 Ni( 2) = 0 ( 3)
, 3 N i = 0 .
2 2 2 2
Luego, la matriz de transformacin entre el espacio original y el espacio principal queda:


1 0 0
1 1
a ij = A = 0
2 2
1 1
0
2 2
Es decir, se debe cumplir que:
C = A C AT
T


1 0 0 1 0 0 1 0 0 1 0 0
0 (1 + k ) 2 1 1 1 1
0 = 0 0 1+ k2 2k 0
2 2 2 2
0 (1 k ) 2 1 + k 2
0 1 1 0 2k 1 1
0 0
2 2 2 2
Luego, en el espacio principal de C , tenemos que:
1 0 0 + 1 0 0

C ij = 0 (1 + k ) 2 0 U ij = 0 + (1 + k ) 2 0
0 0 (1 k ) 2 0 0 2
+ (1 k )

1 0 0

U ij = 0 (1 + k ) 0
0 0 (1 k )

Universidad Castilla- La Mancha Draft Por: Eduardo W. V. Chaves (2012)


Ciudad Real - Espaa
238 PROBLEMAS RESUELTOS DE MECNICA DEL MEDIO CONTINUO

La inversa en el espacio principal:




1 0 0
1
Uij1 = 0 0
(1 + k )
1
0 0
(1 k )

Las componentes del tensor U en el espacio original vienen dadas por:


U 1 = A T U 1 A
T


1 0 0 1 0 0 1 0 0 1 0 0
1 1 1 1 1 1 k
U ij = 0 0 0 0 = 0
2 2 (1 + k ) 2 2 (1 k 2 ) (1 k 2 )

1 1 1 1 1 k 1
0 0 0 0 0
2 2 (1 k ) 2 2 (1 k 2 ) (1 k 2 )

De la descomposicin polar obtenemos que F = R U R = F U 1




1 0 0 1 0 0 1 0 0
1 k
R ij = 0 1 k 0 2
= 0 1 0
(1 k 2 ) (1 k )
0 k 1
0 k 1 0 0 1
(1 k 2 ) (1 k 2 )

Ejemplo 2.64
Dada la siguiente ley de movimiento:
x1 = 1 X 1

x 2 = 3 X 3
x = X
3 2 2

Si pide:
a) Encontrar el volumen deformado para un cubo unitario;
b) Encontrar el rea deformada de un cuadrado unitario en el plano X 1 X 2 , y dibujar el
rea deformada;
c) Aplicar la Descomposicin Polar y obtener los tensores U , V y R
Solucin:
a)
x1 1 0 0 X 1 1 0 0

x2 = 0 0 3 X 2 Fij = 0 0 3 (deformacin Homognea)
x 0 2 0 X 3 0 2 0
3
El determinante de F viene dado por F J = 1 2 3 , y el volumen deformado:

Universidad de Castilla- La Mancha Draft Por: Eduardo W. V. Chaves (2012)


Ciudad Real - Espaa
2 CINEMTICA DEL CONTINUO 239

dV = F dV0 integrando
V final = F Vinicial = 1 2 3

b) Aplicando la relacin de Nanson y teniendo en cuenta que estamos en el caso particular


de deformacin homognea:
r r r r
da = JF T dA integrando
a final = JF T Ainicial

donde
1
0 0
e 1 e 2 e 3 2 3 0 0 1
r 1 0 1
Ainicial = 1 0 0 = e 3 ; Fij1 = 0 1 3 = 0 0
1 2 3 2
0 1 0 0 1 2 0
0 1
0
3
Con lo cual el vector rea deformada queda:
1
0 0
a1 1 0 0
1
a 2 = 1 2 3 0 0
0 = 1 2
a 3
3 1 0
0 1
0
2
Su mdulo queda:
r
a final = ( 1 2 ) 2 = 1 2

X 3 , x3
B (0,0, 2 )
r
a final = 1 2

C (1 ,0, 2 )

O (0,0,0) B (0,1,0) X 2 , x2

r
Ainicial = 1
C (1,1,0)
A(1,0,0)
A( 1 ,0,0)

X 1 , x1

donde los puntos A(1,0,0) , B(0,1,0) y C (1,1,0) se desplazan segn la ley del movimiento:

Universidad Castilla- La Mancha Draft Por: Eduardo W. V. Chaves (2012)


Ciudad Real - Espaa
240 PROBLEMAS RESUELTOS DE MECNICA DEL MEDIO CONTINUO

x1A 1 0 0 1 1 x1B 1 0 0 0 0
A B
x2 = 0 0 3 0 = 0 ; x2 = 0 0 3 1 = 0
x A 0 2 0 0 0 x B 0 2 0 0 2
3 3
x1C 1 0 0 1 1
C
x2 = 0 0 3 1 = 0
xC 0 2 0 0 2
3
c) Segn la definicin de la descomposicin polar F = R U = V R donde
U= C = F T
F y V = b = F F T

1 0 0 1 0 0 21 0 0 1 0 0

C ij = 0 0 2 0 0 3 = 0 22 0 U ij = 0 2 0
0 3 0 0 2 0 0 0 23 0 0 3

1 0 0 1 0 0 21 0 0 1 0 0

bij = 0 0 3 0 0 2 = 0 23 0 Vij = 0 3 0
0 2 0 0 3 0 0 0 22 0 0 2

Verifiquemos que el espacio original coincide con el espacio principal de C . Verifiquemos


tambin que C y b tienen los mismos autovalores pero direcciones principales distintas.
Para obtener el tensor de rotacin de la descomposicin polar R = F U 1 = V 1 F , con lo
cual:
1
0 0
1 0 0 1 1 0 0
1
R ij = 0 0 3 0 0 = 0 0 1
2
0 2 0
0 1 0 1 0
0
3
1
0 0
1 1 0 0 1 0 0
1
R ij = 0 0 0 0 3 = 0 0 1
3
2 0 0 1 0
0 1 0
0
2

Ejemplo 2.65
Determinar para la deformacin homognea:
x1 = 3 X 1

x2 = 2 X 2

x3 = 3 X 3 X 2
el elipsoide de deformacin material que resulta de la deformacin de una esfera material
X 12 + X 22 + X 32 = 1 (ver Figura 2.19). Probar que este elipsoide en el espacio principal del
tensor izquierdo de estiramiento V tiene la forma:

Universidad de Castilla- La Mancha Draft Por: Eduardo W. V. Chaves (2012)


Ciudad Real - Espaa
2 CINEMTICA DEL CONTINUO 241

x1 2 x 2 2 x3 2
+ + =1
21 22 23
donde 1 , 2 , 3 son los estiramientos principales.

X 2 , x2
X 3 , x3
Superficie material
(Siempre constituida por las
mismas partculas)

X 1 , x1
Figura 2.19: Esfera material.

Solucin:
La ley del movimiento y de su inversa vienen dadas por:
3
0 0
x1 3 0 0 X 1 X1 3 x1
1
x2 = 0 2 X 2 =
0 X 2 inversa 0 0 x2
x 0 1 3 X 3 X 2
3 3 3 3 x3
0
6 3
La ly del movimiento en la descripcin espacial viene dada por:
3
X1 = x1
3
x2
X 2 =
2
3 3
X 3 = x2 + x3
6 3
Reemplazando en la ecuacin de la esfera:
X 12 + X 22 + X 32 = 1
2 2 2
3 x 3 3
x1 + 2 + x2 + x3 = 1
3 2 6 3

Tras la simplificacin de la expresin anterior obtenemos que:
x12 + x 22 + x32 + x 2 x3 = 3

Universidad Castilla- La Mancha Draft Por: Eduardo W. V. Chaves (2012)


Ciudad Real - Espaa
242 PROBLEMAS RESUELTOS DE MECNICA DEL MEDIO CONTINUO

Que es la ecuacin de un elipsoide. Tenemos ahora que representar la ecuacin de este


elipsoide en el espacio principal del tensor derecho de estiramiento V . Recordemos que el
tensor V y el tensor b son coaxiales (tienen las mismas direcciones principales), y adems
se cumple que:
V = b = F FT
A continuacin obtenemos las componentes del tensor b , y sus autovalores y autovectores.
T
3 0 0 3 0 0 3 0 0

bij = 0 2 0 0 2 0 = 0 5 3
0 1
3 0 1 3 0 3 3

Verificamos que ya conocemos un autovalor y autovector b1 = 3 , n i(1) = [1 0 0] . Luego,
las otras direcciones principales estarn el plano x 2 x 3 . Obteniendo los dems
autovalores y autovectores
2 2
n (i 2 ) = 0
b2 = 6 autovector
2 2

2 2
n (i 3) = 0
b3 = 2 autovector
2 2

Resultando as que:

1 0 0
3 0 0 2 2
bij = 0 6 0 Matriz
de a ij = 0
Trasnforamcin
2 2
0 0 2 2 2
0
2 2
1 = 3 0 0

Vij = 0 2 = 6 0
0 0 3 = 2

Luego, aplicando la ley de transformacin del sistema x1 , x 2 , x3 al sistema x1 , x 2 , x3 ,
obtenemos que:


T
x1 = x1
x1 1 0 0 x1
2 2 2 2
x2 = 0 x 2 x2 = x 2 + x3
x 2 2 x 2 2
3 2 2 3
0 2 2
2 2 x3 = x 2 + x3
2 2
Con lo cual, la ecuacin del elipsoide en el espacio principal de V viene representada por:
x12 + x 22 + x32 + x 2 x3 = 3
2 2
2 2 2 2 2 2 2 2
(x1 ) 2
+
2
x 2 + x3 +
2
x 2 + x3 +
2
x 2 + x3
2
x 2 + x 3 = 3

2 2 2 2

Universidad de Castilla- La Mancha Draft Por: Eduardo W. V. Chaves (2012)


Ciudad Real - Espaa
2 CINEMTICA DEL CONTINUO 243

Simplificando la expresin anterior obtenemos que:


x1 2 x 2 2 x3 2 x 2 x2 x 2 x2 x2 x2
+ + = 1 2 + 2 2 + 3 2 = 12 + 22 + 32 = 1
3 6 2 ( 3) ( 6) ( 2) 1 2 3

X 3 , x3
X 2 , x2
x3

3 = 2

2 = 6
1 = 3 x 2

x1

X 1 , x1

Figura 2.20: Elipsoide material (configuracin deformada).

x2

x1
x3
R V

x 2
X2 x2

F x1 x1
X1 x3
X3

x3

Figura 2.21: Descomposicin polar por la izquierda.

Universidad Castilla- La Mancha Draft Por: Eduardo W. V. Chaves (2012)


Ciudad Real - Espaa
244 PROBLEMAS RESUELTOS DE MECNICA DEL MEDIO CONTINUO

2.1.4 Deformacin Infinitesimal

Ejemplo 2.66
Dadas las ecuaciones del movimiento
x1 = X 1 + 4 X 1 X 2 t
2
x2 = X 2 + X 2 t (2.156)
2
x3 = X 3 + X 3 t
Se pide:
a) Encontrar el campo de velocidad;
b) Encontrar el campo de deformacin infinitesimal;
c) Para el tiempo t = 1 s , obtener el tensor de deformacin infinitesimal.
Solucin:
a) Velocidad:

r V1 = 4 X 1 X 2
r r dx
V ( X , t) = V 2 = X 22 (2.157)
dt 2
V3 = X 3
b) Aceleracin:
r A1 = 0
r r dV
A( X , t ) = A2 = 0 (2.158)
dt A = 0
3
c) Campo de desplazamientos:
u1 = x1 X 1 = X 1 + 4 X 1 X 2 X 1 = 4 X 1 X 2
2 2
u 2 = x2 X 2 = X 2 + X 2 X 2 = X 2 (2.159)
2 2
u3 = x3 X 3 = X 3 + X 3 X 3 = X 3
Luego, las componentes del tensor de deformacin infinitesimal vienen dadas por:

1 u i u j

ij = + (2.160)
2 x j xi

u1 u1 u1

X 1 X 2 X 3 4 X
2 4X1 0
u i u 2 u 2 u 2
= = 0 2X 2 0 (2.161)
x j X 1 X 2 X 3
2 X 3
u 3 u 3 u 3 0 0
X 1 X 2 X 3

luego:

Universidad de Castilla- La Mancha Draft Por: Eduardo W. V. Chaves (2012)


Ciudad Real - Espaa
2 CINEMTICA DEL CONTINUO 245

4 X 2 2 X1 0
ij = 2 X 1 2X 2 0 (2.162)
0 0 2 X 3

Ejemplo 2.67
Consideren el siguiente tensor de deformacin infinitesimal:

0 0 0
2
X X X
ij = 0 2 2 3 23 (2.163)
l l
X2 X2X3
0 3
l2 l 2
y el tensor de rotacin infinitesimal:

0 0 0

ij = 0

0 ( X 22 X 32

) (2.164)
2l 2

2 2
0 2 X 2 X 3 ( ) 0


2l
Hallar las componentes del campo de desplazamientos.
Solucin:
El gradiente de los desplazamientos viene relacionado con el tensor de deformacin
infinitesimal y el tensor spin como:
u i , j = ij + ij
(2.165)
ij =
1
2
(
ui, j + ui, j ) ; ij =
1
2
(
ui, j ui, j )
luego:
0 0 0

u i , j = 2 0 2X 2 X 3 X 22 2
3X 3 (2.166)
2l
0 (X 22 + X 32 ) 2 X 2 X 3

u1
=0
u1 = 0 (2.167)
x1
u 2
= 2 (2 X 2 X 3 )
x 2 2l
(2.168)

u 2 =

2l (2 X
2 2X3 )x 2 u 2 =
2l 2
[X 22 X 3 + C1 ( X 3 ) ]

Universidad Castilla- La Mancha Draft Por: Eduardo W. V. Chaves (2012)


Ciudad Real - Espaa
246 PROBLEMAS RESUELTOS DE MECNICA DEL MEDIO CONTINUO

u 3
= 2 (2 X 2 X 3 )
x3 2l
(2.169)

u 3 = 2l 2

(2 X 2 X 3 )x3 u 3 =
2l 2
[
X 32 X 2 + C2 ( X 2 ) ]
Para determinar la constante C1 ( X 3 ) del resultado (2.168) derivamos con respecto a X 3 :

u 2
= 2
X 3 2l
2 C1 ( X 3 )
X 2 +
X
2 2
= 2 X 2 3X 3
X
[
C1 ( X 3 )
= 3 X 32 ]
3 2 l 3 (2.170)
C1 ( X 3 ) = X 33

Anlogamente hacemos para determinar la constante C2 ( X 2 ) :

u 3
X 2

= 2
2l
2 C 2 ( X 2 )
X 3 +
X
2 2
= 2 X2 + X3
2 X
[
C 2 ( X 2 )
= X 22 ]
2 l 2
(2.171)
X3
C2 ( X 2 ) = 2
3
Luego, el campo de desplazamientos viene dado por:

u1 = 0 ; u 2 =

2l 2
[X 2
2 X3 X 33 ] ; u3 =


2l 2
X 2
3 X 2 +
X 23
3
(2.172)

Ejemplo 2.68
Demostrar que, para el caso de pequeas deformaciones, la tasa del tensor de deformacin
infinitesimal ( & ) es igual al tensor tasa de deformacin ( D ).
Solucin:
Consideremos la relacin entre la tasa del tensor material de deformacin de Green-
Lagrange ( E& ) y el tensor tasa de deformacin ( D ):
E& = F T D F (2.173)
Para el caso de pequeas deformaciones se cumple que F 1 , y adems se cumple
tambin que E& e& & luego:
E& = & = D (2.174)
Ejemplo 2.69
Dado el movimiento
x1 = X 1
2t
x 2 = X 2 + X 1 exp 1 ( ) (2.175)
3t
x3 = X 3 + X 1 exp 1 ( )
Encontrar el tensor tasa de deformacin ( D ) y compararlo con la tasa del tensor
infinitesimal de deformacin ( & ).
Solucin:

Universidad de Castilla- La Mancha Draft Por: Eduardo W. V. Chaves (2012)


Ciudad Real - Espaa
2 CINEMTICA DEL CONTINUO 247

Por definicin el tensor tasa de deformacin ( D ) es la parte simtrica del tensor gradiente
espacial de la velocidad:
1
D= (l + l T ) r r
2 y ( x , t ) = sym u (2.176)
r
l = xv

El tensor infinitesimal de deformacin por definicin es igual la parte simtrica del


gradiente de los desplazamientos:
r D
= (u) sym & (2.177)
Dt
r r r
El campo de desplazamientos viene dado por u = x X . Considerando las ecuaciones del
movimiento dadas, las componentes del campo de desplazamiento quedan:
u1 = x1 X 1 = X 1 X 1 = 0

(
2t
)
u 2 = x 2 X 2 = X 2 + X 1 exp 1 X 2 = X 1 exp 1
2t
( )

(
3t
)
u 3 = x 3 X 3 = X 3 + X 1 exp 1 X 3 = X 1 exp 1
3t
( )
r
r Du
El campo de velocidades viene definido por v = . Luego, las componentes del campo
Dt
de velocidades, en coordenadas materiales, son:
V1 = 0
2t
V2 = X 1 (2exp ) (2.178)
3t
V3 = X 1 ( 3exp )
Teniendo en cuenta las ecuaciones inversas del movimiento:
x1 = X 1 X 1 = x1
2t 2t
x 2 = X 2 + X 1 (exp 1) X 2 = x 2 x1 (exp 1) (2.179)
3t 3t
x3 = X 3 + X 1 (exp 1) X 3 = x3 x1 (exp 1)
podemos obtener el campo de velocidades en coordenadas espaciales:
v1 = 0
2t
v 2 = 2 x1 exp (2.180)
3t
v3 = 3x1exp

Las componentes del tensor gradiente espacial de la velocidad ( l ) vienen dadas por:

0 0 0
vi
0 0
r
( l ) ij = ( x v ) ij = = 2exp 2t (2.181)
x j
3exp 3t 0 0

Universidad Castilla- La Mancha Draft Por: Eduardo W. V. Chaves (2012)


Ciudad Real - Espaa
248 PROBLEMAS RESUELTOS DE MECNICA DEL MEDIO CONTINUO

0 0 0 0 0 0
T

1 1
(D) ij = ( l ij + l ji ) = 2exp 2t 0 0 + 2exp 2t 0 0
2 2
3exp 3t 0 0 3exp 3t 0 0

3 (2.182)
0 exp 2t exp 3t
2

= exp 2t 0 0
3 exp 3t 0 0
2

Tambin obtenemos el tensor spin W = l anti

3
0 exp 2t exp 3t
2
Wij =
1
2
(
l ij l ji )
= exp 2t 0 0

(2.183)
3 exp 3t 0 0
2

Tensor de deformacin Infinitesimal ( )


Conocido el campo de desplazamiento:
u1 = 0
2t
u 2 = x1 (exp 1) (2.184)
3t
u 3 = x1 (exp 1)
Las componentes del gradiente de desplazamientos vienen dadas por:
0 0 0
r u i
(u)ij = = exp 1 0 0
2t
( ) (2.185)
x j
exp 3t 1 0 0 ( )
r
Podemos decomponer (u) en una parte simtrica y una antisimtrica:

(u)ij = ( symu)ij + ( anti u)ij


r r r
(2.186)
= ( )ij + ()ij

La parte simtrica:
0 0 0 0 0 0
T

1
(sym r
u ij = )
2
(
2t
exp 1 ) 2t
0 0 + exp 1 ( ) 0 0
(
exp 3t 1

) (
0 0 exp 3t 1 ) 0 0


(2.187)
0 exp 2t 1 exp 3t 1
1
= exp 2t 1 0 0 = ij
2 3t
exp 1 0 0
Tambin proporcionamos el tensor spin infinitesimal:

Universidad de Castilla- La Mancha Draft Por: Eduardo W. V. Chaves (2012)


Ciudad Real - Espaa
2 CINEMTICA DEL CONTINUO 249

0 (exp 2t 1) (exp 3t 1)0


1
( )ij = (exp 2t 1) 0 0 (2.188)
2 3t
(exp 1) 0 0

Luego, la tasa de :
0 exp 2t 1 exp 3t 1
D D 1
(& )ij = ( )ij = exp 2t 1 0 0
Dt Dt 2
exp 1 3t
0 0

3 (2.189)
0 exp 2t exp 3t
2

= exp 2t 0 0
3 exp 3t 0 0
2

Con lo que concluimos que:


D = & (2.190)

Ejemplo 2.70
Consideremos un cuerpo material bajo el rgimen de pequeas deformaciones, el cual est
sometido al siguiente campo de desplazamientos:
u1 = (2 x1 + 7 x 2 ) 10 3 ; u 2 = (10 x 2 x1 ) 10 3 ; u 3 = x3 10 3
a) Encontrar el tensor de deformaciones infinitesimales, y el tensor spin infinitesimal;
b) Encontrar los invariantes principales del tensor de deformacin infinitesimal, y las
deformaciones principales;
c) Dibujar el crculo de Mohr en deformaciones, y obtener la deformacin tangencial
mxima;
d) Encontrar la deformacin volumtrica lineal y el tensor de deformacin infinitesimal
desviador.
Solucin
a) El gradiente de los desplazamientos:
u1 u1 u 1

x1 x 2 x 3 2 7 0
u i u 2 u 2 u 2
( u)ij = = = 1 10 0 10 3
x j x1 x 2 x 3
1
u 3 u 3 u 3 0 0
x1 x 2 x 3
Tensor spin infinitesimal:
0 4 0
1 u u j
ij = ( anti
)
u ij = i
2 x j x i
= 4 0 0 10 3

0 0 0

Universidad Castilla- La Mancha Draft Por: Eduardo W. V. Chaves (2012)


Ciudad Real - Espaa
250 PROBLEMAS RESUELTOS DE MECNICA DEL MEDIO CONTINUO

Tensor de deformacin infinitesimal:


2 3 0
1 u u j
ij = ( sym
)
u ij = i +

2 x j xi
= 3 10 0 10 3

0 0 1

b) Las deformaciones principales (autovalores) se obtienen al resolver el determinante
caracterstico:
2 3 0
3 10 0 10 3 = 0

0 0 1

donde = 10 3 . Al desarrollar el determinante anterior obtenemos la ecuacin


caracterstica 3 I 2 + II III = 0 , donde los invariantes principales de vienen
definidos por I = Tr ( ) , II =
1
2
{ }
[Tr()]2 Tr ( 2 ) , III = det() , ver captulo 1. Luego,
para el problema propuesto los invariantes son:
I = Tr ( ) = (2 10 + 1) 10 3 = 11 10 3
2 3 0 2 3 0 2 3 0

1
{ }
II = [Tr ( )] Tr ( ) = 3 10 0 + 3 10 0 + 3 10 0 10 6 = 1 10 6
2
2 2

0 0 1 0 0 1 0 0 1

III = det ( ) = 11 10 9
Resultando en la siguiente ecuacin caracterstica:
3 I 2 + II III = 0
3 + 11 10 3 2 + 11 10 6 11 10 9 = 0
Al resolver la ecuacin anterior obtenemos los autovalores de (deformaciones
principales). Pero, si nos fijamos en el formato de las componentes de verificamos que
33 = 1 10 3 ya es una deformacin principal y que est asociada a la direccin
n i = [0 0 1] . Luego, para obtener los dems autovalores es suficiente resolver el
siguiente sistema:
2 3 3
3 10 = 0 2 + 12 10 3 + 11 10 6 = 0
10
1 = 1,0 10 3
2 + 12 10 3 + 11 10 6 = 0
2 = 11,0 10 3

c) Para dibujar el crculo de Mohr en deformaciones, ver Apndice A, tenemos que


reestructurar las deformaciones principales tal que I II III , i.e.:
I = 1,0 10 3 ; II = 1,0 10 3 ; III = 11,0 10 3
La deformacin tangencial mxima viene dada por:
1 III
max = S max = I = 6 10 3
2 2

Universidad de Castilla- La Mancha Draft Por: Eduardo W. V. Chaves (2012)


Ciudad Real - Espaa
2 CINEMTICA DEL CONTINUO 251

El crculo de Mohr en deformaciones se puede apreciar en la figura abajo.

S (10 3 )

S max = 12 max = 6

II = 1

III = 11
I = 1 N (10 3 )

d) La deformacin volumtrica lineal - V :


V = I = Tr ( ) = 12 10 3
Haciendo la descomposicin aditiva de en una parte esfrica y otra desviadora,
= esf + dev , donde la parte esfrica viene dada por:
4 0 0
Tr ( )
ijesf = ij = 0 4 0 10 3

3
0 0 4
Y, la parte desviadora por:
2 3 0 4 0 0 2 3 0

ijdev = ij ijesf = 3 10 0 0 4 0 10 = 3 6 0 10 3
3

0 0 0 0 0 4 0 0 4

Ejemplo 2.71
En un punto de un slido el gradiente de los desplazamientos viene representado por sus
componentes como:
4 1 4
r
(u) ij = 1 4 2 10 3 (2.191)
4 0 6

Determinar:
a) Las componentes del tensor infinitesimal de deformacin y rotacin;
b) Las componentes de la parte esfrica y desviadora del tensor infinitesimal de
deformacin;
c) Los invariantes principales de : I , II , III ;
d) Los autovalores y autovectores del tensor de deformacin.

Universidad Castilla- La Mancha Draft Por: Eduardo W. V. Chaves (2012)


Ciudad Real - Espaa
252 PROBLEMAS RESUELTOS DE MECNICA DEL MEDIO CONTINUO

Solucin:
a) El tensor infinitesimal de deformacin ( ) viene dado por la parte simtrica del
gradiente de los desplazamientos:
r 1
2
r
[ r
= sym u = (u) + (u) T ] (2.192)

Luego:
4 1 4 4 1 4 8 0 0 4 0 0
1 1

ij = 1 4 2 + 1 4 0 10 = 0 8 2 10 = 0 4 1 10 3
3 3
2 2
4 0 6 4 2 6 0 2 12 0 1 6
r
El tensor spin infinitesimal = anti u
4 1 4 4 1 4 0 2 8 0 1 4
1 1
3
ij = 1 4 2 1 4 0 10 = 2 0
2 10 = 1 0
3
1 10 3
2 2
4 0 6 4 2 6 8 2 0 4 1 0

b) Descomponiendo de forma aditiva el tensor en una parte esfrica y una parte


desviadora:
= esf + dev (2.193)
donde la parte esfrica viene dada por:
2 0 0
Tr ( ) 6 10 3
esf
= 1= 1 = 2 10 31 ijesf = 0 2 0 10 3 (2.194)
3 3
0 0 2
La parte desviadora viene dada por:
4 0 0 2 0 0 2 0 0
ijdev 3
= 0 4 1 10 0 2 0 10 = 0 6 1 10 3
3
(2.195)
0 1 6 0 0 2 0 1 4

c) Los invariantes principales del tensor son:


I = Tr ( ) = 6 10 3
4 1 4 0 4 0
II = + + 10 6 = 17 10 6
(2.196)
1 6 0 6 0 4
III = (4 (4) 6 4 ) 10 9 = 100 10 9

d) Teniendo en cuenta las componentes del tensor de deformacin:


4 0 0
ij = 0 4 1 10 3 (2.197)
0 1 6

Ya verificamos que 1 = 4 10 3 es un autovalor y la direccin [ 1,0,0] es el autovector


asociado a 1 . Para encontrar los dems autovalores hay que obtener la solucin del
determinante caracterstico:

Universidad de Castilla- La Mancha Draft Por: Eduardo W. V. Chaves (2012)


Ciudad Real - Espaa
2 CINEMTICA DEL CONTINUO 253

4 1
=0 (2.198)
1 6

(4 )(6 ) 1 = 0
2 2 25 = 0
2
b b 2 4ac 2 (2) 4 1 (25) 2 4 + 4 25
= = = = 1 26
2a 2 1 2 (2.199)

1 = 6,0990

2 = 4,099
Luego:

1 = 4 10 3 ; 2 = 6,0990 10 3 ; 3 = 4,099 10 3 (2.200)

Reestructurando las deformaciones:

I = 6,0990 10 3 ; II = 4 10 3 ; III = 4,099 10 3 (2.201)

Ejemplo 2.72
Encontrar el tensor de deformacin infinitesimal y el tensor de rotacin infinitesimal para
el siguiente campo de desplazamiento:
x12

u i = x1 x 2
0

Solucin:
Tensor de deformacin Infinitesimal
En el rgimen de pequeas deformaciones, el tensor de deformacin viene dado por:
1 u i u j

E ijL eijL ij = +
2 x j x i

Tenemos que hallar el gradiente del desplazamiento:
u 1 u1 u1

x1 x 2 x 3 2 x
1 0 0
u j u 2 u 2 u 2
= = x x1 0
x k x1 x 3
2
x 2
0 0 0
u 3 u 3 u 3
x1 x 2 x 3
Con eso podemos obtener:

Universidad Castilla- La Mancha Draft Por: Eduardo W. V. Chaves (2012)


Ciudad Real - Espaa
254 PROBLEMAS RESUELTOS DE MECNICA DEL MEDIO CONTINUO

x2
1 u u j 2 x 0 0 2 x1 x2 0 2 x1 0
E ijL eijL ij = i + 1 1 2

2 x j x i = x2 x1 0 + 0 x1 0 = x 2
2 x1 0
0 0 0 0 0 0 2
0 0 0
Tensor de rotacin infinitesimal:
1 u i u j

ij =
2 x j x i

x2
0 0
2 x1 0 0 2 x1 x2 0 2

1 x
= x2 x1 0 0 x1 0 = 2 0 0
2 2
0 0 0 0 0 0 0 0 0

Ejemplo 2.73
En la Figura 2.22 se muestra la transformacin que experimenta el cuadrado ABCD de
lado unitario.

X 2 , x2
x2
D
C
D
1
45
B C
A = A X 1 , x1
1
B
x1
Figura 2.22: Cuerpo sometido a una rotacin.
Se pide:
a) Plantear las ecuaciones del movimiento;
b) Es vlida la teora de pequeas deformaciones? Probar y Justificar;
c) Es vlida la teora de deformacin finita (grandes deformaciones)? Probar.
Solucin:
La ley de transformacin entre los sistemas x x viene dada por:
2 2
0
x1 cos sin 0 x1 x1 2 2 x1
= 45 2 2
x 2 = sin cos 0 x 2 x 2 = 0 x 2
x 0 2 2
3 0 1 x3 x
3 0 0

1 x 3

Universidad de Castilla- La Mancha Draft Por: Eduardo W. V. Chaves (2012)


Ciudad Real - Espaa
2 CINEMTICA DEL CONTINUO 255

Si consideramos los sistemas materiales y espaciales superpuestos, las ecuaciones de


movimiento quedan definidas por la inversa de la expresin anterior, es decir, x x :
2 2
0
x1 2 2 X 1
2 2
x 2 = 0 X 2
x 2 2
3 0 0 1 X 3

2 2
x1 = X1 + X2
2 2

2 2
x 2 = 2 X 1 + 2 X 2

Por ejemplo, el punto C en la configuracin de referencia tiene coordenadas materiales


2 2 2 2
X 1C = 1 , X 2C = 1 . Tras el movimiento: x1C = (1) + (1) = 2 , x 2C = (1) + (1) = 0
2 2 2 2
Campo de desplazamientos:
2 2 2 2
u1 = x1 X 1 = X1 X 2 X 1 = X 1 1 X2
2 2 2 2

2 2 2 2
u 2 = x 2 X 2 = X1 + X2 X2 = X 1 + X 2 1
2 2 2 2
Gradiente material de los desplazamientos:
u1 u1 u1 2 2
1 0
X 1 X 2 X 3 2 2
u i u 2 u 2 u 2 2 2
= = 1 0
X j X 1 X 2 X 3 2 2

u 3 u 3 u 3
X 1 X 2 X 3 0 0 0

El tensor de deformacin infinitesimal viene definido como = symu =


r 1
2
[ r r
]
(u) + (u)T ,
con o obtenemos:
2
1 0 0
2
2
ij = 0 1 0 0 ij
2
0 0 0

Como para un movimiento de slido rgido el tensor de deformacin tiene que ser cero, es
decir, = 0 (tensor de deformacin infinitesimal), E = 0 (tensor de deformacin de
Green-Lagrange), e = 0 (tensor de deformacin de Almansi). Calculando las componentes
del tensor de deformacin de Green-Lagrange:

Universidad Castilla- La Mancha Draft Por: Eduardo W. V. Chaves (2012)


Ciudad Real - Espaa
256 PROBLEMAS RESUELTOS DE MECNICA DEL MEDIO CONTINUO

0 0 0
1 u u j u k u k
= 0 0 0
E ij = i + +
2 X j X i X i X j
0 0 0

Ejemplo 2.74
Un rectngulo de base y altura b se gira en sentido antihorario 30 . Tras el giro el
rectngulo sufre una deformacin de tal forma que la base mantiene su longitud inicial y la
altura se dobla. Calcular el gradiente de deformacin, el tensor derecho de deformacin de
Cauchy-Green, y el tensor de deformacin de Green-Lagrange.
Solucin:
X 2 , x2
x2 C

D C x1
2b 30
B
30 B
A = A b X 1 , x1

Figura 2.23: Cuerpo sometido rotacin/deformacin.

Fijaros que podemos hacer la descomposicin del movimiento por: primero una
deformacin y a continuacin una rotacin:
El movimiento de deformacin
X 2 , x2 viene gobernado por el tensor
derecho de estiramiento de la
descomposicin polar:
D C
1 0 0
U ij = 0 2 0
0 0 1
2b D C
donde hemos aplicado la definicin
del estiramiento. Fijemos que son
los propios estiramientos
principales. A continuacin
B B
aplicamos una rotacin, donde las
b X 1 , x1 componentes del tensor R son las
A = A mismas que la matriz de
r
transformacin del sistema x al
r
sistema x :

Universidad de Castilla- La Mancha Draft Por: Eduardo W. V. Chaves (2012)


Ciudad Real - Espaa
2 CINEMTICA DEL CONTINUO 257

cos sin 0
R ij = sin cos 0
0 0 1

Luego, aplicando la descomposicin polar por la derecha F = R U :


cos sin 0 1 0 0 cos 2 sin 0
Fij = R ik U kj = sin cos 0 0 2 0 = sin 2 cos 0
0 0 1 0 0 1 0 0 1
Para el problema propuesto, tenemos que:
cos 30 2 sin 30 0
Fij = sin 30 2 cos 30 0
0 0 1
r r r
Como se trata de un caso de deformacin homognea se cumple x = F X + c , en este
r r
caso con c = 0 . Por ejemplo, para una partcula que en la configuracin de referencia
ocupaba el punto D , en la configuracin actual estar segn:
x1D cos 30 2 sin 30 0 X 1D cos 30 2 sin 30 0 0 2b sin 30
D D
x 2 = sin 30 2 cos 30 0 X 2 = sin 30 2 cos 30 0 b = 2b cos 30
x D 0 0 1 X 3D 0 0 1 0 0
3
hecho que se puede comprobar fcilmente a travs de la Figura 2.23.
A travs de la definicin del tensor derecho de deformacin de Cauchy-Green, C = F T F ,
podemos obtener las componentes cartesianas:
cos sin 0 cos sin 0 1 0 0
C ij = Fki Fkj = sin cos 0 sin cos 0 = 0 4 0
0 0 1 0 0 1 0 0 1

1
El tensor de deformacin de Green-Lagrange, E = (C + 1) , y sus componentes
2
cartesianas quedan:
1 0 0 1 0 0 0 0 0
1
E ij = 0 4 0 0 1 0 = 0 1,5 0
2
0 0 1 0 0 1 0 0 0

Observemos que el espacio original coincide con el espacio principal de deformacin.
Tambin podamos haber obtenido las componentes de C y E a travs de sus
3 3
1
representaciones espectrales: C = 2a N
(a) N
(a) , E =
(2a 1)N (a ) N ( a) , donde
a =1 a =1 2
a son los estiramientos principales.

Universidad Castilla- La Mancha Draft Por: Eduardo W. V. Chaves (2012)


Ciudad Real - Espaa
258 PROBLEMAS RESUELTOS DE MECNICA DEL MEDIO CONTINUO

2.2 Ejercicios Propuestos

Problema 2.1
Considere el movimiento del continuo dado por las siguientes ecuaciones:

x1 = X 1 (1 + t 3 )

x2 = X 2 (2.202)
x = X
3 3

donde es una constante. Determinar los campos de desplazamiento, velocidad y


aceleracin en las descripciones material y espacial.

Problema 2.2
Un movimiento del medio continuo viene definido por las siguientes componentes de la
velocidad:
3 x1 x2 5 x32
v1 = ; v2 = ; v3 = (2.203)
1+ t 1+ t 1+ t
r r
Asumiendo que la configuracin de referencia (t = 0) se cumple que x = X .
Se pide:
a) Obtener la trayectoria de la partcula;
b) Expresar las componentes de la velocidad en coordenadas materiales;
c) Obtener las componentes de la aceleracin en coordenadas espaciales y materiales.

Problema 2.3
Un cuerpo se halla bajo deformacin homognea.
3
x1 = 2 X 1 + 4 2 X 2

3 1
x2 = X 1 + X 2 + 2 X3 (2.204)
4 4
3 1
x3 = X 1 4 X 2 + 4 2 X 3

Encontrar:
a) La direccin del elemento de lnea en la configuracin deformada cuyo elemento de
lnea en la configuracin de referencia estaba segn la direccin (1,1,1) ;
b) El estiramiento del elemento de lnea.

Universidad de Castilla- La Mancha Draft Por: Eduardo W. V. Chaves (2012)


Ciudad Real - Espaa
2 CINEMTICA DEL CONTINUO 259

Problema 2.4
Dados los siguientes campos de desplazamientos en el plano x y :
r 1 1
a) u = x 2 e 1 + x1 e 2
2 2
r 1 1
b) u = x 2 e 2 + x1 e 2
2 2
r
c) u = x1 e 2
Para cada campo de desplazamiento con << 1 , dibujar los desplazamientos que sufren las
partculas que originalmente estaban en un cuadrado determinado por x1 = 0 , x1 = 1 ,
x2 = 0 y x2 = 1 . Ver Figura 2.24

x2

1 t=0

O 1 x1

Figura 2.24: Dominio en la configuracin de referencia

Problema 2.5
Para un campo de desplazamientos dado en coordenadas cilndricas por:
r
u = a r e r + b r z e + c sin e z (2.205)
donde a , b y c son constantes. Determinar las componentes del tensor de deformacin
infinitesimal en coordenadas cilndricas.

Problema 2.6
r r r
Si el campo de aceleracin a es la derivada material v& del campo de la velocidad v ,
encontrar la aceleracin en el punto (1,1,0) en el tiempo t = 0 si la velocidad en la
descripcin Euleriana viene dada por:
r
[ ]
v = C ( x13 + x1 x22 )e 1 ( x12 x2 + x23 )e 2 exp at (2.206)

donde C y a son constantes.

Problema 2.7
El campo de velocidad de un fluido viene dado por:
r
v = x1e 1 + (2 x1 x 2 )e 2 + ( x1 x3 )e 3 (2.207)

Universidad Castilla- La Mancha Draft Por: Eduardo W. V. Chaves (2012)


Ciudad Real - Espaa
260 PROBLEMAS RESUELTOS DE MECNICA DEL MEDIO CONTINUO

y la distribucin de temperatura es:


T = 3 x 2 + x3 t (2.208)
Encontrar la tasa de cambio en el tiempo de la temperatura.

Problema 2.8
Un movimiento de un fluido viene representado por las siguientes ecuaciones:
x1 = X 1 + X 2 t + X 3t 2
2
x2 = X 2 + X 3t + X 1t (2.209)
2
x3 = X 3 + X 1t + X 2 t
Encontrar la velocidad y aceleracin para los siguientes casos:
a) Para una partcula que estaba en el punto (1,1,1) en la configuracin de referencia;
b) Para una partcula que para el tiempo t se encuentra en (1,1,1) .
Explicar por qu este movimiento pierde sentido fsico cuando t 1 .

Problema 2.9
Encontrar las componentes de los tensores F , C , B , F 1 , C 1 y B 1 para la siguiente
deformacin:
x1 = a1 ( X 1 + X 2 )

x2 = a2 X 2 (2.210)
x = a X
3 3 3

donde a1 , a 2 , a3 y son constantes.

Problema 2.10
Para la siguiente deformacin
u1 = AX 1 + BX 1 ( X 12 + X 22 ) 1
2 2 1
u 2 = AX 2 + BX 2 ( X 1 + X 2 ) (2.211)
u = CX
3 3

donde A , B y C son constantes.


Encontrar las componentes de los tensores F , E y .

Problema 2.11
Dadas las ecuaciones del movimiento:
x1 = X 1 + 4 X 1 X 2 t
2
x2 = X 2 + X 2 t
2
x3 = X 3 + X 3 t

Universidad de Castilla- La Mancha Draft Por: Eduardo W. V. Chaves (2012)


Ciudad Real - Espaa
2 CINEMTICA DEL CONTINUO 261

a) Hallar el campo de velocidad;


b) Hallar el campo de aceleracin;
c) Para el instante de tiempo t = 1s , obtener el tensor de deformacin infinitesimal.

Problema 2.12
Considrese un cuerpo que sufre un movimiento de cuerpo rgido, tal y como se muestra
en la siguiente figura:

x2 , X 2 x2

x1
x 2

X 1 , x1 x1

Figura 2.25: Cuerpo sometido a una rotacin.

Se pide:
a) Plantear las ecuaciones del movimiento
b) Obtener el campo de desplazamientos
c) Obtener el tensor de deformacin infinitesimal
d) Obtener el tensor de deformacin de Green-Lagrange
e) Qu podemos decir con respecto a los tensores de deformacin para un movimiento de
cuerpo rgido?
f) Qu aproximacin tenemos que adoptar para que se cumpla la teora de pequeas
deformaciones para el movimiento dado?

Problema 2.13
Qu se entiende por deformacin homognea?

Problema 2.14
Definir: Estado de Tensin Plana y Estado de Deformacin Plana. Para cada estado, qu
simplificaciones son consideradas? Dar ejemplos prcticos en los que se pueda aplicar uno
y otro estado.

Problema 2.15
Plantear la Descomposicin Polar del Gradiente de Deformacin.

Universidad Castilla- La Mancha Draft Por: Eduardo W. V. Chaves (2012)


Ciudad Real - Espaa
262 PROBLEMAS RESUELTOS DE MECNICA DEL MEDIO CONTINUO

Problema 2.16
Justificar si son ciertas o falsas las siguientes afirmaciones:
a) Si el campo de velocidades es estacionario, entonces el campo de aceleracin
tambin lo es.
b) Si el campo de velocidades es uniforme, entonces el campo de aceleraciones es
siempre nulo;
c) Si el campo de velocidades es estacionario y el medio es incompresible, el campo
de aceleraciones es siempre nulo.

Problema 2.17
Partiendo de las ecuaciones del movimiento de cuerpo rgido, obtener los siguientes
tensores:
F - Gradiente de deformacin
C - Tensor derecho de deformacin de Cauchy-Green
b - Tensor izquierdo de deformacin de Cauchy-Green
E - Tensor de deformacin de Green-Lagrange
e - Tensor de deformacin de Almansi.

Problema 2.18
Obtener la tasa del determinante del Jacobiano
a) ( J& = f ( E& , C ) ) en funcin de la tasa del tensor de deformacin de Green-Lagrange
( E& ) y del tensor derecho de deformacin de Cauchy-Green;
b) ( J& = f (C& , C ) ) en funcin de la tasa del tensor derecho de deformacin de Cauchy-
Green ( C& ),y del tensor derecho de deformacin de Cauchy-Green.
Datos
D r
F J& = F v
Dt r r 1
= J v E ( X , t ) = ( F T F 1)
r
= J Tr (v )
2 E& = F T D F
1
= (C 1)
= J Tr ( l ) 2
= J Tr (D)

Problema 2.19
Dado el campo de velocidad vi = 2 x 2 1i , obtener el tensor tasa de deformacin y el tensor
spin.
Respuesta:
0 1 0 0 1 0
D ij = 1 0 0 ; Wij = 1 0 0
0 0 0 0 0 0

Universidad de Castilla- La Mancha Draft Por: Eduardo W. V. Chaves (2012)


Ciudad Real - Espaa
3 Tensiones
3.1 Ejercicios Resueltos

3.1.1 Fuerza, Tensor de Tensiones, Vector Tensin

Ejemplo 3.1
Ignorando la curvatura de la superficie de la tierra, el campo gravitacional puede suponerse
de la forma como se muestra en la Figura 3.1, donde g es la aceleracin de la gravedad.
Obtener la fuerza resultante que acta en el cuerpo B .

g
B

x3
x2

x1
Figura 3.1: Campo gravitacional.

Solucin:
Todos los cuerpos situados en este campo se encontrarn sometidos a la fuerza:
0
b i ( x , t ) = 0
r

La fuerza msica que acta sobre el cuerpo es:


264 PROBLEMAS RESUELTOS DE MECNICA DEL MEDIO CONTINUO

0

r 0
Fi = b i ( x , t ) dV =

V

V

g dV

kg m kg m
Podemos verificar la unidad de F : [F] = 3 2
dV = 2 = N ( Newton )
V m s s

Ejemplo 3.2
Las componentes del tensor de tensiones en el punto P son:

8 4 1
ij = 4 3 0,5 Pa
1 0,5 2 x3

C (0,0,5)
a) Calcular el vector tensin en el punto P
segn la direccin del plano ABC ,
como se indica en la Figura 3.2.
r B (0,2,0)
b) Obtener el vector tensin normal ( N ) O
r x2
y el vector tensin tangencial ( S ),
ver Apndice A en Chaves(2007, 3 edicin). A(3,0,0)
x1

Figura 3.2: Plano ABC .


Solucin:
En primer lugar, deberemos obtener la direccin normal a este plano, para ello escogemos
dos vectores pertenecientes al plano y hacemos el producto vectorial entre ellos:

BA = OA OB = 3e 1 2e 2 + 0e 3 ; BC = OC OB = 0e 1 2e 2 + 5e 3
El vector normal al plano ABC viene dado a travs del producto vectorial de los vectores
definidos anteriormente:
e 1 e 2 e 3
r
n = BC BA = 0 2 5 = 10e 1 + 15e 2 + 6e 3
3 2 0
r
r n 10 15 6
El versor asociado a n ser: n = r = e 1 + e 2 + e 3
n 19 19 19

Utilizando la ecuacin t i(n) = ij n j , podemos obtener las componentes del vector tensin
de la forma:

Universidad de Castilla- La Mancha Draft Por: Eduardo W. V. Chaves (2012)


Ciudad Real - Espaa
3 TENSIONES 265

t1 8 4 1 10 t1 26
t = 1 4 3 0,5 15 Pa 1
t 2 = 8 Pa

2 19 19
t 3 1 0,5 2 6 t 3 29,5
r
b) El vector tensin t (n) asociado a la direccin n puede descomponerse en una componente
r r
normal N y en otra tangencial S tal como se indica en la Figura 3.3. La suma vectorial
de estos vectores resulta:
r r r r
t (n ) = N + S t (n) = N n + S s
r r
donde N y S son los mdulos de N y de S , respectivamente.
Como visto en el Apndice A, N puede ser obtenido a travs de las siguientes relaciones
r
N = t (n) n = ( n ) n = n n = : (n n )

= t i(n) n i = ( ij n j )n i = n i ij n j = ij (n i n j )

Luego:
10
1
N = t i n i N = 2 [26 8 29,5] 15 1,54 Pa
19
6
r
t (n)
x3
r
N
r
S
s n
P
e 3

e 2 x2
e1

x1

Figura 3.3: Componentes normal y tangencial del vector tensin.

La componente tangencial viene dada por:


r
S = t (n) s = ( n ) s = s n = : (n n )

= t i(n) s i = ( ij n j )s i = s i ij n j = ij (s i n j )
La componente tangencial tambin puede ser obtenida a travs del teorema de Pitgoras:
r 2
t (n) = 2N + 2S 2S = t i(n) t i(n) 2N

donde

Universidad de Castilla- La Mancha Draft Por: Eduardo W. V. Chaves (2012)


Ciudad Real - Espaa
266 PROBLEMAS RESUELTOS DE MECNICA DEL MEDIO CONTINUO

26
1
= 2 [26 8 29,5] 8 4,46

t i(n) t i(n)
19
29,5

Resultando que:

S = t i(n) t i(n) 2N = 4,46 2,3716 2,0884 Pa

Ejemplo 3.3
El estado tensional en un punto del continuo viene representado a travs de las
componentes del tensor de tensiones de Cauchy como:
2 1 0
ij = 1 2 0 Pa
0 0 2

a) Obtener las componentes de en un nuevo sistema x1 , x 2 , x3 , donde la matriz de


transformacin viene dada por a ij , ver Figura 3.4.
b) Obtener los invariantes principales de ;
c) Obtener los autovalores y autovectores de . Verificar tambin si los autovectores
forman una matriz de transformacin de base entre el sistema original y el principal;
d) Obtener la representacin grfica del tensor de tensiones de Cauchy, i.e. el crculo de
Mohr en tensiones (ver Apndice A);
e) Obtener la parte esfrica ( sph ) y la parte desviadora ( dev ) del tensor . Tambin,
obtener los invariantes principales de dev y los autovalores de dev ;
f) Obtener la tensin normal octadrica ( oct
N ) y la tensin tangencial octadrico (o tambin conocida
oct
como tensin de corte octadrica) ( S ).

x3
x 2
3 0 4
1
1 a ij = A = 0 5 0
x3 5
4 0 3
x1
e2 donde
e 3
e3
e1 1 a11 = cos 1
e 2 a12 = cos 1
e1 x2
a13 = cos 1
1
M

x1

Figura 3.4: Matriz de transformacin

Solucin:

Universidad de Castilla- La Mancha Draft Por: Eduardo W. V. Chaves (2012)


Ciudad Real - Espaa
3 TENSIONES 267

a) Como hemos visto en el captulo 1 (Chaves (2007)), la ley de transformacin de las


componentes del tensor de segundo orden viene dada por:
ij = aik a jl kl Forma
Matricial
= A AT
Luego:
T

11 12 13 3 0 4 1 1 0 3 0 4 2 0,6 0
1

12 22 23 = 2 0 5 0 2 2 0 0 5 0 = 0,6 2 0,8

5
13
23 33 4 0 3 0 0 2 4 0 3 0 0,8 2

donde estas componentes se pueden apreciar en la Figura 3.5.


b) Los invariantes principales del tensor de tensiones de Cauchy stress tensor son obtenido
a travs de las expresiones:
I = Tr ( ) = ii = 11 + 22 + 33

II =
1
2
[ 1
] (
( Tr ) 2 Tr ( 2 ) = ii jj ij ij
2
)
2 2
= 11 22 + 11 33 + 33 22 12 13 223
1
III = det ( ) = ijk i1 j 2 k 3 =
6
(
ii jj kk 3 ii jk jk + 2 ij jk ki )
= 11 22 33 + 212 23 13 11 223 22 132
33 122

Reemplazando los valores del problema propuesto obtenemos que:


2 0 2 0 2 1
I = 6 ; II = + + = 11 ; III = 6
0 2 0 2 1 2

c) Los valores principales (autovalores) ( i ) y direcciones principales ( n (i ) ) son obtenidos a


travs del siguiente sistema de ecuaciones:
2 1 0 n1 0
1 2 0 n 2 = 0

0 0 2 n 3 0

Para obtener las soluciones no triviales de n (i ) , tenemos que resolver el siguiente


determinante caracterstico:
2 1 0
ij ij = 1 2 0 =0
0 0 2
Pero, si nos fijamos en las componentes del tensor de tensiones de Cauchy, verificamos
que ya conocemos un autovalor y autovector ya que las componentes tangenciales segn
direccin x3 son iguales a cero, luego:
1 = 2 Direccin
principal
n1(1) = n (21) = 0 , n3(1) = 1
Para obtener los autovalores restantes, es suficiente resolver el determinante:
2 1 2 = 1
= (2 ) 1 = 0
2

1 2 3 = 3
Expresando as las componentes del tensor de tensiones de Cauchy en el espacio principal:

Universidad de Castilla- La Mancha Draft Por: Eduardo W. V. Chaves (2012)


Ciudad Real - Espaa
268 PROBLEMAS RESUELTOS DE MECNICA DEL MEDIO CONTINUO

2 0 0
ij = 0 1 0 Pa
0 0 3

Direccin principal asociada al autovalor 2 = 1 :

2 1 1 0 n1( 2 ) 0
( 2 ) n1 + n 2 = 0
( 2) ( 2)
1 2 1
0 n 2 = 0 ( 2 ) n1( 2 ) = n (22 )
n + n 2 = 0
( 2)

0 0 2 1 n 3( 2 ) 0 1
2 2
Con n 3( 2 ) = 0 y utilizando la restriccin n1( 2 ) + n (22 ) = 1 obtenemos que:
1 1 1
n1( 2 ) = n (22 ) = , luego n i( 2 ) = 0
2 2 2
Direccin principal asociada al autovalor 2 = 3 :

2 3 1 0 n1( 3) 0
( 3) n1 + n 2 = 0
( 3) ( 3)
1 23
0 n 2 = 0 ( 3) n1( 3) = n (23)
n n 2 = 0
( 3)
0 0 2 3 n 3( 3) 0 1
2 2
Con n 3(3) = 0 y utilizando la restriccin n1(3) + n (23) = 1 obtenemos que:
1 1 1
n1( 3) = n (23) = luego n i(3) = 0
2 2 2
Como hemos visto en el captulo 1, los autovectores del tensor constituye una matriz de
transformacin, B , del espacio original al espacio principal, i.e. = B B T . Luego, hay
que cumplir que:
T
0 0 1 0 0 1

1 = 2 0 0 2 1 0
0 1 1 1 1
2 = 1 0 = 0 1 2 0 0
2 2 2 2
0 0 3 = 3 0 0 2 1
1 1 1
0 0
2 2 2 2

Universidad de Castilla- La Mancha Draft Por: Eduardo W. V. Chaves (2012)


Ciudad Real - Espaa
3 TENSIONES 269

x3
x2
x3
x2
P
x1 x1

= A AT
x3

33
x3
33
23 23 23
13 23 22 x2
13
13
12 22
12
x2
13
12

12
11

11

x1
x1

= AT A

Figura 3.5: Ley de transformacin de base.

d) La representacin grfica de un tensor de segundo orden simtrico, i.e. el crculo de


Mohr, puede ser obtenido tal y como se describe en el Apndice A. Para ello, debemos
reestructurar los autovalores de tal forma que I > II > III , resultando:
I = 3 ; II = 2 ; III = 1
Las tres circunferencias son definidas por:
1 1
Crculo 1 ; (centro)C1 = ( II + III ) = 1,5 ; (radio) R1 = ( II III ) = 0,5
2 2
1 1
Crculo 2 ; (centro)C 2 = ( I + III ) = 2,0 ; (radio) R2 = ( I III ) = 1,0
2 2
1 1
Crculo 3 ; (centro)C 3 = ( I + II ) = 2,5 ; (radio) R3 = ( I II ) = 0,5
2 2

Entonces, el crculo de Mohr en tensiones viene representado en la Figura 3.6.

Universidad de Castilla- La Mancha Draft Por: Eduardo W. V. Chaves (2012)


Ciudad Real - Espaa
270 PROBLEMAS RESUELTOS DE MECNICA DEL MEDIO CONTINUO

S max = 1

R2

R1 R3 N
C3
III = 1 C1 II = 2 I = 3 = N max

Figura 3.6: Crculo de Mohr en tensiones.

e) Como definido en al captulo 1, un tensor de segundo orden puede ser descompuesto de


forma aditiva en una parte esfrica y otra desviadora:
Notacin Tensorial Notacin Indicial

= esf + dev 1
ij = ijesf + ijdev = kk ij + ijdev (3.1)
3
= m 1 + dev = m ij + ij dev

La representacin esquemtica de estas componentes se puede apreciar en la Figura 3.7. El


valor de m viene dado por:
11 + 22 + 33 1 + 2 + 3 1 1 I 6
m = = = kk = Tr ( ) = = = 2
3 3 3 3 3 3
Luego, la parte esfrica queda definida por:
2 0 0
ijesf = m ij = 2 ij = 0 2 0
0 0 2

Y, la parte desviadora por:


11 12 13 m 0 0
ijdev = 12 22 23 0 m 0
13 23 33 0 0 m
13 (211 22 33 ) 12 13

= 12 1
3
(2 22 11 33 ) 23
13 23 1
(2 33 11 22 )
3

Luego,
2 2 1 0 0 1 0
ijdev
= 1 22 0 = 1 0 0
0 0 2 2 0 0 0

Universidad de Castilla- La Mancha Draft Por: Eduardo W. V. Chaves (2012)


Ciudad Real - Espaa
3 TENSIONES 271

Los tensores y dev son coaxiales (ver captulo 1 del libro de texto), i.e., presentan las
mismas direcciones principales. Luego, podemos obtener los autovalores de dev
fcilmente si operamos en el espacio principal de :
1 0 0 m 0 0 0 0 0
ijdev = 0 2 0 0
m 0 = 0 1 0
0 0 3 0 0 m 0 0 1

Los invariantes de dev vienen dados por:


I dev = Tr ( dev ) = 0 ; II dev = 1 ; III dev = 0

Tradicionalmente, los invariantes del tensor de tensiones desviador viene denotados por:
J1 = I dev = 0

J 2 = II
dev
1 2
3
= (
I 3 II )
J 3 = III
dev
=
1
27
(
2 I 3 9 I II + 27 III )

x3
33

23
13 23
13
12 22
12
11 x2

x
14414444442444444443
x3 x3
m dev
33

23
13 23
13
m
+ 12
12
dev
22

m x2 dev
11 x2

x1 x1

esf dev

Figura 3.7: Parte esfrica y desviadora de .

Universidad de Castilla- La Mancha Draft Por: Eduardo W. V. Chaves (2012)


Ciudad Real - Espaa
272 PROBLEMAS RESUELTOS DE MECNICA DEL MEDIO CONTINUO

f) Las tensiones normal y tangencial octadricas vienen dadas por:

oct
N =
1
(1 + 2 + 3 ) = 1 ii = I = m
3 3 3

oct oct =
1
2 I 2 6 II =
2
J2 =
( ) + ( ) + ( )
dev 2
1
dev 2
2
dev 2
3
S
3 3 3

Reemplazando los valores del problema propuesto obtenemos que:


2 2
oct
N = m = 6 ; oct = J2 =
3 3

Ejemplo 3.4
Las componentes del tensor de tensiones en un punto P son:
1 2 3
ij = 2 4 6 MPa (3.2)
3 6 1

Encontrar:
r
a) El vector traccin t en P para un plano normal al eje x1 ;
r
b) El vector traccin t en P para un plano cuyo vector normal es (1,1,2) ;
r
c) El vector traccin t en P para un plano paralelo al plano 2 x1 2 x 2 x3 = 0 ;
d) Las tensiones principales en P ;
e) Las direcciones principales de en P .

Solucin:
a) El vector normal al plano es (1,0,0) . Luego el vector traccin viene dado por:

1 2 3 1 1
= 2 4 6 0 = 2

t i(n) (3.3)
3 6 1 0 3

b) El vector unitario (versor) asociado a la direccin (1,1,2) es:

1
1 1
n i = (3.4)
6
2

luego,
1 2 3 1 5
1 1
t i(n) = 2 4 6 1 = 10 (3.5)
6 6
3 6 1 2 1

Universidad de Castilla- La Mancha Draft Por: Eduardo W. V. Chaves (2012)


Ciudad Real - Espaa
3 TENSIONES 273

c)
2 1 2 3 2 5
1 1 1

n i = 2 t i = 2 4 6 2 = 10
)
(n
(3.6)
3 3 3
1 3 6 1 1 7

d) Resolviendo el determinante caracterstico


1 2 3
2 4 6 =0 (3.7)
3 6 1

obtenemos que:
1 = 10 ; 2 = 0 ; 3 = 4 (3.8)
e) Las tensiones principales correspondientes son:
Para 1 = 10

9n1 + 2n 2 + 3n 3 = 0 3
(1)
2n1 6n 2 + 6n 3 = 0 n i = 6 (3.9)
3n + 6n 9n = 0 5
1 2 3

Anlogamente:
2 1
n (i 2 ) = 1 ; n i = 2
( 3)
(3.10)
0 3

Normalizacin de las direcciones principales:


3 2 1
n i(1) 1 n (i 2 ) 1 n (i 3) 1
n i(1) = r = 6 ; n i( 2 ) = r = 1 ; n i(3) = r = 2
n (1) 70 n ( 2) 5 n ( 3) 14
5 0 3

Ejemplo 3.5
r r r
Probar que S = t (n) (1 n n ) , donde t (n) es el vector traccin asociado al plano cuya

r
normal es n y S es la tensin tangencial asociada a este plano.

Solucin 1:
r r r
[
S = t (n) t (n ) n n ]
r r r
S = t (n) t (n) n n
r r
S = t (n) (1 n n )

Solucin 2:
Podemos resolver el problema anterior utilizando slo las componentes de la ecuacin
r
S = t (n) [ : (n n )]n :
r

Universidad de Castilla- La Mancha Draft Por: Eduardo W. V. Chaves (2012)


Ciudad Real - Espaa
274 PROBLEMAS RESUELTOS DE MECNICA DEL MEDIO CONTINUO

Si

[ ]
= t i(n) (n k n l kl ) n i

= t i(n) n i n k t (kn)

= t (kn) ik n i n k t (kn)
= t (kn) ( ik n i n k )

o en forma compacta:
r r
S = t (n) (1 n n )

Ejemplo 3.6
El estado de tensin en un punto P del medio continuo se da esquemticamente por:

x3

1
4
1
4 22

1 1
x2

x1
Se pide:
Determinar el valor de la componente 22 del tensor de tensiones para que exista al menos
un plano que pase por P que est libre de tensiones;
Determinar la direccin de dicho plano.
Solucin:
r r
Buscamos un plano cuya direccin es n tal que t (n ) = 0 . Podemos relacionar el tensor de
tensiones con el vector tensin segn expresin:
r
t (n) = n
luego:
t1(n ) 0 1 4 n1 0
(n )
t 2 = 1 22 1 n 2 = 0
t (n ) 4 1 0 n3 0
3
Resultando en el siguiente sistema de ecuaciones:
1
n2 + 4n3 = 0 n3 = 4 n 2

n1 + 22n 2 + n3 = 0
1
4n1 + n2 = 0 n1 = n 2
4
Combinando las ecuaciones anteriores obtenemos que:

Universidad de Castilla- La Mancha Draft Por: Eduardo W. V. Chaves (2012)


Ciudad Real - Espaa
3 TENSIONES 275

1 1
n1 + 22n 2 + n3 = 0 n2 + 22n 2 n2 = 0
4 4
1 1
+ 22 n 2 = 0
4 4
r r
Luego, para n 0 , tenemos que: + 22 = 0 22 = .
1 1 1
4 4 2
Para determinar la direccin del plano partimos de la restriccin: n i n i = 1 , luego:
nini = 1 n12 + n 22 + n32 = 1
2 2
1 1
n 2 + n 22 + n 2 = 1
4 4
2 2 2
n2 = ; n1 = n3 =
3 6
r r
Obteniendo as la direccin de la normal al plano, cuando se cumple t (n ) = 0 :
1
2 4
n i =
6
1

3.1.2 Ecuacin de Equilibro, Tensiones y Direcciones


Principales

Ejemplo 3.7
El campo de tensin de un medio continuo viene representado por:
1 0 2 x2
ij = 0 1 4 x1 unidades de tensin (3.11)
2 x2 4 x1 1

donde xi son las coordenadas cartesianas.


Se pide:
a) Despreciando las fuerzas msicas, est el cuerpo en equilibrio?
b) Determinar el vector tensin que acta en un punto ( x1 = 1, x2 = 2, x3 = 3) segn el
plano x1 + x 2 + x3 = 6 ;
c) Determinar la proyeccin del vector tensin segn la direccin normal y tangencial
al plano x1 + x 2 + x3 = 6 ;
Solucin:
Ecuacin de equilibrio:
r r
xr + {
b=0
r (3.12)
=0

Universidad de Castilla- La Mancha Draft Por: Eduardo W. V. Chaves (2012)


Ciudad Real - Espaa
276 PROBLEMAS RESUELTOS DE MECNICA DEL MEDIO CONTINUO

ij , j = 0 i (3.13)
expandiendo,
i1,1 + i 2, 2 + i 3,3 = 0 i (3.14)
11,1 + 12, 2 + 13,3 = 0

21,1 + 22, 2 + 23,3 = 0 (3.15)

31,1 + 32, 2 + 33,3 = 0
ya que:
11 12 13
=0 =0 =0
x1 x2 x3
21 22 23
=0 =0 =0 (3.16)
x 2 x2 x3
31 32 33
=0 =0 =0
x3 x2 x3

b) El versor normal al plano x1 + x 2 + x3 = 6 es:

1
1 1
n i = (3.17)
3
1
r
El vector tensin t (n) :
r
t (n) = n (3.18)
1 0 4
ij ( x1 = 1, x 2 = 2, x3 = 3) = 0 1 4 (3.19)
4 4 1

luego,
1 0 4 1 5
1 1
t i(n) = 0 1 4 1 = 5 (3.20)
3 3
4 4 1 1 9

c) Componente normal
1
r (n ) 1
N = t n = 1 1
[5 5 9] 1 = (5 + 5 + 9) = 19 (3.21)
3 3 3 3
1

Componente tangencial
r r
2S = 2N + t (n) t (n) (3.22)
5
r (n ) r (n ) 1
t t = [5 5 9] 5 1 = 131 (3.23)
3 3 3
9

Universidad de Castilla- La Mancha Draft Por: Eduardo W. V. Chaves (2012)


Ciudad Real - Espaa
3 TENSIONES 277

luego
2
19 131 32
2S = + = (3.24)
3 3 9

Ejemplo 3.8
Dado un cuerpo en equilibrio esttico, donde el campo del tensor de tensiones de Cauchy
viene representado a travs de las siguientes componentes cartesianas:
11 = 6 x13 + x 22 ; 12 = x 32
22 = 12 x13 + 60 ; 23 = x 2
33 = 18 x 23 + 6 x33 ; 31 = x12
Determinar el vector de fuerzas msicas (por unidad de volumen) en el punto
( x1 = 2; x 2 = 4; x3 = 2 ).
Solucin:
Ecuacin de equilibrio:
r r
xr + b = 0 (3.25)
11 12 13
+ + + b1 = 0 b1 = 11 12 13
x1 x 2 x3 x1 x 2 x3
21 22 23 21 22 23
+ + + b2 = 0 b2 = (3.26)
x1 x 2 x3 x1 x 2 x3

31 33
x + x + x + b 3 = 0 b 3 = x x x
32 33 31 32

1 2 3 1 2 3

b1 = 18 x12 0 0 18 x12

b 2 = 0 0 0 bi = 0 (3.27)
b = 2 x 1 18 x 2 2 x 1 18 x 2
3 1 2 1 2

Para el punto x1 = 2; x 2 = 4; x3 = 2 obtenemos que:

72
b i = 0 (Fuerza por unidad de volumen) (3.28)
77

Ejemplo 3.9
El campo del tensor de tensiones de Cauchy viene representado por sus componentes
como:
x12 x 2 (a 2 x 22 ) x1 0
1 3
ij = k (a 2 x 22 ) x1 ( x 2 3a 2 x 2 ) 0 (3.29)
3
0 0 2ax32

donde k y a son constantes.

Universidad de Castilla- La Mancha Draft Por: Eduardo W. V. Chaves (2012)


Ciudad Real - Espaa
278 PROBLEMAS RESUELTOS DE MECNICA DEL MEDIO CONTINUO

r
Encontrar el campo de fuerzas msicas b (por unidad de masa) necesario para que el
campo de tensin est en equilibrio.
Solucin:
11 12 13
+ + + b1 = 0 b1 = 2 x1 x 2 k + 2 x1 x 2 k = 0
x1 x 2 x 3
21 22 23 k
+ + + b 2 = 0 b 2 = k (a 2 x 22 ) (3 x 22 3a 2 ) = 0 (3.30)
x1 x 2 x 3 3
31 32 33
x + x + x + b 3 = 0 b 3 = 4kax3
1 2 3

Luego:
0
4kax3
bi = 0 (Fuerza por unidad de masa) (3.31)

1

Ejemplo 3.10
r
Suponga que las fuerzas msicas son b = ge 3 , donde g es una constante. Considere el
siguiente tensor de tensiones:
x2 x3 0
ij = x3 0 x 2 (3.32)
0 x2 p

Encontrar p tal que cumpla con las ecuaciones de equilibrio. Considerar una constante
y el campo homogneo de densidad de masa, es decir, no depende del vector posicin.
Solucin:
Ecuacin de equilibrio:
r r
xr + b = 0 (3.33)
11 12 13
+ + + b1 = 0
x x x 0 + 0 + 0 + b = 0 b = 0
1 2 3
1 1
21 22 23
+ + + b2 = 0 0 + 0 + 0 + b 2 = 0 b 2 = 0 (3.34)
x1 x 2 x 3
31 32 33 0 + 33 + b 3 = 0
x + x + x + b 3 = 0 x3
1 2 3

33 (p ) p p g
= = = b3 =1 +
x3 x3 x 3 x3
(3.35)
g
dp = 1 + dx
3
g g

p = 1 +

x 3 p = 1 +

x
3

Universidad de Castilla- La Mancha Draft Por: Eduardo W. V. Chaves (2012)


Ciudad Real - Espaa
3 TENSIONES 279

Verificacin:
g
+ 1 + g = + + g g = 0 (3.36)

Ejemplo 3.11
Muestre que para el siguiente campo de tensin:
11 = x 22 + ( x12 x 22 ) ; 12 = 2x1 x 2 ; 23 = 13 = 0
2 2 2 2 2
22 = x1 + ( x 2 x1 ) ; 33 = ( x1 + x 2 )
Satisface las ecuaciones de equilibrio con fuerzas msicas iguales a cero.
Solucin:
Ecuaciones de equilibrio:
ij , j + b i = 0 i
{
=0i

ij , j = 0 i (i , j = 1,2,3)
i1,1 + i 2, 2 + i 3,3 = 0 i

11 12 13
+ + =0
i = 1 11,1 + 12 , 2 + 13,3 = 0 x1 x 2 x 3
22 23
i = 2 21,1 + 22 , 2 + 23, 3 = 0 21 + + =0
i = 3 31,1 + 32 , 2 + 33,3 = 0 x1 x 2 x 3
31 32 33
+ + =0
x1 x 2 x 3

Las ecuaciones de equilibrio quedan:


11,1 + 12 , 2 + 31,3 = 2 x1 2x1 = 0

12 ,1 + 22 , 2 + 23, 3 = 2 x 2 + 2x 2 = 0

13,1 + 23, 2 + 33,3 = 0
Con lo cual se comprueba que el cuerpo est en equilibrio.

Ejemplo 3.12
Considrese el siguiente campo de tensiones:
x1 + x 2 12 0
r
ij ( x ) = 12 x1 2 x 2 0
0 0 x 2

Considerando el medio en equilibrio, encontrar 12 , sabiendo que es funcin de x1 , x 2 , i.e.


12 ( x1 , x 2 ) . Se sabe tambin que el medio est libre de fuerzas msicas y que el vector
r
tensin en el plano x1 = 1 viene dado por: t (n) = (1 + x 2 )e 1 + (5 x 2 )e 2 .

Universidad de Castilla- La Mancha Draft Por: Eduardo W. V. Chaves (2012)


Ciudad Real - Espaa
280 PROBLEMAS RESUELTOS DE MECNICA DEL MEDIO CONTINUO

Solucin:
Como el cuerpo est en equilibrio debe satisfacer las ecuaciones de equilibrio:
ij , j + b i = 0 i
{
=0i

ij , j = 0 i (i , j = 1,2,3)
i1,1 + i 2, 2 + i 3,3 = 0 i

Resultando:
11 12 13
+ + = 1 + 12 + 0 = 0
x1 x 2 x 3 x 2
22 23 12
21 + + = 2+0=0
1 x x 2 x 3 x1
31 32 33
+ + =0+0+0=0
x1 x 2 x 3
r
Como dato del problema tenemos que cuando x1 = 1 , t (n) = (1 + x 2 )e 1 + (5 x 2 )e 2 , luego:
1 + x 2 12 0

ij ( x1 = 1, x 2 ) = 12 1 2 x 2 0
0 0 x 2

1 + x 2 12 0 1 1 + x 2
t (n )
= ij ( x1 = 1, x 2 )n j = 12 1 2 x 2 0 0 = 5 x 2 (3.37)
0 0 x 2 0 0

t (n) = ij ( x1 = 1, x 2 ) n j
1 + x2 12 ( x1 = 1, x 2 ) 0 1 1 + x2 1 + x 2

12 ( x1 = 1, x 2 ) 1 2 x2 0 0 = 12 ( x1 = 1, x 2 ) = 5 x 2

0 0 x 2 0 0 0

A travs de las ecuaciones de equilibrio:


12
x1
=2 12
= 2x1 12 ( x1 , x 2 ) = 2 x1 + C ( x 2 )

A travs de la condicin de contorno dada por (3.37) podemos obtener la constante de


integracin:
12 ( x1 = 1, x 2 ) = 5 x 2 = 2 + C ( x 2 ) C ( x2 ) = 3 x2
Luego:
12 ( x1 , x 2 ) = 2 x1 x 2 + 3

Ejemplo 3.13
Obtener las ecuaciones de equilibrio (en notacin ingenieril), partiendo de un elemento
r
diferencial ( dx ), donde la variacin de las tensiones de punto a punto (campo de tensiones)
es la que se muestra en la Figura 3.8.

Universidad de Castilla- La Mancha Draft Por: Eduardo W. V. Chaves (2012)


Ciudad Real - Espaa
3 TENSIONES 281

z Cara oculta
x
xy
z
z + dz
Cara oculta z
yz
yz + dz xz
z
xz
xz + dz
z yz dz
yz + dy
xy bz y
y by y
xz y + dy
xz + dx y
x bx xy
xy + dy y
y
yz xy
xy + dx dx
x
x + x dx
x

xz

x yz

Cara oculta
z

dy

Figura 3.8: Tensiones en un elemento diferencial.


Solucin:
Para obtener las ecuaciones de equilibro partiremos de que la suma de las fuerzas que
actan en el diferencial sea cero. Haciendo el equilibrio de fuerzas segn direccin x :

F x =0

x xy
b x dxdydz + x + dx dydz x dydz + xy + dy dxdz
x y

xy dxdz + xz + xz dz dxdy xz dxdy = 0
z
Simplificando la ecuacin anterior resulta:
x xy
b x dxdydz + dxdydz + dxdydz + xz dxdydz = 0
x y z

x xy xz
b x + + + =0
x y z

Universidad de Castilla- La Mancha Draft Por: Eduardo W. V. Chaves (2012)


Ciudad Real - Espaa
282 PROBLEMAS RESUELTOS DE MECNICA DEL MEDIO CONTINUO

Resultante de fuerzas segn direccin y : Fy = 0


y yz
b y dxdydz + y + dy dxdz y dxdz + yz + dz dxdy
y z
xy
yz dxdy + xy + dx dydz xy dydz = 0
x
Simplificando la ecuacin anterior resulta:
xy y yz
b y + + + =0
x y x z

Resultante de fuerzas segn direccin z : Fz = 0


z
b z dxdydz + z + dz dxdy z dxdy + xz + xz dx dzdy
z x
yz
xz dzdy + yz + dy dxdz yz dxdz = 0
y
Simplificando la ecuacin anterior resulta:
xz yz z
b z + + + =0
x y z
Luego, las ecuaciones de equilibrio son:
x xy xz
+ + + b x = 0
x y z
xy y yz
+ + + b y = 0
x y x z
z
xz + yz + + b z = 0
x y z

Ejemplo 3.14
Dado un medio continuo donde se conoce el estado tensional en un punto y que viene
representado a travs de las componentes del tensor de tensiones de Cauchy:
1 1 0
ij = 1 1 0 Pa
0 0 2

Se pide:
a) Encontrar las tensiones principales y las direcciones donde se producen.
Solucin:
Para obtener las tensiones principales i = i y direcciones principales n (i ) debemos
resolver el siguiente sistema de ecuaciones:

Universidad de Castilla- La Mancha Draft Por: Eduardo W. V. Chaves (2012)


Ciudad Real - Espaa
3 TENSIONES 283

1 1 0 n1 0
1 1 0 n 2 = 0
(3.38)
0 0 2 n 3 0

La obtencin de soluciones no triviales de n (i ) , es equivalente a resolver:

ij ij = 0

Pero si nos fijamos en el formado de la matriz que contiene las componentes del tensor de
tensiones, podemos notar que ya tenemos una solucin principal, ya que las componentes
tangenciales en la direccin 3 son cero, luego:
direccin
1 = 2 n1(1) = n (21) = 0 , n3(1) = 1
Pero obtener las otras dos direcciones es suficiente con resolver:
1 1
= (2 ) = 0
1 1
Podemos fcilmente verificar que las races de la ecuacin anterior son:
2 = 2 , 3 = 0
Expresamos las componentes del tensor de tensiones en este nuevo sistema como:
2 0 0
ij = 0 2 0 Pa
0 0 0

b) Direcciones principales
b.1) Para obtener la direccin principal asociada a la solucin 2 = 2 , sustituimos esta
solucin en la ecuacin (3.38):
1 2 1 0 n1( 2 ) 0
1
1 2 0 n (22 ) = 0
0 0 2 2 n (32 ) 0
n1( 2 ) + n (22 ) = 0
( 2)
n1 n (22 ) = 0
2 2
Resolviendo el sistema obtenemos n 3( 2) = 0 , n1( 2) = n (22) y utilizando n1( 2 ) + n (22 ) = 1 resulta:
1 1 1
n1( 2 ) = n (22 ) = . n ( 2 ) = 0
2 2 2
b.2) Para la solucin 3 = 0 , obtenemos que:

1 0 1 0 n1( 3) 0 n1( 3) + n (23) = 0


1 n ( 3) = 0
1 0 0 2 n1( 3) + n (23) = 0
0 0 2 0 n 3( 3) 0 (3)
2n 3 = 0
2 2
Resolviendo el sistema obtenemos n (33) = 0 , n1(3) = n (23) y utilizando n1(3) + n (23) = 1 ,
1 1 ( 3) 1 1
resulta: n1(3) = , n (23) = . n = 0 .
2 2 2 2

Universidad de Castilla- La Mancha Draft Por: Eduardo W. V. Chaves (2012)


Ciudad Real - Espaa
284 PROBLEMAS RESUELTOS DE MECNICA DEL MEDIO CONTINUO

Como hemos visto, los autovectores constituyen una matriz de transformacin, A , entre
los dos sistemas, es decir, = A A T , As:
T
0 0 1 0 0 1

1 = 2 0 0 1 1 0
0 1 1 1 1
2 = 2 0 = 0 1 1 0 0
2 2 2 2
0 0 3 = 0 1 1 0 0 2 1 1
0 0
2 2 2 2

Ejemplo 3.15
Una presa prismtica est sometida a una presin ejercida por el agua. La presa tiene
espesor b y altura h , ver Figura 3.9. Obtener las restricciones de las componentes
cartesianas del tensor de tensiones de Cauchy en las caras BC , OB y AC .
x2
a - densidad de masa del agua
g - aceleracin de la gravedad

B C

a
a g (h x 2 )
h

O b A x1

Figura 3.9.

Solucin:
La cara BC tiene como normal n (i BC ) = [0 1 0] . Teniendo en cuenta que en esta cara no
hay vector traccin, concluimos que:
11 12 13 0 12 0
t i( BC ) = 0 i = ij n j 22 23 1 = 22 = 0
21
31 32 33 0 32 0

Lo que es lo mismo que i 2 = 0 y debido a la simetra 2i = 0 .


La cara OB tiene como normal n (i BC ) = [ 1 0 0] . Teniendo en cuenta que en esta cara el
vector traccin tiene como componentes t i(OB ) = [ a g (h x 2 ) 0 0] , concluimos que:
a g ( h x 2 ) 11 12 13 1 11 a g (h x 2 )
t i(OB ) = 0 = n
ij j
21 22 23 0 = 21 = 0

0
31 32 33 0 31 0

Lo que es lo mismo que i1 = a g (h x 2 ) i1 .

Universidad de Castilla- La Mancha Draft Por: Eduardo W. V. Chaves (2012)


Ciudad Real - Espaa
3 TENSIONES 285

La cara AC tiene como normal n (i BC ) = [1 0 0] . Teniendo en cuenta que en esta cara no


hay vector traccin, concluimos que:
11 12 13 1 11 0
t i( AC ) = 0 i = ij n j 22 23 0 = 21 = 0
21
31 32 33 0 31 0

Lo que es lo mismo que i1 = 0 y debido a la simetra 1i = 0 .

3.1.3 Otras Medidas de Tensin

Ejemplo 3.16
Demostrar que se cumplen las siguientes relaciones:
P = J dev F T + J m F T ; S = JF 1 dev F T + J m C 1
donde P y S son el primer y segundo tensor de tensiones de Piola-Kirchhoff,
respectivamente, C es el tensor derecho de deformacin de Cauchy-Green, F es el
gradiente de deformacin, J es el determinante del Jacobiano, y m es la tensin media
del tensor de tensiones de Cauchy. Demostrar tambin que se cumplen las siguientes
relaciones:
P : F = S : C = 3J m
Solucin:
Teniendo en cuenta que P = J F T , y la descomposicin de como = esf + dev ,
podemos obtener que:
P = J ( dev + m 1) F T
= J dev F T + J m 1 F T
= J dev F T + J m F T
Consideremos ahora la definicin del segundo tensor de tensiones de Piola-Kirchhoff
S = JF 1 F T , y teniendo en cuenta la descomposicin aditiva de como
= esf + dev , obtenemos:
S = JF 1 F T
= JF 1 ( dev + m 1) F T
= JF 1 dev F T + JF 1 m 1 F T
= JF 1 dev F T + J m C 1
Aplicando en doble producto escalar entre los tensores S y C , obtenemos que:
S : C = JF 1 dev F T : C + J m C 1 : C

donde el trmino JF 1 dev F T : C queda:

Universidad de Castilla- La Mancha Draft Por: Eduardo W. V. Chaves (2012)


Ciudad Real - Espaa
286 PROBLEMAS RESUELTOS DE MECNICA DEL MEDIO CONTINUO

JF 1 dev F T : C = ( JF 1 dev F T ) : {
C
F T F

( JF 1 dev F T ) ij ( F T F ) ij = J Fip1 dev 1


pk F jk Fqi Fqj

= J qp qk dev
pk

pk pk
= J dev
dev
=J
1
424 :1
3 =0
Tr ( dev ) =0

Luego:
S : C = J m C 1 : C = J m Tr (C 1 C ) = J m Tr (1) = 3 J m
Ahora, haciendo el doble producto escalar entre P y F , obtenemos que:
P : F = J dev F T : F + J m F T : F

Analizando el trmino J dev F T : F , concluimos que:


J dev F T : F = ( J dev F T ) ij ( F ) ij

= J ikdev F jk1 Fij = J ikdev ik


dev
=J
1
424 :1 = 0
3
Tr ( dev )=0

Luego,
P : F = J m F T : F = J m Tr ( F T F T ) = J m Tr (1) = 3 J m

3.1.4 Mxima Tensin de Corte, Crculo de Mohr

Ejemplo 3.17
Cul es la tensin de corte mxima cuyo estado tensional en un punto es el siguiente?
x2

30 MPa

20 MPa
x1

x3
Figura 3.10.

Universidad de Castilla- La Mancha Draft Por: Eduardo W. V. Chaves (2012)


Ciudad Real - Espaa
3 TENSIONES 287

Solucin:

Como los ejes xi son ejes
principales, dibujamos el crculo max (MPa)
de Mohr con las tensiones
principales I = 30 MPa ,
II = 20MPa y III = 0 .
N (MPa)
20 30

Figura 3.11.

30 0
max = = 15 MPa (3.39)
2

Ejemplo 3.18
Dado el estado tensional en un punto representado por el elemento infinitesimal mostrado
en la Figura 3.12. Se pide:
x2
a) Dibujar el crculo de Mohr;
b) Obtener la tensin normal mxima,
20 MPa
e indicar el plano en la que se produce;
c) Obtener la tensin tangencial mxima.
5 MPa
x1

10 MPa

x3
Figura 3.12:

Solucin:
S
max = 15
N max = 10MPa
10 (20)
S max = = 15MPa
2

N (MPa)
20 5 10

Universidad de Castilla- La Mancha Draft Por: Eduardo W. V. Chaves (2012)


Ciudad Real - Espaa
288 PROBLEMAS RESUELTOS DE MECNICA DEL MEDIO CONTINUO

Ejemplo 3.19
Determinar para que valores de * son posibles 6
los siguientes estados tensionales en planos que
pasen por P :

*
Caso a) N = 4 ; =2 P

Caso b) N = 4 ; =1 2
Caso c) N = 7 ; =0

Figura 3.13.
Solucin
Para que los pares de valores ( N ; ) sean factibles, tienen que pertenecer a la zona en gris
en el crculo de Mohr, ver Figura 3.14, o pertenecer a las circunferencias.

3 2 1 N

Figura 3.14: Crculo de Mohr.

Universidad de Castilla- La Mancha Draft Por: Eduardo W. V. Chaves (2012)


Ciudad Real - Espaa
3 TENSIONES 289

Caso a)

2 Caso b)

1 Caso c)

2 6 7 N

Figura 3.15: Crculo de Mohr.

Caso a) :En este caso el par ( N = 4; = 2) pertenece al crculo formado por la tensiones
principales 2 y 6 , luego * puede ser cualquiera, ver Figura 3.16.

Caso a)

2
1


* 2 * 6
*
N

Figura 3.16: Crculo de Mohr.

Caso b) En este caso podemos decir que la solucin es:

*( 2) * *(1) (3.40)

donde *( 2) , *(1) estn sealados en la Figura 3.17.

Universidad de Castilla- La Mancha Draft Por: Eduardo W. V. Chaves (2012)


Ciudad Real - Espaa
290 PROBLEMAS RESUELTOS DE MECNICA DEL MEDIO CONTINUO

Casos lmites

2 2
( 4,1) ( 4,1)
1 1

2 * ( x ) 6 N 2 * ( 2) * (1) 6 N

Figura 3.17: Crculo de Mohr.

Partiendo de la ecuacin de la circunferencia:


( x xC ) 2 + ( y y C ) 2 = R 2 (3.41)

( *(1) + 2) ( *(1) 2)
Para el caso *(1) , tenemos: x = 4; x C = ; y = 1; y C = 0; R =
2 2
Reemplazando estos valores en la ecuacin de la circunferencia, resulta:
( x xC ) 2 + ( y y C ) 2 = R 2
2 2
( * + 2) ( * 2)
4 (1) + (1 0)2 = (1) (3.42)
2 2

*(1) = 4,5

(6 + *( 2 ) ) (6 *( 2 ) )
Para el caso *( 2 ) , tenemos: x = 4; xC = ; y = 1; y C = 0; R =
2 2
reemplazando estos valores en la ecuacin de la circunferencia, resulta:
( x xC ) 2 + ( y y C ) 2 = R 2
2 2
(6 + *( 2) ) (6 *( 2 ) )
4 + (1 0)2 = (3.43)
2 2

*( 2 ) = 3,5

luego:

3,5 * 4,5 (3.44)


Caso c) En este caso la nica solucin posible es que N sea una tensin principal, luego

* = 7 (3.45)

Universidad de Castilla- La Mancha Draft Por: Eduardo W. V. Chaves (2012)


Ciudad Real - Espaa
3 TENSIONES 291

2 6 * = 7 N

Figura 3.18: Crculo de Mohr.

Ejemplo 3.20
Obtener la mxima tensin normal y tangencial (de corte) y dibujar el crculo de Mohr
correspondiente para los siguientes estados tensionales:
a)
0
ij = 0 (3.46)
0 0 0

b)
2 0 0
ij = 0 0 (3.47)
0 0

Solucin:
a) Valores principales. Si verificamos el formato de las componentes del tensor de
tensiones de Cauchy, ya podemos decir que un valor principal es (3) = 0 . Luego, es
suficiente obtener solo los dos otros autovalores:

= ( ) 2 2 = 0 = = 0 (3.48)

(1) = 0
( ) 2 2 = 0 2 2 + 2 2 = 0 (2 + ) = 0 (3.49)
( 2) = 2

Universidad de Castilla- La Mancha Draft Por: Eduardo W. V. Chaves (2012)


Ciudad Real - Espaa
292 PROBLEMAS RESUELTOS DE MECNICA DEL MEDIO CONTINUO

max =

2 N

Figura 3.19:

N max = 2
(3.50)
max =

b)

2 N

Figura 3.20.

N max =
(2) 3 (3.51)
max = =
2 2

Universidad de Castilla- La Mancha Draft Por: Eduardo W. V. Chaves (2012)


Ciudad Real - Espaa
3 TENSIONES 293

Ejemplo 3.21
Hacer la representacin del crculo de Mohr para los siguientes casos:
1) Caso unidimensional, estado de carga de traccin
2) Caso unidimensional, estado de carga de compresin
3) Caso bidimensional, estado de carga de traccin
4) Caso triaxial
5) estado de corte puro
Solucin:
1) Caso unidimensional, estado de carga de traccin

x

I 0 0 I N
x 0 0 0

0 0 0

2) Compresin uniaxial

x

0 0 0
0 0
II II N
x 0 0 0

3) Caso biaxial
II

II I N

I 0 0
0 II 0

0 0 0

Universidad de Castilla- La Mancha Draft Por: Eduardo W. V. Chaves (2012)


Ciudad Real - Espaa
294 PROBLEMAS RESUELTOS DE MECNICA DEL MEDIO CONTINUO

4) Caso triaxial

III

I 0 0
II
0 II 0

0 0 III III II I
N
I

5) Corte puro
S

0 0
0 0

0 0 0

3.1.5 Particularidades del Tensor de Tensiones

Ejemplo 3.22
Las componentes del tensor de tensiones de Cauchy en un punto P vienen dadas por:
5 6 7
ij = 6 8 9 GPa (3.52)
7 9 2

Obtener:
a) La tensin media;
b) La parte volumtrica y desviadora del tensor .
Solucin:
kk 5 + 8 + 2
m = = =5 (3.53)
3 3

Universidad de Castilla- La Mancha Draft Por: Eduardo W. V. Chaves (2012)


Ciudad Real - Espaa
3 TENSIONES 295

m 0 0 5 0 0
ijesf = 0 m 0 = 0 5 0
0 0 m 0 0 5
ij = ijesf + ijdev ijdev = ij ijesf (3.54)
0 6 7
ijdev = 6 3 9
7 9 3

Ejemplo 3.23
Considere las componentes del tensor de tensiones:
5 3 2
ij = 3 1 0 unidades de (3.55)
2 0 3 tensin

dadas en el sistema constituido por la base (e 1 , e 2 , e 3 ) .


Dada la ley de transformacin de base entre los sistemas x y x' por:

x'1 x' 2 x '3

3 4
x1 0
5 5

x2 0 1 0

4 3
x3 0
5 5

donde el sistema x' est constituido por la base (e '1 , e ' 2 , e ' 3 ) .
Se pide:
r
a) Obtener el vector tensin t ( e'2 ) segn el plano cuya normal es e ' 2 , expresado segn el
sistema cartesiano (e '1 , e ' 2 , e ' 3 ) con el siguiente formato:
r
t (e'2 ) = ( )e 1 + ( )e 2 + ( )e 3 (3.56)
b) Obtener la parte esfrica y desviadora del tensor de tensiones.

Solucin:
a) Como definimos, la primera fila de la matriz de transformacin est formada por los
cosenos directores del eje x'1 con x1 , x2 y x3 , luego:

3 0 4
1
A = 0 5 0 (3.57)
5
4 0 3

y la ley de transformacin para las componentes de tensor de segundo orden:

Universidad de Castilla- La Mancha Draft Por: Eduardo W. V. Chaves (2012)


Ciudad Real - Espaa
296 PROBLEMAS RESUELTOS DE MECNICA DEL MEDIO CONTINUO

' = A A T (3.58)
luego:
53 0 45 5 3 2 53 0 4
5

9 9 2
1
ij = 0 1 0 3 1 0 0 1 0 = 9 5 12 (3.59)
5
4 0 3 2 0 3 4 0 35 2 12 31
5 5 5
9
1
t i( e' 2 ) = 5
5
12 (3.60)
r 9 12
t (e'2 ) = e 1 + (1)e 2 + e 3
5 5

ya que:

t 1 ( e'1 )
( e ' 2 ) ( e '3 )

11 12 13 t1 t1
( e '3 )
23 = t 2 1
( e ' ) ( e ' 2 )
21 22 t2 t2 (3.61)
33 t 3 1
( e ' ) ( e ' ) ( e '3 )
31 32 t3 2 t3

b)
ij = ijesf + ijdev
I (3.62)
= ij + ijdev
3
I = 5 +1+ 3 = 9 (3.63)
3 0 0
esf
ij = 0 3 0 (3.64)
0 0 3

ijdev = ij ijesf
5 3 3 2 2 3 2
(3.65)
= 3 1 3 0 = 3 2 0
2 0 3 3 2 0 0

Ejemplo 3.24
El estado de tensin en un medio continuo (cuerpo) est dado por el tensor de tensiones
de Cauchy:
0 Cx 3 0
ij = Cx 3 0 Cx1
0 Cx1 0
donde C es una constante. Considrese que el cuerpo est libre de fuerzas msicas. Se
pide:
a) Probar si el cuerpo est en equilibrio;

Universidad de Castilla- La Mancha Draft Por: Eduardo W. V. Chaves (2012)


Ciudad Real - Espaa
3 TENSIONES 297

b) Calcular el vector tensin en el punto P(4,4,7) segn un plano cuya normal viene dada
2 2 1
por n = e 1 + e 2 e 3 .
3 3 3
c) Representar los crculos de Mohr del estado de tensin del punto P .
Solucin:
a) Para que el medio continuo est en equilibrio hay que cumplir las ecuaciones de
equilibrio:
r r
+ b = 0 ; ij,j + b i = 0 i (3.66)
0 Cx 3 0
ij = Cx 3 0 Cx1 (3.67)
0 Cx1 0

Para el problema propuesto b i = 0 i , luego:

i = 1 0 + 0 + 0 = 0
ij i1 i 2 i 3
ij,j = = + + i = 2 0 + 0 + 0 = 0 (3.68)
x j x1 x 2 x3 i = 3 0 + 0 + 0 = 0

ij,j = 0 i luego el cuerpo est en equilibrio.

b) El vector tensin viene dado por:


r
t (n) = n ; t i = ij n j
(n )
(3.69)
0 Cx3 0 0 7C 0
ij ( x1 = 4; x 2 = 4; x3 = 7) = Cx3 0 Cx1 = 7C 0 4C (3.70)
0 Cx1 0 0 4C 0

2
1
n j = 2 (3.71)
3
1

Resultando que:
0 7C 0 2 14C
r (n ) 1 1
t i = ij n j = 7C
0
4C 2 = 18C (3.72)
3 3
0 4C 0 1 8C

c)
0 7 0

ij = C 7 0 4 (3.73)
0 4 0

Los autovalores (tensiones principales) vienen dados por:

Universidad de Castilla- La Mancha Draft Por: Eduardo W. V. Chaves (2012)


Ciudad Real - Espaa
298 PROBLEMAS RESUELTOS DE MECNICA DEL MEDIO CONTINUO


C 0 0
0 7 0 0 0 0 7 0

C 7 0 4 0 0 = 0
C 7 0 4 C 0 0 =0 (3.74)
C
0 4 0 0 0 0 4 0
0 0
C

Considerando que = C , obtenemos:


7 0 7 0
C 7 4 = C 3
7 4 = 0
(3.75)
0 4 0 4

3 + 16 + 49 = 0 2 + 65 = 0 = 65 (3.76)

Con eso, obtenemos que = C 65 .


Resultando as un estado de corte puro, el crculo de Mohr viene representado por:

C 65

III = C 65 I = C 65 N

Ejemplo 3.25
El estado tensional en un punto del cuerpo viene dado por las componentes del tensor de
tensiones de Cauchy representado en el sistema cartesiano como:

x3

r
t ( e 3 ) = 8e 1

r
t ( e 2 ) = 6e 1
e 3
e1
e 2 x2

r
t ( e1 ) = 6e 2 + 8e 3
x1

Figura 3.21:

Universidad de Castilla- La Mancha Draft Por: Eduardo W. V. Chaves (2012)


Ciudad Real - Espaa
3 TENSIONES 299

Se pide:
a) La tensin desviadora;
b) Determinar las tensiones principales ( I , II , III ) y las direcciones principales;
c) Dibujar el crculo de Mohr;
d) Obtener la mxima tensin de corte;
e) Encontrar el vector tensin en un plano que pasa por el punto dado cuya normal a este
6
plano es n = 0,75e 1 + 0,25e 2 e3 ;
4
f) Obtener tambin la tensin normal y tangencial en este plano.
Solucin: Segn la Figura 3.21 podemos obtener las componentes del tensor de tensiones de
Cauchy como:
0 6 8
ij = 6 0 0
8 0 0

a)
ij = ijesf + ijdev

I
La parte esfrica ijesf = ij = 0 ij ya que I = 0 . Luego, la parte desviadora viene dada por:
3
0 6 8
ijdev = ijesf ij = 6 0 0
8 0 0

b) Los autovalores pueden ser determinados por el determinante caracterstico:


6 8
6 0 =0 3 + 100 = 0 ( )
2 + 100 = 0
8 0

Las soluciones son 1 = 0 , 2 = 10 , 3 = 10 , que son las tensiones principales. Las direcciones
principales quedan:
n i(1) = [0 0,8 0,6]
1 = 0 autovalor
n i( 2 ) = [ 0,707 0,424 0,566]
2 = 10 autovalor
n i(3) = [0,707 0,424 0,566]
3 = 10 autovalor
I = 10 , II = 0 , III = 10

Universidad de Castilla- La Mancha Draft Por: Eduardo W. V. Chaves (2012)


Ciudad Real - Espaa
300 PROBLEMAS RESUELTOS DE MECNICA DEL MEDIO CONTINUO

c) El crculo de Mohr viene dibujado en la figura abajo:


S

max = 10

III = 10 II = 0 I = 10 N

d) En el crculo de Mohr se puede obtener directamente la tensin de corte mxima:


max = 10

e) Teniendo en cuenta que t i (n) = ij n j , podemos obtener las componentes del vector tensin
6
en el plano de normal n = 0,75e 1 + 0,25e 2 e3 :
4

t 1 (n ) 0 6 8 0,75 3,39898
(n )
t 2 = 6 0 0 0,25 4,5
t (n) 8 0 0

3 6 6
4

f)
r
t (n)

r
N
r
S
s n
P

r r r
El mdulo de N se puede obtener a travs de la proyeccin N = t (n) n = t i (n) n i ,

luego:

0,75
r
= t i n i [ 3,39898 4,5 6] 0,25 5,09847
(n )
N
6
4
r
El vector N viene dado por:
r r
N = N n = 3,82385e 1 1,27462e 2 + 3,12216e 3

Universidad de Castilla- La Mancha Draft Por: Eduardo W. V. Chaves (2012)


Ciudad Real - Espaa
3 TENSIONES 301

r r r
Adems como se cumple que t (n) = N + S , podemos obtener el vector tangencial a este
plano como:
r r r
S = t (n) N
( 3,39898 + 3,82385)e 1 + (4,5 + 1,27462)e 2 + (6 3,12216)e 3
(0,42487 )e 1 + (5,77462)e 2 + (2,87784)e 3
y su mdulo:
r
S (0,42487 )2 + (5,77462)2 + (2,87784)2
41,808713 = 6,465966
r r r r
= t (n) t (n) N
2 2
OBS.: Tambin podramos haber utilizado la expresin S para
r
obtener el mdulo de S .

Ejemplo 3.26
El campo del tensor de tensiones de Cauchy de un medio continuo viene representado por:
3 x1 5 x 22 0
r
ij ( x ) = 21 3x 2 2 x3
32 0
31
a) Obtener las fuerzas msicas (por unidad de volumen) para que el medio continuo est en
equilibrio.
b) Para un punto particular ( x1 = 1, x 2 = 1, x3 = 0 ). Se pide:
b.1) Dibujar el Crculo de Mohr en tensiones. Obtener la tensin tangencial y normal
mximas.
1 1 1
b.2) Obtener el vector tensin en el plano definido por la normal ni =
3 3 3
b.2.1) Obtener la componente normal y tangencial en este plano.
Solucin:
a) Debido a la simetra del tensor de tensiones de Cauchy tenemos que:
3 x1 5 x 22 0
r 2
ij ( x ) = 5 x 2 3x 2 2 x3
0 2 x3 0

+ 12, 2 + 13,3 = b1 3 + 10 x 2 + 0 = b1
r r componentes 11,1
xr + b = 0 21,1 + 22, 2 + 23,3 = b1 0 + 3 + 2 = b 2
0 + 0 + 0 = b
31,1 + 32, 2 + 33,3 = b1 3

con lo cual obtenemos que:

Universidad de Castilla- La Mancha Draft Por: Eduardo W. V. Chaves (2012)


Ciudad Real - Espaa
302 PROBLEMAS RESUELTOS DE MECNICA DEL MEDIO CONTINUO

10 x 2 3
b i = 5 (Fuerza por unidad de volumen) (3.77)
0

b) Para el punto en particular ( x1 = 1, x 2 = 1, x3 = 0 ) tenemos que:


3 5 0
ij = 5 3 0
0 0 0

donde podemos verificar que 3 = 0 es un valor principal. Para obtener los otros
autovectores es suficiente con resolver:
3 5 1 = 8
=0 (3 ) = (5) 2 3 = 5
5 3 2 = 2
Reestructurando los autovalores:
I = 8 , II = 0 , III = 2
b.1) El crculo de Mohr viene dibujado en la figura abajo:

S
max = 5

III = 2 II = 0 I = 8 N

En el crculo de Mohr se puede obtener directamente la tensin de corte mxima max = 5 y la


tensin normal mxima N max = I = 10 .

e) Teniendo en cuenta que t i (n) = ij n j , podemos obtener las componentes del vector tensin
1 1 1
en el plano de normal n = e1 + e2 + e3 :
3 3 3

t 1 (n ) 3 5 0 1 8
(n ) 1 1
t 2 = 5 3 0 1 = 8
t (n ) 3 3
0 0 0 1 6
3
b.2) La tensin normal:
1
1 1
N = ti
(n )
n i = [8 8 0] 1 = 16
3 3 3
1

Universidad de Castilla- La Mancha Draft Por: Eduardo W. V. Chaves (2012)


Ciudad Real - Espaa
3 TENSIONES 303

r r r r
= t (n) t (n) N
2 2
Para la componente tangencial podemos aplicar directamente S ,
8
r r (n ) r (n ) 1 1
[8 8 0] 8 = 128 . Luego:
2
= t t = ti ti =
(n ) (n )
donde t (n)
3 3 3
0
2
r r r r 128 16 128 128
= t (n) t (n) N
2 2
S = = S =
3 3 9 3

Ejemplo 3.27
El estado tensional en un punto del cuerpo viene dado por las componentes del tensor de
tensiones de Cauchy segn su parte esfrica y desviadora, respectivamente:
1 0 0 0 6 8
ijesf = 0 1 0 ; ijdev = 6 0 0
0 0 1 8 0 0

Se pide:
a) Obtener las componentes del tensor de tensiones de Cauchy;
b) Determinar las tensiones principales ( I , II , III ) y las direcciones principales.
c) Obtener la mxima tensin de corte;
d) Dibujar el crculo de Mohr para: d.1) el tensor de tensiones de Cauchy ( ij ), d.2) Parte
esfrica ( ijesf ) y; d.3) parte desviadora ( ijdev );
Solucin:
1 0 0 0 6 8 1 6 8
a) ij = ijesf + ijdev = 0 1 0 + 6 0 0 = 6 1 0
0 0 1 8 0 0 8 0 1

En el Ejemplo 3.25 hemos obtenido los valores principales del tensor ijdev que es el
mismo del problema propuesto. Como el tensor y su parte desviadora tienen las mismas
direcciones principales, podemos obtener de forma automtica las tensiones principales:
1 0 0 10 0 0 9 0 0
ij = ijesf + ijdev
= 0 1 0 + 0 0 0 = 0 1 0
0 0 1 0 0 10 0 0 11

Las direcciones principales son las mismas del tensor del Ejemplo 3.25.

Universidad de Castilla- La Mancha Draft Por: Eduardo W. V. Chaves (2012)


Ciudad Real - Espaa
304 PROBLEMAS RESUELTOS DE MECNICA DEL MEDIO CONTINUO

d) Crculo de Mohr
S S
max = 10

dev dev
II = 0 dev = 10 dev N
III = 10 I N I = II = III = 1

Parte desviadora Parte esfrica

14444444444444442444444444444444
3

S
max = 10

III = 9 II = 1 I = 11 N

Observemos que la parte esfrica lo que hace es desplazar el crculo de Mohr segn el eje
N , no alterando as el valor de la tensin tangencial mxima.

Ejemplo 3.28
En un punto P del medio continuo el tensor de tensiones de Cauchy viene
representado por sus componentes cartesianas por:
1 1 0
ij = 1 1 0 MPa ,
0 0 2

Se pide:
a) Determinar las tensiones principales y las direcciones principales en el punto P ;
b) Obtener la mxima tensin de corte;
c) Dibujar el crculo de Mohr para: c.1) el tensor de tensiones de Cauchy ( ij ), c.2) Parte
esfrica ( ijesf ) y; c.3) parte desviadora ( ijdev );
d) i.) Encontrar el vector tensin en un plano que pasa por el punto dado cuya
r
direccin normal a este plano es n = 1,0e 1 + 1,0e 2 + 0e 3 ;
ii.) Obtener tambin la tensin normal y tangencial en este plano.

Universidad de Castilla- La Mancha Draft Por: Eduardo W. V. Chaves (2012)


Ciudad Real - Espaa
3 TENSIONES 305

f) Obtener los autovalores y autovectores de la parte desviadora del tensor de tensiones de


Cauchy ( dev ).
Solucin:
a) Ver Ejemplo 3.14. Los autovalores son I = 2 , II = 2 , III = 0
b) y c)
2 0 0 1 0 0 1 0 0
4 2
ijdev = ij ijesf = 0 2 0 0 1 0 = 0 1 0

3 3
0 0 0 0 0 1 0 0 2

S S

max = 1
+
N

III = 1,333 I , II = 0,667 N


I = II = III = 1,333

Parte desviadora Parte esfrica

14444444444444442444444444444444
3

max = 1

III = 0 I , II = 2 N

r
d) El vector tensin se obtiene a partir de t (n) = n , normalizando el vector obtenemos

r
n 1 1
que: n = r = e1 + e 2 + 0e 3 . Vector tensin:
n 2 2

t 1(n ) 1 1 0 1 2
(n ) 1 1
t 2 = 1 1 0 1 = 2 2
t (n ) 0 0 2 2 0 0
3

Universidad de Castilla- La Mancha Draft Por: Eduardo W. V. Chaves (2012)


Ciudad Real - Espaa
306 PROBLEMAS RESUELTOS DE MECNICA DEL MEDIO CONTINUO

Ejemplo 3.29
Las componentes de un estado de tensin en un punto P son:
29 0 0
ij = 0 26 6 Pa

0 6 9

Descompnganse las componentes del tensor de tensiones en una parte esfrica y otra
desviadora, y determnense los valores de las tensiones principales del tensor desviador.
Solucin:
Considerando la descomposicin aditiva del tensor de tensiones en una parte esfrica y
desviadora:
ij = ijdev + ijesf

La parte desviadora viene dada por


11 m 12 13
ijdev = 12 22 m 23

13 23 33 m

siendo la tensin media dada por:


1 ( 29 26 + 9)
m = ii = =4
3 3
Resultando as:
29 4 0 0 25 0 0
ijdev
= 0 26 4
6 = 0 30 6 Pa
0 6 9 4 0 6 5

Las componentes del tensor hidrosttico son:


4 0 0
ijhid esf
ij = 0 4 0 Pa
0 0 4

Para comprobar las operaciones anteriores, la siguiente relacin tiene que verificarse:
25 0 0 4 0 0 29 0 0
ij = ijdev + ijesf
= 0 30 6 + 0 4 0 = 0 26 6 Pa
0 6 5 0 0 4 0 6 9

Obteniendo la ecuacin caracterstica del tensor de tensiones desviador:


ijdev ij = 0
3 J 2 J 3 = 0

Con la solucin de la ecuacin cbica anterior obtenemos las tensiones principales del
tensor desviador:
1dev = 25 Pa
dev
2 = 6 Pa
dev = 31Pa
3

Universidad de Castilla- La Mancha Draft Por: Eduardo W. V. Chaves (2012)


Ciudad Real - Espaa
3 TENSIONES 307

Ejemplo 3.30
Descomponer el tensor de tensiones de Cauchy dado por sus componentes:
12 4 0
ij = 21 9 2 MPa
31 32 3
en su parte esfrica y desviadora.
Obtener los invariantes del tensor desviador
Obtener tambin la tensin normal octadrica, y la tensin media en este punto.
Solucin:
Debido a la simetra del tensor de tensiones de Cauchy:
12 4 0
ij = 4 9 2 MPa

0 2 3

I 12 + 9 + 3 24
Tensin media m = oct = = = = 8.
3 3 3
La parte esfrica y desviadora del tensor de tensiones son:
8 0 0 12 4 0 8 0 0 4 4 0
ijesf = 0 8 0 ; ijdev = ij ijesf
=4
9 2 0 8 0 = 4 1 2

0 0 8 0 2 3 0 0 8 0 2 5

Los invariantes principales del tensor desviador son:


I dev J1 = 4 + 1 5 = 0 , como era de esperar, ya que la traza de cualquier tensor desviador
es cero.
1 2 4 0 4 4
II dev = + + = 41 = J 2
2 5 0 5 4 1

o bien utilizando la definicin: J 2 =


3
(
1 2 1
) (
I 3 II = 24 2 3 151 = 41
3
)
III dev J 3 = det ( dev ) = 44

Ejemplo 3.31
El estado tensional en un punto est dado por el tensor de tensin:
a b
ij = a c
b c

donde a , b , c son constantes y es un valor de tensin. Determinar las constantes a ,


b , c de tal manera que el vector tensin se anule en un plano octadrico.
Solucin:

Universidad de Castilla- La Mancha Draft Por: Eduardo W. V. Chaves (2012)


Ciudad Real - Espaa
308 PROBLEMAS RESUELTOS DE MECNICA DEL MEDIO CONTINUO

1
Un plano octadrico tiene el siguiente versor: n i = [1 1 1] . El vector tensin en este
3
r (n )
plano viene definido por t = n , en componentes:
t 1(n ) a b 1 + a + b 0 a + b = 1
(n ) 1 1
t 2 = a c 1 = a + + c = 0 a + c = 1
t (n ) b c 3 1 3
b + c + 0 b + c = 1
3

1 1 1
resolviendo el sistema anterior obtenemos que, b = , c= , a=
2 2 2

Ejemplo 3.32
En un punto P del medio continuo el tensor de tensiones de Cauchy viene
representado por sus componentes cartesianas por:
57 0 24
ij = 21 50 0 MPa ,
31 32 43
a) Determinar las tensiones principales y las direcciones principales en el punto P ;
b) Obtener la tensin tangencial mxima y la tensin normal mxima;
c) Dibujar el crculo de Mohr del estado tensional correspondiente;
r
d) Obtener el vector tensin t (n) en el plano octadrico del espacio de Haigh-
Westergaard. Obtener tambin la tensin normal octadrica y la tensin tangencial
octadrica.
Solucin:
Teniendo en cuenta que la simetra del tensor de tensiones de Cauchy:
57 0 24
ij = 0 50 0 MPa
24 0 43

Verificamos que la tensin 22 = 50 ya es una tensin principal y est asociada al


autovector n ( 2) = [0 1 0] . Para encontrar las otras tensiones principales resolvemos el
siguiente sistema:
57 24 1 = 25
=0 2 100 + 1875 = 0
24 43 3 = 75
Utilizando la definicin de autovalor-autovector, podemos obtener los siguientes
autovectores:
Asociado al autovalor 1 = 25 n (1) = [m 0,6 0 0,8]

Asociado al autovalor 3 = 75 n (3) = [ 0,8 0 0,6]


Circulo de Mohr en tensiones:
Reestructurando tal que I > II > III :
I = 75 , II = 50 , III = 25

Universidad de Castilla- La Mancha Draft Por: Eduardo W. V. Chaves (2012)


Ciudad Real - Espaa
3 TENSIONES 309

b, c) El crculo de Mohr viene dibujado en la figura abajo:


S
75 25
max = = 25
max = 25 2

III = 25 II = 50 I = 75 = N max N

d) El espacio de Haigh-Westergaard est formado por las tensiones principales luego, el


r
vector tensor en este espacio viene dado por t (n) = n , cuya normal del plano octadrico
1 1 1
tiene como componentes n i = :
3 3 3

t 1(n) 75 0 0 1 75
r (n) 1 1
t (n) = n componente
s t 2 = 0 50 0 1 = 3 50
t (n) 0 0 25 3 1 25
3
Su mdulo viene dado por:

( )
r 2 1 8750 r
t (n) = 75 2 + 50 2 + 25 2 = t (n) = 54,00617
3 3
r
La tensin normal octadrica viene dada por oct = t (n) n :
1
1
oct = [75 50 25] 1 = 50
3 3
1

Podramos haber aplicado directamente la definicin de tensin normal octadrica:


I 75 + 50 + 25
oct = = m = = 50
3 3
La tensin tangencial octadrica se puede obtener a travs del teorema de Pitgoras:
r 2 8750
t (n)
2
oct = oct = 50 2 = 20,4124
3
Tambin podramos haber aplicado la definicin:
1 1
oct = 2 I 2 6 II = 2 150 2 6 6875 = 20,41241
3 3
donde I = 150 , II = 75 50 + 75 25 + 50 25 = 6875 .
e) Componentes de la parte esfrica del tensor:

Universidad de Castilla- La Mancha Draft Por: Eduardo W. V. Chaves (2012)


Ciudad Real - Espaa
310 PROBLEMAS RESUELTOS DE MECNICA DEL MEDIO CONTINUO

50 0 0
Tr ( )
ijesf = ij = m ij = 0 50 0
3
0 0 50

Su parte desviadora:
57 0 24 50 0 0 7 0 24
ijdev = ij ijesf = 0 50 0 0 50 0 = 0 0 0
24 0 43 0 0 50 24 0 7

f) Teniendo en cuenta que el tensor y su parte desviadora son coaxiales, es decir, presentan
las mismas direcciones principales, podemos utilizar el espacio principal para obtener los
valores principales del tensor desviador:
75 0 0 50 0 0 25 0 0
ijdev = ij ijesf
= 0 50 0 0 50 0 = 0 0 0
0 0 25 0 0 50 0 0 25

3.1.6 Estado Tensional en Dos Dimensiones

Ejemplo 3.33
Considere el siguiente estado de tensin:

2
y

6
x

Figura 3.22:

Obtener el estado de tensin en este punto ij .


Solucin:
En el estado de tensn plano ij (i, j = 1,2) se necesitan dos planos para definir
completamente el estado tensional en el punto:
x xy
ij = (3.78)
xy y

Universidad de Castilla- La Mancha Draft Por: Eduardo W. V. Chaves (2012)


Ciudad Real - Espaa
3 TENSIONES 311

Segn la Figura 3.22 verificamos que:


5

x = 4

xy = 2
xy = 2

y = 6

x
Figura 3.23:

Luego:
4 2
ij = (3.79)
2 6

Ejemplo 3.34
Considrese un material compuesto, constituido por matriz y fibras segn direccin de 45
tal como se indica en la Figura 3.24. Este material compuesto puede romper si la tensin de
corte a lo largo de la fibra supera el valor de 3,8 10 6 Pa ( N / m 2 ) .
Para una tensin normal x = 2,8 10 6 Pa , determnese el valor mximo de y para que el
material no rompa.

45 x 45 x

n
y

y
x

Figura 3.24: Material compuesto (matriz-fibra).

Universidad de Castilla- La Mancha Draft Por: Eduardo W. V. Chaves (2012)


Ciudad Real - Espaa
312 PROBLEMAS RESUELTOS DE MECNICA DEL MEDIO CONTINUO

Solucin: Este es un ejemplo tpico de transformacin de coordenadas. Es decir, tenemos


que considerar la tensin de corte mxima segn la direccin = 45 . Para ello realizamos
la transformacin de coordenadas siguiente:
x y
xy ( ) = sin 2 + xy cos 2
2
y 4,8 10 6 Pa (compresin)
2,8 10 6 y
xy ( = 45 ) = sin( 90 ) = 3,8 10 6 Pa
2
Ver Ejemplo 1.98 del captulo 1.

Ejemplo 3.35
Las tensiones que actan en dos planos que pasan por el punto P estn indicadas en la
Figura 3.25. Determnese el valor de la tensin de corte en el plano a a y las tensiones
principales en este punto.
y

a b


80 Pa
45
x
60 Pa 60
a

b
Figura 3.25: Estados tensionales en un punto, segn los planos a y b .
Solucin:
Para obtener el estado de tensin en un punto, en el caso de dos dimensiones,
determinamos las tensiones: x , y , xy , como se indica en la Figura 3.26.

y y

b a b
a y y
xy xy

80 Pa xy 80 Pa xy
x
45 x 45 x

x x
60
60 xy

60 Pa 60 Pa
b a b a
a) b)

Figura 3.26: Estados tensionales en un punto, segn los planos a y b .

Universidad de Castilla- La Mancha Draft Por: Eduardo W. V. Chaves (2012)


Ciudad Real - Espaa
3 TENSIONES 313

Segn la Figura 3.26, podemos determinar directamente x y xy descomponiendo el


vector tensin 60 Pa , ver Figura 3.26(b), i.e.:
x = 60 cos( 30 ) = 51,962 Pa
xy = 60 cos( 60 ) = 30 Pa

Para determinar la componente y , emplearemos las ecuaciones:


x + y x y
x ( ) N = + cos 2 + xy sin 2
2 2
x y
xy S ( ) = sin 2 xy cos 2
2
Reemplazando los valores numricos en las expresiones anteriores:
51,962 + y 51,962 y
( = 45 ) = + cos( 90 ) + 30 sin( 90 ) = 80 Pa
2 2
51,962 y
( = 45 ) = sin( 90 ) 30 cos( 90 )
2
La primera ecuacin nos proporciona el valor de y :
y = 48,038 Pa

Una vez determinado y , podemos determinar (= 45) :


( = 45 ) = 1,96 Pa

Las tensiones principales pueden determinarse a travs de las componentes x , y , xy , tal


como se indica en las ecuaciones:
2
x + y x y
(1, 2 ) =
+ 2xy
2 2

2
51,962 + 48,038 51,962 48,038 = 80,1Pa
(1, 2 ) = + 30 2 1
2 2 2 = 19,9 Pa

Universidad de Castilla- La Mancha Draft Por: Eduardo W. V. Chaves (2012)


Ciudad Real - Espaa
314 PROBLEMAS RESUELTOS DE MECNICA DEL MEDIO CONTINUO

Ejemplo 3.36
Dado el estado de tensiones x = 1Pa , xy = 4 Pa y y = 2 Pa . Obtener una grfica de
ngulo-tensiones ( x , y , xy ), siendo el ngulo de giro de la cua dada en la Figura
3.27.
y

xy = 4 Pa y = 2 Pa
xy = 4 Pa
x x = 1Pa
P
x

xy

Figura 3.27: Estado tensional en un punto.

Solucin:
Calculemos los distintos valores de x , y , xy utilizando las ecuaciones:
x + y x y
x = + cos 2 + xy sin 2
2 2
x y
xy = sin 2 + xy cos 2
2
x + y y x
y = + cos 2 xy sin 2
2 2
Podemos calcular el ngulo correspondiente a la direccin principal a travs de la ecuacin:
2 xy 2 ( 4)
tan 2 = = = 8 ( = 41,437 )
x y 1 2
y las tensiones principales:
2
x + y x y 1 = 5,5311P
1, 2 =
+ 2xy
2 2 2 = 2,5311Pa
Considerando las leyes de transformacin, podemos obtener los distintos valores de
x , y , xy para distintos valores de . Haciendo variar de 0 hasta 360 podemos
representar las tensiones x , y , xy en funcin del ngulo, ver Figura 3.28. Podemos
observar que cuando = 41,437 la tensin tangencial es cero ( xy = 0 ) y las tensiones
principales I = 5,5311Pa y II = 2,5311Pa .

Universidad de Castilla- La Mancha Draft Por: Eduardo W. V. Chaves (2012)


Ciudad Real - Espaa
3 TENSIONES 315

x
1
= 41,437
2

x
= 131,437
8 2
Tensiones

6
1 = 5,5311 y

y 2
xy
x
0
0 50 100 150 200 250 300 350

-2 x
45 x

xy -4 2 = 2,5311
= 86,437

-6

max = 4,0311

Figura 3.28: Tensiones en funcin del ngulo .

Ejemplo 3.37
a) Dado un campo de tensiones ij (i , j = 1,2) , y los siguientes valores:
t t t
2 2 2
m11 =
t
11 x 3 dx 3 ; m12 =
t
12 x 3 dx 3 ; m 22 = t
22 x 3 dx 3


2 2 2

y dado un nuevo sistema x1 x 2 x 3 formado por una rotacin alrededor de x3 de un


ngulo , obtener la ley de transformacin de mij (i, j = 1,2) para este nuevo sistema.
Solucin:
Debido a la simetra de ij = ji , concluimos que m12 = m21 . La matriz de transformacin
del sistema x1 x 2 x 3 al sistema x1 x 2 x 3 viene dada por:
cos sin 0
cos sin
a ij = sin cos 0 2D
A=
0 0 1 sin cos

Universidad de Castilla- La Mancha Draft Por: Eduardo W. V. Chaves (2012)


Ciudad Real - Espaa
316 PROBLEMAS RESUELTOS DE MECNICA DEL MEDIO CONTINUO

Vamos utilizar la notacin de Voigt, luego:


t t t t
2 2 2 2



m11
11
11 11 11
{m } = m22 =

22 x3 dx3 =

22 x3 dx3 = [M] 22 x3dx3 = [M]
22 x3 dx3
m
12 12 12 12 12
t t t t
2 2 2 2

con eso, podemos concluir que:



m11 m11
{m } = m 22 = [M]m 22 = [M]{m}

(3.80)
m m
12 12
donde la matriz [M] es la matriz de transformacin para un tensor de segundo orden
cuando ste est en la notacin de Voigt, ver Ejemplo 1.98, y viene dada por:
a11 2 a12
2
2a11 a12 cos 2 sin 2 2 cos sin

[M] = a 21 2 a 22
2
2a 21 a 22 = sin
2
cos 2
2 sin cos
a a a 22 a12 a11 a 22 + a12 a 21 sin cos cos sin cos 2 sin 2
21 11
Adems considerando que [M]1 = [N ] , obtenemos {m} = [N ] {m }, donde
T T

a11 2 a12
2
a11 a12 cos 2 sin 2 cos sin
[N ] = a 212 a 22
2
a 21 a 22

= sin
2
cos 2 sin cos

2a a 2a 22 a12 a11 a 22 + a12 a 21 2 sin cos 2 cos sin cos 2 sin 2
21 11
El mismo resultado (3.80) podra haber sido obtenido si considerbamos mij como un
tensor de segundo orden en el plano, y a travs de la ley de transformacin de un tensor de
segundo orden obtenemos que:
mij = a ik a jl m kl ; (i, j = 1,2) m = Am A T
m m12 cos sin m11 m12 cos sin (3.81)
11 =

m12 m 22 sin cos m12 m 22 sin cos

x3 = x3

x2
t
x3 =
2
x2

x1
t
x3 =
2 x1

Universidad de Castilla- La Mancha Draft Por: Eduardo W. V. Chaves (2012)


Ciudad Real - Espaa
3 TENSIONES 317

3.1.7 Tensiones en Coordenadas Cilndricas y Esfricas

Ejemplo 3.38
Demustrese que un cilindro cerrado de pared delgada de radio interno r y espesor t
sujeto a una presin interna p , ver Figura 3.29, tiene como estado tensional:
pr pr
r = 0 ; = ; z =
t 2t
NOTA: Las expresiones anteriores slo son vlidas para un cilindro de pared delgada.

z
p

Figura 3.29: Cilindro cerrado bajo presin.


Solucin:
Una vez adoptados los ejes de referencia de la Figura 3.29, planteamos el equilibrio de
fuerzas segn las direcciones z , y y r .
Equilibrio de fuerzas segn direccin z :

F z =0
p z
2
p ( r ) = z ( 2 r )t
pr r pA = p (r 2 )
z =
2t
z

Equilibrio de fuerzas segn direccin y :

L
Fy = 0
2 ( Lt ) = p ( 2 rL )
pr
=
t
pA = p ( 2rL )

r y

Universidad de Castilla- La Mancha Draft Por: Eduardo W. V. Chaves (2012)


Ciudad Real - Espaa
318 PROBLEMAS RESUELTOS DE MECNICA DEL MEDIO CONTINUO

Equilibrio de fuerzas segn direccin r :


Podemos verificar que en la pared interna del cilindro la tensin radial ( r ) es igual a la
presin ( r = p ) y en la pared externa est libre de presin r = 0 , luego:
p r 0

r = p r = 0
r
p

r r <<
para el caso >> 1 luego, r 0 cuando comparado con z , .
t r << z
r
Experimentalmente se ha observado que la relacin para despreciar r es 10 . En esta
t
situacin:
pr pr
z = = 5p ; = = 10 p ; r = p
2t t
Podemos verificar tambin que la tensin es ms grande, > z , es decir, que para un
material homogneo, un cilindro rompera segn direccin de z , como se indica en la
figura siguiente:

Pero si el material estuviera


constituido por un material
heterogneo, como por ejemplo, z
matriz y fibras en la direccin de ,
la forma de rotura ya no estara tan
definida.
Observemos tambin que las
tensiones obtenidas anteriormente
para el cilindro de pared delgada no
r
sern vlidas si < 10 . El error
t
cometido ya ser significativo.

Universidad de Castilla- La Mancha Draft Por: Eduardo W. V. Chaves (2012)


Ciudad Real - Espaa
3 TENSIONES 319

Ejemplo 3.39
Demustrese que una esfera cerrada de pared delgada de radio interno r y espesor t ,
sometida a una presin interna p , ver Figura 3.30, presenta el estado tensional siguiente:
pr pr
r = 0 ; = ; =
2t 2t
NOTA: Las expresiones anteriores slo son vlidas para una esfera de pared delgada.

x3 , z

e r
x3
e
r
e

x1 x2
x2 , y

x1 , x

Figura 3.30: Esfera sometida a presin interna.


Solucin:
Considerando los ejes adoptados en la Figura 3.30, planteamos el equilibrio de fuerzas
segn direccin x , y , r .
Equilibrio de fuerzas segn direccin x :

y
e e r
F x =0
e
p ( r 2 ) + ( 2 r )t = 0
pr
2 =
pA = p ( r ) 2t

2r x

Universidad de Castilla- La Mancha Draft Por: Eduardo W. V. Chaves (2012)


Ciudad Real - Espaa
320 PROBLEMAS RESUELTOS DE MECNICA DEL MEDIO CONTINUO

Equilibro de fuerzas segn direccin y :

pA = p ( r 2 )
x
e e
F y =0
e r
p ( r 2 ) + ( 2 r )t = 0
pr
=
2t

2r

Equilibrio de fuerzas segn direccin r :


Podemos verificar que en la pared interna de la esfera la tensin radial es igual a la presin
( r = p ) y que la pared externa est libre de presin ( r = 0 ), luego:
p r 0

r = p r = 0
r
p

r r <<
Para el caso >> 1 luego r 0 comparado con , .
t r <<
r
Experimentalmente se ha observado que la relacin para despreciar r es 10 , en esta
t
situacin:
pr
= 2t = 5 p

pr
= = 5p
2t
r = p

Universidad de Castilla- La Mancha Draft Por: Eduardo W. V. Chaves (2012)


Ciudad Real - Espaa
3 TENSIONES 321

3.2 Ejercicios Propuestos

Problema 3.1
El tensor de tensiones en la base cartesiana e i viene representado por sus componentes:

0,1 0,6 0,0


ij = 0,6 1,2 0,0 MPa (3.82)
0,0 0,0 0,3

Se pide:
r
a) Encontrar el vector traccin t segn el plano 2 x1 2 x 2 + x 3 = 1 ;
r
b) Encontrar la magnitud del vector traccin t , vector tensin normal y tangencial
segn el plano 2 x1 2 x 2 + x 3 = 1 ;
c) Obtener los invariantes principales I , II , III ;
d) Obtener la tensin de corte octadrica.

Problema 3.2
Dibujar el crculo de Mohr para cada uno de los siguientes casos:
a) 11 = 150 MPa (tensin uniaxial)
b) 22 = 110 MPa (compresin uniaxial)
c) 11 = 50 MPa , 22 = 100 MPa (tensin biaxial)
d) 11 = 50 MPa , 22 = 50 MPa (tensin biaxial)
e) 11 = 8 MPa , 22 = 33 = 40 MPa (tensin triaxial)
f) 11 = 50 MPa , 22 = 10 MPa , 12 = 21 = 40 MPa , 33 = 30 MPa
Para cada caso encontrar tambin la tensin de corte mxima.

Problema 3.3
Las componentes del tensor de tensiones en un punto P , en el sistema de coordenadas
cartesianas, vienen dadas por:
3 2 2
ij = 2 4 0 MPa (3.83)
2 0 2

Encontrar:
a) El vector traccin en el punto P cuyo plano es normal al eje x1 ;

Universidad de Castilla- La Mancha Draft Por: Eduardo W. V. Chaves (2012)


Ciudad Real - Espaa
322 PROBLEMAS RESUELTOS DE MECNICA DEL MEDIO CONTINUO

b) El vector traccin en P cuyo plano tiene normal igual a (1,3,2) ;


c) Las tensiones principales de ;
d) Las direcciones principales de .

Problema 3.4
Consideremos que 13 = 23 = 33 = 0 . Mostrar que si los ejes x1* y x2* son obtenidos a
partir de una rotacin de sobre el eje x3 , las componentes del tensor de tensiones en
este nuevo sistema vienen dadas por:
1 1
*
11 = (11 + 22 ) + (11 22 ) cos 2 + 12 sin 2 (3.84)
2 2
1 1
*22 = (11 + 22 ) (11 22 ) cos 2 12 sin 2
2 2
(3.85)
1
*
12 = (11 22 ) sin 2 + 12 cos 2
2

Problema 3.5
Partiendo de qu principio se demuestra la simetra del tensor de tensiones?

Problema 3.6
Dadas las componentes del tensor de tensiones:
1 1 0
ij = 1 1 0
0 0 2

y sus valores principales:


2 0 0
ij = 0 2 0 (3.86)
0 0 0

Obtener los invariantes J1 , J 2 y J 3 .

Problema 3.7
Sabiendo que un punto se encuentra en un estado de tensin hidrosttico y que la presin
media es ( 4 Pa ), determinar las tensiones principales y las direcciones principales en este
punto.

Problema 3.8
El campo de tensin de un medio continuo viene representado por:

Universidad de Castilla- La Mancha Draft Por: Eduardo W. V. Chaves (2012)


Ciudad Real - Espaa
3 TENSIONES 323

1 0 2 x2
ij = 0 1 4 x1 unidades de tensin (3.87)
2 x 2 4 x1 1

donde xi son las coordenadas cartesianas.


Se pide:
a) Despreciando las fuerzas msicas, est el cuerpo en equilibrio?
b) Determinar el vector tensin que acta en un punto ( x1 = 1, x 2 = 2, x3 = 3) segn el
plano x1 + x 2 + x3 = 6 .
c) Determinar la proyeccin del vector tensin segn la direccin normal y tangencial
al plano x1 + x 2 + x3 = 6

Problema 3.9
Si las componentes del tensor de tensiones de Cauchy en el sistema ortonormal e1 , e 2 , e 3
son:
11 12 13
ij = 12 22 23 (3.88)
13 23 33

Si las tensiones principales son 1 = 30,7259 , 2 = 13,8247 y 3 = 4,4494 y las direcciones


principales son:
para 1 n (1) = [ 0,6720 0,7127 0,2010]
para 2 n ( 2 ) = [ 0 ,4237 0,1474 0,8937] (3.89)
para 3 n (3) = [ 0,6073 0,6858 0,4010]

Obtener las componentes del tensor de tensiones de Cauchy ij en el sistema: e1 , e 2 , e 3 .

Problema 3.10
El estado de tensin en un medio continuo respecto a los ejes cartesianos est dado por:
3 x1 x 2 5 x 22 0

ij = 5 x 22 0 2 x3
0 2 x3 0

Determinar el vector tensin que acta en el punto P(2,1, 3 ) de un plano que es tangente
en P a la superficie cilndrica x 22 + x32 = 4 .

Problema 3.11
Obtener las componentes del tensor de tensiones de Cauchy en el punto representado por
el elemento infinitesimal dado por la figura abajo:

Universidad de Castilla- La Mancha Draft Por: Eduardo W. V. Chaves (2012)


Ciudad Real - Espaa
324 PROBLEMAS RESUELTOS DE MECNICA DEL MEDIO CONTINUO

Problema 3.12
El estado de tensin en un punto P del medio continuo se da esquemticamente por:
x3

1
4
1
4 22

1 1
x2

x1

Se pide:
Determinar el valor de la componente 22 del tensor de tensiones para que exista al menos
un plano que pase por P que est libre de tensiones;
Determinar la direccin de dicho plano.

Problema 3.13

Universidad de Castilla- La Mancha Draft Por: Eduardo W. V. Chaves (2012)


Ciudad Real - Espaa
4 Leyes Fundamentales de
la Mecnica del Medio
Continuo
4.1 Ejercicios Resueltos

Ejemplo 4.1
Demostrar el teorema del transporte de Reynolds partiendo de la siguiente expresin:
D D

Dt V
dV =
Dt V
JdV 0 (4.1)
0

donde V es el volumen en la configuracin actual, V0 es el volumen en la configuracin de


referencia, J es el determinante del Jacobiano y es un campo escalar que describe una
cantidad fsica de una partcula en el espacio por unidad de volumen en un instante de
tiempo t .
Solucin:
D D DJ

Dt V
JdV0 = J
V Dt
+ dV 0
Dt
0 0

D r

= J + J xr v dV 0 (4.2)
V0
Dt

D r

= + xr v dV
V
Dt

Ejemplo 4.2
r
Si PijL ( x , t ) representa alguna propiedad escalar, vectorial o tensorial cualquiera por unidad
de masa de un medio continuo, probar que la siguiente expresin es siempre vlida:
r
D r DPijL ( x , t )

Dt V
PijL ( x , t ) dV =
V
Dt
dV (4.3)
326 PROBLEMAS RESUELTOS DE MECNICA DEL MEDIO CONTINUO

Solucin:
Partiendo del Teorema del transporte de Reynolds:
D r D r r v p
Dt V
V
Dt
( x , t )dV = ( x , t ) + ( x , t )
x p
dV

Haciendo que = PijL y reemplazando en la ecuacin anterior, resulta:
D D v p

Dt V
PijK dV
= ( PijK ) + PijK
Dt
V
dV
x p
D D v

=
V
Dt
PijK + PijK
Dt
+ PijK k
x k
dV

D D v k

=
V
Dt
PijK + PijK
Dt
+
x k
dV

14243
=0
ecuacin de continuida d

Con lo que concluimos que:


D DP
Dt V V

PijK dV = ijK dV
Dt
c.q.d.

Ejemplo 4.3
Probar que la siguiente relacin es vlida:
r r r r
a= ( v ) + xr ( v v ) (4.4)
t
r r r r r
donde ( x , t ) es la densidad de masa, a ( x , t ) es la aceleracin Euleriana, y v ( x , t ) es la
velocidad Euleriana.
Solucin:
Partiendo del teorema del transporte de Reynolds:
D r

dV = dV + (v n ) dS

Dt V V
t S
r
y considerando que = v obtenemos que:
r
D r ( v ) r r
v dV = dV + v (v n ) dS

Dt V V
t S
en notacin indicial queda:
D ( v i )
Dt V vi dV =
V
t
dV + v i (v k n k ) dS
S

D ( v i )
1
V
Dt
23
v dV = i
V
t S

dV + ( v i v k )n k dS
= ai
Aplicando el teorema de la divergencia para la integral de superficie obtenemos que:
( v i ) ( v i )
ai dV =
V V
t
dV + ( vi v k ) ,k dV
V
ai dV = V
+ ( v i v k ) ,k dV
V t
En notacin tensorial:
r r
r ( v ) r r r ( v ) r r
V a dV = V t + xr ( v v ) dV a=
t
+ xr ( v v )
c.q.d.

Universidad de Castilla- La Mancha Draft Por: Eduardo W. V. Chaves (2012)


Ciudad Real Espaa
4 LEYES FUNDAMENTALES DE LA MECNICA DEL MEDIO CONTINUO 327

Ejemplo 4.4
Para un campo de velocidad dado por:
xi
vi = para t 0
1+ t
Se pide:
1) Encontrar la densidad de masa de una partcula en funcin del tiempo;
2) Probar que para el movimiento dado se cumple que x1 x 2 x3 = 0 X 1 X 2 X 3 .
Solucin:
1) Por la conservacin de masa sabemos que:
D v D d v
+ k =0 = = k
Dt x k Dt dt x k
Utilizando el campo de velocidad dado hallamos que:
v i 1 x i ii 3
= = =
x i 1 + t x i 1+ t 1+ t
luego
d 3 d 3dt
= =
dt 1+ t 1+ t
Integrando la relacin anterior obtenemos que:
d 3dt
= 1 + t Ln = 3 Ln(1 + t ) + C1
La constante de integracin C1 se obtiene con la condicin inicial t = 0 donde se cumple
que la densidad de masa es igual a la densidad de masa de referencia ( = 0 ):
Ln 0 = 3 Ln(1 + 0) + C1 C1 = Ln 0
1 0
Ln = 3 Ln(1 + t ) + Ln 0 = Ln 3
+ ln 0 = Ln
(1 + t ) 3


(1 + t )
Con lo que concluimos que:
0
=
(1 + t )3
2) Utilizando la definicin de velocidad y aplicando el campo de velocidad obtenemos:
dx i x dx i dt
vi = = i =
dt 1 + t xi 1 + t
Integrando la ecuacin anterior resulta:
dx i dt
xi
=
1+ t
Lnx i = Ln(1 + t ) + C1 (4.5)
Aplicando la condicin inicial, donde se cumple que: t = 0 xi = X i , luego:
Ln X i = Ln(1 + 0) + C1 C1 = Ln X i
Reemplazando el valor de C1 en la ecuacin (4.5) obtenemos:
Ln x i = Ln(1 + t ) + Ln X i Ln( x i ) = Ln[ X i (1 + t )]
con lo que concluimos que:
xi = X i (1 + t )
Expandiendo la relacin anterior:

Universidad de Castilla- La Mancha Draft Por: Eduardo W. V. Chaves (2012)


Ciudad Real - Espaa
328 PROBLEMAS RESUELTOS DE MECNICA DEL MEDIO CONTINUO

x1
x1 = X 1 (1 + t ) (1 + t ) =
X1
x2
x 2 = X 2 (1 + t ) (1 + t ) =
X2
x
x 3 = X 3 (1 + t ) (1 + t ) = 3
X3
0
y considerando que = (ver apartado 1 de este ejemplo), obtenemos:
(1 + t )3
(1+3
12 t )(1+3
12 t )(1+3
12 t) = 0
x1 x2 x3
X1 X2 X3

x1 x 2 x 3 = 0 X 1 X 2 X 3 c.q.d.

Ejemplo 4.5
r
Considrese un medio continuo y una propiedad ( x , t ) asignada por la densidad, i.e.
unidad de la propiedad por unidad de volumen. Obtener la tasa de esta propiedad de tal
forma que venga descrita por una integral de volumen de control conjuntamente con una
integral de superficie de control.
Solucin:
Recordar de la tasa de una propiedad est relacionada siempre con las mismas partculas. A
travs de la Derivada Material podemos obtener la tasa de una propiedad cuando sta est
en la descripcin espacial. Luego, la variacin total de la propiedad en volumen V que est
delimitado por la superficie S viene dada por:


( x, t ) + ( x, t ) (dV )
D r D D r r D

Dt V
( x, t )dV =
V
Dt
( dV ) = dV
V

Dt Dt

( x, t ) + ( x, t ) xr v dV
D r r r

= dV
V
Dt
(4.6)

D r r r
= ( x , t ) + ( x , t ) xr v dV

V
Dt
Podemos aplicar la definicin de derivada material a la ecuacin anterior, resultando que:

( x, t )dV = ( x, t ) + ( x, t ) xr v dV
D r D r r r
Dt V V
Dt
r
r ( x , t ) r r r r

= ( x, t ) + r v ( x , t ) + ( x , t ) xr v dV
V
t x
r (4.7)
r ( x , t ) r r r

= ( x , t ) dV + r v + ( x , t ) xr v dV
V
t V
x
r r
= ( x , t ) dV + [ xr (v ) ]dV

V
t V

Podemos aplicar el teorema de la divergencia a la segunda integral del lado derecho de la


igualdad y obtener que:

Universidad de Castilla- La Mancha Draft Por: Eduardo W. V. Chaves (2012)


Ciudad Real Espaa
4 LEYES FUNDAMENTALES DE LA MECNICA DEL MEDIO CONTINUO 329

flujo de a travs
de la superficie S
6 4 4744 8
r
D r ( x , t ) r (4.8)
( x, t )dV = dV + ({ v ) n dS

Dt V V 1
t4
42 3 S flujo de
local
r
( x , t )
como el trmino es local, la integral de volumen de la derecha de la igualdad es un
t
volumen de control y la integral de superficie es una superficie de control ya que las
r r
variables (v ) estn en las descripcin Euleriana. El trmino (v ) representa el flujo de la
propiedad . Cuando no hay fuente o sumidero de la propiedad se cumple que
D r
Dt V( x, t )dV = 0 . Verificar tambin que la ecuacin (4.8) cuando la propiedad es la

densidad de masa se denomina de ecuacin de continuidad de masa:

( x, t )dV = ( x, t ) + ( x, t ) xr v dV = 0
D r D r r r
Dt V V

Dt
(4.9)
r r
= ( x , t ) + xr ( v ) dV = 0

V
t

Si la ecuacin anterior es vlida para todo el volumen tambin lo ser localmente, i.e.:
D r r r
( x, t ) + ( x, t ) xr v = 0 Ecuacin de continuidad de masa (4.10)
Dt
or
r r
( x, t ) + xr ( v ) = 0 Ecuacin de continuidad de masa (4.11)
t

volumen de control r
(v )
S
r r
V q n = [(v ) n ] n
n
r
r ( x , t )
x
t
superficie de control

Figura 4.1: Volumen y superficie de control.

Universidad de Castilla- La Mancha Draft Por: Eduardo W. V. Chaves (2012)


Ciudad Real - Espaa
330 PROBLEMAS RESUELTOS DE MECNICA DEL MEDIO CONTINUO

Volumen material Volumen de control Superficie de control

t=0

v0

XP
X*

Superficie de control
Volumen material Volumen de control
t1

r
v( x * , t1 )

xP
x*
Superficie de control
Volumen material Volumen de control
t2

r
v( x * , t 2 )

xP x*

Figura 4.2: Volumen material vs. volumen y superficie de control.

Ejemplo 4.6
Las componentes del campo del tensor de tensiones de un medio continuo en equilibrio
vienen dadas por:
11 = x12 ; 22 = x 22 ; 33 = x12 + x 22
12 = 21 = 2 x1 x 2 ; 23 = 32 = 31 = 13 = 0
Encontrar la fuerza msica (por unidad de volumen) que acta en el continuo.
Solucin: Aplicando las ecuaciones de equilibrio podemos obtener que:
r r
xr + b = 0
11 12 13
+ + + b1 = 0
x1 x 2 x 3
2 x1 + 2 x1 + b 1 = 0
21 22 23
+ + + b2 = 0 2 x 2 + 2 x 2 + b 2 = 0
x1 x 2 x 3 b = 0
31 32 33 3
+ + + b3 = 0
x1 x 2 x 3
Con lo que concluimos que para satisfacer las ecuaciones de equilibrio hay que cumplir que:

Universidad de Castilla- La Mancha Draft Por: Eduardo W. V. Chaves (2012)


Ciudad Real Espaa
4 LEYES FUNDAMENTALES DE LA MECNICA DEL MEDIO CONTINUO 331

4 x1 = b 1 b 1 = 4 x1
4 x 2 = b 2 b 2 = 4 x 2
b3 = 0
r
b = 4( x1 e 1 + x 2 e 2 )

Ejemplo 4.7
Sea el movimiento del continuo dado por las siguientes ecuaciones:
x1 = X 1 + tX 3

x 2 = X 2 + tX 3
x = X t ( X + X )
3 3 1 2
donde es una constante. Encontrar la densidad de masa en la configuracin actual ( )
en funcin de la densidad de masa en la configuracin de referencia ( 0 ) .
Solucin:
Queremos encontrar una funcin densidad de masa tal que: = ( 0 ) . Sabemos que la
ecuacin de continuidad de masa en la forma Lagrangiana viene dada por 0 = J , donde
el determinante del Jacobiano puede ser obtenido directamente de las ecuaciones de
movimiento:
x1 x1 x1
X 1 X 2 X 3
1 0 t
x i x 2 x 2 x 2
J= F = = = 0 1 t = 1 + 2(t ) 2
X j X 1 X 2 X 3
x 3 x 3 x 3 t t 1
X 1 X 2 X 3
Resultando as:
0 0
= =
J 1 + 2(t ) 2

Ejemplo 4.8
Dado el campo de velocidad:
v1 = x1 x3 ; v 2 = x 22 t ; v3 = x 2 x 3t
y el campo de tensiones:
x 2 x1 x 2 x3 0
ij = x 2 x3 x 22 x 2
0 x2 x32
donde es una constante. Encontrar las fuerzas msicas (por unidad de volumen) de tal
forma que cumpla con el principio de la conservacin del momento lineal.
Solucin:
Del principio de la conservacin del momento lineal obtenemos las ecuaciones del
movimiento:
r r r r r
xr + b = v& = a b = a xr
El campo de aceleracin:

Universidad de Castilla- La Mancha Draft Por: Eduardo W. V. Chaves (2012)


Ciudad Real - Espaa
332 PROBLEMAS RESUELTOS DE MECNICA DEL MEDIO CONTINUO

r r r r
r v ( x , t ) v ( x , t ) r r v i v i
a= + r v ( x, t ) ; ai = + vj
t x t x j

donde
0 x3 0 x1
v i 2 v i
= x2 ; = 0 2x2 t 0
t x j
x 2 x3 0 x3t x 2 t
Luego:
v i v i
ai = + vj
t x j
0 x3 0 x1 x1 x 3 0 x1 x 32 + x1 x 2 x 3 t

= x 22 + 0 2 x2t 0 x 22 t = x 22 + 2 x 23 t
x 2 x 3 0 x3t x 2 t x 2 x 3 t x 2 x 3 x 3 x 22 t 2 + x 22 x 3 t 2
x1 x 32 + x1 x 2 x 3 t
2 3
= x2 + 2x2t
x x + x x 2t 2 + x 2 x t 2
2 3 3 2 2 3
La divergencia del tensor de tensiones de Cauchy viene dada por:
11 12 13
+ + = ( x 2 x3 )
x1 x 2 x 3
21 22 23
+ + = (2 x2 )
x1 x 2 x 3
31 32 33
+ + = ( 2 x 3 1)
x1 x 2 x 3

Con lo cual, las fuerzas msicas quedan:


r r
b = a xr
b i = a i ij , j
x1 x 32 + x1 x 2 x 3 t x 2 x3

bi = 2
x2 + 2 x2 t 3
2 x2
x 2 x 3 + x 3 x 22 t 2 + x 22 x 3 t 2 2 x 3 1

Ejemplo 4.9
Teniendo en cuenta el Principio de la conservacin del momento angular, demostrar que:

[r r r r
( x (a b) (a b) x dV =
r r
] [ ( xr tr *
r r
]
t * x dS
V S

donde
r r r r r r
x - vector posicin; ( x , t ) -densidad de masa; a ( x , t ) -aceleracin; b( x , t ) -fuerzas msicas
r r
(por unidad de masa); t * ( x , t ) - vector traccin prescrito (fuerza de superficie) en la
superficie S .

Universidad de Castilla- La Mancha Draft Por: Eduardo W. V. Chaves (2012)


Ciudad Real Espaa
4 LEYES FUNDAMENTALES DE LA MECNICA DEL MEDIO CONTINUO 333

Solucin:
El principio de la conservacin del momento angular establece que:
r r r r D r r r r

S V

( x t * )dS + ( x b)dV =
Dt V
( x v )dV = ( x a )dV
V

A continuacin hacemos el producto vectorial de la expresin anterior con un vector
r r
arbitrario z , independiente de x , resultando:
r r r r r r r r r

V

z ( x a )dV = z
S
( x t * )dS + z ( x b)dV
V
r r r r r r r r r

z ( x a )dV =
V

S

z ( x t * )dS + z ( x b)dV
V

r r r
Se ha demostrado en el captulo 1 que dados tres vectores a , b , c , se cumple que
r r r r r r r r
a (b c ) = (b c c b) a . Luego, la expresin anterior puede ser reescrita como:
r r r r r r r r r r r r r r r
( x a a x ) z dV = ( x t * t * x ) z dS + ( x b b x ) z dV

V S V
r r r r r r r r r r r r r r r
( x a a x ) z dV ( x b b x ) z dV = ( x t * t * x ) z dS

V V S
r r r
[ r r r r r r r r r
x (a b) (a b) x z dV = ( x t * t * x ) z dS
]
V S

r r r r

V
r r r
[ r r r

r r
]
x (a b) (a b) x dV z = ( x t * t * x ) dS z
S
Con eso, concluimos que:

[x (a b) (a b) x ] dV = ( x t
r r r r r r r r* r r
t * x ) dS
V S

Ejemplo 4.10
1) Teniendo en cuenta la definicin del tensor tensin media ( ) de un medio continuo:
V = dV
V

Y partiendo del principio de que el continuo est en equilibrio esttico, demostrar que se
cumple que:

=
1
2V
[r r
x b + b x dV +
r r
] 1
2V
r r r r
( x t * + t * x ) dS
V S

2) Teniendo en cuenta ahora que el volumen del continuo viene dado por V = V (1) V ( 2) ,
ver Figura 4.3. Dicho continuo est sometido a una presin p (1) en la superficie S (1) , y una
presin p ( 2) en la superficie S ( 2) . Considerando que el continuo est libre de fuerzas
msicas, verificar que se cumple la relacin:
1
= (1) ( 2)
( p (1)V (1) p ( 2)V ( 2 ) )1
(V V )

Universidad de Castilla- La Mancha Draft Por: Eduardo W. V. Chaves (2012)


Ciudad Real - Espaa
334 PROBLEMAS RESUELTOS DE MECNICA DEL MEDIO CONTINUO

n (1)
S (1)

V (1) p (1)

S (2)

V ( 2)
n ( 2 )

p (2)

Figura 4.3

Solucin:
r r r
Teniendo en cuenta la ecuacin de equilibrio xr + b = a = 0 (principio de la
conservacin del momento lineal). Para todo el continuo hay que cumplir que:


r
[ r
] r
x xr + b dV = 0
V
r r r r (4.12)
x xr dV + x b dV = 0

V V

En el captulo 1, hemos demostrado que se cumplen las siguientes relaciones:

r r r* r
( ) x dV = ( n ) x dS dV = t
V S V S
x dS dV
V
(4.13)

r r r r
x ( ) dV = x ( n ) dS
T
dV = x t * dS T dV (4.14)
V S V S V
r
donde hemos tenido en cuenta que el vector tensin prescrito, t * = n . Reemplazando
(4.14) en la expresin (4.12), obtenemos que:

Universidad de Castilla- La Mancha Draft Por: Eduardo W. V. Chaves (2012)


Ciudad Real Espaa
4 LEYES FUNDAMENTALES DE LA MECNICA DEL MEDIO CONTINUO 335

r r r r
x xr dV + x b dV = 0
V V

r r r r r

S

x t * dS T dV + x b dV = 0
V

V
(4.15)

r r r r

T dV = x t * dS + x b dV
V S

V

Tambin se cumple que:


r r r r

V

dV = t * x dS + b x dV
S

V
(4.16)

r r r r
Observemos que los tensores resultantes de las operaciones x t * , x b , no son
tensores simtricos. Esto quiere decir que en la expresin (4.12) no ha tenido en cuenta el
principio de la conservacin del momento angular (simetra del tensor de tensiones de
Cauchy). Para garantizar que sea simtrico, hacemos que:

+ T 1 r r r r 1 r r r r
2
dV = t * x dS + b x dV + x t * dS + x b dV
2 S 2 S

V V V
(4.17)


sym dV =
1
2V
r r r r
[
x b + b x dV +
1 r r* r* r
2S
x t + t x dS] [ ]
V

Teniendo en cuenta la definicin del tensor tensin media, concluimos que:

+ T 1 r r r r 1 r r r r
2
dV = t * x dS + b x dV + x t * dS + x b dV
2 S 2 S

V V V


sym dV =
1
2V
r r r r
[
x b + b x dV +
1 r r* r* r
2S
x t + t x dS] [ ]
V
(4.18)
V =
1
2V [
r r r r
x b + b x dV +
1 r r* r* r
2S
]
x t + t x dS [ ]
=
1
2V
[r r r
x b + b x dV +
r
] 1
2V
[ xr tr *
r r
+ t * x dS ]
V S

Si adems el cuerpo est libre de fuerzas msicas, la expresin anterior se resume a:

=
1
2V
[ xr tr *
r r
+ t * x dS] (4.19)
S

Para el caso particular de la Figura 4.3 tenemos que V = V (1) V ( 2) , S = S (1) + S ( 2) ,


r (1) r ( 2)
t * = p (1) n (1) , t * = p ( 2) n ( 2) . En este caso, la expresin (4.19) queda:

Universidad de Castilla- La Mancha Draft Por: Eduardo W. V. Chaves (2012)


Ciudad Real - Espaa
336 PROBLEMAS RESUELTOS DE MECNICA DEL MEDIO CONTINUO

r r
[ ] [ ]
1 *
r* r (1) r r* r* r (2)
= x t + t x dS + x t + t x dS
2(V (1) V ( 2 ) ) S (1) S ( 2)

=
2(V (1)
1
( 2)
V ) S (1)
r
[ r
] r
[ r
p (1) x n (1) + n (1) x dS (1) + p ( 2 ) x n ( 2) + n ( 2 ) x dS ( 2 ) ]
S (2)

[ ] [ ]
1 (1) r (1) (1) r (1) ( 2) r ( 2) (2) r ( 2)
= p x n + n x dS + p x n + n x dS
2(V (1) V ( 2 ) ) S ( 1) S ( 2)

r r
Hemos demostrado en el captulo 1 que se cumple ( x n + n x ) dS = 2V 1 , donde n es
S

el versor normal exterior a la superficie S (ver Ejemplo 1.121). Para este ejemplo n ( 2) es
normal interior a la superficie S (2) , debido a eso tenemos que

[x n ]
r r
( 2)
+ n ( 2 ) x dS ( 2 ) = 2V ( 2 ) 1 , resultando que:
(2)
S


[ ] [ ]
1 (1) r
(1) + n (1) x dS (1) + p ( 2 ) x n ( 2) + n ( 2 ) x dS ( 2 )
r r r
= p x n
2(V (1) V ( 2 ) ) S (1 ) S (2)

=
1
(1)
( 2)
{
p (1) 2V (1) 1 p ( 2) 2V ( 2 ) 1 }
2(V V )

= (1)
1
( 2)
{
p (1)V (1) p ( 2 )V ( 2 ) 1 }
(V V )

Ejemplo 4.11
Dado el campo de velocidad:
v1 = ax1 bx 2
v 2 = bx1 ax 2
v3 = c x12 + x 22
donde a , b y c son constantes, se pide:
a) Comprobar si se cumple la ecuacin de continuidad de masa;
b) Es un movimiento isocrico?, es decir, es un medio incompresible?
Solucin:
Ecuacin de continuidad de masa:
D r
+ ( xr v ) = 0
Dt
donde:
r
xr v = vi ,i = v1,1 + v 2, 2 + v3,3
=aa+0=0
r
El movimiento es isocrico, ya que xr v = 0

Universidad de Castilla- La Mancha Draft Por: Eduardo W. V. Chaves (2012)


Ciudad Real Espaa
4 LEYES FUNDAMENTALES DE LA MECNICA DEL MEDIO CONTINUO 337

4.1.1 Problemas de Flujo

Ejemplo 4.12
Obtener la ecuacin de energa para un movimiento de slido rgido. Considrese que el
r
flujo de calor viene dado por q = K (T ) xr T , donde K (T ) es un tensor de segundo orden
denominado de tensor de conductividad trmica (propiedad del material), y considrese
u
tambin que c = , donde c es el calor especfico (propiedad del material). Proporcionar
T
tambin la unidad del tensor K .
Solucin: Para un movimiento de cuerpo rgido la potencia tensional es igual a cero, luego, la
ecuacin de energa se resume a:
u T r r
u& = = { : D xr q + r = xr q + r
T t =0
T r
c = xr q + r
t
T
c = xr [ K (T ) xr T ] + r
t
Resultando as que:
T
xr [K (T ) xr T ] + r = c
t
La ecuacin anterior se conoce como ecuacin de flujo de calor que se aplica a problema de
conduccin trmica.
DT T
NOTA: Cuando no hay transporte de masa se cumple T& = .
Dt t
T K
Teniendo en cuenta las siguientes unidades: [q] =
r J W
= 2 , xr T = r = , podemos
2
m s m x m
verificar que para que haya compatibilidad de unidades hay que cumplir que:
[qr ] = [K ] [ xr T ]
J W J W K
m 2 s = m 2 = s m K = m K m

J W
Luego, concluimos que [K ] = = .
s m K m K
NOTA: Tambin es interesante destacar que debido a que la potencia tensional es nula,
podemos desacoplar el problema mecnico del problema trmico, es decir, podemos
analizar los dos problemas separadamente.

Universidad de Castilla- La Mancha Draft Por: Eduardo W. V. Chaves (2012)


Ciudad Real - Espaa
338 PROBLEMAS RESUELTOS DE MECNICA DEL MEDIO CONTINUO

Ejemplo 4.13
Consideremos un problema de conduccin trmica, ver Ejemplo 4.12, donde tenemos una
pared de espesor h en la cual la temperatura en la cara exterior ( x1 = 0 ) es igual a 38 C y
la temperatura en la cara interior ( x1 = h ) es igual a 21 C , ver Figura 4.4. Obtener el flujo
de calor para el caso definido por: problema estacionario, el campo de temperatura segn
las direcciones x 2 y x3 es homogneo, no hay fuente interna de calor, y el material es
homogneo e istropo.
x2

T ( A) = 38 C Datos:
h = 0,04m
T ( B ) = 21 C W
K = 0,19
mK
(Exterior) (Interior)

x1

r
q
h

Figura 4.4
NOTA: Cuando decimos que un problema es estacionario, nos referimos que el campo de
la incgnita es estacionario. En el caso de la ecuacin de flujo de calor, la incgnita es la
temperatura.
Solucin:
Como hemos visto en el Ejemplo 4.12 la ecuacin de gobierno para este problema es la
T
ecuacin xr [K xr T ] + r = c . Si consideramos el problema estacionario, tenemos
t
T
que = 0 . Si no hay fuente interna de calor esto implica que r = 0 . Con estas
t
simplificaciones la ecuacin de flujo de calor se resume a xr [K xr T ] = 0 , adems si el
r
material es homogneo, el tensor con las propiedades trmicas K no vara con x
resultando que xr [K xr T ] = K : xr [ xr T ] = 0 o en notacin indicial
[K ij T , j ],
i
= K ij ,i T , j + K ij T , ji = K ij T , ji = 0 . Expandiendo sta ltima expresin, obtenemos
123
=0
que:
2T 2T 2T 2T 2T 2T
K 11 + K 12 + K 13 + K 21 + K 22 + K 23 +
x12 x 2 x1 x3 x1 x1x 2 x 22 x3 x 2
(4.20)
2T 2T 2T
+ K 31 + K 32 + K 33 2 = 0
x1x3 x 2 x3 x3

Si el campo de temperatura segn las direcciones x 2 y x3 es homogneo, eso implica que


las componentes del gradiente de temperatura segn estas direcciones son iguales a cero, i.e.

Universidad de Castilla- La Mancha Draft Por: Eduardo W. V. Chaves (2012)


Ciudad Real Espaa
4 LEYES FUNDAMENTALES DE LA MECNICA DEL MEDIO CONTINUO 339

T T
= = 0 . Para un material istropo, las componentes del tensor de conductividad
x 2 x3
trmica, ver Captulo 5 del libro texto, vienen dadas por:
K 0 0
K ij = 0 K 0
0 0 K

Con estas consideraciones la expresin (4.20) se reduce a:


2T 11 =K
2T
K 11 =0 K K =0 (4.21)
x12 x12

2T
Integrando una vez la expresin K = 0 obtenemos que:
x12

2T T dT
K =0 integrando
K + q1 = 0 q1 = K
x12 x1 dx1

Que es la ecuacin de flujo de calor de Fourier. Observar que para este caso q1 es una
constate, es decir, es independiente de x1 . Integrando una vez ms:
q1 q1
dT = K
dx1 T ( x1 ) =
K
x1 + C

Aplicando la condicin de contorno, x1 = 0 T = T ( A) , obteniendo as la constante de


q1
integracin C = T ( A) . Resultando que T ( x1 ) = x1 + T ( A) . Adems para x1 = h tenemos
K
que
q1 (T ( B ) T ( A) )
T ( x1 = h) = T ( B ) = h + T ( A) q1 = K
K h
Fijemos que en este caso (unidimensional) el gradiente de temperatura es la pendiente de la
recta definida por la temperatura, y la temperatura vara linealmente en la pared, ver Figura
4.4.
Reemplazando los datos del problema, ver Figura 4.4, obtenemos el flujo:
(T ( B ) T ( A) ) W (21 38)( K ) W J
q1 = K = 0,19 = 80,75 2 = 80,75 2
h mK 0,04(m) m m s
Observar que la transformacin de temperatura de grados Celsius para Kelvin viene dada
por K = C + 273,15 , luego la diferencia de temperatura sea en grados Celsius o en Kelvin
ser la misma. Observar tambin que el flujo de calor va en el sentido de mayor
temperatura hacia la regin de menor temperatura.

Universidad de Castilla- La Mancha Draft Por: Eduardo W. V. Chaves (2012)


Ciudad Real - Espaa
340 PROBLEMAS RESUELTOS DE MECNICA DEL MEDIO CONTINUO

NOTA: Supongamos que ahora tengamos dos paredes con propiedades distintas tal y
como se muestra en la Figura 4.5.

T ( A)

T (B )

T (C )
1 2

K (1) K ( 2) r
q
x1
h (1) h (2)

Figura 4.5
(T ( B ) T ( A) )
Observemos que la expresin q1 = K (1) sigue siendo vlida. Tambin es
h (1)
(T (C ) T ( B ) )
vlido para el material 2 : q1 = K ( 2) . Para obtener el flujo tenemos que
h (2)
aplicar la compatibilidad de temperatura en la cara B , es decir:
(T ( B _ 1) T ( A) ) q1 h (1)
q1 = K (1) T ( B _ 1) = T ( A)
h (1) K (1)
(T (C ) T ( B _ 2 ) ) q1 h ( 2)
q1 = K ( 2) T ( B _ 2) = T (C ) +
h ( 2) K ( 2)
T ( B _ 1) = T ( B _ 2 )
q1 h (1) q1 h ( 2 )
T ( A) = T (C ) +
K (1) K ( 2)
Resultando que:
(T (C ) T ( A) )
q1 =
h (1) h ( 2 )

K (1) + K ( 2 )

Ejemplo 4.14
Considrese un problema de conduccin trmica estacionario sin fuente interna de calor y
r r
donde el flujo de calor viene gobernado por la ley de Fourier ( q = K ( x ) xr T ), donde el
r
campo de tensor de conductividad trmica viene representado por K ( x ) , y es un tensor de
segundo orden arbitrario (no necesariamente simtrico).
a) Demostrar que el tensor de conductividad trmica es un tensor semi-definido positivo;
r
b) Verificar en que situacin la parte antisimtrica de K ( x ) no afecta en el resultado del
problema de conduccin de calor. Considerar la potencial tensional igual a cero.

Universidad de Castilla- La Mancha Draft Por: Eduardo W. V. Chaves (2012)


Ciudad Real Espaa
4 LEYES FUNDAMENTALES DE LA MECNICA DEL MEDIO CONTINUO 341

c) Que formato presenta K si el material es istropo.


Solucin:
a) Partimos de la desigualdad de conduccin de calor que hay que cumplir siempre:
r
q xr T 0 q i T,i 0
r
(K ( x ) xr T ) xr T 0 ( K ij T, j )T,i 0
r
xr T K ( x ) xr T 0 T,i K ij T, j 0

Recordar que un tensor arbitrario A es semi-definido positivo si se cumple que


r r r r r
x A x 0 para todo x 0 . Demostrando as que K ( x ) es un tensor semi-definido
r
positivo. Luego, como consecuencia los autovalores de K ( x ) sern todos valores reales
r
mayores o iguales a cero, i.e. K 1 0 , K 2 0 , K 3 0 . Tambin recordar que, ya que K ( x )
r
no es simtrico, el espacio principal de K ( x ) no constituye una base ortonormal.
Es interesante constatar que la parte antisimtrica no afecta en la desigualdad de
conduccin de calor, ya que:
r
xr T K ( x ) xr T 0
[ ]
xr T K sym + K anti xr T 0
xr T K sym
xr T + xr T K anti
xr T 0
xr T K sym xr T + K anti : ( xr T xr T ) 0

Note que K anti : ( xr T xr T ) = 0 ya que el doble producto escalar entre un tensor


antisimtrico ( K anti ) y otro simtrico ( xr T xr T ) resulta ser igual a cero, luego:
r
0 xr T K ( x ) xr T = xr T K sym xr T 0
r
Es decir, siempre se cumplir la desigualdad de conduccin de calor, sea K ( x ) simtrico o
no.
b) Para el problema planteado, la nica ecuacin de gobierno que queda es la ecuacin de
energa:
Du r r
u& = : D xr q + r = xr q
Dt
donde u es la energa interna especfica, : D es la potencial tensional, y r es la fuente
interna de calor por unidad de volumen. Luego:
u& = q i ,i
= (K ij T, j ) ,i
= K ij ,i T, j + K ij T, ji
= ( xr K T ) ( xr T ) + K : xr ( xr T )
[ ]
= ( xr K T ) ( xr T ) + K sym + K anti : xr ( xr T )
= ( xr K T ) ( xr T ) + K sym : xr ( xr T ) + K anti : xr ( xr T )
= ( xr K T ) ( xr T ) + K sym : xr ( xr T )

Universidad de Castilla- La Mancha Draft Por: Eduardo W. V. Chaves (2012)


Ciudad Real - Espaa
342 PROBLEMAS RESUELTOS DE MECNICA DEL MEDIO CONTINUO

donde hemos considerado la simetra de [ xr ( xr T )]ij = T,ij = T, ji . Si el material es


r
homogneo eso implica que el campo de K no depende de ( x ) con lo cual el trmino
K ij ,i = 0 j . En esta situacin quedamos con:

u& = K sym : xr ( xr T )
Luego, cuando el material es homogneo la parte antisimtrica de K no afecta en el
resultado.
c) Un material istropo tiene la caracterstica que sus propiedades en un punto no cambian
si hacemos un cambio de base. Un tensor de segundo orden istropo es un tensor esfrico,
ver captulo 1, luego el tensor K tiene que ser del tipo: K = K1 , donde K es una constante.
En forma de matriz:
1 0 0
K ij = K 0 1 0
0 0 1

Ejemplo 4.15
Supongamos un medio continuo donde en un punto material hay dos cantidades fsicas por
unidad de volumen c s y c f tal que c = c f + c s , ver Figura 4.6, y se cumple tambin que
r r r
v = v f + v s . Considerando que es un proceso isotrmico, un medio incompresible, que la
propiedad c s no afecta en la velocidad del material f y que el campo c f es homogneo, y
no hay fuente del material f . Demostrar que:

r c s Ecuacin Conveccin-
Q s xr (v f c s ) + xr (D xr c s ) = (4.22)
t Difusin
r
donde el flujo de la propiedad s viene dado por q ( D ) = D xr c s .

r
vf
Volumen de control c f

cs
r
v r
dV vs

Figura 4.6: Medio heterogneo.

Solucin:
Partiendo de la ecuacin de continuidad de esta cantidad fsica:

Q=

t
+ xr
r
(v ) Q=
(c f + c s )
t
[ r r
+ r (c f + c s )(v f + v s )
x
] (4.23)

con Q = Q s + Q f . Luego:

Universidad de Castilla- La Mancha Draft Por: Eduardo W. V. Chaves (2012)


Ciudad Real Espaa
4 LEYES FUNDAMENTALES DE LA MECNICA DEL MEDIO CONTINUO 343

Qs + Q f =
(c f + c s )
t
[ r
+ r (c f + c s )(v f + v s )
x
r
]
[ ]
f s
(c + c ) f r f r r r
Qs + Q f = + r c v + c f v s + csv f + csv s
t x
(4.24)
[ ]
f s
c c r r r r
Qs + Q f = + + xr c f v f + c f v s + c s v f + c s v s
t t
c r c s
[ ]
f
r r r
Qs + Q f = + xr (c f v f ) + + xr (c s v f ) + xr c f v s + c s v s
t t

c f r
Si suponemos que para el material ( f ) no hay fuente, luego + xr (c f v f ) = 0 , y
t
Q f = 0 , que es la ecuacin de continuidad de la cantidad c f . Con eso quedamos con:

Qs =
c s
t
r
[ r r
+ xr (c s v f ) + xr c f v s + c s v s ] (4.25)

c s r r r
Qs = + xr (c s v f ) + xr (c s v s ) + xr (c f v s ) (4.26)
t
c s r r r r
Qs = + xr (c s v f ) + xr (c s v s ) + xr c f v s + c f xr v s (4.27)
t
r
Si la cantidad fsica c f no cambia con x , campo homogneo, luego el gradiente
r r
xr c f = 0 . Si adems, para el medio ( s ) lo consideramos incompresible xr v s = 0 . Estas
simplificaciones nos indican que el material ( s ) no afecta en las propiedades y ni en el
campo de velocidad del material ( f ). Lgicamente si la cantidad del material ( s ) es
significativa esta aproximacin ya no ser vlida. Con estas aproximaciones quedamos con:
c s r r c s r r
Qs = + xr (c s v f ) + xr (c s v s ) = + xr (c s v f ) + xr q ( D ) (4.28)
t t
r r
Observemos que el trmino (c s v s ) q ( D ) representa el flujo debido a la concentracin del
r r
material ( s ), trmino difusivo. El trmino (c s v f ) q (C ) est relacionado con transporte de
r
masa, trmino convectivo. Teniendo en cuenta que q ( D ) = D xr c s la expresin (4.28)
queda:
c s r r
Qs = + xr (c s v f ) + xr q ( D )
t
c s r
Qs = + xr (c s v f ) + xr (D xr c s ) (4.29)
t
r c s
Q s xr (c s v f ) + xr (D xr c s ) =
t
Demostrando as (4.22).

Universidad de Castilla- La Mancha Draft Por: Eduardo W. V. Chaves (2012)


Ciudad Real - Espaa
344 PROBLEMAS RESUELTOS DE MECNICA DEL MEDIO CONTINUO

Ejemplo 4.16
Considrese un tanque lleno de agua
con concentracin de sedimentos, ver
Figura 4.7. La concentracin de
sedimentos (densidad de
concentracin) viene dada por
( kx t ) h
c( x3 , t ) = C t exp 3 , por unidad de

volumen, donde C y k son constantes


positivas. Se pide: a) Obtener la masa
total de sedimentos en el tanque; b)
Obtener el flujo de sedimentos
sabiendo que ste es solo funcin de
r r
x3 y del tiempo t , i.e. q = q( x3 , t ) . x3 x2
b

x1

Figura 4.7: Tanque con sedimentos.


Solucin:
Para obtener la masa total tenemos que resolver la integral:
h b a h
( kx3t ) ( kx3t )
C t exp
s
M = c dV = dx1 dx 2 dx3 = ab C t exp dx3
V 0 0 0 0
h
C
= ab
k
exp
( kx3t )


C
= ab
k
C abC
exp ( kht ) + =
k k
exp ( kht ) 1 [ ]
0

Para obtener el flujo, podemos aplicar la ecuacin de continuidad de la concentracin:


c s r r c s
Q= + xr q xr q = qi ,i = (4.30)
t t
donde hemos tenido en cuenta que no hay fuente de sedimentos Q = 0 . Para este problema
el flujo no es dependiente de x 2 y x1 . Con estas condiciones q1,1 = q 2, 2 = 0 . Luego:

q1 q 2 q 3 c s
q i ,i = q1,1 + q 2, 2 + q 3,3 = + + =
x1 x 2 x3 t
(4.31)
q c s
3 =
x3 t

donde
c s
t
=
t
[
C t exp
( kx3t )
]
= C exp
( kx3t )
C t k x 3 exp
( kx3t )
y reemplazando en la
expresin (4.31) obtenemos que:

Universidad de Castilla- La Mancha Draft Por: Eduardo W. V. Chaves (2012)


Ciudad Real Espaa
4 LEYES FUNDAMENTALES DE LA MECNICA DEL MEDIO CONTINUO 345

dq 3 c s ( kx3t ) ( kx3t )
= = C exp + C t k x3 exp
dx3 t

[ C exp ]dx
( kx3t ) ( kx3t )

dq 3 = + C t k x3 exp 3
(4.32)
C ( kx3t ) C ( kx3t ) C k x3 t ( kx3t )
q 3 = exp exp exp + K3
kt kt kt
( kx3t )
q 3 = C x3 exp + K3
{
=0

r ( kx3t )
El vector flujo viene dado por q = C x3 exp e 3 .

Universidad de Castilla- La Mancha Draft Por: Eduardo W. V. Chaves (2012)


Ciudad Real - Espaa
346 PROBLEMAS RESUELTOS DE MECNICA DEL MEDIO CONTINUO

4.1.2 Movimiento de Slido Rgido

Ejemplo 4.17
Obtener el momento lineal y el momento angular para un slido sometido a un
movimiento de cuerpo rgido.
x3
Slido rgido
Bt
C.M .
x2
r
r v
x3 x
x1

O
x2
x1

Figura 4.8
Solucin:
Para un movimiento de cuerpo rgido todas las partculas estarn sometidas a una misma
r
velocidad v . En el captulo 2 obtuvimos que la velocidad para un medio continuo
sometido a un movimiento de cuerpo rgido viene dada por:
r r r r r
v = c& + ( x c )
r
donde es el vector axil (velocidad angular) asociado al tensor antisimtrico W (tensor
spin).
Momento lineal:
r r
L = v dV
V
r r
( r r
= c& + ( x c ) dV
)
V
r r r r r
= c& dV + x dV c dV

V V V
En notacin indicial:
r& r
( r r
c + ( x c) ) i
dV = c i dV + ijk j x k dV ijk j c k dV
V V V V


= c i dV + ijk j x k dV ijk j c k dV
V V

V

= mc i + ijk j m x k ijk j c k m
[
= m c i + ijk j ( x k c k ) ]
r r r r r r
donde notamos que:
V
x dV = x dV = (m x ) , siendo m la masa total y
V
r
x k el vector posicin del centro de masa (C.M.). As obtenemos que:

Universidad de Castilla- La Mancha Draft Por: Eduardo W. V. Chaves (2012)


Ciudad Real Espaa
4 LEYES FUNDAMENTALES DE LA MECNICA DEL MEDIO CONTINUO 347

r r r r r
L = m c& + ( x c )
r
[
(Momento lineal de slido rgido)
] (4.33)
= mv
r r r r r
donde v = c& + ( x c ) es la velocidad del centro de masa.

Momento angular:
r r r

H O = ( x v ) dV
V

[x (c + ( x c))]dV
r r& r r r
=
V
r r r r r r r r (4.34)
= x c& dV + x ( x ) dV x ( c ) dV

V V V

r r r r r r r r
= x dV c& + x ( x ) dV x dV ( c )

V V V
r r r
Del captulo 1 obtuvimos que: dados tres vectores a , b , c la siguiente relacin se cumple
r r r r r r r r r r r r r
a (b c ) = (a c )b (a b)c . Luego, a (b a) = [(a a) 1 (a a) ] b , si a = c . Con
r r r r r r
r r r r r r r r r
es x ( x ) dV = [( x x ) ( x ) x ] dV y en notacin indicial:

V V

[x
V
k ] [
x k i x p p x i dV = x k x k p pi x p p x i dV
V
]
= [x k ]
x k pi x p x i p dV
V

[
= x k x k pi x p x i dV p
V
]
= I O ip p
En notacin tensorial queda:
r r r r r r r r
x ( x ) dV = [( x x ) 1 ( x x )] dV
V V
r
= IO
r r r r
donde I O = [( x x ) 1 ( x x )] dV es el tensor de inercia relativo al punto O . Como
V
podemos observar I O es un pseudo-tensor de segundo orden y simtrico. Es un pseudo-
tensor porque depende del sistema de referencia adoptado. Aunque sea un pseudo-tensor,
por comodidad nos vamos a referir simplemente por tensor de inercia. Las, cuyas
componentes son I O ij = [x k x k ij xi x j ] dV , y explcitamente:
V

I O 11 = [( x1 x1 + x 2 x 2 + x 3 x 3 ) 11 x1 x1 ] dV = x 22 + x 32 dV

V
[
V
]
[
I O 22 = x12 + x 32 dV
V
] ; [
I O 33 = x12 + x 22 dV
V
]
I O 12 = [( x1 x1 + x 2 x 2 + x 3 x 3 ) 12 x1 x 2 ] dV = [x1 x 2 ] dV = I O 12

V V

Universidad de Castilla- La Mancha Draft Por: Eduardo W. V. Chaves (2012)


Ciudad Real - Espaa
348 PROBLEMAS RESUELTOS DE MECNICA DEL MEDIO CONTINUO

I O 13 = [x1 x 3 ] dV = I O 13
; I O 23 = [x 2 x 3 ] dV = I O 23

V V

donde I O11 , I O 22 , I O 33 , son conocidos como momentos de inercia, y I O12 , I O13 , I O 23 ,


son conocidos como productos de inercia en la mecnica clsica. Luego:

[
x2 + x3 dV] [x1 x 2 ] dV
[x1 x3 ] dV

2 2

V V V I O 11 I O 12 I O13
[ + ] dV

I Oij = [x1 x 2 ] dV
x12 x32 [x 2 x3 ] dV = I O12
I O 22 I O 23
V V V I O13 I O 23 I O 33
[x1 x3 ] dV
[x x ] dV [2 3 ]
x12 + x22 dV
V V V

(4.35)
Retomando la ecuacin (4.34) podemos decir que:
r r r r r r r r r
H O = x dV c& + x ( x ) dV x dV ( c )

V V V
r r& r r r r
= m x c + I O m x ( c )
r r r r
(
= m x c& c + I O
r r r
r
)
Sumando y restando el trmino m x x en la relacin anterior obtenemos:
r r r r r
H O = m x c& c + I O ( r
)
r r r r r
[ r r r
= m x c& + ( x c ) m x x + I O
r
]
r r r r r r r
[
= m x v m ( x x) 1 ( x x) + IO
r
]
r r r r r r
{[
= m x v + m ( x x) ( x x) 1 + IO
r r r r r
r
] }
r
= m x v + I = m x v + HG
Definiendo as el Momento angular con respecto al punto O :
r r r r Momento angular con respecto al sistema
HO = m x v + HG (4.36)
que pasa por O
r r r r
NOTA: Verifiquemos que cuando c = 0 tenemos que H O = I O .
Definimos tambin el tensor de inercia con respecto al centro de masa:
r r
[ r r
I = IO + m ( x x) ( x x) 1 ]
Tensor de inercia con respecto al centro de
masa
(4.37)

Las componentes de Iij = I O ij + m[x i x j ( x12 + x 22 + x 32 ) ij ] vienen dadas explcitamente


por:
I11 = I O 11 m( x 22 + x 32 ) ; I12 = I O 12 + m( x1 x 2 )
Teorema de los ejes
I 22 = I O 22 m( x12 + x 32 ) ; I 23 = I O 23 + m( x 2 x 3 ) (4.38)
paralelos
I33 = I O 33 m( x12 + x 22 ) ; I13 = I O 13 + m( x1 x3 )
Observemos que las expresiones anteriores constituyen el teorema de los ejes paralelos
(Teorema de Steiner) de la Mecnica Clsica.

Ejemplo 4.18
Considrese un paraleleppedo de dimensiones a b c , ver Figura 4.9, cuyo campo de
r
densidad masa, ( x ) , es homogneo. Obtener el tensor de inercial con respecto al sistema
que pasa por el centro de gravedad.

Universidad de Castilla- La Mancha Draft Por: Eduardo W. V. Chaves (2012)


Ciudad Real Espaa
4 LEYES FUNDAMENTALES DE LA MECNICA DEL MEDIO CONTINUO 349

x3

x2
O
c
x1

b
Figura 4.9: Paraleleppedo.
Solucin:
Utilizaremos la ecuacin (4.35):

[
x2 + x3 dV] [x1 x 2 ] dV
[x1 x3 ] dV

2 2

V V V I O 11 I O 12 I O13
[ + ] dV

I Oij = [x1 x 2 ] dV
x12 x32 [x 2 x3 ] dV = I O12
I O 22 I O 23
V V V I O13 I O 23 I O 33
[x1 x3 ] dV
[x x ] dV [
2 3 ]
x12 + x22 dV
V V V
r
Notar que la densidad de masa para un material homogneo no vara con x , y adems se
cumple que:
m = dV =
V

V
dV = V = abc

El momento de inercia I O11 queda:


c b a

[ ] [x ]
2 2 2
abc 2 m
I O11 = x22 + x32 dV = 2
2 + x32 dx1dx 2 dx3 = (b + c 2 ) = (b 2 + c 2 )
V c b a
12 12
2 2 2

m 2 m 2
Anlogamente podemos obtener que I O 22 = (a + c 2 ) ; I O 33 = (a + b 2 ) .
12 12
Dejamos al lector demostrar que I O12 = I O 13 = I O 23 = 0 . Recordar que el tensor de inercia
nos dar la informacin de cmo la masa est distribuida segn los ejes adoptados, y fijar
que la masa est distribuida de forma igual con respecto al plano x1 x2 , luego
[x x ] dV = 0 . Resultando que, los ejes adoptados son ejes principales de inercia:
V
1 2

m 2 2
12 (b + c ) 0 0
m 2
I Oij = 0 (a + c 2 ) 0
12
m 2 2
0 0 ( a + b )
12

Universidad de Castilla- La Mancha Draft Por: Eduardo W. V. Chaves (2012)


Ciudad Real - Espaa
350 PROBLEMAS RESUELTOS DE MECNICA DEL MEDIO CONTINUO

Ejemplo 4.19
Obtener el principio de la conservacin del momento lineal y del momento angular para un
slido sometido a un movimiento de cuerpo rgido.
Solucin: Para un movimiento de cuerpo rgido, ver Figura 4.8, todas las partculas estarn
r
sometidas a una misma velocidad v . Partiendo de la definicin de principio de la
conservacin del momento lineal:
r r D r r

t * dS + b dV = v dV = L&
S V
Dt V
Recurrimos ahora la expresin del momento lineal para un movimiento de slido rgido
r r
obtenido en el ejemplo anterior como L = m v , luego:
r r D r r r r
t * dS + b dV = v dV = L& = m v& = m a

S
Dt V
14442V 444 3
r
F
Resultando:
r r
F =ma
Consideremos ahora el principio de la conservacin del momento angular:
r r* r r D r r D r r&
(x t
S

)dS + ( x b)dV =
Dt V
( x v )dV =
Dt
HO HO
1 444442V44444 3
r
MO
Resultando:
r r& r r&
M O = H O , tambin es vlido que M G = HG
r
donde la expresin del momento angular H O para un movimiento de slido rgido fue
r r
obtenida en el ejemplo anterior. Los conjuntos de ecuaciones anteriores, F=ma y
r r&
M G = H G , nos informan que los siguientes sistemas son equivalentes:
r&
r r HG
F( n ) F( 2 )

G G
= r
ma

r
F(1) G - centro de masa

NOTA: Cuando estamos tratando de un movimiento de slido rgido, las ecuaciones de


gobierno del problema se resumen a:
r r r r& Ecuaciones de gobierno para
F =ma y M O = HO
movimiento de slido rgido
(4.39)

Universidad de Castilla- La Mancha Draft Por: Eduardo W. V. Chaves (2012)


Ciudad Real Espaa
4 LEYES FUNDAMENTALES DE LA MECNICA DEL MEDIO CONTINUO 351

Ejemplo 4.20
Consideremos una viga con las siguientes condiciones de carga y de apoyo:
P P


= HA A B
A B

L L VA VB
2 2

Figura 4.10: Viga isosttica.


Obtener las reacciones de apoyo VA , VB , H A .
Solucin:
Aunque en la viga haya deformacin (rgimen de pequeas deformaciones) y tensin, para
efecto de clculo de las reacciones de apoyo de una viga isosttica podemos considerar que
de un cuerpo rgido se tratara, y las ecuaciones necesarias, ver ecuaciones (4.39), son:
F =0
F
x
r r r = H A + P cos = 0 H A = P cos
F
x
F=ma =0 =0
F
y
= V A + V B P sin = 0 V A = V B + P sin
F
y
z =0

r


M x =0
r r& L P sin
MA = HA =0 M y =0 M


z = V B L P sin
2
= 0 VB =
2

M z =0
P sin
pudiendo ahora determinar V A = V B + P sin = . Notar que tenamos 3 ecuaciones
2
y tres incgnitas. Si tenemos un sistema donde hay ms incgnitas que ecuaciones, sistema
hiperesttico, este procedimiento ya no es vlido ya que las reacciones dependern de la
deformada de la viga y sta a su vez depender de la rigidez de la viga.

Ejemplo 4.21
Obtener la energa cintica para un slido sometido a un movimiento de cuerpo rgido, ver
Figura 4.8.

Solucin:
Para un movimiento de cuerpo rgido todas las partculas estarn sometidas a una misma
r r r r r
velocidad v = c& + ( x c ) . Luego la energa cintica viene dada por:
1
K (t ) =
2V
r r 1 r r r r r r
{[ r r
(v v )dV = c& + ( x c) c& + ( x c ) dV
2V ][ ]}
r r r r
Haciendo la siguiente suma de vectores x = x + x , donde x es el vector posicin del
r
centro de masa, y x es el vector posicin de las partculas del slido con respecto al
sistema que pasa por el centro de masa.

Universidad de Castilla- La Mancha Draft Por: Eduardo W. V. Chaves (2012)


Ciudad Real - Espaa
352 PROBLEMAS RESUELTOS DE MECNICA DEL MEDIO CONTINUO

K (t ) =
1
2V
r r
{[ r r r r r r r r
][
c& + (( x + x ) c) c& + (( x + x ) c ) dV
]}
=
1
2V
r r
{[( r r r r r r
) r r r r
] [(
c& + ( x c ) + ( x ) c& + ( x c ) + ( x ) dV
) ]}
Resultando as que:
K (t ) =
1
2V
r r
( r r r r
)(
r r
c& + ( x c ) c& + ( x c ) dV + )
+
1
2V
r r
( r r r r
2 c& + ( x c ) ( x ) dV + )
1 r r r r
( x ) ( x ) dV
+
2V
r r& r r r
Observemos que v = c + ( x c) es la velocidad del centro de masa, resultando as que:
1 r r 1 r r r 1 r r r r
K (t ) = v v dV + 2 v ( x ) dV + ( x ) ( x ) dV

2V 2V 2 V
r r r r
A continuacin analizaremos separadamente el trmino ( x ) ( x ) dV .
V
r r r r
( [( x ) ( x )])
V
i
dV = ijk j x k ipq p x q dV
V


= ( jp kq jq kp ) j x k p x q dV
V


= ( jp kq j x k p x q jq kp j x k p x q ) dV
V


= ( j x k j x k j x k k x j ) dV
V


= k ( x s x s kp x k x j ) j dV
V


= k ( x s x s kj x k x j ) dV j


V
= k I kj j
En notacin tensorial:
r r r r r r r r r r
[( x ) ( x )] dV = [( x x ) 1 ( x x )] dV
V V
r r
= I
donde I es el tensor de inercia segn el sistema que pasa por el centro de masa.
r r r
Analizamos ahora el trmino 2 v ( x ) dV :
V

r r r r r r r r r
2 v ( x ) dV = 2v x dV = 2v m {xr = 0
V V =0
r
Observemos que el sistema x pasa por el centro de masa luego, el vector posicin del
r
centro de masa para el sistema x es el vector nulo.
Resultando as que la expresin de la energa cintica para un movimiento de slido rgido:

Universidad de Castilla- La Mancha Draft Por: Eduardo W. V. Chaves (2012)


Ciudad Real Espaa
4 LEYES FUNDAMENTALES DE LA MECNICA DEL MEDIO CONTINUO 353

1 r r 1 r r r 1 r r r r
K (t ) = v v dV + 2 v ( x ) dV + ( x ) ( x ) dV

2V 2 2V
1V44424443
=0

1 1r r
= mv 2 + I
2 2
Energa cintica para un
1 1r r
K(t ) = mv 2 + I movimiento de slido (4.40)
2 2 rgido
Representando las componentes del tensor de inercia como:

[
x 2 + x3 dV ] [x1 x 2 ] dV
[x1 x3 ] dV

2 2

V V V I11 I12 I13


[x + x ] dV

Iij = [x1 x 2 ] dV
1
2
3
2
[x 2 x 3 ] dV = I12
I 22 I 23
V V V I13 I 23 I 33
[x1 x3 ] dV
[x x ] dV [ ]
x1 + x 2 dV
2 2
2 3
V V V
La forma explcita de la energa cintica queda:
1 1
K(t ) = mv 2 + k Ikj j
2 2
I11 I12 I13 1
1 1
= mv 2 + [1 2 3 ] I12 I 22 I 23 2
2 2 I
13 I 23 I 33 3

=
1
2
1
[
mv 2 + I1112 + I 22 22 + I 33 32 2 I12 1 2 2 I13 1 3 2 I 23 2 3
2
]
1
K(t ) = mv 2 +
2
1
2
[
I1112 + I 22 22 + I 33 32 2 I12 1 2 2 I13 13 2 I 23 2 3 ] (4.41)

Ejemplo 4.22
Considrese el pseudo-tensor de inercia, I O , con respecto al sistema ortonormal x1 x 2 x3 ,
ver Figura 4.11. a) Hacer la interpretacin fsica del tensor de inercia. b) Dado otro sistema
ortonormal, representado por x1* x 2* x3* , obtener las componentes del tensor de inercia en
este nuevo sistema. c) Demostrar que el tensor de inercia es un tensor definido positivo.
DI O &
Para un slido en movimiento, en que situaciones la tasa del tensor de inercia, IO ,
Dt
es cero?
x3
x2*
x3*
x1*

O x2

x1
Figura 4.11

Universidad de Castilla- La Mancha Draft Por: Eduardo W. V. Chaves (2012)


Ciudad Real - Espaa
354 PROBLEMAS RESUELTOS DE MECNICA DEL MEDIO CONTINUO

Solucin:
El pseudo-tensor de inercia depende del sistema de coordenadas adoptado. Y por
definicin viene dado por:

[ ]
r r r r
I O = [( x x ) 1 ( x x )] dV
; I O ij = x k x k ij xi x j dV
V V

En componentes

[ ]
x 2 + x3 dV [x1 x 2 ] dV
[x1 x 3 ] dV

2 2

V V V
I ij = [x1 x 2 ] dV
[x + x ]dV [x 2 x3 ] dV

2 2
1 3
V V V
[x1 x3 ] dV
[x x ] dV [
2 3 ]
x12 + x 22 dV
V V V

a) El tensor de inercia nos dar informacin de como la masa del slido est distribuida
segn el sistema adoptado.
El trmino [x1 x 2 ] dV nos indica como la masa del slido est distribuida segn el plano
V
x1 x 2 . Luego, Si el material es homogneo, i.e. campo de densidad de masa constante, y
x1 x 2 es un plano de simetra, es decir si la masa est distribuida por igual con respecto al
plano x1 x 2 , el trmino [x1 x 2 ] dV es igual a cero. Con eso, concluimos tambin que si
V
los planos x1 x 2 , x1 x 3 , x 2 x3 , son planos de simetra, la matriz de inercia ser una
matriz diagonal.
Como ejemplo de ilustracin, vamos considerar un estudiante atado a un disco y con los
brazos estirados donde en cada mano sujeta un peso (ver Figura 4.12 sistema inicial). El
r r
disco est girando con un velocidad angular (i ) y el tensor de inercia segn el sistema x
viene dado por I O(i ) . Si consideramos un sistema sin disipacin de energa, qu pasar al
sistema cuando el estudiante mueva sus brazos haca dentro?, ver Figura 4.12 sistema
final. Como estamos tratando con un sistema conservativo, el momento angular se
conserva, i.e.:
r r
H O( i ) = H O( f )
r r
I (Oi ) ( i ) = I O( f ) ( f )

Ya que para el sistema final la masa est ms concentrada segn el eje de rotacin, la
r r
desigualdad I O( f ) < I (Oi ) se cumple, y como consecuencia ( f ) > (i ) .

Universidad de Castilla- La Mancha Draft Por: Eduardo W. V. Chaves (2012)


Ciudad Real Espaa
4 LEYES FUNDAMENTALES DE LA MECNICA DEL MEDIO CONTINUO 355

x3 x3
Initial system Final system

r r
(i ) ( f )

I O(i ) I O( f )

Figura 4.12:

b) Vamos suponer que los sistemas dados estn relacionados por la ley de transformacin
xi* = Aij x j , donde Aij es la matriz ortogonal de transformacin de base, luego, se cumple
que xi = A ji x *j . Pudiendo as expresar I O ij de la siguiente manera:

[
I O ij = x k x k ij x i x j dV
V
]
[ ]
= ( x k* x k* )Aip pq A jq Aip x *p A jq x q* dV
V

{[ ]}
= Aip ( x k* x k* ) pq x *p x q* A jq dV
V


[ ]
= Aip ( x k* x k* ) pq x *p x q* dV A jq
V
= Aip I *O ij A jq
r r
Recordar que el trmino ( x x = x k x k = x k* x k* ) es un invariante, i.e. no depende del sistema
adoptado, y tambin que la matriz de transformacin, igual que un tensor ortogonal, slo
es dependiente, si es el caso, del tiempo.

Abusando un poco de la notacin, utilizamos tambin notacin tensorial, pero hay que
tener en cuenta que estamos trabajando con las componentes del tensor, y no haciendo una
transformacin ortogonal.

Universidad de Castilla- La Mancha Draft Por: Eduardo W. V. Chaves (2012)


Ciudad Real - Espaa
356 PROBLEMAS RESUELTOS DE MECNICA DEL MEDIO CONTINUO

r r r r
I O = [( x x ) 1 ( x x )] dV

V

[ ]
r r r r
= ( x * x * )A T 1 A (A T x * A T x * ) dV
V

[ ]
r r r r
= ( x * x * )A T 1 A (A T x * x * A ) dV
V

{ [( x * x * )1 ( x * x * )]} A dV
r r r r

= AT
V


[ ]
r r r r
= A T ( x * x * )1 ( x * x * ) dV A
V
= A T I *O A

I O = A T I *O A Componentes del tensor de inercia tras un


(4.42)
I O ij = Aip I *O ij A jq cambio de base (rotacin)

Luego, es vlido que I *O = A I O A T , que son las nuevas componentes del tensor de
inercia en el sistema x1* x 2* x3* . Verifiquemos que es la misma ley de transformacin de las
componentes de un tensor de segundo orden, donde A es la matriz de transformacin del
sistema x1 x 2 x3 al sistema x1* x 2* x3* .
c) Un tensor definido positivo, por definicin, sus autovalores son mayores que cero.
Partiremos de la expresin de la energa cintica obtenida en el Ejemplo 4.21:
1
K(t ) = mv 2 +
2
1
2
[
I1112 + I 22 22 + I 33 32 2 I12 1 2 2 I13 13 2 I 23 2 3 ]
La energa cintica es un escalar y siempre positivo, slo en dos situaciones la energa ser
igual a cero, cuando no haya masa o cuando el cuerpo est en reposo. Vamos adoptar un
sistema tal que el origen est situado en el centro de masa y los ejes adoptados son ejes de
simetra (ejes principales de inercia) y que el cuerpo est girando alrededor del origen,
centro de masa. En esta situacin la expresin de la energa se resume a:
I1 0 0 1
1
K(t ) = [1 2 3 ] 0 I 2 0 2

2
0 0 I 3 3
1 44424443
Autovalores del
tensor de inercia

=
1
2
[
I112 + I 2 22 + I 3 32 > 0]
1
Si adems tenemos un movimiento tal que 2 = 3 = 0 , nos quedamos con K(t ) = I112 ,
2
luego, la nica forma que la energa cintica sea siempre positiva es que I1 > 0 .
Anlogamente, podemos concluir que I 2 > 0 , I 3 > 0 . Con eso concluimos que el tensor
de inercia es un tensor definido positivo.
d) Como el tensor de inercia depende del sistema adoptado, en las siguientes situaciones el
tensor de inercia para un slido en movimiento no cambia con el tiempo:
1) Si el sistema adoptado est unido al slido.

Universidad de Castilla- La Mancha Draft Por: Eduardo W. V. Chaves (2012)


Ciudad Real Espaa
4 LEYES FUNDAMENTALES DE LA MECNICA DEL MEDIO CONTINUO 357

2) Si el slido est girando alrededor de un eje de simetra, por ejemplo si un cilindro est
girando alrededor del eje prismtico, luego, durante el movimiento la distribucin de masa,
con respecto a los ejes adoptados, no cambia con el tiempo:

r
sistema de referencia fijo en el
espacio

Ejemplo 4.23
Considrese un cilindro homogneo de radio r y altura h = 3r con masa total igual a m .
Encontrar el tensor de inercia en el sistema Ox1 x 2 x3 . El sistema Ox1 x 2 x3 viene dado por
una rotacin del sistema Ox1x 2 x3 de 45 a lo largo del eje x1 . Los sistemas Gx1 x 2 x3 y
Ox1x 2 x3 tienen las mismas orientaciones.

x3

x3
x3 G x2
x1 h = 3r

r x2
rG

45
O x2

x1, x1

Datos: Para el sistema de referencia Gx1 x 2 x3 se conoce el tensor de inercia y viene dado
por:
1 2 2
12 m(3r + h ) 0 0
1 mr 2 2 0 0
I G ij = 0 2 2
m(3r + h ) 0 = 0 2 0
12 2
1 2 2

0 0 1
0 0 m ( 3r + h )
12
Solucin:

Universidad de Castilla- La Mancha Draft Por: Eduardo W. V. Chaves (2012)


Ciudad Real - Espaa
358 PROBLEMAS RESUELTOS DE MECNICA DEL MEDIO CONTINUO

Primero obtenemos el tensor de inercia en el sistema Ox1x 2 x3 a travs del teorema de


Steiner, ver ecuacin (4.38) del Ejemplo 4.17. Despus aplicamos una rotacin al tensor
segn ecuacin (4.42) del Ejemplo 4.22.
A travs de las ecuaciones (4.38) podemos decir que:
I11 + m( x 22 + x 32 ) = I O 11 ; I12 m( x1 x 2 ) = I O 12
I 22 + m( x12 + x 32 ) = I O 22 ; I 23 m( x 2 x3 ) = I O 23 (4.43)
I33 + m( x12 + x 22 ) = I O 33 ; I13 m( x1 x 3 ) = I O 13

donde ( x1 , x 2 , x 3 ) son las coordenadas del centro de masa con respecto al sistema Ox1x 2 x3 .
r 3
Luego, definimos el vector rG = x1e 1 + x 2 e 2 + x 3 e 3 = 0e 1 + re 2 + re 3 . Con lo cual
2
obtenemos que:
3 17
I O 11 = I11 + m( x 22 + x 32 ) = mr 2 + m r 2 + ( r ) 2 = mr 2
2 4
3 13
I O 22 = I 22 + m( x12 + x 32 ) = mr 2 + m 0 2 + ( r ) 2 = mr 2
2 4
1
[
I O 33 = I33 + m( x12 + x 22 ) = mr 2 + m 0 2 + r 2 = mr 2
2
] 3
2
I O 12 = I12 m( x1 x 2 ) = 0
I O 23 = I 23 m( x 2 x3 ) = 0
3 3
I O 13 = I13 m( x1 x3 ) = m(r )( r ) = mr 2
2 2
Resultando as en las siguientes componentes del tensor de inercia en el sistema Ox1x 2 x3 :
34 0 0
mr 2
I O ij = 0 13 6
4
0 6 6

Teniendo en cuenta la matriz de transformacin entre los sistemas Ox1x 2 x3 y Ox1 x 2 x3 :


1 0 0
A = 0 cos 45 sin 45

0 sin 45 cos 45

y aplicando la relacin (4.42) obtenemos que:


34 0 0
mr 2
I O ij = A I O A = Aip I O ij A jq
T
= 0 7 7
8
0 7 31

Ejemplo 4.24
r r r r r r r
Teniendo en cuenta el momento angular H O = m x v + I = m x v + H G , encontrar la
tasa del momento angular de tal forma que no tenga la necesidad de calcular en cada
instante de tiempo el tensor de inercia.

Universidad de Castilla- La Mancha Draft Por: Eduardo W. V. Chaves (2012)


Ciudad Real Espaa
4 LEYES FUNDAMENTALES DE LA MECNICA DEL MEDIO CONTINUO 359

r
r r
- velocidad angular del slido HG r
r r
- velocidad angular del sistema x *
x3
x2*
r x3*
HO x1*
x3
G x 2
r
x

O x2 x1
G - centro de masas

x1

Figura 4.13
Solucin:
Recurriendo a la derivada material podemos decir que:
D r
Dt
r&
HO HO =
D
Dt
[
r r r
m x v + HG ]
=
D
Dt
[ r r
m x v +
D r
Dt
]
HG [ ]
r r
Dx r r Dv r&
=m v +m x + HG
Dt Dt
r r r r r&
= m v12
3 v + m x a + HG
r
=0

Luego, obtenemos que:


D r r& r r r&
HO HO = m x a + HG (4.44)
Dt
r r
donde a es la aceleracin del centro de masa. A continuacin analizamos el trmino H& G .
Adoptamos un sistema mvil x1 x 2 x3 pero con orientacin fija y siempre paralelo al sistema
r
fijo en el espacio x1 x 2 x3 , ver Figura 4.13. Expresando las componentes de I y en el
sistema x1 x 2 x3 , obtenemos que:
r r D r r& & r r&
H G = I tasa

H G H G = I + I
Dt
r
Fijemos que como el slido gira con respecto al sistema x , con lo cual la distribucin de
r
masa, con respecto al sistema x , cambia, y a su vez el tensor de inercia tambin cambia.
Luego, a cada instante de tiempo tenemos que calcular el tensor de inercia. Este
procedimiento es muy costoso. Para solventar este problema adoptamos un nuevo sistema
r
x * , que tambin tiene origen en el centro de masa, ver Figura 4.13. A travs de ley de
transformacin de las componentes de los tensores, las siguientes relaciones son vlidas:

Universidad de Castilla- La Mancha Draft Por: Eduardo W. V. Chaves (2012)


Ciudad Real - Espaa
360 PROBLEMAS RESUELTOS DE MECNICA DEL MEDIO CONTINUO

r r r r
H G* = A H G ; H G = A T H G*
r r r r
Componente s * = A ; = A T *
*
I O = A I O A I O = A T I O* A
T
;
r r
donde A es la matriz de transformacin del sistema x al sistema x * .
r r
La tasa de H G = A T H G* resulta:
D r
Dt
r&
H G H G =
D
Dt
[ r
] r r&
A T H G* = A& T H G* + A T H G* (4.45)

Haciendo una analoga con la tasa de un tensor ortogonal, ver captulo 2 del libro texto,
podemos decir que = A& A T A& T = A T T , donde T es un tensor antisimtrico
r r
y representa el tensor tasa de rotacin del sistema x * con respecto al sistema x . Pudiendo
as expresar (4.34) como:
r& r r&
H G = A T T H G* + A T H G*
r r& (componentes) (4.46)
= A T T H G* + H G*

r r r r
Recurriendo a la propiedad del tensor antisimtrico tal que T H G* = H G* , donde
r r
es el vector axil asociado al tensor antisimtrico T , es decir, = (t ) es la velocidad
r
angular del sistema mvil x * (ver NOTA 3). Resultando que (4.46) an puede ser escrito
como:
r& r r&
H G = A T T H G* + H G*

r r (componentes) (4.47)
T r * &*
= A HG + HG *

donde
r*
r& *
HG =
Dt
[
D * r * DI * r *
I =
Dt
] * D
+ I
Dt
DI *
Para que el trmino sea igual a cero, podemos tener dos posibilidades:
Dt
DI * r r r
1) = 0 si el sistema x * est unido al slido. En este caso se cumple que = , es
Dt
decir, la velocidad del sistema mvil es igual a la velocidad angular del slido.
DI *
2) = 0 si el slido gira alrededor de un eje prismtico, ver Ejemplo 4.22.
Dt

NOTA 1: La ecuacin (4.47) puede ser reescrita como:


r& r r r&
H G = A T * H G* + H G*

r& r* r& * (componentes) (4.48)
T r* r * r * r& *
A H G = A A H G + H G = H G + H G

Universidad de Castilla- La Mancha Draft Por: Eduardo W. V. Chaves (2012)


Ciudad Real Espaa
4 LEYES FUNDAMENTALES DE LA MECNICA DEL MEDIO CONTINUO 361

r r r
Notar que el trmino A H& G representa las componentes de H& G en el sistema x * , y notar
r r
tambin que A H& G H& G* , luego:
r * r r
A H& = H& * + r * H * (componentes) (4.49)
G G G

Tambin podemos expresar la ecuacin anterior en notacin tensorial:


r r
DH G DH G r r
=
Dt Dt + H G (notacin tensorial) (4.50)
f r
r r
DH G r DH G

donde representa la tasa de H G con respecto al sistema fijo,
Dt
Dt f r
r r
representa la tasa de H G con respecto al que est rotando con una velocidad angular .
NOTA 2: La ecuacin (4.50) es vlida para cualquier vector, i.e. la tasa de cambio de un
r r r
vector b respecto a un sistema fijo x es igual a la tasa de cambio del vector b con
r
respecto al sistema mvil x * ms el producto vectorial entre la velocidad angular del
r r
sistema mvil ( ) y el vector b :
r r
Db Db r r
= + b (4.51)
Dt
fijo Dt mvil
r
D D D Db
Adoptaremos la siguiente nomenclatura , .
Dt fijo Dt fl Dt mvil Dt r

r
b r

x3
x2*
x3*
x1*

x 2

x1

Figura 4.14

Notar tambin que:


r r r
D D r r D
= +1
=
2r3 Dt
Dt f Dt r r
=0

Universidad de Castilla- La Mancha Draft Por: Eduardo W. V. Chaves (2012)


Ciudad Real - Espaa
362 PROBLEMAS RESUELTOS DE MECNICA DEL MEDIO CONTINUO

NOTA 3: Vamos exponer un simple ejemplo para la obtencin de T . Vamos asumir que
el sistema ei es girando alrededor del sistema fijo ei , ver Figura 4.15, luego para obtener
T procedimos como sigue. Obtenemos la matriz de transformacin del sistema ei al ei :

cos sin 0
A = sin cos 0 (4.52)
0 0 1

cuya tasa de cambio viene dada por:


d (cos ) d (sin )
0
& & cos 0 sin
sin cos
dt dt 0
d (A ) d (sin ) d (cos )
&
A = 0 = & cos & sin 0 = & cos sin 0
dt dt dt
0 0 0 0 0 0 0 0 0


sin cos 0 cos sin 0 0 1 0 0 & 0



= A A = cos
& T & sin 0 sin cos 0 = & 1 0 0 = & 0 0
0 0 0 0 0 1 0 0 0 0 0 0

Con lo cual:
0 & 0 0 3 2 0

T = & 0 0 = 3 0 1 i = 0
0 0 0 2 3 0 &

r
donde es el vector axil asociado con el tensor antisimtrico T .

3 = &

e3 e3
e2
e1 e 2
e1

Figura 4.15

Universidad de Castilla- La Mancha Draft Por: Eduardo W. V. Chaves (2012)


Ciudad Real Espaa
4 LEYES FUNDAMENTALES DE LA MECNICA DEL MEDIO CONTINUO 363

Ejemplo 4.25
r
Demostrar que la aceleracin en un sistema fijo a f puede ser expresada por:
r r r r r r r
a f = ar + 2( v r ) + ( x ) (4.53)
r r
donde a r y v r son, respectivamente, la aceleracin y la velocidad de una partcula con
r
respecto a un observador que est girando con el sistema x * , (ver Figura 4.14). Considerar
r r r
tambin que = es la velocidad angular del sistema x * , y dicha velocidad es consntante
en el tiempo.
Solucin:
Utilizaremos directamente la ecuacin (4.51) para obtener la velocidad:
r r
Dx Dx r r r r r r
= + x v f = vr + x
Dt f Dt r
Aplicaremos la misma definicin con el objeto de obtener la aceleracin, i.e.:
r r r r r r r
Dv f D[v r + x ] D[v r + x ] r r r r
= = + [v r + x ]
Dt f Dt f Dt r
r r r
r Dv D[ x ] r r r r r
af = r + + v r + ( x )
Dt r Dt r
r r r
r Dv r D r r Dx r r r r r
af = + x+ + v r + ( x )
Dt r Dt r Dt r
r r r& r r r r r r r r
a = a + x + v + v + ( x )
f r r r
r r r& r r r r r r
a f = ar + x + 2( vr ) + ( x )
r r
Como estamos asumiendo una velocidad angular constante, tenemos que & = 0 , i.e. la
aceleracin angular es cero, con eso obtenemos la ecuacin (4.53). Luego, podemos
concluir que:
r r r& r r r r r r
a f = ar + x + 2( v r ) + ( x ) (4.54)

Notar que para obtener la ecuacin anterior no hemos tenido que recurrir a ningn
principio de conservacin. La ecuacin anterior es nada ms y nada menos que relacionar la
aceleracin en un sistema fijo con parmetros definidos en el sistema mvil.
r r r r r r r r r
NOTA 1: Utilizando la identidad a (b c ) = (a c )b (a b )c , (ver Ejemplo 1.17),
r r r r r r r r r r r r r r
podemos concluir que ( x ) = ( x ) ( ) x = ( x ) x . Notar que, si
2

r
= 3e 3 , (ver Figura 4.16), y si tambin adoptamos el sistema ( e r , e , e3 ) y teniendo en
r r r r r r r r r r r r
cuenta que r = 0 , podemos obtener que ( r ) = ( r ) r = r , el cual
2 2

representa la aceleracin centrpeta, (ver Ejemplo 2.57). La Tierra gira con una tasa de
rad 2 rad rad r r r
3 = 2 = 0.727 10 4 . Notar que el trmino ( x ) es muy
day 86400 s s
r r
pequeo cuando comparado con 2( vr ) .

Universidad de Castilla- La Mancha Draft Por: Eduardo W. V. Chaves (2012)


Ciudad Real - Espaa
364 PROBLEMAS RESUELTOS DE MECNICA DEL MEDIO CONTINUO

x3 , z
r
= 3e 3
3

e3
r
x3 r e
r
x e r


x2 , y

x1 , x

Figura 4.16

r r
NOTA 2: El trmino 2( v r ) , el cual fue establecido por Gustave-Gaspard Coriolis en
1835, est asociado con la fuerza ficticia conocida como fuerza de Coriolis. A continuacin,
r r
representaremos 2( v r ) en el sistema e i de la Figura 4.17.

N
e2
e3
Latitude
r e1
x
e 3 , e 2

e 2 , e 1
Equator

e 1 , e 3

Figura 4.17

Universidad de Castilla- La Mancha Draft Por: Eduardo W. V. Chaves (2012)


Ciudad Real Espaa
4 LEYES FUNDAMENTALES DE LA MECNICA DEL MEDIO CONTINUO 365

La ley de transformacin del sistema e i al e i viene dada por:

e 1 0 1 0 e 1 0 1 0

e2 = sin 0 cos e 2 B = sin 0 cos

(4.55)
e cos 0 sin e cos 0 sin
3 3

r
= 3 cos( )e 2 + 3 sin( )e 3
3 3e3

N
e2
e3

Figura 4.18

r r
El trmino 2( v r ) puede ser obtenido como sigue:

e 1 e 2 e 3
r r
2( vr ) = 0 3 cos( ) 3 sin( )
vr1 vr 2 vr 3 (4.56)
= 2e 1 [3 cos( )vr 3 3 sin( )vr 2 ] 2e 2 [ 3 sin( )vr1 ] + 2e 3 [ 3 cos( )vr1 ]
= 2[3 cos( )vr 3 3 sin( )vr 2 ]e 1 + 2[3 sin( )vr1 ]e 2 2[3 cos( )vr1 ]e 3

El parmetro f = 23 sin( ) es conocido como parmetro de Coriolis. Para pequeos valores


de vr 3 la ecuacin anterior se reduce a:
r
Dv r r r
= 2( v r ) = [23 sin( )vr 2 ]e 1 + [ 23 sin( )vr1 ]e 2 = [ f vr 2 ]e 1 + [ f vr1 ]e 2
Dt r
Dv r 1
Dt = f vr 2

Dv r 2 = f v
Dt
r1

Universidad de Castilla- La Mancha Draft Por: Eduardo W. V. Chaves (2012)


Ciudad Real - Espaa
366 PROBLEMAS RESUELTOS DE MECNICA DEL MEDIO CONTINUO

Figura 4.19: Efecto de Coriolis (Ref.: Wikipedia Coriolis effect).

NOTA 3: Deflexin de un cuerpo en cada vertical


Un aplicacin muy sencilla del efecto de Coriolis se presenta a continuacin. Vamos
considerar un observador en la superficie de la Tierra. Vamos considerar tambin que un
slido de masa m est en cada libre con las siguientes condiciones de contorno e inicial: en
d
t = 0 . ( x3 = h) , ( x1 = 0) , (
x3 = v3 = 0) , (v1 = 0) , (v2 = 0) . Como el slido est cayendo,
dt
calcularemos la deflexin del slido, i.e. vamos obtener x1 relacionada con el observador el
cual est atado al sistema que est girando con la Tierra. Adoptaremos el ismo sistema
descrito en la Figura 4.17.
r r
Teniendo en cuenta la segundo Ley de Newton ( F = ma f ) (aplicada para un sistema de
referencia inercial), podemos decir que
r r r r r r r r r r r r r
F = m[ar + 2( v r ) + ( x )] ma r = F 2m( v r ) = mge 3 2m( v r )

2[3 cos( )vr 3 3 sin( )vr 2 ] 23 cos( )vr 3


r r r r
ar = ge 2( vr ) (a r ) i = 2[3 sin( )vr1 ] = 0

2[3 cos( )vr1 ] g
g

r
donde la aceleracin a f viene dada por (4.54), y estamos considerando que el trmino
r r r r r
( x ) es muy pequeo cuando comparado con el trmino 2( vr ) cuyas
componentes vienen dadas por (4.56). Luego
d 2 x1
2
ar1 dt2 23 cos( )vr 3
r d x
(ar )i = ar1 = 22 = 0 (4.57)
a dt2 g
r1 d x3
dt 2

Notar que
d 2 x3 dx
2
= g integrando
3 = gt + C1 vr 3 = gt
dt dt
dx t2 t2
x3 = g + C 2
3 = gt integrando x3 = g + h
dt 2 2

Universidad de Castilla- La Mancha Draft Por: Eduardo W. V. Chaves (2012)


Ciudad Real Espaa
4 LEYES FUNDAMENTALES DE LA MECNICA DEL MEDIO CONTINUO 367

donde hemos tenido en cuenta las condiciones iniciales i.e. en


t = 0 (vr 3 = 0) C1 = 0 , y t = 0 ( x3 = h) C2 = h . Notar que
t2 gt 2
x3 = g +h=0h= .
2 2
Considerando la aceleracin vr 3 = gt dentro de la primera componente de (4.57)
podemos obtener que:
d 2 x'1 dx '1 t2
= 2 3 cos( ) v r3 = 2 3 gt cos( ) integrando
= 2 3 g cos( ) + C1 = vr1
dt 2 dt 2
donde la constante de integracin se obtiene a travs de la condicin inicial:
at (t = 0) {vr1 = 0 C1 = 0

dx '1 t3
= vr1 = 3 g cos( )t 2 integratin
g x'1 = 3 g cos( ) + C2
dt 3
Notar tambin que C2 = 0 , con lo cual obtenemos que:
1
x'1 = 3 g cos( )t 3
3
Como el slido empez su cada a partir de una altura h , podemos decir que la siguiente
1 2 2h
relacin se cumple h = gt t = , con lo cual, la ecuacin anterior queda:
2 g
3
1 g 2h 2
x'1 = 3 g cos( )t 3 = 3 cos( )
3 3 g

Universidad de Castilla- La Mancha Draft Por: Eduardo W. V. Chaves (2012)


Ciudad Real - Espaa
368 PROBLEMAS RESUELTOS DE MECNICA DEL MEDIO CONTINUO

NOTE 4: Aceleracin debido a la esfericidad


Sistema local e1 (este)- e2 (norte)- e3 (radialmente haca fuera)

N North
S South
E East
N W West
N
e2
r e3
r
Latitud
E
W e1
r S
x r r
e3 r = x cos
e3
e1 e 2

Ecuador
e1

Polo Sur

e3 r d r d
v r1 = r = x cos ( e1 )
vr 2 dt dt
N
PN
r e2
r e3
r
x
S e2
PN
r v r1
r E

e1
PS
W
r d e1
vr 2 = x ( e2 )
dt

Figura 4.20

Universidad de Castilla- La Mancha Draft Por: Eduardo W. V. Chaves (2012)


Ciudad Real Espaa
4 LEYES FUNDAMENTALES DE LA MECNICA DEL MEDIO CONTINUO 369

Previamente hemos obtenido que la ley de transformacin del sistema ei al ei , (ver


ecuacin (4.52)), viene dada por:
cos sin 0
A = sin cos 0 (4.58)
0 0 1

y la ley de transformacin del sistema e i al e i (ver ecuacin (4.55)) viene dada por:

0 1 0
B = sin 0 cos

(4.59)
cos 0 sin

Luego, la ley de transformacin del sistema ei al e i viene dada por:


0 1 0 cos sin 0 sin cos 0
C = BA = sin 0 cos sin
cos 0 = sin cos sin sin cos
cos 0 sin 0 0 1 cos cos cos sin sin

La tasa de cambio de C viene dada por:


& cos & sin 0
d (C ) & &
C = ( cos cos + & sin sin ) ( & cos sin & sin cos ) & sin
dt ( & sin cos & cos sin ) ( & sin sin + & cos cos ) & cos

Efectuando la operacin algebraica = C&C T podemos obtener que:


v r1 v
0 r sin r r1 cos
0 & sin & cos x cos x cos
v vr 2
= C&C T = & sin 0 & = r r1 sin 0 r
& cos & 0 x cos x
v r1 vr 2
xr cos cos r
x
0

0 vr1 tan vr1
1
= r vr1 tan 0 vr 2
x
vr1 vr 2 0

como era de esperar, resulta ser una matriz antisimtrica. Notar que, segn la Figura 4.20
d v d v r 2
las siguientes relaciones se cumplen: & = r r1 y & = r .
dt x cos dt x
Aplicamos la definicin (ver ecuacin (4.50))
r r
Dv
=
Dv r r

Dt + vr
f Dt r
r r r
Notar tambin que T v r = v r se cumple, luego:

Universidad de Castilla- La Mancha Draft Por: Eduardo W. V. Chaves (2012)


Ciudad Real - Espaa
370 PROBLEMAS RESUELTOS DE MECNICA DEL MEDIO CONTINUO

0 vr1 tan v r1 v r1 vr1vr 2 tan + vr1vr 3


r 1 1 2
v r = r vr1 tan
T
0 vr 2 vr 2 = r vr1 tan + vr 2 vr 3
x x
vr1 vr 2 0 vr 3 vr21 vr22

(4.60)
vr1vr 2 tan + vr1vr 3 vr1vr 2 tan + vr1vr 3
r r 1 2 r r 1 2
a f = a r + r vr1 tan + vr 2 vr 3 a r = a f r vr1 tan + vr 2 vr 3
x x
vr21 vr22 vr21 vr22

NOTA 4: Aceleracin de Coriolis + aceleracin de Curvatura


La aceleracin relacionada con el trmino de Coriolis (ver Ec. (4.56) y (4.54)) y el trmino
debido a la curvatura vienen dados por:
r r r r r r r r
a f = a r + 2( v r ) + T vr + ( x ) (4.61)
donde

2[3 vr 3 cos( ) 3 vr 2 sin( )] vr1vr 2 tan( ) + vr1vr 3


r r T r 1 2
2( v r ) + v r = 2[3 vr1 sin( )] + r vr1 tan( ) + vr 2 vr 3 (4.62)
2[3 vr1 cos( ) ] x vr21 vr22

Universidad de Castilla- La Mancha Draft Por: Eduardo W. V. Chaves (2012)


Ciudad Real Espaa
4 LEYES FUNDAMENTALES DE LA MECNICA DEL MEDIO CONTINUO 371

4.2 Ejercicios Propuestos

Problema 4.1

Las ecuaciones obtenidas a travs de las leyes fundamentales de la mecnica del medio
continuo se pueden resumir de la siguiente manera:
D r
Principio de la conservacin de la masa + ( v ) = 0 (63)
Dt
Principio de la conservacin del r r
+ b = v& (64)
momento lineal
Principio de la conservacin del
= T (65)
momento angular
Principio de la conservacin de la r
u& = : D q + r (66)
Energa
Principio de la Irreversibilidad r 1 1 1 r
& ( x, t ) + : D u& 2 q xr T 0 (67)
(Desigualdad de Entropa) T T T
r
donde es la densidad de masa, v es la velocidad, es el tensor de tensiones de Cauchy,
r
b son las fuerzas msicas, u es la energa interna especfica o densidad de energa interna
por unidad de masa, es la densidad de entropa por unidad de masa y por unidad de
r r
tiempo, T es la temperatura, q( x, t ) es el flujo de calor o vector del flujo no convectivo,
r
D es el tensor velocidad de deformacin, r ( x , t ) es una funcin escalar que describe en
forma espacial el calor generado por las fuentes internas por unidad de masa y por unidad
de tiempo.
Se pide:
a) Simplificar las ecuaciones anteriores (63) a (67) para el caso ESTTICO, PROCESO
ISOTRMICO Y ADIABTICO.
b) Hacer un estudio del planteamiento del problema si el problema est bien planteado. En
caso contrario, qu ecuaciones deben ser adicionadas al problema para que est bien
planteado?
c) Escribir en notacin indicial las ecuaciones (63) a (67).

Problema 4.2
Citar los principios fundamentales de la Mecnica del Medio Continuo. De cada principio,
qu ecuaciones se obtienen?

Universidad de Castilla- La Mancha Draft Por: Eduardo W. V. Chaves (2012)


Ciudad Real - Espaa
372 PROBLEMAS RESUELTOS DE MECNICA DEL MEDIO CONTINUO

Problema 4.3

Universidad de Castilla- La Mancha Draft Por: Eduardo W. V. Chaves (2012)


Ciudad Real Espaa
5 Introduccin a las
Ecuaciones Constitutivas
y PVCI
5.1 Ejercicios Resueltos

Ejemplo 5.1
Para un material termoelstico simple cules son las ecuaciones constitutivas y variables
libres si tenemos en cuenta la energa libre de Helmholtz ?
Solucin:
Las ecuaciones constitutivas para un material simples estn en funcin de las siguientes
variables libres:
= (F ,T )
( F , T )
P(F , T ) = 0
F
( F , T )
(F ,T ) =
T
r r
q0 = q0 ( F , T , Xr T )

(ver libro texto). Tambin se pueden presentar en funcin de las siguientes variables
= (F ,T )
= ( E , T )
( F , T ) T
( E , T ) = F
S = 0 F
E ( F , T )
; (F ,T ) =
( E , T )
( E ,T ) = T
T r 1 r
r r q = J q0 ( F , T , Xr T ) F T
q 0 = q0 ( E , T , Xr T ) r
= J 1 F q0 ( F , T , Xr T )

Ejemplo 5.2
Para un determinado material elstico se conoce la expresin de la densidad de energa (por
unidad de volumen), y viene dada por:
374 PROBLEMAS RESUELTOS DE MECNICA DEL MEDIO CONTINUO

1
( I E , II E ) = ( + 2 )I E2 2 II E
2
donde , son constantes del material. I E = I E (E ) , II E = II E (E ) son los invariantes
principales, el primer y segundo invariante principal del tensor de deformacin de Green-
Lagrange respectivamente. Cules son las ecuaciones constitutivas para este problema?,
justificar. Obtener tambin las expresiones explcitas de las ecuaciones constitutivas en
funcin de , , I E , II E .
Formulario
I E = I E ( E ) = Tr ( E )

II E = II E ( E ) =
1
2
[
( TrE ) 2 Tr ( E 2 ) ]
I E
=1
E
II E
= Tr ( E )1 E T
E
Solucin:
La expresin de la energa est SOLO en funcin del tensor de deformacin de Green-
Lagrange (grandes deformaciones). Sabemos que las ecuaciones constitutivas son:
= ( E , T )
( E , T )
S = 0
E
( E , T )
( E ,T ) =
T
r r
q 0 = q0 ( E , T , Xr T )

Teniendo en cuenta la expresin de la energa dada, concluimos que el problema es


independiente de la temperatura, ya que en la expresin de la energa dada no est en
funcin de la temperatura. Luego, slo me quedo con la ecuacin constitutiva de la tensin
y que podemos obtener como:
( E ) ( I E , II E ) ( I E , II E ) I E ( I E , II E ) II E
S = 0 = = +
E E I E E II E E
2
(
= ( + 2 )I E (1) + ( 2 ) Tr ( E )1 E T )
2
Simplificando la expresin anterior, y teniendo en cuenta que E T = E , I E = Tr (E ) ,
obtenemos:
S = I E 1 + 2 E
Ejemplo 5.3
1
Considerando la energa libre de Gibbs especfica G(S, T ) = ( E , T ) S : E como
0
ecuacin constitutiva de energa, obtener las dems ecuaciones constitutivas para un
material termoelstico simple.
NOTA: Partir del principio de que G(S, T ) no depende del gradiente de la temperatura.

Universidad de Castilla- La Mancha Draft Por: Eduardo W. V. Chaves (2012)


Ciudad Real - Espaa
5 INTRODUCCIN A LAS ECUACIONES CONSTITUTIVAS Y PVCI 375

Solucin:
Partimos de la forma alternativa de la desigualdad de Clausius-Duhem en la configuracin
de referencia:

[ ]
1r
S : E& 0 & + T& q0 Xr T 0
T
(5.1)

Teniendo en cuenta la energa libre de Gibbs especfica podemos decir que:


& (S, T ) = & ( E , T ) 1 & 1
G S:E S : E&
0 0
& (S, T ) + 1 S& : E + 1 S : E&
& ( E , T ) = G
0 0
y reemplazando en la desigualdad (5.1) obtenemos que:
& 1 & 1 1r
S : E& 0 G (S, T ) + S:E+ S : E& + T& q0 Xr T 0
0 0 T (5.2)
& (S, T ) S& : E T& 1 qr
0G 0 XT 0
r
0
T

Notar que S& : E = E : S& se cumple. La desigualdad anterior nos sugiere que para una
variacin de la energa libre de Gibbs tenemos que tener las siguientes relaciones:
Deformacin para una variacin de tensin; Entropa para una variacin de temperatura;
Flujo de calor para un gradiente de temperatura.
& (S , T ) puede ser expresado como:
El trmino G
DG(S, T ) & G(S, T ) & G(S, T ) &
G(S, T ) = :S + T
Dt S T
Y reemplazando en la ecuacin (5.2) obtenemos que:
& (S , T ) E : S& T& 1 q r
0G 0 XT 0
r
0
T
G(S , T ) & G(S, T ) & 1r
0 :S 0 T E : S& 0T& q0 Xr T 0 (5.3)
S T T
G(S , T ) G(S, T ) 1 r
0 + E : S& 0 + T& q 0 Xr T 0
S T T

La desigualdad anterior debe cumplir para todo proceso termodinmicamente admisible. Si


r r
tenemos un proceso isotrmico (T& = 0) y adiabtico (q 0 = 0) , la desigualdad de entropa
queda:
G(S , T )
0 + E : S& 0 (5.4)
S
cuya desigualdad tambin debe cumplir para todo proceso. Luego si para una dada
variacin S& se cumple la desigualdad anterior, podemos aplicar un proceso tal que S& = S& ,
violando as el principio de la desigualdad de entropa. Luego, la desigualdad anterior nunca
ser violada si y solo si
G(S, T ) G(S, T )
0 + E =0 E = 0
S S

Universidad de Castilla- La Mancha Draft Por: Eduardo W. V. Chaves (2012)


Ciudad Real - Espaa
376 PROBLEMAS RESUELTOS DE MECNICA DEL MEDIO CONTINUO

Teniendo en cuenta la ecuacin anterior (ecuacin constitutiva de deformacin) en la


desigualdad (5.3), obtenemos que:
G(S, T ) G(S, T ) 1r
0 + E : S& 0 + T& q0 Xr T 0
S T T
(5.5)
G(S, T ) & 1 r
0 + T q0 X T 0
r
T T
r
Si ahora tenemos un proceso donde Xr T = 0 (campo de temperatura homogneo), la
desigualdad queda:
G(S, T )
0 + T& 0
T
La desigualdad anterior nunca ser violada si y solo si
G(S, T ) G(S, T )
+=0 =
T T
Que es la ecuacin constitutiva de entropa. Luego, las ecuaciones constitutivas son:
Ecuacin constitutiva de energa G(S, T )
G(S, T ) g(S, T )
Ecuacin constitutiva de deformacin E = 0 =
S S
(5.6)
G(S, T )
Ecuacin constitutiva de entropa = T
r r
Ecuacin constitutiva de flujo de calor q0 = q0 ( Xr T )
donde g = 0 G . Notar que nuestras variables libres son (S , T ) .

Ejemplo 5.4
Demostrar que para un proceso adiabtico e isotrmico, y sin tasa de cambio de tensin, la
energa libre de Gibbs especfica no puede crecer.
Solucin:
Empezaremos directamente a partir de la ecuacin (5.3):
r
& (S , T ) E : S& T& 1 q
0G 0 XT 0 (5.7)
r
0
T
Teniendo en cuenta un proceso isotrmico y adiabtico las siguientes relaciones se cumplen
r r
T& = 0 , q0 = 0 , y si consideramos que no hay tasa de cambio de tensin S& = 0 se cumple.
Con lo cual la desigualdad en (5.7) reduce a :
& (S, T ) 0
0G (5.8)
& (S, T ) 0 para que no la desigualdad
ya que 0 > 0 , concluimos que se debe cumplir que G
de entropa se cumpla.

Universidad de Castilla- La Mancha Draft Por: Eduardo W. V. Chaves (2012)


Ciudad Real - Espaa
5 INTRODUCCIN A LAS ECUACIONES CONSTITUTIVAS Y PVCI 377

Ejemplo 5.5
a) Hacer el planteamiento de las ecuaciones de gobierno para un problema de slidos con
las siguientes caractersticas: Proceso isotrmico y adiabtico, rgimen de pequeas
deformaciones, y relacin lineal entre tensin y deformacin.
b) Una vez establecida la relacin lineal entre tensin-deformacin, obtener dicha relacin
para que cumpla que ( ) sea una funcin-de-tensores istropa de valor tensor de
segundo orden.
Solucin:
Para un proceso isotrmico y adiabtico la temperatura y la entropa no juega ningn papel.
Para un rgimen de pequea deformaciones tenemos que:
r
Tensor de deformaciones: E e = sym u
Tensor de Tensiones: P S
F 1 ; 0 ; Xr xr , con esta aproximacin la densidad de masa deja
de ser incgnita.
Teniendo en cuenta las ecuaciones fundamentales:
Ecuaciones Fundamentales de la Mecnica del Medio Continuo
(Configuracin Actual)
Ecuacin de continuidad de masa D r
+ ( xr v ) = 0 (1 ecuacin) (5.9)
(Principio de la conservacin de la masa) Dt
Ecuaciones del Movimiento r r
(Principio de la conservacin del momento xr + b = v& (3 ecuaciones) (5.10)
lineal)
Simetra del tensor de Tensiones de Cauchy
(Principio de la conservacin del momento = T (5.11)
angular)
Ecuacin de Energa r
u& = : D xr q + r (1 ecuacin) (5.12)
(Principio de la conservacin de la Energa)
Desigualdad de Entropa r 1 1 1 r
& ( x, t ) + : D u& 2 q xr T 0 (5.13)
(Principio de la Irreversibilidad) T T T

Solo quedamos con las siguientes ecuaciones:


1) Ecuaciones de Movimiento
r r
+ b = v&
2) Ecuacin de Energa
r r r
0 u& ( X , t ) = S : E& Xr q 0 + 0 r ( X , t )
u& = : &
Du D
o en funcin de la energa libre de Helmholtz = [ + T] = & :
Dt Dt
& = & e = : &

Universidad de Castilla- La Mancha Draft Por: Eduardo W. V. Chaves (2012)


Ciudad Real - Espaa
378 PROBLEMAS RESUELTOS DE MECNICA DEL MEDIO CONTINUO

donde e es la densidad de energa. Verificamos a travs de la desigualdad de entropa que


es un proceso sin disipacin de energa, es decir, toda energa que se almacena debido al
incremento de se recuperar con la disminucin de .
3) De las Ecuaciones Constitutivas (ver Ejemplo 5.1) solo quedamos con:
= ( )
( ) e ( )
S= = = ( )

es decir, la energa ( ) y la tensin son funciones solamente de la deformacin. Si
( ) &
calculamos la tasa de la energa libre de Helmholtz & () = : , y reemplazamos en la

expresin de la energa & = & e = : & , concluimos que:
( ) & & e ( ) & e ( )
: = : = : & =

Luego, la ecuacin de energa es una ecuacin redundante, es decir, si conozco la tensin
puedo conocer la energa y vise-versa. Resumimos as las ecuaciones de gobierno para el
problema propuesto:
Ecuaciones de Movimiento:
r r r &r& (3 ecuaciones)
+ b = a = v& = u
Ecuacin Constitutiva en Tensin:
( ) (5.14)
( ) = (6 ecuaciones)

Ecuaciones Cinemticas:
r
= sym u (6 ecuaciones)
r
Como incgnitas tenemos: (6), u (3), (6), un total de 15 incgnitas y 15 ecuaciones
luego, el problemas est bien planteado. Para que el conjunto de ecuaciones en derivada
parciales anteriores tenga solucin nica es necesario introducir las condiciones de
contorno e inicial, constituyendo as en un Problema de Valor de Contorno Inicial (PVCI). El
problema que acabamos de plantear es el Problema Elstico Lineal que es el tema del prximo
captulo. Las condiciones de contorno e inicial para este problema son:
Condiciones de contorno en desplazamiento, en S u :
r r r r r r
u( x , t ) =u* ( x , t ) ui ( x, t ) = u i * ( x, t ) (5.15)
Condiciones de contorno en tensiones, en S :
r r r r
( x , t ) n = t * ( x , n , t ) jk n k = t j * ( x, t ) (5.16)
Condiciones iniciales ( t = 0 ):
r r r
u( x , t = 0) = u 0 r r
r r u i ( x , t = 0) = u 0 i ( x )
u 0 ( x , t ) r r r r r (5.17)
= u& 0 ( x, t ) = v 0 ( x ) u& 0 i ( x ) = v 0 i
t t =0

En el caso de un problema esttico o casi-esttico, las ecuaciones de Cauchy recaen en las


ecuaciones de equilibrio y las condiciones iniciales son redundantes.

Universidad de Castilla- La Mancha Draft Por: Eduardo W. V. Chaves (2012)


Ciudad Real - Espaa
5 INTRODUCCIN A LAS ECUACIONES CONSTITUTIVAS Y PVCI 379

B S
Su
r r
dV t * ( x)
r r
b( x )
n

Figura 5.1: Slido sobre acciones externas.


En el apartado Serie de Tensores, captulo 1, hemos visto que podemos aproximar un
tensor a travs de la serie:
1 1 ( 0 ) 1 2 ( 0 )
( ) ( 0 ) + : ( 0 ) + ( 0 ) T : : ( 0 ) + L
0! 1! 2!
( 0 ) 1 2 ( 0 )
0 + : ( 0 ) + ( 0 ) T : : ( 0 ) + L
2
Considerando el punto de aplicacin 0 = 0 , y ( 0 ) = 0 = 0 , y adems teniendo en
cuenta que la relacin - es lineal, podemos despreciar los trminos de orden superior,
obteniendo entonces que:
( 0 ) 2 e ( 0 ) ij 2 e ( 0 )
( ) = : = : = Ce : ij = kl = kl = C eijkl kl
kl ij kl

2 e ( 0 )
donde C e = es un tensor de cuarto orden simtrico y es conocido como tensor

constitutivo elstico, que contiene las propiedades mecnicas del material.
Observemos que la energa tiene que ser de orden cuadrtica para que la relacin - sea
lineal, ver ecuacin (5.14). Utilizamos la expansin en serie para representar la densidad de
energa, obtenemos que:
1 1 e ( 0 ) 1 2 e ( 0 )
e ( ) = e ( 0 ) + : ( 0 ) + ( 0 ) T : : ( 0 ) + L
0! 1! 2!
1 2 e ( 0 )
= e 0 + 0 : ( 0 ) + ( 0 ) T : : ( 0 ) + L
2
1 2 e ( 0 )
= : :
2
1
= : Ce :
2
donde tambin hemos considerado que 0 = 0 e0 = 0, 0 = 0 .
NOTA 1: Aunque la ecuacin de energa es redundante, a la hora de establecer un mtodo
sea analtico o numrico para resolver el problema el punto de partida es a travs de
principios energticos, de ah la importancia de estudiar la energa de un sistema.
NOTA 2: La simetra de C e se comprueba fcilmente. Presenta simetra menor debido a
la simetra de y :
ij = ji C eijkl = C ejikl ; kl = lk e
C ijkl e
= C ijlk

Universidad de Castilla- La Mancha Draft Por: Eduardo W. V. Chaves (2012)


Ciudad Real - Espaa
380 PROBLEMAS RESUELTOS DE MECNICA DEL MEDIO CONTINUO

y la simetra mayor es debido a que:


2 e ( ) 2 e ( )
Ceijkl = = = C eklij (simetra mayor)
ij kl kl ij
NOTA 3: Para una mejor ilustracin del problema planteado, consideremos un caso
particular (caso unidimensional) donde las componentes del tensor de tensiones y de
deformaciones vienen dadas por:
0 0 0 0
ij = 0 0 0 ; ij = 0 0 0 11 = C1111
e
11 = E
0 0 0 0 0 0

En este caso la relacin lineal tensin-deformacin viene dada por = E y la densidad de


1 1 2 e
energa e = = E , y = =E.
2 2

estado actual
e () ()
1
e e = E
2
energa almacenada
1
e =
E 2
1
0 = 0
e0 = 0 0 = 0

Figura 5.2: Relacin tensin-deformacin (caso unidimensional).


NOTA 4: Debemos enfatizar que en el caso de un proceso elstico la ecuacin constitutiva
( ) es nicamente dependiente del valor actual de , i.e. es independiente de la historia
de deformacin.
b) La funcin-de-tensor ( ) ser istropa si se cumple que:
ij ( kl ) = ij ( kl )

Teniendo en cuenta que la relacin entre - viene dada, en notacin indicial, por
e
ij ( ) = C ijkl kl , concluimos que:

ij ( kl ) = ij ( kl )
C ijkl
e e
= C ijkl
C ijkl
e
kl = C ijkl
e
kl

Es decir, el tensor de cuarto orden C e es un tensor istropo. Un tensor de cuarto orden


e
istropo simtrico tiene el formato C ijkl = ij kl + ( ik jl + il jk ) . Que en notacin
tensorial viene dado por C e = 1 1 + 2 I , donde I I sym es el tensor identidad
simtrico de cuarto orden, y los parmetros y son conocidos como las constantes de
Lam. Como hemos visto en el Captulo 1, un tensor de cuarto orden simtrico viene dado
en funcin de dos constantes ( , ). En el Captulo 7 del libro de texto (Elasticidad

Universidad de Castilla- La Mancha Draft Por: Eduardo W. V. Chaves (2012)


Ciudad Real - Espaa
5 INTRODUCCIN A LAS ECUACIONES CONSTITUTIVAS Y PVCI 381

Lineal) veremos que es posible expresar C e en funcin de otros parmetros, e.g. ( E , ),


( , G ), donde E es el mdulo de Young, es el coeficiente de Poisson, es el mdulo volumtrico,
y G = es el mdulo de elasticidad transversal. Los significados fsicos de estos parmetros
sern dados en el mismo captulo 7.
NOTA 5: En la Figura 5.3 se muestra la relacin tensin-deformacin para un material
istropo. Es interesante observar que debido a que C e es independiente de la direccin los
tensores y comparten las mismas direcciones principales. Luego, para un material
istropo tenemos que ( ) = (1 1 + 2 I) : = Tr ( )1 + 2 :
e ( )
( ) = lineal
( ) = C e : istropo
( ) = Tr ( )1 + 2
14424 4
3
Elstico

Es interesante comparar con la ecuacin constitutiva en tensin del Ejemplo 5.2 donde se
considera grandes deformaciones, pero mantiene una relacin lineal entre la tensin y
deformacin.

22
11

12 x1

ij = C ijkl
e
kl P
ij = a ip a jq pq
22
12
11 ij = a ip a jq pq

22 e
ij = C ijkl kl 22
12 12
11 11
P P

P
x1
22

ij = C ijkl
e kl
P

11
22

Espacio
principal
Material istropo
P
e
C ijkl = C ijkl
e
= C ijkl
e
11

e (kl ) = e ( kl )
x1

Figura 5.3: Relacin tensin-deformacin material istropo.

Universidad de Castilla- La Mancha Draft Por: Eduardo W. V. Chaves (2012)


Ciudad Real - Espaa
382 PROBLEMAS RESUELTOS DE MECNICA DEL MEDIO CONTINUO

NOTA 6: Denotamos la densidad de energa de deformacin complementaria por e ( ) el cual es


una funcin de , (ver Figura 5.4), y viene dada por:
1 1 e ( 0 ) 1 2 e ( 0 )
e ( ) = e ( 0 ) + : ( 0 ) + ( 0 ) : : ( 0 ) + L
0! 1! 2!
1 2 e ( 0 )
= 0e + 0 : ( 0 ) + ( 0 ) : : ( 0 ) + L
2
1 2 e ( 0 ) 1 1 1
= : : = :De : = : Ce :
2 2 2
Note que, si estamos tratando con un material elstico lineal se cumple que e ( ) = e ( )
e ( )
y = .

a) Material elstico lineal. Densidad de energa de deformacin complementaria


( ) 1
e () = E 1
2

e ( ) = e ( ) Densidad de energa - e () =
1
E
2
E
1
0 = 0
0 = 0

b) Material elstico no-lineal.

Densidad de energa de deformacin complementaria e ()

()

Densidad de energa - e ()
e ( ) e ( )

0 = 0
0 = 0

Figura 5.4: Densidad de energa de deformacin complementaria (caso unidimensional).


NOTA 7: Notar que

e () = e () tensorial
e ( ) = : e ( ) = 0 G( ) = g( )

Universidad de Castilla- La Mancha Draft Por: Eduardo W. V. Chaves (2012)


Ciudad Real - Espaa
5 INTRODUCCIN A LAS ECUACIONES CONSTITUTIVAS Y PVCI 383

donde g( ) = 0 G( ) es la densidad de energa libre de Gibbs (por unidad de volumen)


con signo contrario, (ver ecuaciones en (5.6) del Ejemplo 5.2).

Ejemplo 5.6
En un ensayo de traccin simple los siguientes valores fueron obtenidos para la tensin-
deformacin:
Punto ( Pa ) (10 3 )
1 6,67 0,667
2 13,3 1,33
3 20 2
4 24 3
5 22 3,6
Determinar el mdulo de Young E y los puntos lmites.
Solucin:
Podemos verificar que los tres primeros puntos mantienen la misma proporcionalidad:
(1) ( 2 ) ( 3) 20
E= = ( 2 ) = ( 3) = = 10 000 Pa = 10 kPa
(1)
2 10 3
La grfica tensin-deformacin con los puntos dados se puede apreciar en la Figura 5.5. En
esta figura se sealan los punto: e - lmite elstico; Y - punto de fluencia; u - punto de
tensin ltima. r - punto de ruptura.

( Pa ) 30
u
25 Y r
3; 24
e 3,6; 22
20 2; 20

15
1,33; 13,3
10
0,667; 6,67
5

0 0; 0
0 0,5 1 1,5 2 2,5 3 3,5 4
0, 2% 3
(10 )
Figura 5.5: Curva tensin-deformacin.

Ejemplo 5.7
a.1) Escribir la densidad de energa de deformacin a.2) y las ecuaciones constitutivas de
tensin de un material elstico linear e istropo en notacin de Voigt, a.2.1) en funcin de
E E
( , ), a.2.2) y en funcin de ( E , ) donde = y = . b) Si
(1 + )(1 2 ) 2(1 + )

Universidad de Castilla- La Mancha Draft Por: Eduardo W. V. Chaves (2012)


Ciudad Real - Espaa
384 PROBLEMAS RESUELTOS DE MECNICA DEL MEDIO CONTINUO

expresamos el tensor de deformacin en notacin de Voigt tal que { } = L(1) [ ] {u },


[ ]
obtener la matriz L(1) .
c) Escribir las ecuaciones de movimiento en notacin de Voigt.
Solucin:
a.1) La densidad de energa de deformacin viene dada por:
1 1 1 1
e () = : C e : = : = : = ij ij
2 2 2 2
donde hemos utilizado que = C e : . Notar que
ij ij = 1 j 1 j + 2 j 2 j + 3 j 3 j
123 123 123
1111 21 21 3131
+ + +
1212 22 22 3232
+ + +
1313 23 23 3333

luego
1 1
e () = ij ij = (1111 + 22 22 + 33 33 + 212 12 + 2 23 23 + 21313 )
2 2
y
11

22
1 1 33 1
e ( ) = ij ij = [11 22 33 12 23 13 ] = { } { }
T

2 2 2
12 2
2 23

213
Luego, los tensores y en notacin de Voigt estn almacenados como sigue:
11 11

22 22

{ } = 33 ; { } = 33
12 212
23 2 23

13 213
a.2.1) La ecuacin constitutiva de tensin en notacin de Voigt queda:
11 + 2 0 0 0 11
+ 2 0 0 0 22
22
33 + 2 0 0 0 33
= C e : Voigt
= { } = [C ] { }
12 0 0 0 0 0 2 12
23 0 0 0 0 0 2 23

13 0 0 0 0 0 213

Universidad de Castilla- La Mancha Draft Por: Eduardo W. V. Chaves (2012)


Ciudad Real - Espaa
5 INTRODUCCIN A LAS ECUACIONES CONSTITUTIVAS Y PVCI 385

(5.18)
Para mayores detalles de la ecuacin anterior ver Ejemplo 1.97 en el captulo 1, donde
tambin hemos obtenido que:
1
= Tr ( )1
2 2 ( 2 + 3 )
y
+
( 2 + 3 ) 0 0 0
2 ( 2 + 3 ) 2 ( 2 + 3 )

+
11
0 0 0 11

2 ( 2 + 3 ) ( 2 + 3 ) 2 ( 2 + 3 )
22
22 +
33 2 ( 2 + 3 ) 0 0 0
2 ( 2 + 3 ) ( 2 + 3 ) 33
= 1
2 12 0 0 0 0 0 12 (5.19)
2 23 23
1
2 13 0 0 0 0 0 13


1
0 0 0 0 0

{ } = [C ] 1 { }
a.2.2) Note que
E E E
+ 2 = +2 = (1 )
(1 + )(1 2) 2(1 + ) (1 + )(1 2)
E
= ( )
(1 + )(1 2 )
E E (1 2 )
= =
2(1 + ) (1 + )(1 2) 2
Luego, la ecuacin (5.18) puede ser reescrita como:
(1 ) 0 0 0
11 0 11
(1 ) 0 0

22 (1 ) 0 0 0 22
33 E (1 2)
= 0 0 0 0 0 33 (5.20)
2
12 (1 + )(1 2) (1 2 )
2 12
23 0 0 0 0 0 2 23
2
13 (1 2) 2 13
0 0 0 0 0
2
Note que

Universidad de Castilla- La Mancha Draft Por: Eduardo W. V. Chaves (2012)


Ciudad Real - Espaa
386 PROBLEMAS RESUELTOS DE MECNICA DEL MEDIO CONTINUO

E E E
+ = + =
(1 + )(1 2) 2(1 + ) 2(1 + )(1 2)
E E E E2
(2 + 3 ) = 2 + 3 =
2(1 + ) 2(1 + ) (1 + )(1 2 ) 2(1 + )(1 2)
+ E 2(1 + )(1 2) 1
= =
(2 + 3 ) 2(1 + )(1 2) E2 E
E (1 + )(1 2 )
= =
2 (2 + 3 ) (1 + )(1 2) E2 E
1 2(1 + ) 1
= = 2(1 + )
E E
Luego, la ecuacin (5.19) puede ser reescrita como:
11 1 0 0 0 11

22 1 0 0 0 22
33 1 1 0 0 0 33
= (5.21)
2 12 E 0 0 0 2(1 + ) 0 0 12
2 23 0 0 0 0 2(1 + ) 0 23

2 13 0 0 0 0 0 2(1 + ) 13

b) Segn la definicin ij = 12 (u i , j + u j ,i ) obtenemos que:

u1 1 u1 u 2 1 u1 u 3
+ +
x1 2 x 2 x1 2 x3 x1
1 u u u 2 1 u 2 u 3
ij = 1 + 2 +
2 x 2 x1 x 2 2 x3 x 2
1 u u 1 u 2 u 3 u 3
1 + 3

+
2 x3 x1 2 x3 x 2 x 3
u1
0 0
x1
x1
u 0
11
2
0
x 2 x 2

22 u 3 0
0 u1
33 x 3
{ } = = u u =
x3
u 2 { } = [L(1) ]{u }
212 1 + 2 0 u
2 23 x 2 x1 x 2 x1 3

u 2 u 3
2 13 + 0
x3 x 2 x3 x 2

u1 u 3
x + x x 0
3 1 3 x1

NOTA: Si adoptamos la notacin ingenieril, i.e. x1 = x , x 2 = y , x3 = z , u1 = u , u 2 = v ,


u 3 = w , 11 = x , 22 = y , 33 = z , 2 12 = xy , 2 23 = yz , 2 13 = xz , la ecuacin anterior
queda:

Universidad de Castilla- La Mancha Draft Por: Eduardo W. V. Chaves (2012)


Ciudad Real - Espaa
5 INTRODUCCIN A LAS ECUACIONES CONSTITUTIVAS Y PVCI 387

u
x x 0 0
v
11 x 0 0
y y
22 y w u
33 z z 0 0
{ } = = = u v =
z v
{ } = [L(1) ]{u } (5.22)
2 12 xy + 0 w
2 23 yz y x y x
v w
2 13 xz + 0
z y z y
u + w 0

z x z x
r r r
c) Teniendo en cuenta las ecuaciones de movimiento + b = v& = u && (ver ecuacin
(5.14) en notacin indicial ij , j + b i = u
&& i y su forma explcita:

ij , j + b i = i1,1 + i 2, 2 + i 3,3 + b i = u
&& i
11 12 13
+ + + b1 = &u&1
x x x
11,1 + 12, 2 + 13,3 + b1 = u &&1 1 2 3
21 22 23
21,1 + 22, 2 + 23,3 + b 2 = &u& 2 + + + b2 = u && 2
x1 x 2 x 3
31,1 + 32, 2 + 33,3 + b 3 = u 3
&&
31 32 33
+ + + b3 = u && 3
x1 x 2 x3
Luego, si consideramos las componentes del tensor de tensiones en notacin de Voigt, la
ecuacin anterior queda:


11
0 0 0
x1 x 2 x 3 22 b u &&1
0
33
1
&&
0 0 + b 2 = u 2
x 2 x1 x3 (5.23)
12 b 3 u
&& 3
0
0 0 23
x 3 x 2 x1
13
[ ] { } + { b } = { u&&}
L(1)
T

Ejemplo 5.8
Considerando un material elstico lineal homogneo e istropo descrito en el Ejemplo 5.5,
obtener las ecuaciones de gobierno de tal forma que resulte en un sistema de tres
ecuaciones y tres incgnitas, a saber: u1 , u 2 , u 3 , (formulacin en desplazamientos).

Universidad de Castilla- La Mancha Draft Por: Eduardo W. V. Chaves (2012)


Ciudad Real - Espaa
388 PROBLEMAS RESUELTOS DE MECNICA DEL MEDIO CONTINUO

Solucin:
Como visto en el Ejemplo 5.5 las ecuaciones de gobierno para un material elstico linear
homogneo e istropo en rgimen de pequeas deformaciones son:
Notacin tensorial Notacin indicial
Ecuaciones de Movimiento: Ecuaciones de Movimiento:
r r &r& (3 ecuaciones)
+ b = v& = u ij , j + b i = u
&& i (3 ecuaciones)
Ecuacin Constitutiva en Tensin: Ecuacin Constitutiva en Tensin:
( ) = Tr ( )1 + 2 (6 ecuaciones) ij = kk ij + 2 ij (6 ecuaciones) (5.24)

Ecuaciones Cinemticas: Ecuaciones Cinemticas:


r 1 u i u j
= sym u (6 ecuaciones) ij = + (6 ecuaciones)
2 x j x i

Resultando en un sistema de 15 ecuaciones y 15 incgnitas.
La divergencia del tensor de tensiones de Cauchy ( ) se puede obtener a travs de las
ecuaciones constitutivas en tensin, i.e.:
ij = kk ij + 2 ij
ij , j = ( kk ij + 2 ij ) , j
ij , j = , j kk ij + kk , j ij + kk ij , j + 2 , j ij + 2 ij , j
{ { { (5.25)
=0 j = 0i =0 j

ij , j = kk , j ij + 2 ij , j
ij , j = kk ,i + 2 ij , j
r
Si las propiedades mecnicas y son constantes en el material, i.e. no varan con x

(material homogneo), luego, , j = 0 j y ,j = 0 j . Podemos tambin expresar
x j x j
los trminos kk ,i y ij, j en funcin de los desplazamientos. Para ello utilizamos las
ecuaciones cinemticas:
1 u i u j 1
ij = +
2 x j xi 2
( )
u i , j + u j ,i divergenci 1
(
a ij , j = u i , jj + u j ,ij
2
)

Notar que
u i
2ui
=

x j x j x j

x j
r r
[ ] r
u i , jj [ (u)]i 2 u (Laplaciano del vector u )
i

2u j 2u j u j r
u j ,ij = = u j , ji [ ( u)]i
x j xi x i x j x i x j

1 u k u k u k
kk = + = gradiente
u k ,k kk ,i = u k , ki = u j , ji
2 x k x k x k
Con eso la ecuacin (5.25) puede ser reescrita como:

ij , j = kk ,i + 2 ij , j ij , j = u j , ji + 2
1
2
(
u i , jj + u j , ji )
ij , j = ( + )u j , ji + u i , jj

Universidad de Castilla- La Mancha Draft Por: Eduardo W. V. Chaves (2012)


Ciudad Real - Espaa
5 INTRODUCCIN A LAS ECUACIONES CONSTITUTIVAS Y PVCI 389

Reemplazando las ecuaciones anteriores en ij , j + b i = u


&& i (ecuaciones de movimiento),
obtenemos que:
ij , j + b i = &u& i ( + )u j , ji + u i , jj + b i = u
&& i

Resultando as en 3 ecuaciones y 3 incgnitas ( u1 , u 2 , u 3 ):

( + )u j , ji + u i , jj + b i = u
&& i
Ecuaciones de Navier (5.26)
r r r
&r&
( + )[ ( u)] + [ (u)] + b = u

NOTA 1: Las ecuaciones anteriores son conocidas como Ecuaciones de Navier o tambin
como Ecuaciones de Navier-Lam. La forma explcita de las ecuaciones (5.26) se presentan
a continuacin:
( + )u j , ji + u i , jj + b i = ( + )(u1,1i + u 2, 2i + u 3,3i ) + (u i ,11 + u i , 22 + u i ,33 ) + b i = u
&& i

( + )(u1,11 + u 2, 21 + u 3,31 ) + (u1,11 + u1, 22 + u1,33 ) + b1 = u &&1



( + )(u1,12 + u 2, 22 + u 3,32 ) + (u 2,11 + u 2, 22 + u 2,33 ) + b 2 = u&& 2

( + )(u1,13 + u 2, 23 + u 3,33 ) + (u 3,11 + u 3, 22 + u 3,33 ) + b 3 = u 3
&&

o an:
u1 u 2 u 3 2u 2 u1 2 u1
( + ) + + + 21 + + + b1 = u
&&1
x1 x1 x 2 x3 x x 22 x32
1

u1 u 2 u 3 2u 2 2u 2 2u 2
( + ) +
x 2 + x 2 + x 2 + b 2 = u 2
+ + &&
x 2 x1 x 2 x 3 1 2 3

( + ) u1 + u 2 + u 3 + u 3 + u 3 + u 3 + b = u
2 2 2
&& 3
x 3 x1 x 2 x 3 x 2 x32
3
1 x 22
NOTA 2: Hemos demostrado en el Ejemplo 1.105 que se cumple la siguiente relacin:
r r r r r
( a) = ( a) 2 a indicial
ilq qjk a k , jl = a j , ji a i , jj
Luego, tambin se cumple que
r r r r r r
(u) 2 u = ( u) ( u) indicial
u i , jj = u j , ji ilq qjk u k , jl
Con lo cual la ecuacin (5.26) tambin se puede escribir como:
( + )u j , ji + u i , jj + b i = u
&& i
( + )u j , ji + (u j , ji ilq qjk u k , jl ) + b i = u
&& i
( + 2 )u j , ji ilq qjk u k , jl + b i = u
&& i
Lo equivalente en notacin tensorial:
r r r
&r&
( + )[ ( u)] + [ (u)] + b = u
[ ]
r r r r r r
&r&
( + )[ ( u)] + ( u) ( u) + b = u
[ ]
r r r r r
( + 2 )[ ( u)] ( u) + b = u &r&
Resume:
[ ]
r r r r r
&r&
( + 2 )[ ( u)] ( u) + b = u
(5.27)
( + 2 )u j , ji ilq qjk u k , jl + b i = u
&&i
En el sistema Cartesiano tenemos que:

Universidad de Castilla- La Mancha Draft Por: Eduardo W. V. Chaves (2012)


Ciudad Real - Espaa
390 PROBLEMAS RESUELTOS DE MECNICA DEL MEDIO CONTINUO

r
u = u i e i = u1 e 1 + u 2 e 2 + u 3 e 3
r r r r u u u u u u
( u) rot (u) = (rot (u) )i e i = 3 2 e 1 + 1 3 e 2 + 2 1 e 3
x 2 x3
1442r443
x3 x1
1442r443
x1 x 2
1442r443
= (rot (u) )1 = (rot (u) )2 = (rot (u) )3
r r r r r r
r r r (rot (u) )3 (rot (u) )2 (rot (u) )1 (rot (u) )3 (rot (u) )2 (rot (u) )1
( u) = e 1 +


x e 2 +

e 3
x 2 x 3 3 x1 x 1 x 2

r r
(rot (u) )3 (rot (u) )2 u 2 u1 u1 u 3

x x x x x x
x x 2 2 3 1
r
2 3 1 3
r
(rot (u) ) ( ) )3 u 3 u 2 u 2 u1
[r r r
( u) i = ] 1

rot (u
=





x 3 x1 x 3 x 2 x 3 x1 x1 x 2
(rot (ur r
) )2 (rot (u) )1 u1 u 3 u 3 u 2


x1 x 2 x1 x3 x1 x 2 x 2 x3

NOTA 3: Si estamos tratando con un material heterogneo, las ecuaciones en (5.25)


quedan:
ij = kk ij + 2 ij
ij , j = ( kk ij + 2 ij ) , j
ij , j = ( kk ) , j ij + (2 ij ) , j = ( kk ) ,i + ( 2 ij ) , j
Teniendo en cuenta que 2 ij = u i , j + u j ,i , kk = u k ,k , la ecuacin anterior queda:
ij , j = ( kk ) ,i + (2 ij ) , j
[ ]
ij , j = (u k , k ) ,i + (u i , j + u j ,i ) , j
Con lo cual
ij , j + b i = &u& i [ ]
(u k ,k ) ,i + (u i , j + u j ,i ) , j + b i = &u& i (5.28)
Notar que
r r
u k , k = Tr (u) = ( u)
Du& i u& i u& i u& u& u& u& y sus componentes
&& i =
u = + v j = i + i v1 + i v 2 + i v3
Dt t x j t x1 x 2 x3
u& 1 u& 1 u& u&
+ v1 + 1 v 2 + 1 v3
t x1 x 2 x3
u& 2 u& 2 u& 2 u& 2
&& i =
u + v1 + v2 + v3
t x1 x 2 x 3
u& u& u& u&
3 + 3 v1 + 3 v 2 + 3 v3
t x1 x 2 x3
[ (u i, j ]
+ u j ,i ) , j =

x j
[
(u i , j + u j ,i ) ]

=
x1
[
(u i ,1 + u1,i ) +]x 2
[
(u i , 2 + u 2,i ) +
x3
] [
(u i ,3 + u 3,i ) ]

Universidad de Castilla- La Mancha Draft Por: Eduardo W. V. Chaves (2012)


Ciudad Real - Espaa
5 INTRODUCCIN A LAS ECUACIONES CONSTITUTIVAS Y PVCI 391


[ ]
2 (u1,1 ) + [
(u1, 2 + u 2,1 ) + ] [
(u1,3 + u 3,1 ) ]
x1 x 2 x3

[ ]
(u i , j + u j ,i ) , j = [ (u 2,1 + u1, 2 ) + ]
[
2 (u 2, 2 ) +

] [
(u 2,3 + u 3, 2 ) ]
x1 x 2 x3

[ (u 3,1 + u1,3 ) + ] [
(u 3, 2 + u 2,3 ) + ]
2 (u 3,3 ) [ ]
x1 x 2 x3
Explcitamente las tres ecuaciones dadas por (5.28) ( i = 1,2,3 ) quedan:

x
r
[( u)] +
x
[
2 (u1,1 ) +
x
] [
(u1, 2 + u2,1 ) +
x
] [
(u1,3 + u3,1 ) + b1 = u&&1 ]
1 1 2 3

[( ur )]+ (u2,1 + u1,2 ) + 2 (u2,2 ) + (u2,3 + u3,2 ) + b 2 = u&&2
[ ] [ ] [ ]
x2 x1 x2 x3

[( ur )] + (u3,1 + u1,3 ) + (u3,2 + u2,3 ) + 2 (u3,3 ) + b3 = u&&3
[ ] [ ] [ ]
x3 x1 x2 x3


[ r
( u) + 2 (u1,1 ) + ] [
(u1, 2 + u2,1 ) + ] [
(u1,3 + u3,1 ) + b1 = u&&1 ]
x1 x2 x3

[ r
( u) + 2 (u2, 2 ) + ] [
(u2,1 + u1, 2 ) + ] [
(u2,3 + u3, 2 ) + b 2 = u&&2 ]
x2 x1 x3

[ r
( u) + 2 (u3,3 ) + ] [
(u3,1 + u1,3 ) + ] [
(u3, 2 + u2,3 ) + b3 = &u&3 ]
x3 x1 x2

NOTE 4: Ecuaciones de Onda


Si aplicamos la divergencia a la ecuacin (5.27) obtenemos que:
[ ]
r r r r r
&r&
( + 2 ) [ ( u)] ( u) + b = u
1442443
=0
r r
( + 2 ) [ ( u)] + b = u &r&
r r
( + 2 ) 2 ( u) + b = u &r&
r
&r& = ( + 2 ) 2 ( u
u
r
) + b

D2 r ( + 2 ) 2 r r (5.29)
( u ) = ( u ) + b
Dt 2
D 2 ( + 2 ) 2 r
= + b
Dt 2

D2 r
2
= 2 2 + b
Dt
r
&& = 2 2 + b
r r r
donde hemos considerado que = u y ( v ) = 0 (ver Ejemplo 1.106). En notacin
indicial queda:

Universidad de Castilla- La Mancha Draft Por: Eduardo W. V. Chaves (2012)


Ciudad Real - Espaa
392 PROBLEMAS RESUELTOS DE MECNICA DEL MEDIO CONTINUO

( + 2 )u j , jii ilq qjk u k , jli + bi ,i = u


&&i ,i
( + 2 )u j , jii + b i ,i = u
&&i ,i
( + 2 )
u
&&i ,i = u j , jii + bi ,i

(5.30)
D2 ui ( + 2 ) 2 u j b i
2 = +
x j xi
Dt xi xi xi
D 2 2
2
b
= + i
Dt 2
xi xi xi
donde
( + 2 )
= velocidad de onda-P (5.31)

r r
Si las fuerzas msicas no cambian en el espacio, se cumple que b = 0 , con eso la
ecuacin (5.29) se reduce a:
D 2
= 2 2 Ecuacin de onda P (o Primaria) (5.32)
Dt 2
La onda-P no tiene rotacin.
r
Si ahora aplicamos el rotacional ( ) a la ecuacin (5.27), obtenemos que:
r r r r r r
[
r r r &r&
( + 2 ) [ ( u)] ( u) + b = u ]
r r
[ r r r r
]
( u) + b = u
r &r&

r r
[ r r r r
]
( u) + b = 2 ( u)
D2 r r
Dt
r r r r r 2 r
D
( ) + b =
Dt 2
r r r r
D 2 (5.33)
( ) =
Dt 2
r r r r
D 2
= ( )
Dt 2
r
D 2 r r r
= ( )
Dt 2
r r r r
D 2
2
= 2 ( )
Dt
r r r r
donde hemos considerado que = u , y que el campo vectorial b es un campo
r r r r r r r
conservativo, luego b = 0 se cumple. Notar que [ ( u)] = [ ] = 0 (ver
Ejemplo 1.106), y

= Velocidad de onda-S (5.34)

r r r r r r r r r r r r
Notar que 2 = ( ) ( ) 2 = ( ) , ya que ( u) = 0
r r r r
( ) = ( ( u)) = 0 (ver Ejemplo 1.106). Con lo cual la ecuacin (5.33) queda:

Universidad de Castilla- La Mancha Draft Por: Eduardo W. V. Chaves (2012)


Ciudad Real - Espaa
5 INTRODUCCIN A LAS ECUACIONES CONSTITUTIVAS Y PVCI 393

r
D 2 r Ecuacin de onda-S (o Secundaria, o
= 2 2 (5.35)
Dt 2 onda de corte)
Las ondas de corte no tienen cambio de volumen, solamente presentan cambio de forma.
En el caso que = 0 , la ecuacin (5.32) se reduce a la ecuacin de onda acstica:
D 2
2
= c 2 2 Ecuacin de onda acstica (5.36)
Dt
con

c= Velocidad de propagacin (5.37)

r r r
Notar que el campo de desplazamientos puede ser representado por: u = + con
r r r r r r
= 0 . Podemos probar eso a travs de la identidad ( a) = ( a) 2 a . Si
r r r r r r
consideramos los vectores u = 2a y = a , y el escalar = a . Con lo cual
r r r
obtenemos que u = + , y se cumple que:
r r r r r r r r r r r r
u = + ( ) = y u = + ( ) = ( )

a)

b)

Figura 5.6: Desplazamientos debido a una onda-P harmnica plana (a) y una onda-S (b). La
onda-P no tiene rotacin y la onda-S no tiene cambio de volumen.

E E
Si consideramos que = y = , podemos obtener que:
(1 + )(1 2) 2(1 + )
( + 2 ) E E
+2
( + 2 ) (1 + )(1 2) 2(1 + ) (2 2 )
= = = =
E (1 2)
2(1 + )
Con lo cual concluimos que la relacin entre las velocidades de onda-P y onda-S depende
nicamente del coeficiente de Poisson.

Universidad de Castilla- La Mancha Draft Por: Eduardo W. V. Chaves (2012)


Ciudad Real - Espaa
394 PROBLEMAS RESUELTOS DE MECNICA DEL MEDIO CONTINUO

Ejemplo 5.9
Consideremos el campo de deformacin infinitesimal . a) Demostrar que:
r r
qjk til ij ,kl = 0 qt ( )T = 0 (5.38)

donde ijk es el smbolo de permutacin.


b) Demostrar que:
ij , kl + kl ,ij il , jk jk ,il = O ijkl (5.39)
c) Expresar las ecuaciones en (5.38) de forma explcita.
Solucin:
1 u 1 u
El tensor de deformacin infinitesimal viene dado por ij = + i = (u j ,i + u i , j ) .
j

2 xi x j 2
r
Si derivamos con respecto a ( x ) obtenemos:
ij 1
= ij ,k = (u j ,ik + u i , jk )
x k 2
Notar que u i , jk = u i ,kj es simtrico en jk si multiplicamos por un tensor que es
antisimtrico en jk , i.e. qjk = qkj , ste se anula: u i , jk qjk = 0 iq , luego:
1 1
qjk ij ,k = (u j ,ik + u i , jk ) qjk = u j ,ik qjk
2 2
r
Derivamos una vez ms con respecto a ( x ) y obtenemos que:
1
( qjk ij ,k ) = qjk ij , kl = u j ,ikl qjk
x l 2
Notar que u j ,ikl = u j ,kil = u j ,kli es simtrico en il y til = tli es antisimtrico en il y si
multiplicamos ambos lados de la igualdad por til obtenemos que:
1
til qjk ij ,kl = u j ,ikl til qjk = 0 jkt qjk = 0 qt
2
b) Si en la ecuacin anterior multiplicamos por ambos lados de la igualdad por tab qmn ,
obtenemos que:
tab qmn til qjk ij ,kl = 0 qt tab qmn = O abmn
Recordar que se cumple que tab til = ai bl al bi y qmn qjk = mj nk mk nj , con lo
cual:
tab qmn til qjk ij ,kl = O abmn
( ai bl al bi )( mj nk mk nj ) ij , kl = O abmn
( ai bl mj nk ai bl mk nj al bi mj nk + al bi mk nj ) ij ,kl = O abmn
am ,nb an,mb bm ,na + bn,ma = O abmn
que es lo mismo que:
am,bn + bn,am an,mb mb,an = O ambn

Universidad de Castilla- La Mancha Draft Por: Eduardo W. V. Chaves (2012)


Ciudad Real - Espaa
5 INTRODUCCIN A LAS ECUACIONES CONSTITUTIVAS Y PVCI 395

c) Notar que en (5.38) tenemos 6 ecuaciones independientes ya que 0 qt es simtrico.


Para el caso q = 1, t = 1 tenemos que 1 jk 1il ij ,kl y expandiendo el subndice l obtenemos:
1 jk 1il ij ,kl = 1 jk 1i1 ij , k1 + 1 jk 1i 2 ij ,k 2 + 1 jk 1i 3 ij ,k 3 = 1 jk 1i 2 ij ,k 2 + 1 jk 1i 3 ij ,k 3
Expandiendo el subndice i
1 jk 1il ij ,kl = 1 jk 1i 2 ij ,k 2 + 1 jk 1i 3 ij ,k 3 = 1 jk 132 3 j , k 2 + 1 jk 123 2 j ,k 3 = 1 jk 3 j , k 2 + 1 jk 2 j ,k 3
Expandiendo los dems subndices obtenemos:
1 jk 1il ij ,kl = 1 jk 3 j ,k 2 + 1 jk 2 j , k 3 = 123 32,32 132 33, 22 + 123 22,33 + 132 23, 23
= 32,32 + 33, 22 + 22,33 23, 23 = 33, 22 + 22,33 2 23, 23 = 0
2 33 2 22 2 23
= + 2 =0
x 22 x32 x 2 x3

notar que 23, 23 = 32,32 . Dejamos para el lector las siguientes demostraciones:
Para el caso q = 2, t = 2
2 jk 2il ij ,kl = 31,31 + 33,11 + 11,33 13,13 = 33,11 + 11,33 213,13 = 0
2 33 2 11 2 13
= + 2 =0
x12 x32 x1x3
Para el caso q = 3, t = 3
3 jk 3il ij ,kl = 11, 22 12,12 21, 21 + 22,11 = 11, 22 + 22,11 212,12 = 0
2 11 2 22 2 12
= + 2 =0
x 22 x12 x1 x 2
Para el caso q = 1, t = 2
1 jk 2il ij ,kl = 12,33 + 13, 23 + 32,31 33, 21 = 13, 23 + 23,13 33,12 12,33 = 0
2 13 2 23 2 33 2 12 23 13 12 2 33
= + = + =0
x 2 x3 x1 x3 x1 x 2 x3 x3 x3 x1 x 2 x3 x1x 2
Para el caso q = 2, t = 3
2 jk 3il ij ,kl = 11,32 + 13,12 + 21,31 23,11 = 13,12 + 12,13 23,11 11, 23 = 0
2 13 2 12 2 23 2 11 13 12 23 2 11
= + = + =0
x1 x 2 x1 x3 x1x1 x 2 x 3 x1 x 2 x3 x1 x 2 x 3
Para el caso q = 1, t = 3
1 jk 3il ij ,kl = 12,32 13, 22 22,31 + 23, 21 = 12, 23 13, 22 22,13 + 23,12 = 0
2 12 2 13 2 22 2 23 12 13 23 2 22
= + = + =0
x 2 x3 x 2 x 2 x1x 3 x1x 2 x 2 x3 x 2 x1 x1x3
Reagrupando las 6 ecuaciones:

Universidad de Castilla- La Mancha Draft Por: Eduardo W. V. Chaves (2012)


Ciudad Real - Espaa
396 PROBLEMAS RESUELTOS DE MECNICA DEL MEDIO CONTINUO

2 33 2 22 2 23
S11 = + 2 =0
x 22 x32 x 2 x3
2 2 2
S 22 = 33 + 11 2 13 = 0
x12 x32 x1 x3

2 11 2 22 2 12
S 33 = + 2 =0
x 22 x12 x1 x 2 Ecuaciones de
(5.40)
S = 23 + 13 12 33 = 0
2 Compatibilidad 3D
12 x x x 2 x3 x1 x 2
3
1

23 13 12 2 11
S 23 = + + =0
x1 x1 x 2 x 3 x 2 x3

S = 23 13 + 12 22 = 0
2

13 x 2 x1 x 2 x 3 x1 x3

Las ecuaciones anteriores en notacin de Voigt quedan:


2 2 2
0 0 0
x32 x 22 x 2 x3
2 2 2
0 0 0
S11 x 2 x12

x1x3 11 0
S 3

22
2
2 2 22 0
0 0 0
S 33 x 2 x12 x1x 2
0
=
2 33 =
S
12 2 2 2 2
212 0 (5.41)
0 0 12 2 1 1
S 23 x1x 2 x3
2
x1x3 2
x 2 x 3 2 23 0

S13 2 2 13 0
2 2
2
0 0 1
2
12 2 1
2
x 2 x 3 x1 x3 x1 x1 x 2
2 2 2 2
0 0 1
2
1
2
12 2
x1x3 x 2 x3 x1x 2 x 2

{S } = [L( 2) ] { } = {0}
NOTA 1: Las ecuaciones (5.40) son conocidas como Ecuaciones de Compatibilidad. Las
ecuaciones de compatibilidad nos garantizan que el campo de desplazamiento es nico y
continuo. En otras palabras, las 6 componentes del tensor de deformacin no son
independientes y no pueden ser arbitrarias.

2 2
1 3 3
1
4 5 6 5
4 6
7 8 9
7 8 9

No cumple con las ecuaciones de


compatibilidad

Cumple con las ecuaciones de 1 2 3


compatibilidad 4 5 6
7 8 9

Universidad de Castilla- La Mancha Draft Por: Eduardo W. V. Chaves (2012)


Ciudad Real - Espaa
5 INTRODUCCIN A LAS ECUACIONES CONSTITUTIVAS Y PVCI 397

NOTA 2: Cuando utilizamos un mtodo numrico para la obtencin de la solucin, e.g.


mtodo de los elementos finitos, la forma de garantizar el cumplimiento de las ecuaciones
de compatibilidad es a travs de la continuidad del campo de desplazamientos. Con lo que
respecta la tcnica de los elementos finitos, al hacer el ensamblaje de los elementos finitos
(atamos los nodos) estamos de cierta forma garantizando que las ecuaciones de
compatibilidad se cumplan.
NOTA 3: Cuando el campo de desplazamiento no depende de una direccin, i.e.
r r
u = u( x1 , x 2 ) , las ecuaciones de compatibilidad reducen a:

2 11 2 22 2 12 Ecuacin de
S 33 = + 2 =0 (5.42)
x 22 x12 x1x 2 Compatibilidad 2D

ya que i 3 = 3i = 0 .
NOTE 4: Para mejor ilustracin de la condicin de compatibilidad vamos considerar un
ejemplo en dos dimensiones (2D), donde tenemos un campo escalar = ( x1 , x2 ) y que

conocemos las derivadas: = x1 + 3x2 y = x12 , podemos ver claramente que este
x1 x2
campo escalar es incompatible ya que
2 ( x1 + 3 x2 )
= x1 + 3 x2 = F1 = = =3
x1 x2 x1 x2 x1 x2
2 ( x12 )
= x12 = F2 = = = 2 x1
x2 x1 x2 x1x2 x2

El campo escalar = ( x1 , x2 ) ser compatible si y solo si:


= F1 ( x1 , x 2 )
x1 compatible sii F1 F2
= (5.43)
x2 x1
= F2 ( x1 , x2 )
x 2

Si consideramos el teorema de Green (ver Captulo 1 del libro de texto) que establece:
r r r r F F
F d = ( F) e 3 dS
F1 dx1 + F2 dx2 = 2 1 dS 3
r componentes
x
x1 x 2

r
x2 dS = dSe 3

x3
e3

x1

Figura 5.7: Teorema de Green.

Universidad de Castilla- La Mancha Draft Por: Eduardo W. V. Chaves (2012)


Ciudad Real - Espaa
398 PROBLEMAS RESUELTOS DE MECNICA DEL MEDIO CONTINUO

r r
y considerando tambin la ecuacin (5.43), podemos concluir que: si F = xr , es
r r r r r r r
compatible si y solo si F d = ( xr F) e 3dS = 0 xr F = 0 .

r r r r r
NOTA 5: Vamos considerar que F = ( xr ) a = a ( xr )T , donde es un campo
r r
tensorial de segundo orden y a es un vector arbitrario independiente de x (constante).
Notar tambin que las siguientes relaciones son vlidas:
r r r r r r
r r
[ r r
] r
[
(a) F d = ( xr ) a d = a ( xr )T d = a ( xr )T d ]

y

{ [( ]} { [a ( ]} dSr
r r r r r r r r r r
( xr F) dS = r
x
r
x ) a dS = r
x
r
x )T

(b)
{ } { } dS
r r r r r r r r
= a xr ( xr )T dS = a
T T




r
x ( xr )T

En notacin indicial
r r r r r r r
(a) Fi (d )i = ( xr )ij a j (d )i = a j ( xr )ij (d )i = a j ( xr )ij (d )i

(b)

[ [ ]]
r r r r r r
(

r
x
F) i (dS ) i = ijk Fk , j (dS )i = ijk a p ( xr )T

kp , j (dS ) i

r

r
[ r
] [ r
] r
= ijk a p , j ( xr )T kp + a p ( xr )T kp, j (dS ) i = ijk a p ( xr )T kp , j (dS ) i
{ [ ]
=0

[ ]
r
[ ]
r r r

T
= a p ijk ( xr ) kp, j (dS ) i = a p ijk psq qk ,s , j (dS ) i = a p ijk psq qk ,sj (dS )i

[ ]
r r r
= ap r
x ( xr )T ip (dS )i

{ } dS
r r r r
= a
T
r
x ( xr )T

Valdra la pena revisar el Ejemplo 1.108, donde hemos demostrado que la relacin
r r
( xr ) = ksq qp , s e k e p se cumple, luego ( xr )T = psq qk , s e k e p tambin. En el
r r
Ejemplo 1.108 hemos demostrado que xr ( xr )T = ipq tsj qj , ps e t e i , el cual es
r r
equivalente a xr ( xr )T = psq ijk qk ,sj e i e p .
Considerando el Teorema de Stokes (ver Captulo 1 del libro de texto) concluimos que:
r r r r r
F d = (

r
x F ) dS
14444244443


r
{ } dS
r r r r r r
a ( xr )T d = a xr ( xr )T
T




r
{ } dS
r r r r
( xr )T d =
T


xr ( xr )T

Universidad de Castilla- La Mancha Draft Por: Eduardo W. V. Chaves (2012)


Ciudad Real - Espaa
5 INTRODUCCIN A LAS ECUACIONES CONSTITUTIVAS Y PVCI 399

Luego, para un campo compatible se debe cumplir que:


{ }
r r T r r
r
x ( xr )T =0 xr ( xr )T = 0
r r
r x
Consideremos ahora que A = F a donde F es el gradiente de deformacin, F = r , y
X
aplicando el teorema de Stokes obtenemos que:
r r r r r r r r r r
A d = ( xr A ) dS
( F a) d = ( xr ( F a)) dS


1444444444444442444444444444443


r r r
{ } { } dS
r r r r r
a F T d = a dS d
T T
F
T
xr F = r
x F

r
Luego, el campo x ser compatible si y solo si:
{ }
r T r
r
x F =0 xr F = 0
Para mayores detalles acerca de las manipulaciones algebraicas ver Ejemplo 1.108.

Ejemplo 5.10
a) Demostrar que las ecuaciones fundamentales para un material elstico lineal e istropo
dadas por (5.24) pueden ser reemplazadas por seis ecuaciones y seis incgnitas ( ij ),
(formulacin en tensin), cuyas ecuaciones son:
2( + )
ij ,kk +
(2 + 3 )
kk ,ij
( 2 + 3 )
ll ,kk ij = 2 ( u
&& i ) , j [ ]
sym
[
2 ( b i ) , j ] sym
(5.44)

b) o por
2( + )
ij ,kk +
(2 + 3 )
kk ,ij =
( 2 + )
[
( b k ) , k ( u
&& k ) , k ij + 2 ( u]
&& i ) , j [ ]sym
[
2 ( b i ) , j ]
sym

(5.45)
E E
c) Considerando que = , = , expresar las ecuaciones (5.44) y
(1 + )(1 2 ) 2(1 + )
(5.45) en funcin ( E , ) .
r
Consejo: Las ecuaciones cinemticas = sym u pueden ser reemplazadas por
ij , kl + kl ,ij il , jk jk ,il = O ijkl (5.46)
Ver ejemplo Ejemplo 5.9.
Solucin:
a) Obtenemos la inversa de la ecuacin constitutiva en tensin ( = C e : ) para obtener:
1 1 1
Ce : = Ce : C e : = I sym : = sym = = Ce :
1
Recordar que el tensor C e para un material istropo viene dado por:
1 1
= Tr ( )1 indicial
ij = ij ss ij .
2 2 (2 + 3 ) 2 2 (2 + 3 )

Universidad de Castilla- La Mancha Draft Por: Eduardo W. V. Chaves (2012)


Ciudad Real - Espaa
400 PROBLEMAS RESUELTOS DE MECNICA DEL MEDIO CONTINUO

r
Si consideramos que las propiedades mecnicas no varan con x , i.e. ,i = 0i y
x i

,i = 0 i obtenemos que:
xi

2 ij 1 1
ij , kl = ij ss ij = ij ,kl ss , kl ij (5.47)
x k xl 2 2 (2 + 3 ) ,kl 2 2 (2 + 3 )

Adems, si multiplicamos la ecuacin (5.46) por jk obtenemos que:

ij , kl jk + kl ,ij jk il , jk jk jk ,il jk = O ijkl jk


(5.48)
ik ,kl + kl ,ik il , kk kk ,il = 0 il

Observar que, segn la ecuacin (5.47) se cumplen que:


1 1
ik ,kl = ik ,kl ss ,kl ik = ik ,kl ss ,il
2 2 (2 + 3 ) 2 2 (2 + 3 )
1 1
kl ,ik = kl ,ik ss ,ik kl = lk ,ki ss ,il
2 2 (2 + 3 ) 2 2 (2 + 3 )
1
il ,kk = il ,kk ss ,kk il
2 2 (2 + 3 )
1 1 3
kk ,il = kk ,il ss ,il kk = kk ,il ss ,il
2 2 (2 + 3 ) {
=3
2 2 ( 2 + 3 )
1 3 1 3 2
= ss ,il ss ,il = ss ,il = ss ,il
2 2 (2 + 3 ) 2 2 (2 + 3 ) 2 (2 + 3 )

2 ij 1
ij ,kl = ij ,kl ss ,kl ij
x k x l 2 2 (2 + 3 )
Con lo cual la ecuacin (5.48) queda:
ik ,kl + kl ,ik il ,kk kk ,il = 0 il
1 2 2
ik ,kl ss ,il + lk ,ki il , kk + ss ,kk il ss ,il = 0 il
2 (2 + 3 ) (2 + 3 ) (2 + 3 )
2 2
ik ,kl + ss ,il + lk , ki il ,kk + ss , kk il = 0 il
(2 + 3 ) (2 + 3 ) (2 + 3 )
2( + )
ik ,kl ss ,il + lk , ki il ,kk + ss , kk il = 0 il
(2 + 3 ) (2 + 3 )

2( + )
ss ,il il ,kk + ss ,kk il = ik , kl lk ,ki (5.49)
(2 + 3 ) (2 + 3 )

De las ecuaciones de movimiento ij , j + b i = u


&& i podemos obtener que:

ij , jk + ( b i ) , k = ( u
&& i ) ,k

Con lo cual se cumplen que:


ik ,kl + ( b i ) ,l = ( u
&& i ) ,l ik ,kl = ( b i ) ,l ( u
&& i ) ,l

Universidad de Castilla- La Mancha Draft Por: Eduardo W. V. Chaves (2012)


Ciudad Real - Espaa
5 INTRODUCCIN A LAS ECUACIONES CONSTITUTIVAS Y PVCI 401

lk , ki + ( b l ) ,i = ( u
&& l ) ,i lk ,ki = ( b l ) ,i ( u
&& l ) ,i .

Y notar que ik ,kl lk , ki = ( b i ) ,l ( u&& i ) ,l + ( b l ) ,i ( u&& l ) ,i = 2[( b i ) ,l ]sym 2[( u&& i ) ,l ]sym
Reemplazando la ecuacin anterior (5.49) obtenemos que:
2( + )

(2 + 3 )
ss ,il il ,kk +
(2 + 3 )
ss ,kk il = 2 ( b i ) ,l [ ]
sym
[
2 ( &u& i ) ,l ] sym

Reestructurando la ecuacin anterior y por hacer ( l = j ) obtenemos que:


2( + )
ij ,kk +
(2 + 3 )
kk ,ij
(2 + 3 )
ll ,kk ij = 2 ( u
&& i ) , j[ ]sym
[
2 ( b i ) , j ] sym

Cuya ecuacin es la misma que (5.44).


b) Partiendo de la ecuacin anterior obtenemos que:
2( + )
ij ,kk +
(2 + 3 )
kk ,ij =
(2 + 3 )
ll ,kk ij + 2 ( u [
&& i ) , j ]
sym
[
2 ( b i ) , j ]
sym
(5.50)

Nuestro objetivo ahora es obtener una expresin para ll, kk . Si multiplicamos la ecuacin
(5.46) por jk li obtenemos que:
ij , kl jk li + kl ,ij jk li il , jk jk li jk ,il jk li = O ijkl jk li
ij , ji + ji ,ij ii , jj jj ,ii = 2 ij ,ij 2 ii , jj = 0 (5.51)
ij ,ij ii , jj = 0
Si recurrimos a la ecuacin constitutiva inversa (ver ecuacin (5.47)), podemos decir que:
1 1
ij ,ij = ij ,ij ss ,ij ij = ij ,ij ss ,ii
2 2 (2 + 3 ) 2 2 (2 + 3 )
(5.52)
1 2
ii , kk = ii ,kk ss ,kk ii = ii , kk
2 2 (2 + 3 ) 2 (2 + 3 )
Con lo cual la ecuacin (5.51) queda:
ij ,ij ii , jj = 0
1 2
ij ,ij ss ,ii ii ,kk = 0
2 2 (2 + 3 ) 2 (2 + 3 )
2 (5.53)
ij ,ij + ii ,kk = 0
(2 + 3 ) (2 + 3 )
2 +
ij ,ij = ii , kk
(2 + 3 )
Si ahora recurrimos a las ecuaciones del movimiento ij , j + b i = u
&& i podemos obtener
que:
ij , ji + ( b i ) ,i = ( u
&& i ) ,i ij , ji = ( u
&& i ) ,i ( b i ) ,i

Con lo cual la ecuacin en (5.53) queda:

Universidad de Castilla- La Mancha Draft Por: Eduardo W. V. Chaves (2012)


Ciudad Real - Espaa
402 PROBLEMAS RESUELTOS DE MECNICA DEL MEDIO CONTINUO

2 +
ij ,ij = ii ,kk
(2 + 3 )
2 +
( u && i ) ,i ( b i ) ,i =
(2 + 3 ) ii ,kk (5.54)

(2 + 3 ) (2 + 3 )
ii ,kk = ll ,kk =
2 +
( u[
&& k ) ,k ( b k ) ,k =
2 +
] ( b k ) , k ( u
&& k ) , k[ ]
Reemplazando la ecuacin (5.54) en la ecuacin (5.50), obtenemos que:
2( + )
ij ,kk +
(2 + 3 )
kk ,ij =
(2 + 3 )
ll ,kk ij + 2 ( u
&& i ) , j[ ]
sym
[
2 ( b i ) , j ]
sym

2( + ) (2 + 3 )
ij , kk +
(2 + 3 )
kk ,ij =
(2 + 3 ) 2 +
( b k ) , k ( u [
&& k ) ,k ij + 2 ( u
&& i ) , j ] [ ] sym
[
2 ( b i ) , j ]
sym

2( + )
ij , kk +
(2 + 3 )
kk ,ij =
( 2 + )
[
( b k ) , k ( u
&& k ) , k ij + 2 ( u ]
&& i ) , j [ ] sym
[
2 ( b i ) , j ]
sym

(5.55)
Obteniendo as la ecuacin en (5.45)
c) Tras algunas manipulaciones algebraicas podemos obtener que:
1 (1 2)
=
(2 + 3 ) E
(1 2 ) E
= =
(2 + 3 ) E (1 + )(1 2) (1 + )
(1 2) E (1 2)
= =
(2 + 3 ) E 2(1 + ) 2(1 + )
2( + ) (1 2) 1
=2 +2 =
(2 + 3 ) (1 + ) 2(1 + ) (1 + )
E E E (1 )
(2 + ) = 2 + =
2(1 + ) (1 + )(1 2) (1 + )(1 2)
E (1 + )(1 2)
= =
(2 + ) (1 + )(1 2) E (1 ) (1 )
con lo cual la ecuacin (5.44):

ij ,kk +
1
(1 + )
kk ,ij

(1 + )
ll ,kk ij = 2 ( u [
&& i ) , j ]
sym
[
2 ( b i ) , j ]
sym
(5.56)

y (5.45) queda:

ij ,kk +
1
(1 + )
kk ,ij =

(1 )
[
( b k ) , k ( u
&& k ) ,k ij + 2 ( u ]
&& i ) , j [ ] sym
[
2 ( b i ) , j ]
sym
(5.57)

NOTA: Para un problema esttico la ecuacin anterior quedan:

ij ,kk +
1
(1 + )
kk ,ij =

(1 )
[ ] [
( b k ) , k ij 2 ( b i ) , j ]
sym
Ecuaciones de Michell (5.58)

Universidad de Castilla- La Mancha Draft Por: Eduardo W. V. Chaves (2012)


Ciudad Real - Espaa
5 INTRODUCCIN A LAS ECUACIONES CONSTITUTIVAS Y PVCI 403

que son conocidas como ecuaciones de Michell.


r
Si las fuerzas msicas no varan con x las ecuaciones de Michell se reducen a:
1
ij ,kk + kk ,ij = 0 Ecuaciones de Beltrami (5.59)
(1 + )

que son las conocidas ecuaciones de Beltrami.


&& k = 0 k ) , la ecuacin (5.54) queda:
NOTA 2: Para un problema esttico (u
(2 + 3 ) (1 + ) (1 + ) r
ll ,kk = ( b k ) , k = ( b k ) , k { [Tr ( )]} = ( b )
2 + (1 ) (1 )
r (5.60)
(1 + )
2 [Tr ( )] = ( b)
(1 )
La ecuacin anterior tambin puede ser obtenida a partir de la ecuacin (5.58) con ( i = j ),
i.e.:

ii ,kk +
1
(1 + )
kk ,ii =
(1 )
[ ] [
( b k ) , k ii 2 ( b i ) ,i
{
]
=3

1 3
1 +
(1 + )
ii ,kk = [
2 ( b k ) , k ]
(1 )
(5.61)
( 2 + ) (2 + )
ii ,kk = [
( b k ) ,k ]
(1 + ) (1 )
(1 + )
ii ,kk =
(1 )
[
( b k ) , k ]
Notar que ii ,kk = kk ,ii y ( b k ) ,k = ( b i ) ,i .

Ejemplo 5.11
a) Demostrar que:
Indicial notation Tensorial notation
, iijj = 0 (i, j = 1,2) { [ ( )]} = 0
2 2 = 0
(5.62)
4 4 4
4 +2 2 2 + 4 =0
x1 x1 x2 x2 4 = 0

donde
2 2 2
11 = ; 22 = ; 12 = 21 = (5.63)
x 22 x12 x1 x 2
Considerar un material elstico lineal, un problema esttico y sin fuerzas msicas.
Considerar tambin que el tensor de tensiones de Cauchy es dependiente nicamente de x1
y x 2 , i.e. = ( x1 , x 2 ) .
b) Demostrar si las ecuaciones de equilibrio se cumplen.
Solucin:
a) En el Ejemplo 5.10 (ver ecuacin (5.61)) hemos demostrado que:

Universidad de Castilla- La Mancha Draft Por: Eduardo W. V. Chaves (2012)


Ciudad Real - Espaa
404 PROBLEMAS RESUELTOS DE MECNICA DEL MEDIO CONTINUO

(1 + )
ii ,kk =
(1 )
[
( b k ) , k = 0 ]
donde hemos considerado que ( b k ) ,k = 0 . Para el problema propuesto tenemos que
i, k = 1,2 , con lo cual:
ii ,kk = 0
ii ,11 + ii , 22 = 0
11,11 + 22,11 + 11, 22 + 22, 22 = 0
2 11 2 22 2 11 2 22
+ + + =0
x12 x12 x 22 x 22
Utilizando la definicin (5.63), concluimos que:
2 11 2 22 2 11 2 22
+ + + =0
x12 x12 x 22 x 22
2 2 2 2 2 2 2 2
+ + + =0
x12 x 22 x12 x12 x 22 x 22 x 22 x12
4 4 4
+ 2 + =0
x14 x12 x 22 x 24
Q.E.D.
NOTA: En la literatura es conocida como funcin de tensin de Airy.
b) Para el caso bidimensional las ecuaciones de equilibrio (sin fuerzas msicas) se reducen a:
11 12
x + x = 0
1 2
ij , j = 0 i i1,1 + i 2, 2 = 0 i

21 + 22 = 0
x1 x 2
Utilizando la definicin (5.63), obtenemos que:
11 12 2 2
x + x = 0 =0
x1 x 2 x 2 x1 x 2
2
1 2


21 + 22 = 0
2
+ = 0
2

x1 x 2 x x x x 2 x12
1 1 2

Con lo cual demostramos que las expresiones de las tensiones dadas por (5.63) cumplen
con las ecuaciones de equilibrio.

Universidad de Castilla- La Mancha Draft Por: Eduardo W. V. Chaves (2012)


Ciudad Real - Espaa
5 INTRODUCCIN A LAS ECUACIONES CONSTITUTIVAS Y PVCI 405

Ejemplo 5.12
Considerando las ecuaciones de gobierno para un material elstico y lineal, obtener una
r
formulacin equivalente solamente en funcin de desplazamientos- u y tensiones-
(Formulacin Mixta). Utilizar la notacin de Voigt.
Solucin:
Teniendo en cuenta las ecuaciones de gobierno para un problema elstico y lineal:
Notacin tensorial Notacin de Voigt
Ecuaciones de Movimiento: Ecuaciones de Movimiento:
r r &r& (3 ecuaciones)
+ b = v& = u [L ] { } + { b } = { u&&} (3 ecuaciones)
(1) T

Ecuacin Constitutiva en Tensin: Ecuacin Constitutiva en Tensin:


(5.64)
( ) = C e : (6 ecuaciones) { } = [C ]{ } (6 ecuaciones)
Ecuaciones Cinemticas: Ecuaciones Cinemticas:
{ } = [L(1) ] {u } (6 ecuaciones)
r
= sym u (6 ecuaciones)

donde las ecuaciones de notacin de Voigt fueron obtenidas en el Ejemplo 5.7, con

0 0 0
x1 x 2 x 3
[L ]
(1) T
= 0


x 2
0

x1

x3
0


0
0 0
x3 x 2 x1

Para eliminar la deformacin reemplazamos las ecuaciones cinemticas en la ecuacin


constitutiva, resultando:
{ } = [C ]{ }
{ } = [C ] [L(1) ]{u }
[C ] { } = [1
1
C4]24
1
C ] [L(1) ]{u }
[3
=[1 ]
[C ] { } L
1
[ ]{u } = {0 }
(1)

Con lo cual quedamos con el siguiente sistema de ecuaciones:


[ ]
L(1) T { } + { b } = { u&&}
1 (1)
[ ]
[C ] { } L {u } = {0 }

Tambin podemos expresar como:


[0 ]
[L ] {u } = { b } + { u&&}
(1) T

[ ]
L
(1)
[C ]1 { } {0 }

NOTA: La formulacin anterior se conoce como Formulacin Mixta. Y es interesante


observar que en la formulacin en desplazamiento y en tensin obtenidas en los Ejemplo
5.8 y Ejemplo 5.10 respectivamente tenemos derivada segunda de la variable. En la
formulacin mixta tenemos solamente derivada primera de los desplazamientos y tensiones
y notar tambin que no involucra derivadas de los parmetros mecnicos del material.

Universidad de Castilla- La Mancha Draft Por: Eduardo W. V. Chaves (2012)


Ciudad Real - Espaa
406 PROBLEMAS RESUELTOS DE MECNICA DEL MEDIO CONTINUO

Ejemplo 5.13
Dados dos sistemas constituidos por el mismo material elstico lineal y con condiciones de
cargas distintas:
r
Sistema I Fuerzas de superficie - t *
r
Fuerzas msicas especfica- b
B S
Su
r r
r dV t * ( x) Campo de tensin -
u* r r
b( x ) Campo de deformacin -
r
n Campo de desplazamiento - u

r
Sistema II Fuerzas de superficie - t *
r
Fuerzas msicas especfica- b
Su B S
r r r
dV t * ( x) Campo de tensin -
u* r
b Campo de deformacin -
r
n Campo de desplazamiento - u

Figura 5.8: Dos sistemas bajo cargas externas.

Demostrar el Teorema de Betti:

: dV = : dV
V V
Teorema de Betti (5.65)

Solucin:
Teniendo en cuenta la ecuacin constitutiva de tensin, = C e : , en notacin indicial:
e
ij = C ijkl kl

Si a ambos lados de la ecuacin multiplicamos por el campo obtenemos que:


ij ij = ij C eijkl kl Simetra
Mayor de C
e
ij ij = ij C ijkl kl = kl C eklij ij
e
donde hemos aplicado la simetra mayor del tensor constitutivo elstico ( C ijkl = C eklij ).
Como los dos sistemas estn constituidos por el mismo material se cumple que = C e : .
Con lo cual la relacin anterior queda:
Notacin tensorial
ij ij = ij C eijkl kl = kl C eklij ij = kl kl : = :

Si ahora integramos sobre todo el volumen, obtenemos el teorema de Betti:

: dV = : dV
V V
(5.66)

Universidad de Castilla- La Mancha Draft Por: Eduardo W. V. Chaves (2012)


Ciudad Real - Espaa
5 INTRODUCCIN A LAS ECUACIONES CONSTITUTIVAS Y PVCI 407

e
NOTA 1: La ecuacin anterior solo se cumple si C ijkl = C eklij , es decir, si C e presenta
e
simetra mayor. En otras palabras, la condicin C ijkl = C eklij impone la existencia de una
funcin de energa almacenada, tal que:
e 2 e 2 e
C ijkl = = = C eklij
ij kl kl ij
NOTA 2: El teorema de Betti (Teorema de la Reciprocidad) es el punto de partida para la
obtencin de la formulacin de la tcnica de los Elementos de Contorno.
NOTA 3: El teorema de Betti tambin puede ser expresada de otra forma que
demostramos a continuacin.
1 u u j 1
Recordar que ij = i + = (u i , j + u j ,i ) , con eso para el sistema II tambin se
2 x j xi 2
1
cumple que ij = ( ui , j + u j ,i ) . Con lo cual:
2


V
ij ij dV
= ij ij dV
V

1 1
2V
ij (u i , j + u j ,i )dV =
2V
ij ( ui , j + u j ,i )dV (5.67)

u
V
ij i , j dV
= ij ui , j dV
V

donde ij u i , j = ij u j ,i y ij ui , j = ij u j ,i se cumplen debido la simetra de y de ,


respectivamente. Y adems notar que:
( ij u i ), j = ij , j u i + ij u i , j ij u i , j = ( ij u i ), j ij , j u i
( ij ui ), j = ij , j ui + ij ui , j ij ui , j = ( ij ui ), j ij , j ui

Con lo cual la ecuacin (5.67) queda:

u
V
ij i , j dV
V

= ij ui , j dV

( u ),
V
ij i j
ij , j u i dV = ( ij ui ), j ij , j ui dV
V
(5.68)

( u ),
V
ij i j
V

dV ij , j u i dV = ( ij ui ), j dV ij , j ui dV
V

V

Aplicando el teorema de la divergencia a las primeras integrales de cada lada de la ecuacin,


obtenemos que:

u n dS
S
ij i j
V
ij , j u i dV
S

= ij ui n j dS ij , j ui dV
V
(5.69)

t i u i dS ij , j u i dV = t i ui dS ij , j ui dV
S V

S V

r r
donde hemos aplicado la definicin n = t y n = t . La ecuacin anterior en notacin
tensorial queda:
r r r r r r
t udS ( ) udV = t u dS ( ) u dV
(5.70)
S V S V

Universidad de Castilla- La Mancha Draft Por: Eduardo W. V. Chaves (2012)


Ciudad Real - Espaa
408 PROBLEMAS RESUELTOS DE MECNICA DEL MEDIO CONTINUO

Si recurrimos a las ecuaciones de movimiento se cumple que:


r r r r r r
&r& && &r&
+ b = u = (b u ) y + b = u = (b u
&&)

Con lo cual la ecuacin (5.70) queda:


r r r r r r r r r r
&&
t udS + (b u ) udV = t u dS + (b u

&&) u dV
Teorema de Betti (5.71)
S V S V

Notar que, si consideramos S = S u + S podemos decir que:


r r r r r r
t udS = t * udS + t u* dS

S S Su
r r r r r r (5.72)
t u dS = t * u dS + t u * dS

S S Su

Para el caso particular cuando el sistema est en equilibrio y en la ausencia de fuerzas


msicas, la ecuacin (5.71) queda:
r r r r
t udS = t u dS
(5.73)
S S

Si adems las fuerzas de superficies son fuerzas concentradas, podemos decir que:
r r r r
Fi loc u loc
i = Filoc uiloc F loc u loc = F loc u loc (5.74)

Ejemplo 5.14
Considerando el problema planteado en la Figura 5.8, demostrar el Principio del Trabajo
Virtual:
r r r r r
t * u dS + (b u

&&) u dV = : dV

S
1444442444443 V V
14243 Principio del Trabajo Virtual (5.75)
Trabajo externo virtual total Trabajo interno
virtual total

r r
con u = u* en S u .
Solucin:
Partimos directamente de la relacin:
1

V
ij ij dV =
2V
ij ( ui , j + u j ,i )dV = ij ui , j dV
V
(5.76)

Notar que ( ij ui ), j = ij , j ui + ij ui , j ij ui , j = ( ij ui ), j ij , j ui , luego:


V
ij ij dV
= ij ui , j dV = ( ij ui ), j ij , j ui dV
V V
(5.77)

ij ij dV = ( ij ui ), j dV ij , j ui dV
V V

V

Aplicando el teorema de la divergencia a la primera integral del lado derecho, obtenemos


que:

Universidad de Castilla- La Mancha Draft Por: Eduardo W. V. Chaves (2012)


Ciudad Real - Espaa
5 INTRODUCCIN A LAS ECUACIONES CONSTITUTIVAS Y PVCI 409


V
ij ij dV
V

V

= ( ij ui ), j dV ij , j ui dV = ij ui n j dS ij , j ui dV
S

V
(5.78)

= t *i ui dS ij , j ui dV
S V
r
donde hemos aplicado la definicin n = t * . La ecuacin anterior en notacin tensorial
viene dada por:
r r r
: dV = t * u dS ( ) u dV
(5.79)
V S V
r r r r
Notar que + b = u&& = (b u
&&) (ecuaciones de movimiento), con lo
cual, la ecuacin (5.79) queda:
r r r r r
t * u dS + (b u
&&) u dV = : dV

S V V
14243
144444244444
3
Total external virtual work Total internal
virtual work

que es conocido como el Principio del Trabajo Virtual. Observar que no hemos tenido que
recurrir la simetra mayor de C e para su demostracin.
Para el caso particular cuando el sistema est en equilibrio y en la ausencia de fuerzas
msicas, la ecuacin anterior queda:
r r
t * u dS = : dV
(5.80)
S V

Si adems las fuerzas de superficies son fuerzas concentradas, podemos decir que:
Notacin Tensorial Notacin de Voigt
r
{F } {u }= { } { } dV
r (5.81)
F loc u loc = : dV

loc T loc T

V V

donde {F loc }= {F1 , F2 ,..., Fn }T , {u loc }= {U1 , U2 ,..., Un }T , y la direccin de la componente uiloc
es igual a direccin de la componente Filoc .

F1 F2
F3
u
u

, ,

REAL VIRTUAL

NOTA 1: En otras palabras, el Principio del trabajo Virtual afirma que: Una estructura
est en equilibrio, bajo la accin de un sistema de fuerzas exteriores, si y solo si el trabajo
externo virtual total es igual al trabajo interno virtual total, cualquier que sea el campo de
r
desplazamiento virtual ( u ).

Universidad de Castilla- La Mancha Draft Por: Eduardo W. V. Chaves (2012)


Ciudad Real - Espaa
410 PROBLEMAS RESUELTOS DE MECNICA DEL MEDIO CONTINUO

NOTA 2: El Principio del Trabajo Virtual se utiliza en las tcnicas de discretizacin del
problema como por ejemplo la Tcnica de los Elementos Finitos, en el cual la incgnita
fundamental es el campo de desplazamiento.
NOTA 3: Es de fcil demostracin que la ecuacin (5.75) tambin es vlida para las tasas
& r
de los campos virtuales u , & , i.e.:
r r& r r r&
t * u dS + (b u

&&) u dV = : & dV

S
1 44444V2444443 V
14243 Principio del trabajo virtual (5.82)
Trabajo externo virtual total Trabajo interno
virtual total

r
Tambin es valida para una variacin de los campos u , , i.e.:
r r r r r
t * u dS + (b u

&&) u dV = : dV

S
1444444V2444444
3 V
14243 Principio del trabajo virtual (5.83)
Trabajo externo virtual total Trabajo interno
virtual total

NOTA 4: Podemos definir tambin el Principio del trabajo virtual complementario donde
las incgnitas fundamentales son tensiones (fuerzas):
r r &r& r
t u* dSur + (b u ) udV =
: dV Principio del trabajo virtual
Sur
1444442444443 V V
14243 (5.84)
Total internal
complementario
Total external complementary virtual work
complementary virtual work

r
con n = t * en S .

Ejemplo 5.15
Consideremos un sub-dominio caracterizado por un material homogneo, elstico linear e
istropo. Consideremos tambin que en unos puntos del contorno del sub-dominio hay
unas fuerzas concentradas aplicadas {F ( e ) } {F loc } y que el campo de desplazamiento es
aproximado a travs de {u( x )} = [N ( x )]{u ( e ) } donde {u (e ) } {u loc } son los desplazamientos
r r

en los puntos donde se aplican las fuerzas concentradas. Demostrar que las ecuaciones
fundamentales para un problema elstico linear e istropo pueden ser reemplazadas por:

{F }= [K ]{u }
(e) (e) (e)
donde [K ] = [B] [C ][B] dV
(e) T
(5.85)
V

donde [C ] es la matriz constitutiva elstica en notacin de Voigt, y obtener la expresin de


[B( xr )] .
Nota: Usar el Principio del Trabajo Virtual.
Solucin:
Podemos partir directamente de la ecuacin (5.81), el cual es equivalente a:

Universidad de Castilla- La Mancha Draft Por: Eduardo W. V. Chaves (2012)


Ciudad Real - Espaa
5 INTRODUCCIN A LAS ECUACIONES CONSTITUTIVAS Y PVCI 411

r r r
F loc u loc = : dV = : ( sym u ) dV

V V
r r r (5.86)
u loc F loc = ( sym u ) : dV

V

La ecuacin anterior en notacin de Voigt queda:


r r
{ } {F }= { } { } dV
r
u loc F loc = ( sym u ) : dV Voigt
T

T
u ( e ) (e)
(5.87)
V V

Notar que en la ecuacin anterior ya est teniendo en cuenta la ecuacin de equilibrio (ver
ecuaciones (5.79)-(5.81)). La ecuacin constitutiva de tensin, en notacin de Voigt, viene dada
r r
por { ( x )} = [C ]{ ( x )} , donde el campo del tensor de deformacin viene dado por
r r
( x ) = sym u (Ecuaciones cinemticas). Tenemos que hacer la representacin de la parte
simtrica del gradiente del campo de desplazamiento ij = 12 (u i , j + u j ,i ) en notacin de
Voigt. En el Ejemplo 5.7 hemos demostrado que:
u1
0 0
x1
x1
u 0
11
2
0
x 2 x 2

22 u 3 0
0 u1
33
r x3
{ ( x )} = = u u =
x3
u 2 { ( xr )} = [L(1) ]{u( xr )}
2 12 1 + 2 0 u
2 23 x 2 x1 x 2 x1 3

u u
2 13 2 + 3 0
x 3 x 2
x3 x 2
u1 u 3
x + x x 0
3 1 3 x1

Luego
{ ( xr )} = [L(1) ]{u( xr )} = [L(1) ][N ( xr )]{u (e) }= [B( xr )] {u (e) }
donde definimos que:
[B( xr )] = [L(1) ][N ( xr )] (5.88)
Observar que {u (e ) } no depende de x , y el campo de tensin se puede obtener por:
r

{ ( xr )} = [C ]{ ( xr )} = [C ][B( xr )]{u (e ) }
Podemos utilizar la misma aproximacin del campo de desplazamiento para aproximar el
campo de desplazamiento virtual, con lo cual tambin se cumple que:
{u ( xr )} = [N ( xr )]{u (e ) } { ( xr )} = [B( xr )] {u (e) }
Luego, la ecuacin (5.87) queda:

{u } {F }= { } { } dV = {[B( xr )]{u }} [C ][B( xr )]{u }dV


(e) T (e) T (e) T (e)

V V

o an:

Universidad de Castilla- La Mancha Draft Por: Eduardo W. V. Chaves (2012)


Ciudad Real - Espaa
412 PROBLEMAS RESUELTOS DE MECNICA DEL MEDIO CONTINUO

{u } {F }= {u } [B( xr )] [C ][B( xr )]{u }dV


(e) T (e) (e ) T T (e)
(5.89)
V

Observar que ni {u (e ) } ni {u (e ) } son dependientes de x , con lo cual:


r


{u } {F }= {u } [B( xr )] [C ][B( xr )] dV {u }
(e) T (e) (e) T T (e)

V
(5.90)
{ }
r T r
F ( e ) = [B ( x )] [C ][B ( x )] dV u ( e )

{ }
V
{F } = [K ]{u }
(e) (e) (e)

[ ]
NOTA: K (e ) es conocida como matriz de rigidez del sub-dominio (elemento finito), y la
matriz [N ( x )] de la relacin {u( x )} = [N ( x )]{u ( e ) } se conoce como la matriz que contiene
r r r

las funciones de forma. Las funciones de forma son las funciones definidas en un dominio que
r
nos permite obtener el valor de una funcin incgnita {u( x )} en cualquier punto del
dominio a travs de los valores nodales conocidos {u (e ) } de la funcin incgnita. Poe
ejemplo, supongamos que el sub-dominio ( ) viene caracterizado por un cuadriltero,
luego podemos obtener {u( x )} a travs de sus valores nodales {u (e ) } como sigue:
r

U(e ) _ 2
U(e ) _ 1
U (e ) _ 3
U (e ) _ 1 (e ) _ 2
r U
{u( xr )} {u( xr )} = [N ( xr )]{u(e) }
= [N ( x )] (e ) _ 3
U
(e ) _ 4
U
U(e ) _ 4

Ejemplo 5.16
Si consideramos la energa potencial elstica total dada por:
r 1 r r r r
(u) = : dV t * udS ( b) udV
Energa Potencial Total (5.91)
V
2 S V

donde
1
U int = 2 : dV =
e
( ) dV (5.92)
V V

y
r r r r
U ext = t * udS + ( b) udV
(5.93)
S V

Consideremos tambin que la primera variacin de , denotada por , es igual a cero


para un valor estacionario de . Demostrar que, si = 0 es equivalente al valor
r
estacionario de , entonce (u) corresponde a un valor mnimo.

Universidad de Castilla- La Mancha Draft Por: Eduardo W. V. Chaves (2012)


Ciudad Real - Espaa
5 INTRODUCCIN A LAS ECUACIONES CONSTITUTIVAS Y PVCI 413

r r
NOTA: Considerar que durante el proceso de deformacin las acciones externas ( t * , b )
no varan con la deformada. Considerar tambin un material elstico lineal.
Solucin:
La primera variacin ( ) podemos obtener como sigue:
1 r r r r
= : dV t * udS ( b) udV

2
V S V
1 r r r r
= : dV t * udS ( b) udV
(5.94)
V
2 S V

1 r r r r
= ( : ) dV t * udS ( b) udV

V
2 S V

Notar que:

e ( ) =
1
2
1 1
[
( : ) = ( : + : ) = (C e : ) : + :
2 2
]
1
[ 1
2
] [ 1
= (C e : ) : + : = : C e : + : = [ : + : ]
2 2
] (5.95)
= :
e
= :

e
donde hemos considerado que = (ver Ejemplo 5.5). Para rgimen de pequeas

deformaciones, la ecuacin anterior tambin puede ser escrita como:
e r r r
e ( ) = : = : = : ( symu) = : ( sym u) = : (u) (5.96)

donde hemos utilizado la propiedad A sym : B = A sym : (B sym + B skew ) = A sym : B sym , Con lo
cual la ecuacin (5.94) queda:
1 r r r r
= ( : ) dV t * udS ( b) udV

V
2 S V
r* r r r
= : dV t udS ( b) udV

V S V
r r r r
= e dV t * udS ( b) udV

V S V
r r
La expresin (u + u) puede ser obtenida como
r r r r r r r r
(u + u) = e ( + ) dV t * (u + u)dS ( b) (u + u)dV
(5.97)
V S V

Utilizamos serie de Taylor para aproximar el trmino e ( + ) , resultando:


e ( ) 1 2 e ( )
e ( + ) = e () + : + : : + ... (5.98)
2

Universidad de Castilla- La Mancha Draft Por: Eduardo W. V. Chaves (2012)


Ciudad Real - Espaa
414 PROBLEMAS RESUELTOS DE MECNICA DEL MEDIO CONTINUO

e ( ) 2 e ( )
Notar que : = e (ver ecuacin (5.95)) y C e = (ver Ejemplo 5.5).

Con lo cual la ecuacin (5.98) queda:
e ( ) 1 2 e ( )
e ( + ) = e () + : + : : + ...
2
1
e ( ) + e + : C e :
2
Con lo cual la ecuacin (5.97) puede ser reescrita como:
r r r r r r r r
(u + u) = e ( + ) dV t * (u + u)dS ( b) (u + u)dV

V S V

1 r r r r r r

= e ( ) dV + e dV + : C e : dV t * (u + u)dS ( b) (u + u)dV

V V V
2 S V
r r r r
= e ( ) dV t * udS ( b) udV +

V S V
r r r r 1
+ e dV t * udS ( b) udV +

: C e : dV
V S V V
2

Notar que:
r r r r r
(u) = e ( ) dV t * udS ( b) udV

V S V

y
r r r r
= e dV t * udS ( b) udV = 0

V S V

Con lo cual:
r r r 1
(u + u) = (u) + +
V
2
: C e : dV

r r r 1
(u + u) (u) = +
V
2
: C e : dV

r r r 1
(u + u) (u) =
V
2
: C e : dV

donde hemos considerado que = 0 . Notar que el trmino : C e : > 0 siempre ser
positivo para cualquier valor de ya que C e es un tensor definido positivo (ver Captulo
1). Con lo cual garantizamos que:
r r r 1 r r r
= (u + u) (u) =
V
2
: C e : dV > 0 (u + u) > (u)
r
= 0 (u) es un mnimo
NOTA 1: Para un sistema constituido un material elstico linear el punto de equilibrio
corresponde al valor mnimo de . Este es el conocido Principio de la Energa Potencial
Mnima.
NOTA 2: Para el caso particular donde las acciones externas vienen constituida por fuerzas
concentradas y en ausencia de fuerzas msicas, tenemos que:

Universidad de Castilla- La Mancha Draft Por: Eduardo W. V. Chaves (2012)


Ciudad Real - Espaa
5 INTRODUCCIN A LAS ECUACIONES CONSTITUTIVAS Y PVCI 415

r
(u) = U int + U ext =
1
2
: dV F loc { } {u } T loc
Energa Potencial Total (5.99)
V

(u )
F


(u ( 2 ) )

(u )
= 0
=0
(u ( 3 ) ) u
Deformada correspondiente al
equilibrio

u (2) u u ( 3)

NOTA 3: A travs de la ecuacin (5.99) podemos demostrar el Teorema de Castigliano- Parte


I:
r
(u) U int U ext U int
{ }{ }
F loc uloc
T

= + = =0
{ } {u } {u } {u }
uloc loc loc loc
{ }
u loc

{F } =
int
U loc

{u } loc

donde {F }= {F , F ,..., F } , {u }= {U , U ,..., U } . Notar que en la expresin anterior el


loc
1 2 n
T loc
1 2 n
T

trmino U tiene que estar en funcin de {u } .


int loc

NOTA 4: Para una mejor ilustracin del problema planteado, vamos considerar una barra
de longitud L y con rea de la seccin transversal A . Consideremos tambin que el campo
de tensin y de deformacin son campos homogneos y dados por:
0 0 0 0
ij = 0 0 0 ; ij = 0 0 0 11 = C1111
e
11 = E
0 0 0 0 0 0

Consideremos tambin que el campo de desplazamiento viene representado por una


aproximacin lineal ( u ( x) = a1 + a2 x ) y en las extremidades de la barra tenemos que:

Universidad de Castilla- La Mancha Draft Por: Eduardo W. V. Chaves (2012)


Ciudad Real - Espaa
416 PROBLEMAS RESUELTOS DE MECNICA DEL MEDIO CONTINUO

A
V = AL (volumen)
(3D )


F = dA
A
(1) (1) ( 2) ( 2)
F ,U F ,U
(1D )
1 2 x

El objetivo ahora es escribir el energa potencial total en funcin de U(1) ,U( 2) . Verificar que,
debido a las cargas concentradas, tenemos que:
U(1)
U ext = F loc { } {u }= {F
T loc (1)
}
F ( 2 ) ( 2) = F (1)U(1) + F ( 2 )U( 2) (5.100)
U
En este caso la relacin lineal tensin-deformacin viene dada por = E (ver Ejemplo
1 1
5.5 NOTA 3) y la densidad de energa e = = E . Luego, la energa interna total
2 2
viene dada por:
1 1 1 1
2 : dV
1D
U int = U int = dV = E dV = E 2 dV
V
2V 2V 2V

u1 u ( x)
notar que 11 = = = , con lo cual:
x1 x
2
1 1 u ( x)

int
U = E 2 dV = E dV (5.101)
2V 2 V x

Nuestro objetivo ahora es expresar el campo de desplazamiento en funcin de sus valores


nodales U(1) ,U( 2) . Para ello vamos utilizar la aproximacin adoptada u ( x) = a1 + a2 x , donde:
u ( x = 0) = U(1) = a1 U(1) 1 0 a1
( 2) =
u ( x = L ) = U( 2 ) = a1 + a2 L U 1 L a2

Ahora nuestro objetivo es definir los coeficientes a1 , a 2 . Para ello, obtenemos la inversa
de la relacin anterior:
a1 = U (1)
U (1) 1 0 a1 inversa a1 1 L 0 U (1)
(2) = =

U 1 L a 2
( 2)
a 2 L 1 1 U

(
1 ( 2)
a 2 = U U
L
(1)
)
con lo cual:

Universidad de Castilla- La Mancha Draft Por: Eduardo W. V. Chaves (2012)


Ciudad Real - Espaa
5 INTRODUCCIN A LAS ECUACIONES CONSTITUTIVAS Y PVCI 417

u ( x) = a1 + a2 x = U(1) + (
1 (2)
L
x x
)
U U(1) x = 1 U(1) + U( 2 )
L L
(5.102)
x x U
(1)
r
u ( x) = 1 ( 2 ) = [N ( x )] u(e ) { }
L L U
Con lo cual la ecuacin (5.101) queda:
2 2

( )
1 u ( x) E 1 (2) (1) E
U U dV = 2 U 2U(1)U( 2 ) + U(1) dV
( 2) 2 2
U int = E
2 V x
dV =
2VL 2L V
Notar que U(1) y U( 2) no dependen de x , con lo cual:
E (2) 2 E
2U(1)U( 2 ) + U(1) dV = 2 U( 2 ) 2U(1)U( 2 ) + U(1) V
2 2 2
U int =
2L
U
2
V
2L
(5.103)
EAL ( 2) 2 EA ( 2 ) 2
2U(1)U( 2 ) + U(1) = 2U(1)U( 2 ) + U(1)
2 2
= 2
U U
2L 2L
Luego, la energa potencial total (ver ecuacin (5.99)) viene dada por las ecuaciones (5.100)
y (5.103), i.e.:
r
(u) = U int U ext =
EA ( 2 ) 2
U
2L
2


( )
2U(1)U( 2 ) + U(1) F (1)U(1) + F ( 2 )U( 2 ) = (U(1) , U( 2 ) )

Como buscamos un estado estacionario, hay que cumplir que:


(U(1) , U( 2 ) )

U(1)
=
EA ( 2 ) 2
(1)
u 2 L
(
U 2U(1)U( 2 ) + U(1) F (1)U(1) + F ( 2 )U( 2) = 0
2
)


=
EA
2L
( )
2U( 2) + 2U(1) F (1) = 0


=
L
(
EA (1)
)
U U( 2 ) F (1) = 0

( )
(1) ( 2)
(U , U ) EA ( 2 ) 2
= ( 2)

U 2U(1)U( 2 ) + U(1) F (1)U(1) + F ( 2 )U( 2 ) = 0
2
U( 2 ) U 2 L



=
EA
2L
( )
2U( 2 ) 2U(1) F ( 2 ) = 0


=
L
(
EA ( 2 )
)
U U(1) F ( 2 ) = 0

Reestructurando las ecuaciones anteriores en forma matricial obtenemos que:


EA 1 1 U F
(1) (1)

=

L 1 1 U( 2 ) F ( 2)
[ ]{ } { }
K ( e ) u( e ) = F ( e )

Notar que K (e ) [ ] no tiene inversa ya que det[K ]


(e)
= 0 . Para que el problema tenga
solucin nica, tenemos que introducir las condiciones de contorno.
[ ]
Notar que la matriz K (e ) de la expresin anterior podra haber sido obtenida a travs de
la ecuacin (5.90) (ver Ejemplo 5.15), donde para este caso particular tenemos que
[C ] = E , y a travs de la ecuacin (5.102) podemos obtener que:

Universidad de Castilla- La Mancha Draft Por: Eduardo W. V. Chaves (2012)


Ciudad Real - Espaa
418 PROBLEMAS RESUELTOS DE MECNICA DEL MEDIO CONTINUO

[B( xr )] = [L(1) ][N ( xr )] =



x x 1 1
1 =
x L L L L
Luego:
1

[K ]
(e ) r T r
= [B ( x )] [C ][B ( x )] dV =
L 1 1

1 1 1
E dV = E 2
1 L L dV
L 1 1
V
V
V

L
1 1
E E 1 1 EA 1 1

=
L2
V L

1 1 dV = 2 1 1 V =
L 1 1
r
NOTA 5: Analizando [N ( x )]
T
r r r x x
Note que las funciones de forma son [N ( x )] = [N1 ( x ) N 2 ( x )]T = 1 . Si
L L
dibujamos estas funciones dentro del dominio obtenemos que:

N1 x N1 ( x = 0) = 1
N2 N1 ( x ) = 1
L N1 ( x = L ) = 0
x N 2 ( x = 0) = 0
N 2 ( x) =
1 L N 2 ( x = L) = 1
1

x N1 ( x ) + N 2 ( x ) = 1
1 2
L

r
La aproximacin adoptada para [N ( x )] depender del problema. Para el problema
analizado en este ejemplo tenemos que la deformacin es constante dentro del dominio,
luego, es suficiente adoptar una aproximacin linear para el campo de desplazamientos ya
u ( x)
que por definicin = . Como consecuencia necesitamos apenas dos nodos para
x
r
definir [N ( x )] . Si el problema requiere una funcin cbica para el campo de
r
desplazamientos, necesitaremos tres puntos para definir [N ( x )] , y as sucesivamente.

NOTA 5:
Principio del Potencial de Energa Estacionario
En este ejemplo hemos establecido el Principio de la Energa Potencial Estacionaria, (ver
ecuacin (5.91)):
r r r r r
(u) = e ( ) dV t * udS ( b) udV
(5.104)
V S V

Universidad de Castilla- La Mancha Draft Por: Eduardo W. V. Chaves (2012)


Ciudad Real - Espaa
5 INTRODUCCIN A LAS ECUACIONES CONSTITUTIVAS Y PVCI 419

1
donde hemos tenido en cuenta que e ( ) = : . El funcional es estacionario si y solo si
2
r
ur (u) = 0 .

Principio Variacional de Hellinger-Reissner


En el Ejemplo 5.5 en la NOTA 7 hemos establecido que
e () = e () tensorial
e ( ) = : e ( ) = 0 G( ) = g( )
(5.105)
e ( ) = : g( )

donde g( ) es la densidad de energa libre de Gibbs con el signo invertido.

g() - Energa almacenada complementaria

e () - Energa almacenada


Figura 5.9: Energas almacenadas.

Reemplazando el valor e ( ) = : g( ) en el funcional (5.104) obtenemos que:


r r r r r
(u) = e ( ) dV t * udS ( b) udV

V S V
r r r r r (5.106)
HR (u, ) = : g( ) dV t * udS ( b) udV

V S V
r r
Notar que : = : ( sym u) = : (u) . Con lo cual podemos obtener que:
r r r r r r
HR (u, ) = : (u) g( ) dV t * udS ( b) udV
(5.107)
V S V

r
El Funcional (5.106) es estacionario para una variacin arbitraria de u y desapareciendo en
S ur si y solo si satisface las ecuaciones de equilibrio. Y es estacionario para una variacin
de si y solo si satisface la ecuacin constitutiva (tensin-deformacin). Este principio es
conocido como Principio Variacional de Hellinger-Reissner.

Universidad de Castilla- La Mancha Draft Por: Eduardo W. V. Chaves (2012)


Ciudad Real - Espaa
420 PROBLEMAS RESUELTOS DE MECNICA DEL MEDIO CONTINUO

r r r r r r
ur HR (u, ) = : (u) dV t * udS ( b) udV = 0

V S V
r

= ij (u) i , j dV t *i (u) i dS ( b) i (u) i dV = 0
V S V
r (5.108)

= ij (u) i n j dS ij , j (u) i dV t *i (u) i dS ( b) i (u) i dV = 0
S V S V

[ ] [ ]
r
= ij , j + ( b) i (u) i dV + t *i (u) i dS = 0
ij n j
V S

En el volumen obtenemos las ecuaciones de equilibrio: ij , j + ( b) i = 0 i .

En la superficie S la condicin de contorno: ij n j t *i = 0 i


r r

HR (u, ) = : ( sym u) g( ) dV = 0
V
r g( )
V

= ( sym u) :

: dV = 0 (5.109)

r g( )
V

= ( sym u)

: dV = 0

r g( )
En el volumen obtenemos las ecuaciones constitutivas: ( sym u) =0.

Principio Variacional de Hu-Washizu


El Principio de Hu-Washizu es una generalizacin del Principio de Hellinger-Reissner,
r
donde el funcional adems de depender de (u, ) tambin depende del campo de
deformacin , y viene dado por:

[ () : ( ]
r r r r r
sym r r r
HW (u, , ) = e
u) ( b) u dV ( n ) (u* u)dS t * udS

V S ur S

(5.110)
y estacionario para las siguientes situaciones:
r
ur HW (u, , ) = 0 Ecuaciones de equilibrio
r r
u = 0 En S ur

r Ecuaciones cinemticas
HW (u, , ) = 0
Condicin de contorno en S ur
r
HW (u, , ) = 0 Ecuaciones constitutivas tensin - deformacin

Es decir:
r
ur HW (u, , ) = 0

Universidad de Castilla- La Mancha Draft Por: Eduardo W. V. Chaves (2012)


Ciudad Real - Espaa
5 INTRODUCCIN A LAS ECUACIONES CONSTITUTIVAS Y PVCI 421

r
ur HW (u, , ) = [ : ( ur ) ( br ) ur ] dV tr ur dS
sym *

V S

(u, , ) = [( ) (u) ( b) u] dV t udS


r r r r r r *
ur HW
V S

(u, , ) = [( ) ( b)] (u) dV t udS


r r r r r *
ur HW
V S

r
HW (u, , ) = 0

[ ]
r r r r
HW (u, , ) = : ( sym u) dV ( n ) (u* u)dS = 0

V S ur

[ ] [n (u ]
r r r* r
HW (u, , ) = ( sym u) : dV u) : dS = 0
V S ur

r
HW (u, , ) = 0

[ ]
r
HW (u, , ) = e ( ) : ( ) dV = 0
V

r e ( ) e ( )
HW (u, , ) =
V
: : ( ) dV =
V


: dV = 0

Universidad de Castilla- La Mancha Draft Por: Eduardo W. V. Chaves (2012)


Ciudad Real - Espaa
422 PROBLEMAS RESUELTOS DE MECNICA DEL MEDIO CONTINUO

Ejemplo 5.17
Demostrar que:

t
r* r r& r r
[
( X , t ) u dS 0 + 0 b( X , t ) u

r
0 ]X
r
&r&( Xr , t ) u& dV = P : r u& dV
0 (5.111)
S V0 V0

r
donde u es un campo virtual de desplazamientos, y P es el primer tensor de tensiones de
Piola-Kirchhoff.

configuracin
configuracin de F actual
referencia S 0
S
V0
V
B0 dV0 r r r
t * ( X , t ) = t *0 B r r
r r dV t * ( x, t )
u( X , t )
r r
r r r u( x , t )
0 b( X , t ) = 0 b 0 r r
b( x , t )

Solucin:
r r r r
Aunque las variables t * ( X , t ) y b( X , t ) no sean variables intrnsecas de la configuracin de
referencia como las variables 0 , S 0 , V0 entre otras, por simplicidad vamos denotar
r r r r r r
t * ( X , t ) = t *0 y b( X , t ) = b 0 y para el campo de desplazamiento Lagrangeano.
Recordemos tambin que (ver Captulo 2 del libro de texto):
r r r
D x ( X , t ) x ( X , t ) u& i ( X , t) r
Fij F&ij = i
= i
= = u& i , J ( X , t )
Dt t X j X j 142 t 43 X j
x&i

r r
F& = l F = Xr u& ( X , t )
r r
y = F& F 1 = Xr u& ( X , t ) F 1
l
r r r r
F& 1 = F 1 l = F 1 Xr u& ( X , t ) F 1 = F 1 xr u& ( x , t )

Teniendo en cuenta las relaciones anteriores, tambin se cumple para un campo virtual de
desplazamiento:
r& r r& r r& r
F& = Xr u ( X , t ) y F& 1 = F 1 Xr u ( X , t ) F 1 = F 1 xr u ( x , t )

Con lo cual podemos decir que:


& r
P : F dV = P 0 iJ F&iJ dV0 = PiJ u& i , J ( X , t ) dV0

V0 V0 V0

(PiJ u& i ) , J = PiJ , J u& i + PiJ u& i , J PiJ u& i , J = (PiJ u& i ) , J PiJ , J u& i
con lo cual:

Universidad de Castilla- La Mancha Draft Por: Eduardo W. V. Chaves (2012)


Ciudad Real - Espaa
5 INTRODUCCIN A LAS ECUACIONES CONSTITUTIVAS Y PVCI 423

r r r
P : F& dV0 = PiJ u& i , J ( X , t ) dV0 = (PiJ u& i ( X , t )) , J PiJ , J u& i ( X , t ) dV0

V0 V0 V0

& r r
P : F dV = (P
V0
0
V0
iJ
u& i ( X , t )) , J dV0 PiJ , J u& i ( X , t ) dV0
V0

& r r
P : F dV = P
V0
0
S0
iJ
V0

u& i ( X , t )n J dS 0 PiJ , J u& i ( X , t ) dV0

donde hemos aplicado el teorema de la divergencia. En notacin tensorial queda:

r r
P : F dV = (P n ) u& ( X , t ) dS (
& r P) u& i ( X , t ) dV0
0 i 0 X
V0 S0 V0

Recordar que la ecuacin de movimiento en la configuracin de referencia viene dada por:


r
&r&( Xr , t )
Xr P + 0 b 0 = 0 u
r
[
&r&( Xr , t )
Xr P = 0 b 0 u ]
r& r
y teniendo en cuenta que donde por definicin se cumplen que F& = Xr u ( X , t ) y
r
t *0 = P n obtenemos:
r r
P : F& dV0 = (P n ) u& i ( X , t ) dS 0 ( Xr P ) u& i ( X , t ) dV0

V0 S0 V0

P : r
X
r& r r &r& r
u ( X , t ) dV0 = t *0 u
r
&r&( Xr , t ) u& ( Xr , t ) dV
( X , t ) dS 0 + 0 b 0 u i 0 [ ]
V0 S0 V0

Recordatorio: Recordemos del captulo 4 del libro de texto que la potencial tensional
puede ser expresada de distintas formas, a saber:
1 1
V

w int (t ) = P : F& dV0 = S : E& dV0 = P : F& dV0 =
V V
2 V

S : C& dV0 =
V
J
P : F& dV
0 0 0 0


=
V 0
P : F& dV = : D dV = {
J : D dV0 = : D dV0
V V0


V0

y D=l sym
[ r r sym
= xr u& ( x , t) ]
(ver Ejemplo 2.37)
NOTA 1: Recordar que ni P ni F& estn en ninguna configuracin, pero el escalar P : F&
si que estn en la configuracin de referencia.
NOTA 2: Teniendo en cuenta lo anterior. El trabajo interno virtual total tambin se puede
expresar como:
r& r sym r& r r& r
: D dV = : xr u ( x , t ) dV = : xr u ( x , t ) dV = P : F& dV0 = P : Xr u ( X , t ) dV0


V V V V V 0 0

r& r sym r& r


Note que, debido a la simetra de se cumple que : xr u ( x , t ) = : xr u ( x , t ) .

NOTA 3: La ecuacin (5.111) tambin es vlida para una variacin del campo virtual:


r r r r r
t * ( X , t ) u dS 0 + 0 b( X , t ) u
[ r
0 X
r
&r&( Xr , t ) u dV = P : r u dV
0 ] (5.112)
S V0 V0

Universidad de Castilla- La Mancha Draft Por: Eduardo W. V. Chaves (2012)


Ciudad Real - Espaa
424 PROBLEMAS RESUELTOS DE MECNICA DEL MEDIO CONTINUO

Universidad de Castilla- La Mancha Draft Por: Eduardo W. V. Chaves (2012)


Ciudad Real - Espaa
7 Elasticidad Lineal
7.1 Ejercicios Resueltos

Ejemplo 7.1
El cilindro indefinido de la Figura 7.1 constituido por un material elstico lineal istropo,
est sometido al siguiente estado de deformacin (en coordenadas cilndricas):
err = e = a sin
a cos
e r = (7.1)
2
e zz = ez = erz = 0

con eij son las componentes del Tensor de Almansi. Se pide:


r
Calcular el vector traccin t en el contorno, en coordenadas cilndricas.
Hiptesis: 1) , son las constantes de Lam; 2) Rgimen de pequeas deformaciones.

x3

e z
e

t e r

x2
r

x1

Figura 7.1.
426 PROBLEMAS RESUELTOS DE MECNICA DEL MEDIO CONTINUO

Solucin:
Rgimen de pequeas deformaciones: e E
a cos
a sin 0
rr r rz 2
a cos
( r , , z ) = r z = a sin 0 (7.2)
2
rz z zz 0 0 0

= Tr ( )1 + 2 (7.3)
Tr ( ) = 2a sin (7.4)
luego,
a cos
a sin 2
0
1 0 0 a cos
= 2a sin 0 1 0 + 2 a sin 0 (7.5)
2
0 0 1 0 0 0

2a sin + 2 a sin a cos 0



(r,r, = a cos 2a sin + 2 a sin 0
(7.6)
0 0 2a sin
r
El vector traccin t :
r
t (n) = n (7.7)
n = (1,0,0)

t 1(n ) 2a sin + 2 a sin


(n )
t 2 = a cos (7.8)
t (n ) 0
3

Ejemplo 7.2
El paraleleppedo de la Figura 7.2 se deforma de la manera indicada por las lneas de trazo.
Los desplazamientos vienen dados por las siguientes relaciones:
u = C1 xyz ; v = C 2 xyz ; = C 3 xyz (7.9)
Se pide:
a) Determinar el estado de deformacin en el punto E , cuando las coordenadas del
punto E en el cuerpo deformado son E (1,503; 1,001; 1,997) ;
b) Determinar la deformacin normal en E en la direccin de la lnea EA ;
c) Calcular la distorsin angular en E del ngulo recto formado por las lneas EA y
EF .
d) Determinar el incremento de volumen y la deformacin volumtrica media.

Universidad de Castilla- La Mancha Draft Por: Eduardo W. V. Chaves (2012)


Ciudad Real - Espaa
7 ELASTICIDAD LINEAL 427

G G F F

E
D D 2m

C C
O O
y

1,5m
A A
B B
1m
x

Figura 7.2
Solucin:
a) El estado de deformacin en funcin de los desplazamientos es:
1 u i u j

ij = + (7.10)
2 x j x i

Explcitamente en notacin ingenieril:
u 1 v u 1 u
1 1 + +
x xy xz
x 2 x y 2 x z
2 2
1 1 1 v u v 1 v
ij = xy y yz = + + (7.11)
2 2 2 x y y 2 y z
1 1
yz z 1 u 1 v
2 xz 2 x + z +
2 y z
2
z
Para determinar el estado de deformacin en cualquier punto necesitamos a priori
determinar el campo de desplazamientos.
Determinacin de las constantes:
Reemplazando los valores dados para el punto E (1,5; 1,0; 2,0) , resulta:
u ( E ) = 1,503 1,5 = C1 (1,5)(1,0)(2,0) C1 = 0,001
0,001
v ( E ) = 1,001 1,0 = C 2 (1,5)(1,0)(2,0) C 2 = (7.12)
3
w(E ) = 1,997 2,0 = C 3 (1,5)(1,0)(2,0) C 3 = 0,001

donde el desplazamiento del punto E : u ( E ) = (1,503 1,5; 1,001 1,0; 1,997 2,0) fue
utilizado.
Podemos escribir el Campo de Desplazamiento:

Universidad de Castilla- La Mancha Draft Por: Eduardo W. V. Chaves (2012)


Ciudad Real - Espaa
428 PROBLEMAS RESUELTOS DE MECNICA DEL MEDIO CONTINUO

Notacin Ingenieril Notacin Cientfica


u = 0,001 xyz u1 = 0,001 X 1 X 2 X 3
0,001 0,001 (7.13)
v= xyz u2 = X1 X 2 X 3
3 3
w = 0,001 xyz u 3 = 0,001 X 1 X 2 X 3

u
x = = 0,001 yz = 0,002 = 11
x
v 0,001
y = = xz = 0,001 = 22
y 3
w
z = = 0,001xy = 0,0015 = 33
z
v u 0,001 0,011
xy = + = yz + 0,001xz = = 2 12
x y 3 3
w u
xz = + = 0,001yz + 0,001xy = 0,0005 = 213
x z
w v 0,001
yz = + = 0,001xz + xy = 0,0025 = 2 23
y z 3
El campo de deformacin queda:

yz
1 yz
+ xz
1
(xy yz )
2 3 2
1 yz xz 1 xy
ij = 0,001 + xz xz
2 3 3 2 3

1
(xy yz ) 1 xy
xz xy
2 2 3

Estado de deformacin en el punto E ( x = 1,5; y = 1,0; z = 2,0) :

1 1 0,011


0,00025
x xy xz 0,002
2 2 6
1 1 0,011
ij = xy y
yz = 0,00125
E 0,001
(7.14)
2 2 6
1 1
2 xz yz z 0,00025 0,00125 0,0015
2
b) Como visto la componente normal:
M
M = M componente
s M = ij M i M j (7.15)
Expandiendo la expresin anterior y considerando la simetra del tensor de deformacin:
M = 11 M 12 + 22 M 22 + 33 M 32 + 2 12 M 1 M 2 + 2 13 M 1 M 3 + 2 23 M 2 M 3 (7.16)
en notacin Ingenieril:
M = x M 12 + y M 22 + z M 32 + xy M 1 M 2 + xz M 1 M 3 + yz M 2 M 3 (7.17)
La normal ser dada por los cosenos directores de la direccin de la lnea EA :

Universidad de Castilla- La Mancha Draft Por: Eduardo W. V. Chaves (2012)


Ciudad Real - Espaa
7 ELASTICIDAD LINEAL 429

1 2
M 1 = 0 ; M 2 = ; M 3 = (7.18)
5 5
Reemplazando los correspondientes valores en la ecuacin (7.17), resulta:
M = y M 22 + z M 32 + yz M 2 M 3
1 4 2
M = 0,001 + (0,0015) + (0,0025) (7.19)
5 5 5
3
M = 2 10
c) Para el caso de pequeas deformaciones, la distorsin en E del ngulo recto formado
por las lneas EA y EF , = 90 ), ser:

1 1 2M N
M N = M N = N
=M componente
s M N = ij M i N j (7.20)
2 2 sin

Para mayores detalles de la expresin anterior ver Captulo 2- Cinemtica del Continuo
(pequeas deformaciones). Expandiendo la expresin anterior y considerando la simetra
del tensor de deformacin:
(
M N = 11 M 1 N 1 + 22 M 2 N 2 + 33 M 3 N 3 + 12 M 1 N 2 + M 2 N 1 + )
(7.21)
( ) (
+ 13 M 1 N 3 + M 3 N 1 + 23 M 2 N 3 + M 3 N 2 )
o en notacin ingenieril:
M N
2
= x M 1 N 1 + y M 2 N 2 + z M 3 N 3 +
xy
2
(M 1 )
N 2 + M 2 N 1 +
(7.22)

(
+ xz M 1 N 3 + M 3 N 1 +
2
yz
2
) (
M 2 N 3 + M 3 N 2 )
donde:
1 2
M i = 0 ; N i = [ 1 0 0] (7.23)
5 5
resultando:
M N 0,011 1 2
= 12 M 2 N 1 + 13 M 3 N 1 = (1) + (0,00025)(1)
2 6 5 5
M N (7.24)
= 5,96284793998 10 4
2
M N = 1,1925696 10 3

Solucin Alternativa
Podemos crea una base ortonormal asociada a los versores M y N a travs del producto
vectorial P = M N . Luego, obtenemos as las componentes del versor P :
e 1 e 2 e 3
1 2 2 1 2 1
P = M
N = 0 = e2 e3 Pi = 0 (7.25)
5 5 5 5 5 5
1 0 0

Universidad de Castilla- La Mancha Draft Por: Eduardo W. V. Chaves (2012)


Ciudad Real - Espaa
430 PROBLEMAS RESUELTOS DE MECNICA DEL MEDIO CONTINUO

Luego, la matriz de transformacin del sistema X 1 X 2 X 3 para la base M , N , P viene


dada por:
1 2
M 1 M 2 M 3 0 5 5


A = a ij = N 1 N 2 N 3 = 1 0 0 (7.26)
P 2 1
1 P
2 P3 0
5 5

Aplicando la ley de transformacin de las componentes de un tensor de segundo orden, es
decir, ij = a ik a jl kl o en forma matricial = A A T :

0,011 T
1 2 0,002 0,00025 1 2
0 6
0
5 5 0,011 5 5
= 1 0 0 0,001 0,00125 1 0 0 (7.27)
2 1 6 2 1
0 0,00025 0,00125 0,0015 0
5 5 5 5


Resultando:
M N
M M N =
2


2 10 3 5,96284794 10 4 2,5 10 4

3
ij = 5,96284794 10 4
2 10 3 1,75158658 10 (7.28)
4
2,5 10 1,75158658 10 3 1,5 10 3

NOTA: Observar que no se trata de un caso de deformacin homognea, es decir, las


aristas que en la configuracin inicial son rectas, en la configuracin deformada no mas
sern rectas. Para obtener los versores deformados tenemos que aplicar la transformacin
lineal m = F M y n = F N , donde F es el gradiente de deformacin.
( dV )
d) Deformacin volumtrica por definicin es: V = donde dV es un diferencial de
dV
volumen
Caso de pequeas deformaciones:
(dV )
V = = x + y + z (dV ) = x + y + z dV (7.29)
dV
integrando podemos obtener el incremento de volumen:
2, 0 1 1, 5

( ) xz
V =
V
x + y + z dV = 0,001 yz +
z =0 y =0 x =0
3
xy dxdydz

(7.30)

resultando:

V = 1,125 10 3 m 3 (7.31)

Universidad de Castilla- La Mancha Draft Por: Eduardo W. V. Chaves (2012)


Ciudad Real - Espaa
7 ELASTICIDAD LINEAL 431

Luego:
( dV ) 1,125 10 3
V = = = 0,375 10 3 (7.32)
dV 1,5 1,0 2,0

Ejemplo 7.3
El estado de tensiones en un punto de una estructura que est constituida por un material
elstico, lineal e istrropo, viene dado por:
6 2 0
ij = 2 3 0 MPa
0 0 0

a) Determinar las componentes del tensor de deformacin ingenieril. Considrese que el


mdulo de elasticidad longitudinal ( E = 207GPa ) y el mdulo de elasticidad transversal
( G = 80GPa )
b) Si un cubo de 5cm de lado est sometido a este estado tensional. Cual ser su cambio
de volumen?
Solucin:
Las deformaciones pueden ser obtenidas partiendo de las siguientes relaciones:

x =
1
E
[ ( )]
x y + z = 3,333 10 5

1
[ ]
y = y ( x + z ) = 2,318 10 5
E
1
[ ( )]
z = z x + y = 4,348 10 6
E
(7.33)
1
xy = xy = 2,5 10 5
G
1
xy = xy = 0
G
1
xy = xy = 0
G
donde el coeficiente de Poisson puede ser obtenido partiendo de la relacin:
E E 207
G= = 1= 1 0,29375
2(1 + ) 2G 160
Luego:
33,24 12,5 0

ij = 12,5 23,01 0 10 6
0 0 4,257

Solucin Alternativa
1 (1 + )
En el libro texto (Chaves(2007)) hemos demostrado que C e = I 1 1 , con
E E
eso obtenemos que:

Universidad de Castilla- La Mancha Draft Por: Eduardo W. V. Chaves (2012)


Ciudad Real - Espaa
432 PROBLEMAS RESUELTOS DE MECNICA DEL MEDIO CONTINUO

1 (1 + ) (1 + ) (1 + )
= Ce : = I 1 1 : = I : 1 1: = Tr ( )1
E E E E E E
En componentes:
11 12 13 1 0 0
(1 + )
ij = 12 22 23 Tr ( ) 0 1 0

E E
13 23 33 0 0 1
6 2 0 1 0 0 33,24 12,5 0
6 6
= 6,251 10 2 3 0 4,2609 10 0 1 0 = 12,5 23,01 0 10 6
0 0 0 0 0 1 0 0 4,257

(1 + ) 1
where we have used = 6,25 10 6 , Tr ( ) = 4,25725 10 6 .
E MPa E
En el rgimen de pequeas deformaciones la deformacin volumtrica (lineal) es igual a la
traza del tensor de deformacin:
DVL V = I = (33,24 23,01 4,257 ) 10 6 = 5,973 10 6
Luego, la variacin de volumen queda:
V = V V0 = 5,973 10 6 (5 5 5) = 7,466 10 4 cm 3

Ejemplo 7.4
Un paraleleppedo de dimensiones a = 3cm , b = 3cm , c = 4cm , constituido por un material
homogneo elstico y lineal se aloja en una cavidad de la misma forma y dimensiones,
cuyas paredes son de un material lo suficientemente rgido para poderlo suponer
indeformable. Sobre la abertura de la cavidad de dimensiones a b y a travs de una placa
rgida de peso y rozamiento despreciables se aplica, perpendicularmente a ella, una fuerza
F = 200 N que comprime al bloque elstico. Si el coeficiente de Poisson es = 0,3 y el
mdulo de elasticidad E = 2 10 4 N / cm 2 , calcular:
a) Las fuerzas laterales ejercidas por las paredes de la cavidad sobre el paraleleppedo;
b) La variacin de altura experimentada por el mismo.

F
z

y
c

b
Figura 7.3

Universidad de Castilla- La Mancha Draft Por: Eduardo W. V. Chaves (2012)


Ciudad Real - Espaa
7 ELASTICIDAD LINEAL 433

Solucin:
En cualquier punto del cuerpo elstico habrn slo tensiones normales, x , y y z . La
tensin z viene dada por:
200 200 200 N
z = = = (7.34)
ab 3 3 9 cm 2
Observemos que debido a la simetra las tensiones x y y sern iguales, luego:

x = y =
1
E
[ (
x y + z = 0 )]
1
E
[ x ( x + z )] = 0
x ( x + z ) = 0 (7.35)
z
x =
(1 )

obteniendo as:
z 0,3 200 200 N
x = = = (7.36)
(1 ) (1 0,3) 9 21 cm 2

La fuerza que ejerce la pared sobre el cuerpo elstico viene dado por:
200
Fy = y a c = 3 4 = 114,28 N
21
(7.37)
200
Fx = x b c = 3 4 = 114,28 N
21
La deformacin z viene dada por:

z =
1
E
[ (
z x + y )]
1
= [ z 2 x ] (7.38)
E
1 200 200
= 4
+ 2 0,3 = 8,25 10 4
2 10 9 21

Luego, la variacin de altura viene dada por:


c = z c = 8,25 10 4 4 = 0,0033cm (7.39)

Ejemplo 7.5
En la Figura 1a se representa un dispositivo de apoyo en fajan de una mquina. Dicho
aparato de apoyo est constituido por un bloque de neopreno de dimensiones ( 50 20cm ),
representado en la Figura 7.4b por el elemento ABCD.

Universidad de Castilla- La Mancha Draft Por: Eduardo W. V. Chaves (2012)


Ciudad Real - Espaa
434 PROBLEMAS RESUELTOS DE MECNICA DEL MEDIO CONTINUO

a) b)
1,1
1,2
D
D C
C 1 20
A A B B
1 1,1 x
50

Dimensiones en centmetros - cm

Figura 7.4

Bajo accin de cargas vertical y horizontal que transmite la mquina al apoyo, el bloque de
neopreno se deforma como se indica en la Figura 1b (ABCD) y an se puede considerar
que el campo de desplazamiento ( u, v) est dado por unas ecuaciones lineales del tipo:
u = a1 x + b1 y + c1
v = a2 x + b2 y + c2
donde a1 , b1 , c1 , a 2 , b2 , c 2 son constantes a determinar.
Hiptesis:
1 Material elstico lineal istropo con el Mdulo de elasticidad longitudinal igual a
1
1000 N / cm 2 y el mdulo de elasticidad transversal igual a N / cm 2 .
0,0028
2 Se supondr que se trata de un estado plano de deformacin.
Se pide:
a) Calcular las componentes del tensor de deformacin y la deformacin volumtrica
en cualquier punto;
b) Calcular las tensiones en cualquier punto;
c) Mxima tensin normal;
d) Determinar el Alargamiento unitario en la direccin de la diagonal AC .

Solucin:
u = a1 x + b1 y + c1
(7.40)
v = a 2 x + b2 y + c 2
Segn Figura 7.4 sacamos que:
u (0;0) = 1 = c1
u (50;0) = 1,1 = 50a1 + 1 a1 = 0,002 (7.41)
u (0;20) = 1,1 = 20b1 + 1 b1 = 0,005

luego

Universidad de Castilla- La Mancha Draft Por: Eduardo W. V. Chaves (2012)


Ciudad Real - Espaa
7 ELASTICIDAD LINEAL 435

u = 0,002 x + 0,005 y + 1 (7.42)


Para desplazamiento vertical:
v(0;0) = 0 = c 2
u (50;0) = 0 = 50a 2 a 2 = 0 (7.43)
u (0;20) = 1 = 20b2 b2 = 0,05
v = 0,05 y (7.44)
Luego:
u = 0,002 x + 0,005 y + 1
(7.45)
v = 0,05 y
a) Deformaciones
u v u v
x = = 0,002 ; y = = 0,05 ; xy = + = 0,005 (7.46)
x y y x

Deformacin volumtrica lineal (pequeas deformaciones):


DVL = V = x + y + z = I = 0,048 (7.47)
b) Tensiones
E E
G= = 1 = 0,4
2(1 + ) 2G

x =
E
(1 + )(1 2v)
[
(1 ) x + y ]
= 3571,4286 [(0,6) 0,002 0,4 0,05] = 67,1428

y =
E
(1 + )(1 2v)
[
(1 ) y + x ]
(7.48)
= 3571,4286 [(0,6) (0,05) + 0,4 0,002] = 104,2857
xy = G xy
1
= 0,005 = 1,785714
0,0028
ETr ( ) E
Una solucin alternativa es utilizar: ij = ij + ij , donde:
(1 + )(1 2 ) (1 + )

x 1
2
xy 1
2
xz 0,002 1
2
(0,005) 0

ij = 12 xy y 1
2
yz = 12 (0,005) 0,05 0
1 xz 1
yz z 0 0 0
2 2

ETr ( ) N E N
= 68,571429 2 , = 714,285714 2
(1 + )(1 2) cm (1 + ) cm

Universidad de Castilla- La Mancha Draft Por: Eduardo W. V. Chaves (2012)


Ciudad Real - Espaa
436 PROBLEMAS RESUELTOS DE MECNICA DEL MEDIO CONTINUO

1 0 0 0,002 1
2
(0,005) 0

ij = 68,5714290 1 0 + 714,285714 12 (0,005) 0,05 0
0 0 1 0 0 0

67,1428 1,785714 0
N
1,785714 104,2857 0
cm 2
0 0 68,571

c) Tensiones principales
2
x + y x y
(1,2) = + 2xy

(7.49)
2 2
2
67,1428 104,2857 67,1428 + 104,2857 2
(1,2) = + 5,35714 =
2 2 (7.50)
= 171,4285 19,328675
1 = 152,099824 N cm 2
2 (7.51)
2 = 190,757175 N cm
d) Alargamiento unitario
La diagonal ( AC ) mide inicialmente:

L0 = AC = 50 2 + 20 2 = 53,852cm (7.52)
Diagonal deformada

AC = 50,2 2 + 19 2 = 53,675cm (7.53)


L = AC AC = 0,177cm (7.54)
El alargamiento unitario es:
L 0,177
= = = 0,0033 (7.55)
L0 53,852

Ejemplo 7.6
Considrese una barra de seccin cuadrada de lado a y longitud L . Las constantes
elsticas del material se suponen conocidas ( E y = 0,25 ). Se pide:
a) Para el caso de carga de la Figura 7.5(a), calcular la energa almacenada (densidad de
energa de deformacin) en la barra durante la deformacin y la energa total de
deformacin;
b) Determinar la energa almacenada en el cambio de volumen y la correspondiente al
cambio de forma;
c) Mismo apartado a) para el caso de la Figura 7.5(b).

Universidad de Castilla- La Mancha Draft Por: Eduardo W. V. Chaves (2012)


Ciudad Real - Espaa
7 ELASTICIDAD LINEAL 437

M
M
P P

L a
a
b) M
a) a
seccin

Figura 7.5:
Solucin:
Considerando un caso unidimensional:
x
x = E x x = (7.56)
E
Sabemos que la energa de deformacin por unidad de volumen viene dada por:
1 1 1 x 1 P2
e = : unidimensi
onal e = x x = x
= (7.57)
2 2 2 E 2 EA 2
Luego, la energa total U viene dada por:
P2
ex (volumen) = L A
2 EA 2
(7.58)
P2L
U =
2 EA
La energa de deformacin (por unidad de volumen) tambin puede ser expresada por:
1 1
e = I 2 II dev
6(3 + 2 ) 2 (7.59)
144244 3 1424 3
e
vol e
forma

Considerando:
x 0 0
P
ij = 0 0 0 I = x = (7.60)
A
0 0 0

Clculo de II dev :
2
1 I2
II dev = (3 II I 2 ) = = x (7.61)
3 3 3
Luego, la energa de deformacin asociada al cambio de volumen:
1 (1 2) 2 (1 2) 2
e vol = I 2 = I = x (7.62)
6(3 + 2 ) 6E 6E

(1 2 ) P 2
e vol = (por unidad de volumen) (7.63)
6E A 2

Universidad de Castilla- La Mancha Draft Por: Eduardo W. V. Chaves (2012)


Ciudad Real - Espaa
438 PROBLEMAS RESUELTOS DE MECNICA DEL MEDIO CONTINUO

Energa de deformacin asociada al cambio de forma:


1 1 2(1 + )
e forma = II dev = II dev
2 2 E
(7.64)
(1 + ) x
2
=
E 3
2
(1 + ) x (1 + ) P 2
e forma = = (por unidad de volumen) (7.65)
E 3 3E A 2
Comprobacin:
(1 2 ) P 2 (1 + ) P 2
e vol + e forma = + =
6E A 2 3E A 2
P2
= [(1 2) + 2(1 + )] =
6 EA 2
P2
= [1 2 + 2 + 2] =
6 EA 2
P2
= = e
2 EA 2
En el caso de slo a flexin, y adems teniendo las siguientes relaciones:
M y a4
y = I=
I 12
12 M y
y =
a4
y
y = E y y =
E
Energa de deformacin:

1 1 12 M y y
e = y y = 4

2 2 a E
1 12 M y 12 M y
= (7.66)
2 a4 Ea 4
72 M 2 y 2
=
Ea 8
72 M 2 y 2
e = (7.67)
Ea 8

Ejemplo 7.7
Bajo la restriccin de la teora de pequeas deformaciones, para un campo de
desplazamientos dado por:
r
u = ( x1 x3 ) 2 10 3 e 1 + ( x 2 + x3 ) 2 10 3 e 2 x1 x 2 10 3 e 3

Universidad de Castilla- La Mancha Draft Por: Eduardo W. V. Chaves (2012)


Ciudad Real - Espaa
7 ELASTICIDAD LINEAL 439

a) Determinar el tensor de deformacin infinitesimal, el tensor de rotacin


infinitesimal en el punto P (0,2 1) ;
Solucin:
u1 u1 u1

x1 x 2 x3 2( x x )
1 3 0 2( x1 x3 )
u i u 2 u 2 u 2
= = 0 2( x 2 + x 3 ) 2( x 2 + x3 ) 10 3
x j x1 x 2 x3

u 3 u 3 u 3 x 2 x1 0
x1 x 2 x3
Aplicado al punto P(0,2 1)
2( x1 x 3 ) 0 2( x1 x 3 ) 2 0 2
u i
= 0 2( x 2 + x3 ) 2( x 2 + x 3 ) = 0 2 2 10 3
x j
P x 2 x1 0 2 0 0

u i
= ij + ij
x j
cuyas componentes:
Tensor de deformacin infinitesimal Tensor spin infinitesimal
2 0 2 0 0 0
1 u u j
= 0 2 1 10 3 ij = 1 u i
u j
= 0 0 1 10 3
(7.68)
ij = i +
2 x j xi 2 x j xi
2 1 0 0 1 0

Ejemplo 7.8
Bajo la restriccin de la teora de pequeas deformaciones, para un campo de
desplazamientos dado por:
r
u = a ( x12 5 x 22 ) e 1 + (2 a x1 x 2 )e 2 (0) e 3
a) Determinar el tensor de deformacin lineal, el tensor de rotacin lineal;
b) Obtener las deformaciones principales y las tensiones principales;
c) Dado el mdulo de elasticidad transversal G , qu valor toma el mdulo de Young E
para que haya equilibrio en cualquier punto?
NOTA: Las fuerzas msicas son despreciables.
Solucin:
a) Considerando que u1 = a ( x12 5 x 22 ) , u 2 = 2 a x1 x 2 , u 3 = 0 , las componentes del
gradiente de los desplazamientos son:
2 x1a 10ax2 0
r u
( xr u) ij = i
= 2ax2 2ax1 0
x j
0 0 0

Descomponiendo de forma aditiva el gradiente de los desplazamientos en una parte


simtrica (tensor de deformacin lineal - ij ) y en una parte antisimtrica (tensor spin
infinitesimal- ij ):

Universidad de Castilla- La Mancha Draft Por: Eduardo W. V. Chaves (2012)


Ciudad Real - Espaa
440 PROBLEMAS RESUELTOS DE MECNICA DEL MEDIO CONTINUO

u i
= ij + ij
x j
donde
2x a 10ax2 0 2 x1a 2ax2 0
1 u u j 1 1
ij = i + = 2ax2 2ax1 0 + 10ax2 2ax1 0
2 x j xi 2

0 0 0 0 0 0
2 x1a 4ax2 0
= 4ax2 2ax1 0
0 0 0
y
2x a 10ax 2 0 2 x1 a 2ax 2 0 0 6ax 2 0
1 u u j 1 1
ij = i = 2ax 2 2ax1 0 10ax 2 2ax1 0 = 6ax 2 0 0
2 x j xi 2

0 0 0 0 0 0 0 0 0

b) Deformaciones principales.
2 x1a 4ax2
=0
4ax2 2ax1
(2 x1a ) (4ax2 ) = 0
2 2

(2 x1a ) = (4ax2 )
2 2

1 = 2 x1a + 4ax2
2 x1a = 4ax2
2 = 2 x1a 4ax2
2 x1a + 4ax2 0 0

ij = 0 2 x1a 4ax2 0 (deformaciones principales)
0 0 0
Ya que las deformaciones y tensiones comparten el mismo espacio principal podemos
utilizar la relacin ij = 4 x1a ij + 2 ij en el espacio principal:
1 0 0 2 x1a + 4ax2 0 0

ij = 4 x1a ij + 2 ij = 4 x1a 0 1 0 + 2 0 2 x1a 4ax2 0
0 0 1 0 0 0
4 x1a + 2 (2 x1a + 4ax2 ) 0 0
= 0 4 x1a + 2 ( 2 x1a 4ax2 ) 0
0 0 4 x1a
c) Partiendo de la ecuacin de equilibrio:
r r
+ {
b = 0 Indicial
ij , j = 0 i
r
=0

Expandiendo:

Universidad de Castilla- La Mancha Draft Por: Eduardo W. V. Chaves (2012)


Ciudad Real - Espaa
7 ELASTICIDAD LINEAL 441

11 12 13
+ + =0
x x x
11,1 + 12, 2 + 13,3 = 0 1 2 3
21 22 23
21,1 + 22, 2 + 23,3 = 0 + + =0
x1 x 2 x 3
31,1 + 32, 2 + 33,3 = 0 31 32 33
+ + =0
x1 x 2 x3

donde las componentes del tensor de tensiones son ij = kk ij + 2 ij , siendo kk = 4 x1 a


luego ij = 4 x1 a ij + 2 ij
11 = 4 x1 a 11 + 2 11 = 4 x1 a + 2 (2 x1 a ) = 4 x1 a ( + )
12 = 4 x1 a 12 + 2 12 = 2 (4ax 2 ) = 8 ax 2
13 = 0

1 0 0 11 12 13
ij = 4 x1a ij
+ 2 ij = 4 x1a 0 1 0 + 2 12 22 23

0 0 1 13 23 33
1 0 0 2 x1a 4ax2 0 4 x1a ( + ) 8 ax2 0
= 4 x1a 0
1 0 + 2 4ax2 2ax1 0 = 8 ax2 4 x1a ( + ) 0
0 0 1 0 0 0 0 0 4 x1a
Luego, la primera ecuacin de equilibrio queda:
11 12 13
+ + =0
x1 x 2 x3
4a( + ) 8 a = 0
+ = 2
= = G
G (3 + 2G )
Adems teniendo en cuenta la relacin E = , dicha relacin se puede obtener a
+G
E E
travs de las relaciones = , =G = . Luego, concluimos que:
(1 + )(1 2) 2(1 + )
G (3 + 2G ) G (3G + 2G )
E= = = 2,5G
+G G+G

Ejemplo 7.9
En un punto de un suelo que podemos considerar como un slido elstico lineal se conoce
la deformacin volumtrica V = 2 10 3 , la deformacin tangencial 12 = 3 10 3 y la
deformacin horizontal que es nula 11 = 0 . El suelo est sometido a un estado de
deformacin plana en el plano x1 x 2 . Se pide:
a) Componentes cartesianas del tensor de deformacin. Obtener las deformaciones
principales y la orientacin de las mismas, definiendo el ngulo que forman con lo
sejes ( x1 , x 2 , x 3 ) .

Universidad de Castilla- La Mancha Draft Por: Eduardo W. V. Chaves (2012)


Ciudad Real - Espaa
442 PROBLEMAS RESUELTOS DE MECNICA DEL MEDIO CONTINUO

1
b) Suponiendo que las constante elsticas son E = 50MPa , = , obtener las
4
componentes del tensor de tensiones y sus valores principales. Obtener asimismo
las direcciones en las que las tensiones normales y tangenciales son mximas o
mnimas y sus valores.
c) Obtener la densidad de energa elstica de deformacin por unidad de volumen.
NOTA: Se recuerdan las expresiones de la elasticidad lineal:
= Tr ( )1 + 2
E E
con = , =
(1 + )(1 2) 2(1 + )
Solucin:
a) Las componentes del tensor de deformacin infinitesimal son:
0 3 10 3 0

ij = 3 10 3 22 0
0 0 0

Deformacin volumtrica DVL V = I = 11 + 22 + 33 = 2 10 3 22 = 2 10 3 . Con
lo cual:
0 3 0
0 3
ij = 3 2 0 10 3 deformaci
n plana
ij = 10
3

0 3 2
0 0

Deformaciones principales:
0 3 1 = 1
=0 2 + 2 3 = 0
3 2 2 = 3
Las deformaciones principales son:
1 = 1 10 3 1 10 3 0
ij = 3
2 = 3 10 3 0 3 10

b) y

1 x
y xy
yy

Universidad de Castilla- La Mancha Draft Por: Eduardo W. V. Chaves (2012)


Ciudad Real - Espaa
7 ELASTICIDAD LINEAL 443

Crculo de Mohr
S = (10 3 )
2

( N = 0; S = 3 )

III = 3 I =1
N 10 3

( N = 0; S = 2)

( N = 2; S = 3 )

Observemos que el radio es R = (1 (3)) / 2 = 2 . Luego:


3
tg(2) = 2 = arctg( 3 ) = 30
1
E
b) Aplicando ij = Tr ( ) ij + 2 ij , donde = = 20MPa ,
(1 + )(1 2)
E
= = 20 MPa , Tr ( ) = 2 10 3 . Luego:
2(1 + )

1 0 0 0 3 0

ij = Tr ( ) 0 1 0 + 2 3 2 0 10 3
0 0 1 0 0 0

40 0 0 0 3 0

= 0 40 0 + 40 3 2 0 10 3
142 MPa
4
3

0 0 40 0 0 0 3
=10 Pa

Resultando:
40 40 3 0

ij = 40 3 120 0 kPa
0 0 40

Como el material es istropo, las direcciones principales de las tensiones coinciden con las
direcciones principales de las deformaciones. Y adems, recordemos que los autovalores de
y estn relacionados, cuya expresin se demuestra a continuacin.
Reemplazando el valor de = Tr ( )1 + 2 en la definicin de autovalor, autovector:
n = n

Universidad de Castilla- La Mancha Draft Por: Eduardo W. V. Chaves (2012)


Ciudad Real - Espaa
444 PROBLEMAS RESUELTOS DE MECNICA DEL MEDIO CONTINUO

(Tr ( )1 + 2 ) n = n
Tr ( )1 n + 2 n = n
Tr ( )n + 2 n = n
2 n = n Tr ( )n
2 n = ( Tr ( ) )n
Tr ( )
n = n
2
n = n

Luego:
Tr ( )
= = 2 + Tr ( )
2
Pudiendo as obtener los autovalores de :
(1) I = 2 (1) + Tr ( ) = (40 10 6 ) (1 10 3 ) + (20 10 6 ) (2 10 3 ) = 0
( 2 ) II = 2 ( 2 ) + Tr ( ) = (40 10 6 ) (0) + (20 10 6 ) (2 10 3 ) = 40 10 3 Pa
(3) III = 2 (3) + Tr ( ) = (40 10 6 ) (3 10 3 ) + (20 10 6 ) (2 10 3 ) = 160 10 3 Pa

Tambin podemos utilizar la expresin = Tr ( )1 + 2 en el espacio principal:


40 0 0 1 0 0 0 0 0

ij = 0 40 0 + 40 0 3 0 10
142
3
3 = 0 160
MPa
4 0 kPa
0 0 40 0 0 0 =103 Pa 0 0 40

Crculo de Mohr en tensiones:

160 40 0 N (kPa)

1
c) La densidad de energa e = : . Podemos utilizar el espacio principal para obtener
2
la densidad de energa, i.e.:
1 0 0 0 0 0 1 0 0 1 0 0
0 = 0 160 m
2 0 0 10 3 Pa ; 0
2 0 = 0 3 0 10 3 m

0 0 3 0 0 40 0 0 3 0 0 0

Universidad de Castilla- La Mancha Draft Por: Eduardo W. V. Chaves (2012)


Ciudad Real - Espaa
7 ELASTICIDAD LINEAL 445

1 1
e = ij ij = (11 + 2 2 + 3 3 )
2 2
1
[ m N m
= (0)(1) + (160 10 3 )(3 10 3 ) + ( 40 10 3 )(0) = 240 Pa = 240 2
2 m m m
] J
= 240 3
m

Ejemplo 7.10
Un slido se halla sometido a deformacin plana, siendo las componentes del tensor de
deformacin lineal en un determinado punto:
2 3 0
ij = 3 10 0 10 3

0 0 0
Considrese que el slido tiene un comportamiento elstico lineal e istropo, definido por
mdulo elstico de Young E = 10MPa y coeficiente de Poisson = 0,25 .
Se pide:
a) Obtener la deformacin volumtrica y el tensor de deformacin desviadora;
b) Obtener las deformaciones principales y las direcciones en que se producen;
c) Obtener las componentes del tensor de tensiones de Cauchy;
d) Obtener las mximas y mnimas tensiones normales;
e) Se sabe que el material rompe cuando en algn plano se alcanza una tensin tangencial
que supere 40 kPa . Verificar si se produce la rotura.
Solucin:
a) Deformacin volumtrica ( V ):
V = I = Tr ( ) = ( 2 10) 10 3 = 12 10 3

Descomposicin aditiva del tensor de deformacin = esf + dev , donde la parte esfrica:
4 0 0
Tr ( )
ijesf = ij = 0 4 0 10 3

3
0 0 4
y la parte desviadora queda:
2 3 0 4 0 0 2 3 0

ijdev = ij ijesf = 3 10 0 0 4 0 10 = 3 6 0 10 3
3

0 0 0 0 0 4 0 0 4

b) Las deformaciones principales obtenemos al resolver el determinante caracterstico:
2 3
=0 2 + 12 + 11 = 0
3 10
Solucin de la ecuacin cuadrtica:
(12) (12) 2 4(1)(11) 12 10 (1) = 1,0
(1, 2 ) = =
2(1) 2 ( 2 ) = 11

Universidad de Castilla- La Mancha Draft Por: Eduardo W. V. Chaves (2012)


Ciudad Real - Espaa
446 PROBLEMAS RESUELTOS DE MECNICA DEL MEDIO CONTINUO

Luego, las deformaciones principales son:


1 = 1,0 10 3 ; 2 = 11,0 10 3

Direcciones principales ( ij ij )n (j ) = 0 i (i, j = 1,2)

Direccin principal asociada al valor principal (1) = 1,0 :

n1(1) 0 n1 + 3n 2 = 0 n1 = 3n 2
(1) (1) (1) (1)
2 (1) 3
=
3 10 (1) n (21) 0 3n1(1) 9n (21) = 0

2 2 2 1
restriccin n1(1) + n (21) = 1 , con eso obtenemos que (3n (21) ) 2 + n (21) = 1 n (21) = , y que
10
3
n1(1) =
10
Direccin principal asociada al valor principal (1) = 11,0 :

n1( 2) 0 9n1 + 3n 2 = 0
( 2) ( 2)
2 (11) 3
=
3 10 (11) n (22) 0 3n1( 2 ) + n (22 ) = 0 n (22) = 3n1( 2 )

2 2 1 3
Con la restriccin n1( 2) + n (22) = 1 , obtenemos que n1( 2) = , y que n (22) =
10 10
Resumiendo as que:
3 1
1 = 1 10 3 direccin
principal
n (i1) = 0
10 10
1 3
2 = 11 10 3 direccin
principal
n i( 2 ) = 0
10 10
1 = 0 direccin
principal
n i(3) = [0 0 1]
c) Las componentes del tensor de tensiones de Cauchy vienen dadas por:
ij = Tr ( ) ij + 2 ij

E E
donde = = 4MPa , = G = = 4 MPa , Tr ( ) = 12 10 3 :
(1 + )(1 2 ) 2(1 + )

1 0 0 2 3 0


ij = 4 (12) 0 1 0 + 2 (4) 3 10 0 10 3 MPa

0 0 1 0 0 0

64 24 0

= 24 128 0 kPa
0 0 48

Como el material es istropo las direcciones principales de las tensiones y deformaciones


coinciden. Las tensiones principales obtenemos trabajando en el espacio principal
ij = Tr ( ) ij + 2 ij :

Universidad de Castilla- La Mancha Draft Por: Eduardo W. V. Chaves (2012)


Ciudad Real - Espaa
7 ELASTICIDAD LINEAL 447

1 0 0 1 0 0


ij = 4 (12) 0 1 0 + 2 (4) 0 11 0 10 3 MPa
0 0 1 0 0 0

56 0 0

= 0 136 0 kPa
0 0 48

d) Dibujamos el crculo de Mohr en tensiones con I = 48kPa , II = 56kPa ,


III = 136kPa :

S (kPa)

S max = 44

II = 56

III = 136 N (kPa)


I = 48

Podemos obtener la tensin tangencial mxima como:


I III (48) (136)
S max = = = 44kPa
2 2
Luego, el material alcanzar la rotura.

Ejemplo 7.11
El estado tensional en un punto del medio continuo viene dado a travs de las
componentes del tensor de tensiones de Cauchy por:
26 6 0
ij = 6 9 0 kPa
0 0 29
Considerando un material elstico, lineal, homogneo e istropo. Se pide:
a) Obtener los invariantes principales de ;
b) Obtener la parte esfrica y desviadora de ;
c) Obtener los autovalores y autovectores de ;
d) Dibujar el crculo de Mohr en tensiones. Obtener tambin la tensin normal y la tensin
tangencial mxima.
e) Considerando un rgimen de pequeas deformaciones y teniendo en cuenta que el
material que constituye el medio continuo tiene como propiedades mecnicas

Universidad de Castilla- La Mancha Draft Por: Eduardo W. V. Chaves (2012)


Ciudad Real - Espaa
448 PROBLEMAS RESUELTOS DE MECNICA DEL MEDIO CONTINUO

= 20000kPa y = 20000kPa ( , son las constantes de Lam). Obtener las


componentes del tensor de deformacin infinitesimal;
f) Obtener los autovalores y autovectores de .
Solucin:
I = 12 10 3

9 0 26 0 26 6
II = 10 6 + 10 6 + 10 6 = 763 10 6
0 29 0 29 6 9

III = det ( ) = 7830 10 9

Parte esfrica y desviadora ij = ijdev + ijesf :


1 ( 29 26 + 9)
m = ii = = 4 10 3 Pa
3 3
4 0 0
ijhid esf
ij = 0 4 0 kPa
0 0 4

26 4 6 0 30 6 0
ijdev
= 6 94 0 = 6 5 0 kPa
0 0 29 4 0 0 25

Resolviendo la ecuacin caracterstica, los autovalores son:


I = 29kPa , II = 10kPa , III = 27 kPa :
Los autovectores:
I = 29kPa direccin
principal
n i(1) = [0 0 1]
II = 10kPa direccin
principal
n i( 2 ) = [0,1644 0,98639 0]
III = 27 kPa direccin
principal
n i(3) = [0,98639 0,1644 0]

S (kPa)
29 (27)
S max = = 28
2

N (kPa)

III = 27
I = N max = 29

II = 10

Universidad de Castilla- La Mancha Draft Por: Eduardo W. V. Chaves (2012)


Ciudad Real - Espaa
7 ELASTICIDAD LINEAL 449

1
ij = Tr ( ) ij + 2 ij inversa
ij = Tr ( ) ij + ij
2 (3 + 2 ) 2

donde = 5 10 9 ( Pa ) 1 , Tr ( ) = 1,2 10 4 ( Pa)
2 (3 + 2 )

1 0 0 26 6 0
9 4 8
ij = (5 10 )(1,2 10 ) 0 1 0 + 2,5 10 6 9 0 103
0 0 1 0 0 29
1 0 0 26 6 0 7,1 1,5 0
5 5
= 6 10 0 1 0 + 2,5 10 6 9 0 = 1,5 1,65 0 10 4
0 0 1 0 0 29 0 0 6,65
Como el material es istropo, el tensor de tensiones y el tensor de deformaciones tienen el
mismo espacio principal (mismas direcciones principales), luego:
1
ij = Tr ( ) ij + ij
2 (3 + 2 ) 2
1 0 0 29 0 0 66,5 0 0
5 5
= 6 10 0 1 0 + 2,5 10 0 10 0 = 0 19 0 10 5
0 0 1 0 0 27 0 0 73,5
Las direcciones principales del tensor de deformaciones son las mismas que las del tensor
de tensiones para un material istropo.

Ejemplo 7.12
Demostrar que las ecuaciones constitutivas de tensin, para un material elstico lineal
homogneo e istropo, se puede representar por el conjunto de ecuaciones:
dev = 2 dev

Tr ( ) = 3Tr ( )
donde = G es la constante de Lam, es el coeficiente de deformacin volumtrica.
Solucin:
= Ce :
= [1 1 + 2 I] :
= Tr ( )1 + 2

= dev + esf = Tr ( )1 + 2 ( dev + esf )


Tr ( )
dev + 1 = Tr ( )1 + 2 ( dev + esf )
3
Tr ( ) Tr ( )
dev = Tr ( )1 + 2 dev + 2 1
3 3
2 Tr ( )
dev = + Tr ( )1 + 2 dev 1
3 3
La traza del tensor de tensiones:

Universidad de Castilla- La Mancha Draft Por: Eduardo W. V. Chaves (2012)


Ciudad Real - Espaa
450 PROBLEMAS RESUELTOS DE MECNICA DEL MEDIO CONTINUO

Tr ( ) = : 1 = [Tr ( )1 + 2 ] : 1 = Tr ( )3 + 2 Tr ( ) = (3 + 2 )Tr ( )
Con lo cual:
2 Tr ( )
dev = + Tr ( )1 + 2
dev
1
3 3

dev = +
2 (3 + 2 )Tr() 1
Tr ( )1 + 2
dev

3 3

dev = +
2 (3 + 2 )Tr() 1 + 2 dev
Tr ( )1
14434444 424444444
3
3
=0

A las ecuaciones dev = 2 dev tenemos que aadir la restriccin:


2 (3 + 2 )Tr()
+ Tr ( )1 1=0
3 3
2 Tr ( )
+ Tr ( )1 1=0
3 3
2
Tr ( )1 = 3 + Tr ( )1
3
Tr ( )1 = 3 Tr ( )1
o
Tr ( ) = 3 Tr ( )

33 m dev
33

23 23
13 23 = + 13 23

13 13
12 22 m 12 dev
22
12 12
11 m dev
11

ij = Tr ( ) ij + 2 ij
= Tr ( ) ij = 3 Tr ( ) ij
+ ijdev = 2 ijdev

33 m dev
33

23 23
13 23 = + 13 23

13 13
12 22 m 12 dev
22
12 12
11 m dev
11

Universidad de Castilla- La Mancha Draft Por: Eduardo W. V. Chaves (2012)


Ciudad Real - Espaa
7 ELASTICIDAD LINEAL 451

Solucin Alternativa:
Partiendo de la ecuacin constitutiva en tensin para un material elstico, lineal,
homogneo e istropo: = ( ) = Tr ( )1 + 2 . Adems como estamos en rgimen
linear se cumple que = () = ( esf + dev ) = ( esf ) + ( dev ) , donde:
( esf ) = Tr ( esf )1 + 2 esf
Tr ( ) Tr ( ) Tr ( ) 2
esf = Tr 11 + 2 1 = Tr ( )1 + 2 1 = ( + ) Tr ( )1 = Tr ( )1
3 3 3 3
Tr ( )
1 = Tr ( )1
3
Tr ( )1 = 3 Tr ( )1

( dev ) = Tr ( dev )1 + 2 dev = 2 dev


1424 3
=0

[
Observar que se cumple que Tr ( esf ) = Tr esf ] [ ] = Tr[ ] .
NOTA: Es interesante observar que: si en un material istropo tenemos un estado
puramente esfrico de comprensin:
p
p>0

p 0 0
p
ij = 0 p 0
Tr ( ) = 3 p < 0
p 0 0 p

E
tenemos que Tr ( ) = 3 Tr ( ) < 0 , y teniendo en cuenta que = , podemos
3(1 2)
concluir que: si > 0,5 eso implica que < 0 y como consecuencia Tr ( ) > 0 , es decir, una
expansin, lo que no tiene sentido fsico que en un material istropo sometido a un estado
de compresin haya una expansin.

Ejemplo 7.13
Un paraleleppedo de cierto material elstico (coeficiente de Poisson = 0,3 y mdulo de
elasticidad E = 2 10 6 N / m 2 que a cierta temperatura tiene de dimensiones a = 0,10m ,
b = 0,20m , c = 0,30m , ver Figura 7.6, se introduce en una cavidad de anchura b de paredes
rgidas, planas y perfectamente lisas, de tal forma que dos caras opuestas del paraleleppedo
estn en contacto con las paredes de la cavidad.
Una vez en esta posicin se eleva la temperatura del prisma en T = 30 C .
Se pide:
1) Calcular los valores de las tensiones principales en los puntos del paraleleppedo.
2) Hallar las componentes de las deformaciones.

Dato: coeficiente de expansin trmico del material es igual a 1,25 10 5 C 1 .


Solucin:

Universidad de Castilla- La Mancha Draft Por: Eduardo W. V. Chaves (2012)


Ciudad Real - Espaa
452 PROBLEMAS RESUELTOS DE MECNICA DEL MEDIO CONTINUO

Como el slido puede deformarse libremente segn las direcciones x , z , luego est libre
de tensiones normales x = z = 0 . El slido est restringido al movimiento segn la
direccin y luego y = 0 :

y =
1
E
[ ] 1
[ ]
y ( x + z ) + T = y + T = 0
E
y = ET

resultando que:
N
y = ET = 2 10 6 1,25 10 5 (30) = 750
m2
Componentes del tensor de tensiones:
0 0 0
ij = 0 750 0 Pa

0 0 0
y
b) x = z = + T = 4,875 10 4
E
El tensor de deformacin queda:
4,875 0 0

ij = 0 0 0 10 4
0 0 4,875

Datos:
a
a = 0,10m
b = 0,20 m
c = 0,30 m
E = 2 10 6 N / m 2
= 0,3
T = 30 C
= 1,25 10 5 C 1
c
y

x
Figura 7.6.

Ejemplo 7.14
En el fondo de un recipiente, cuyo hueco interior es prisma recto, de base cuadrada, de
0,10 0,10m , se coloca un bloque de caucho sinttico de 0,10 0,10 0,5m , tal como se
indica en la figura. El bloque ajusta perfectamente en el recipiente de paredes rgidas lisas.
Las caractersticas elsticas del caucho sinttico son E = 2,94 10 6 N / m 2 y = 0,1 .

Universidad de Castilla- La Mancha Draft Por: Eduardo W. V. Chaves (2012)


Ciudad Real - Espaa
7 ELASTICIDAD LINEAL 453

Sobre el caucho se vierten 0,004m 3 de mercurio, cuya densidad de masa es 13580kg / m 3 .


Se pide:
a) La altura H que alcanza el mercurio sobre el fondo del recipiente;
b) El estado tensional en un punto genrico del bloque de caucho
NOTA: Desprciese el peso del caucho. Considrese la aceleracin de la gravedad
g = 10m / s 2 .

z Paredes rgidas
Paredes rgidas

Mercurio

Caucho
L0 = 0,5m

Figura 7.7.
Solucin:
Primero calculamos la fuerza total del mercurio ejerce sobre el caucho:
kg m kgm
F = V g = 0,004(m 3 ) 13580 3 10 2 = 543,20 2 N
m s s
La tensin normal segn la direccin z viene dada por:
F 543,20 N
z = = = 54,320 10 3 2
A (0,1 0,1) m
Segn las direcciones x , y el caucho no se deforma: x = y = 0

x =
1
E
[ ]
x ( y + z ) = 0 x = ( y + z )

1
[
y = y ( x + z ) = 0
E
] y = ( x + z )

Universidad de Castilla- La Mancha Draft Por: Eduardo W. V. Chaves (2012)


Ciudad Real - Espaa
454 PROBLEMAS RESUELTOS DE MECNICA DEL MEDIO CONTINUO

y = ( x + z )
{[ ]
y = ( y + z ) + z }
( 2 + )
y = z = z = 6035,55Pa = x
2
(1 ) (1 )
La deformacin normal segn la direccin z :

z =
1
E
[
z ( x + y ) ]
1
= { 54320 0,1[2(6035,55)]} = 0,0180656
2,94 10 6
b) Variacin de la longitud del caucho:
L = L0 z = 0,5 ( 0,018656 ) = 0,00903m
La altura H que alcanza el mercurio sobre el fondo del recipiente:
H = hmer + ( L0 L)
donde hmer viene dado por:
0,004
Vmer = b 2 hmer = 0,004 hmer = = 0,4m
0,1 0,1
Resultando as que:
H = hmer + ( L0 L)
= 0,4 + (0,5 0,00903)
= 0,891m

Ejemplo 7.15
En un ensayo de laboratorio de un material se han obtenido las siguientes relaciones:

1 21 31
x = x + y + z
E1 E2 E3
12 1 32
y = x + y + z (7.69)
E1 E2 E3
13 23 1
z = x + y + z
E1 E2 E3

donde 12 = 0,2 , 13 = 0,3 , 23 = 0,25 , E1 = 1000 MPa , E 2 = 2000MPa , E3 = 1500MPa .


Sabiendo que el material analizado es un MATERIAL ORTTROPO. Obtener los valores
de 21 , 31 , 32 .
Solucin:
La matriz constitutiva para un material orttropo presenta el siguiente formato:

Universidad de Castilla- La Mancha Draft Por: Eduardo W. V. Chaves (2012)


Ciudad Real - Espaa
7 ELASTICIDAD LINEAL 455

C11 C12 C13 0 0 0


C C22 C23 0 0 0
12
C C23 C33 0 0 0 Simetra Orttropa
[C ] = 13 (7.70)
0 0 0 C44 0 0 9 constantes independientes
0 0 0 0 C55 0

0 0 0 0 0 C66

Reestructurando (7.69) obtenemos que:


1 21 31
0 0 0
E E2 E3
xx 1 xx
12 1 32
yy E 0 0 0 yy
zz 1 E2 E3
= 13 23 1 zz (7.71)
2 xy 0 0 0 xy
2 yz E1 E2 E3
0 0 0 C44 0 0 yz
2 xz
0 0 0 0 C55 0 xz
0 0 0 0 0 C66

Luego para un material orttropo debe cumplir que:
21 12 31 13 32 23
= ; = ; =
E 2 E1 E 3 E1 E3 E2
obteniendo as que
21 12 E 2 12 2000 0,2
= 21 = = = 0,4
E2 E1 E1 1000
31 13 E 3 13 1500 0,3
= 31 = = = 0,45
E3 E1 E1 1000
32 23 E 3 23 1500 0,25
= 32 = = = 0,1875
E3 E2 E2 2000

Ejemplo 7.16
Dado un material elstico lineal, homogneo e istropo con las siguientes propiedades
elsticas:
E = 71 GPa
G = 26,6 GPa
Determinar las componentes del tensor de deformacin y la densidad de energa de
deformacin en un punto del cuerpo si las componentes del tensor de tensiones en este
punto son:
20 4 5
ij = 4 0 10 MPa
5 10 15

Universidad de Castilla- La Mancha Draft Por: Eduardo W. V. Chaves (2012)


Ciudad Real - Espaa
456 PROBLEMAS RESUELTOS DE MECNICA DEL MEDIO CONTINUO

Solucin:
E
Obtenemos el coeficiente de Poisson partiendo de la relacin: G=
2(1 + )
E
= 1 = 0,335
2G
1
11 = [11 ( 22 + 33 )] = 1 9 [20 0,335 (0 + 15 )]10 6 = 211 10 6
E 71 10
1 1
22 = [ 22 ( 11 + 33 )] = [0 0,335 (20 + 15 )]10 6 = 165 10 6
E 71 10 9
1 1
33 = [ 33 (11 + 22 )] = 9
[15 0,335 (20 + 0 )]10 6 = 117 10 6
E 71 10
1+ 1 + 0,335
12 = 12 = ( 4 10 6 ) = 75 10 6
E 71 10 9
1+ 1 + 0,335
13 = 13 = 9
(5 10 6 ) = 94 10 6
E 71 10
1+ 1 + 0,335
23 = 23 = (10 10 6 ) = 188 10 6
E 71 10 9
luego:
211 75 94
ij = 75 165 188 10 6
94 188 117

La densidad de energa de deformacin para un material elstico lineal e istropo viene


dada por:
1 1 1
e = : C e : = : indicial
e = ij ij
2 2 2
Considerando la simetra de los tensores de tensin y de deformacin la densidad de
energa de deformacin resulta:
1
e = [1111 + 22 22 + 33 33 + 212 12 + 2 23 23 + 21313 ]
2
1
= [( 211)( 20) + ( 165)(0) + (117 )(15) + 2( 75)( 4) + 2(188)(10) + 2(94)(5) ]
2
= 5637 ,5 J / m 3
Podemos tambin obtener la energa de deformacin utilizando la ecuacin:
1 1 1 1
e= I 2 II dev = I 2 + J 2
6(3 + 2 ) 2 6(3 + 2 ) 2

y considerando que: I = 3,5 10 7 ; II = 2,4933 1014 ; 5,3804 10 10 Pa ; = G ,


obtenemos:
e 5638 ,03 J / m 3
La diferencia entre los resultados obtenidos es debida a la aproximacin numrica.

Universidad de Castilla- La Mancha Draft Por: Eduardo W. V. Chaves (2012)


Ciudad Real - Espaa
7 ELASTICIDAD LINEAL 457

Ejemplo 7.17
Expresar la energa de deformacin en funcin de los invariantes principales de .
Solucin:
1
e = :
2
1
= : [Tr ( )1 + 2 ]
2
Tr ( )
= 2:3 1 + :
2 1 Tr ( )
[Tr ( )]2 [Tr ( )]2
= + : = + Tr ( T )
2 2
[Tr ( )]2 [Tr ( )]2
= + Tr ( ) = + Tr ( 2 )
2 2
Podemos sumar y restar el trmino [Tr ( )]2 sin alterar la expresin:
[Tr ( )]2
e= + [Tr ( ) ] + Tr ( 2 ) [Tr ( ) ]
2 2

2
1
{
= ( + 2 )[Tr ( )] [Tr ( )] Tr ( 2 )
2
2 2
}
Considerando que los invariantes principales de son I = Tr ( ) ,

II =
1
2
{ }
[Tr ()]2 Tr ( 2 ) , obtenemos que:
1
e = ( + 2 )I 2 2 II = e ( I , II )
2

Ejemplo 7.18
Se conocen las respuestas de un slido termoelstico lineal en equilibrio a un sistema de
r r (I ) r (I )
acciones I (b ( I ) , t * en S ; u* ; en S ur ; T ( I ) ) y a otro sistema de acciones
r r ( II ) r ( II )
II (b ( II ) , t * en S ; u* ; en S ur ; T ( II ) ) . Justificar (demostrar) cul sera la respuesta
al sistema I + II .
Solucin:
Como estamos en el rgimen lineal se cumplen que:
r r r
b = b ( I ) + b ( II )
T = T ( I ) + T ( II )
r r ( I ) r ( II )
t* = t* + t*
r r ( I ) r ( II )
u* = u* + u*
Lo mismo para los campos:
r r r
u = u ( I ) + u ( II )
= ( I ) + ( II )
= ( I ) + ( II )
Partiendo de las ecuaciones de gobierno del problema termoelstico lineal en equilibrio:

Universidad de Castilla- La Mancha Draft Por: Eduardo W. V. Chaves (2012)


Ciudad Real - Espaa
458 PROBLEMAS RESUELTOS DE MECNICA DEL MEDIO CONTINUO

Ecuaciones de Equilibrio:
r r r
[ r
] [ r
] r
xr + b = xr ( ( I ) + ( II ) ) + (b ( I ) + b ( II ) ) = xr ( I ) + b ( I ) + xr ( II ) + b ( II ) = 0

Ecuaciones Cinemticas:

= ( I ) + ( II ) =
2
[
1 r r (I ) r
] [
1 r r
x u + ( xr u ( I ) ) T + xr u ( II ) + ( xr u ( II ) ) T
2
]
=
1
2
{[ r
xr u ( I ) + xr u ] [
r ( II ) r (I ) r ( II ) T
+ xr u + xr u ]}
2
{ [r r
] [ r r
]} {
T 1
2
r rT
= xr u ( I ) + u ( II ) + xr (u ( I ) + u ( II ) ) = xr u + [ xr u] =
1
}
Ecuacin Constitutiva:
= C e : + MT
donde M es el tensor de tensiones trmicas
= C e : + MT = C e : ( ( I ) + ( II ) ) + M (T ( I ) + T ( II ) )
= (C e : ( I ) + MT ( I ) ) + (C e : ( II ) + MT ( II ) )
= ( I ) + ( II )
Se comprobando as que se cumplen todas las condiciones. Luego, tambin se puede
aplicar el principio de la superposicin al problema termoelstico lineal, como era de
esperar ya que estamos en el rgimen lineal.

Ejemplo 7.19
Considrese una barra de 7,5m de longitud y 0,1m de dimetro que est constituida por un
1
material cuyas propiedades son: E = 2,0 10 11 Pa y = 20 10 6 .
C
Inicialmente la barra est a 15 C y la temperatura aumenta a 50 C .
Se pide:
1) Determinar el alargamiento de la barra considerando que la barra pueda expandirse
libremente;
2) Suponga que la barra ya no puede alongarse libremente porque en sus extremos se han
colocado bloques de hormign, ver Figura 7.8(b). Obtener la tensin en la barra.
Nota: Considerar el problema en una dimensin.

x
L

a) b)

Figura 7.8: Barra bajo efecto trmico.

Universidad de Castilla- La Mancha Draft Por: Eduardo W. V. Chaves (2012)


Ciudad Real - Espaa
7 ELASTICIDAD LINEAL 459

Solucin:
1) Para obtener el alargamiento debemos calcular previamente la deformacin segn la
direccin del eje de la barra:
ij = (T ) ij

Como se trata de un caso unidimensional slo consideraremos la componente de la


deformacin segn el eje x , 11 = x , luego:
11 = x = 20 10 6 (50 15) = 7 10 4
El alargamiento se obtiene segn la integral:
L


L = x dx = x L = 7 10 4 7,5 = 5,25 10 3 m
0

Observar que como la barra puede expandirse libremente, sta est libre de tensin.
2) Si las extremidades no pueden moverse, surgirn tensiones uniformes que vienen dadas
por:
x = E x = 2,0 1011 7 10 4 = 1,4 10 8 Pa

Ejemplo 7.20
Dado un material elstico lineal, homogneo e istropo con las siguientes propiedades
mecnicas: E = 10 6 Pa (mdulo de Young), = 0,25 (coeficiente de Poisson),
= 20 10 6 C 1 (Coeficiente de dilatancia trmica).
Considrese que en un determinado punto del slido se conoce el estado tensional y que
viene dado por:
12 0 4
ij = 0 0 0 Pa
4 0 6

Se pide:
a) Determinar las tensiones y direcciones principales; Obtener la tensin tangencial
mxima.
b) Las deformaciones en dicho punto. Determinar tambin las deformaciones y direcciones
principales;
c) Determinar la densidad de energa.
d) Si ahora, a ste slido sufre una variacin de temperatura T = 50 C , Cual es el estado
de deformacin final en este punto?
e) Se puede decir que estamos ante un caso de Tensin Plana?
Solucin:
Obtenemos los autovalores al resolver el determinante caracterstico. Adems observemos
que ya se conoce un autovalor 2 = 0 que est asociado a la direccin n i( 2) = [0 1 0] .
Luego, es suficiente resolver el determinante:
12 4
=0 2 18 + 56 = 0
4 6

Universidad de Castilla- La Mancha Draft Por: Eduardo W. V. Chaves (2012)


Ciudad Real - Espaa
460 PROBLEMAS RESUELTOS DE MECNICA DEL MEDIO CONTINUO

Resolviendo la ecuacin cuadrtica obtenemos que:


18 324 224 1 = 14
(1,3) =
2 3 = 4
14 0 0
' ij = 0 0 0 Pa
0 0 4

A continuacin obtenemos las direcciones principales (autovectores que deben ser


versores, vectores unitarios), resultando:
2 1
1 = 14 autovector
n i(1) = 0 = [0,8944 0 0,4472]
5 5
2 = 0 autovector
n i( 2) = [0 1 0]
1 2
3 = 4 autovector
n i(3) = 0 = [0,4472 0 0,8944]
5 5
Haciendo el cambio de nomenclatura tal que I > II > III , tenemos que I = 14 ,
II = 4 , III = 0 .
S (Pa )

S max = 7

N (Pa)

III = 0 II = 4
I = 14

Podemos obtener la tensin tangencial mxima como:


I III (14) (0)
S max = = = 7 Pa
2 2
Las componentes del tensor de tensiones de Cauchy vienen dadas por:
1
ij = Tr ( ) ij + 2 ij inversa
ij = Tr ( ) ij + ij
2 (3 + 2 ) 2
1
Recordar que = C e : , la inversa = C e : .
E E
donde = = 4 10 5 Pa , =G = = 4 10 5 Pa , Tr ( ) = 18 ,
(1 + )(1 2 ) 2(1 + )

= 2,5 10 7 Pa
2 (3 + 2 )

Universidad de Castilla- La Mancha Draft Por: Eduardo W. V. Chaves (2012)


Ciudad Real - Espaa
7 ELASTICIDAD LINEAL 461

1 0 0 12 0 4

6
ij = 4,5 10 0 1 0 + 1,25 10 0 10 0
6

0 0 1 4 0 6

10,5 0 5
= 0 4,5 0 10 6

5 0 3
Para un material istropo lineal las direcciones principales de las tensiones y deformaciones
coinciden. Las tensiones principales obtenemos trabajando en el espacio principal
1
ij = Tr ( ) + ij :
2 (3 + 2 ) 2

1 0 0 14 0 0 13 0 0

ij = 4,5 10 0 1 0 + 1,25 10 0 0 0 = 0 4,5 0 10 6
6 6
0 0 1 0 0 4 0 0 0,5

1 1
La energa de deformacin viene dada por e = : = ij ij . Podemos utilizar el
2 2
espacio principal donde se cumple que:
13 0 0 14 0 0
ij = 0 4,5 0 10 6
; ij = 0 0 0
0 0 0,5 0 0 4

Luego, podemos utilizar el espacio principal para obtener la densidad de energa:


1 1
e = ij ij = [11 11 + 3333 ] = 92 10 6 J3
2 2 m
Utilizando el principio de la superposicin, podemos decir que:
ij = ij ( ) + ij (T )
= ij ( ) + T ij
Luego,
10,5 0 5 1 0 0 1010,5 0 5
6 6
ij = 0 4,5 0 10 + 20 10 (50) 0 1 0 = 0 995,5 0 10 6
5 0 3 0 0 1 5 0 1003
Las direcciones principales del tensor de deformacin infinitesimal son las mismas del
tensor de tensiones.
e) No podemos decir que se trata de un estado de tensin plana ya que no tenemos
informacin del estado tensional de todo el medio continuo. Slo estaremos en el caso de
tensin plana cuando el CAMPO de tensin es independiente de una direccin.

Ejemplo 7.21
Considrese una barra donde en una de las extremidades se aplica una fuerza igual a
6000 N como se indica en la figura siguiente:

Universidad de Castilla- La Mancha Draft Por: Eduardo W. V. Chaves (2012)


Ciudad Real - Espaa
462 PROBLEMAS RESUELTOS DE MECNICA DEL MEDIO CONTINUO

1m
100 m
1m =

y, v
6000
y =
11

x, u F = 6000 N

z, w

Determinar x , y , z y el cambio de longitud en las dimensiones de la barra. Considere


que la barra est constituida por un material cuyas propiedades elsticas son:
Mdulo de Young: E = 10 7 Pa ;
Coeficiente de Poisson: = 0,3
Considerar el material elstico, lineal, homogneo e istropo y rgimen de pequeas
deformaciones.
Solucin:
Utilizando las expresiones de las deformaciones normales:

x =
1
[ (
x y + z = y = )] (0,3)(6000 )
= 0,00018
E E 10 7
y 6000
1
[
y = y ( x + z ) = ]
= = 0,0006
E E 10 7
1
[ (
)]
z = z x + y = y = 0,00018
E E
Los cambios de longitud en las dimensiones de la seccin son:
u = w = 0,00018 1 = 1,8 10 4 m
y de la longitud:
v = 0,0006 100 = 6,0 10 2 m

Ejemplo 7.22
Una roseta a 45 , como se indica en la Figura 7.9, destinada a medir la deformacin en una
parte de una estructura, proporciona las siguientes lecturas:
x = 0,33 10 3 ; x = 0,22 10 3 ; y = 0,05 10 3

Universidad de Castilla- La Mancha Draft Por: Eduardo W. V. Chaves (2012)


Ciudad Real - Espaa
7 ELASTICIDAD LINEAL 463

Cul es la tensin de corte mxima en el punto en cuestin? Sabiendo que el material


(elstico lineal e istropo) que constituye la estructura tiene las siguientes propiedades
elsticas: E = 29000 Pa (Mdulo de Young); = 0,3 (Coeficiente de Poisson). Se pide:
a) Determinar las deformaciones principales y las direcciones principales de las
deformaciones;
b) Determinar las tensiones principales y las direcciones principales de las tensiones.
c) Que conclusin se puede sacar de las direcciones principales de las tensiones y de las
deformaciones?
Nota: Considerar el caso de deformacin plana y el material elstico, lineal e istropo.

45
45

x
Figura 7.9: Roseta.

Solucin:
Primero tenemos que obtener las componentes del tensor de deformacin en el sistema
x, y, z . Para ello, utilizaremos la ley de transformacin de coordenadas para obtener la
componente xy = 212 . Recordando que en el caso bidimensional la componente normal
puede obtenerse como:
11 + 22 11 22
=
11 + cos( 2) + 12 sin( 2) (7.72)
2 2

cuya expresin fue obtenido a travs de la transformacin de coordenadas:



11
12 0 cos sin 0 11 12 0 cos sin 0
22 0 = sin cos 0 12 22 0 sin cos 0
12
0 0 0 0 0 1 0 0 0 0 0 1
Es interesante ver tambin el Ejemplo 1.96 (captulo 1)
La expresin (7.72) en notacin ingenieril:
x + y x y xy
x = + cos( 2) + sin( 2)
2 2 2
Despejando xy obtenemos:

x x cos 2 ( 2) y sin 2 ( 2)
xy = 2 = 0,16 10 3
sin( 2 )

Luego:

Universidad de Castilla- La Mancha Draft Por: Eduardo W. V. Chaves (2012)


Ciudad Real - Espaa
464 PROBLEMAS RESUELTOS DE MECNICA DEL MEDIO CONTINUO

0,33 0,08 0
ij = 0,08 0,05 0 10 3
0 0 0

Las tensiones:

x =
E
(1 + )(1 2 )
[ ]
(1 2 ) x + y = 12,0462 Pa

y =
E
(1 + )(1 2 )
[ ]
(1 2 ) y + x = 3,5692 Pa

E
xy = xy = 1,7846 Pa
2(1 + )

z =
E
(1 + )(1 2 )
[ ]
x + y = 4,684 Pa

Tensin de corte mxima:


2
x + y
max = + 2xy = 4,5988 Pa

2
a) La ecuacin caracterstica para el tensor de deformacin es:
2 0,28 2,29 10 2 = 0 ( 10 3 )
Los autovalores (las deformaciones principales) vienen dados por:
1 = 0,346155 10 3
2 = 0,06615528 10 3
Los autovectores del tensor de deformacin:
1 0,9802 0,1979 0
2 0,1979 0,9802 0
3 0 0 1

b) Dadas las componentes del tensor de tensin:


12,0462 1,7846 0
ij = 1,7846 3,5692 0 Pa
0 0 4,684

A travs del determinante caracterstico podemos obtener los autovalores, tensiones


principales:
1 = 12,40654
2 = 3,208843
3 = 4,684
Los autovectores del tensor de tensiones son:
1 0,9802 0,1979 0
2 0,1979 0,9802 0
3 0 0 1

Universidad de Castilla- La Mancha Draft Por: Eduardo W. V. Chaves (2012)


Ciudad Real - Espaa
7 ELASTICIDAD LINEAL 465

Comparando los autovectores del tensor de tensiones y de deformaciones concluimos que


son los mismos.
Las direcciones principales de tensin y deformacin son coincidentes slo para el caso de
material istropo.
b) Solucin Alternativa para la Componentes del tensor de tensiones:
Conocidas las componentes del tensor de deformaciones:
0,33 0,08 0
ij = 0,08 0,05 0 10 3
0 0 0

Aplicamos la ecuacin constitutiva: ij = Tr ( ) ij + 2 ij , donde las constantes de Lam


vienen dadas por:
E
= = 16,7307692 10 3 Pa
(1 + )(1 2 )
E
= = 11,15384615 10 3 Pa
2(1 + )

y Tr ( ) = 0,27999972 10 3 0,28 10 3 , con lo cual: ij = Tr ( ) ij + 2 ij viene dado


por:
1 0 0 11 12 13 1 0 0 0,33 0,08 0

ij = Tr ( ) 0 1 0 + 2 12 22 23 = Tr ( ) 0 1 0 + 2 0,08 0,05 0 10 3

0 0 1 13 23 33 0 0 1 0 0 0
12,0461 1,784615 0

= 1,784615 3,5692 0 Pa
0 0 4,6846
Como el material es istropo y comparten las mismas direcciones principales, luego
podemos utilizar la misma expresin ij = Tr ( ) ij + 2 ij en el espacio principal, i.e.:

1 0 0 1 0 0 1 0 0 0,346155 0 0

ij = Tr ( ) 0 1 0 + 2 0 2 0 = Tr ( ) 0 1 0 + 2 0 0,0662 0 10 3
0 0 1 0 0 3 0 0 1 0 0 0
12,40752 0 0
= 0 3,20783 0 Pa
0 0 4,6846

Universidad de Castilla- La Mancha Draft Por: Eduardo W. V. Chaves (2012)


Ciudad Real - Espaa
466 PROBLEMAS RESUELTOS DE MECNICA DEL MEDIO CONTINUO

Ejemplo 7.23
Una delta de roseta (aparato para obtener la deformacin) tiene la forma de un tringulo
equiltero, y registra deformaciones longitudinales en las direcciones x1 , x1 y x1 como se
muestra en la Figura 10.

x2
x1 x1

30
60
30

60 60
x1
Figura 10
Si las deformaciones medidas en estas direcciones son:
11 = 4 10 4
= 1 10 4
11
= 4 10 4
11
Determinar 22 = y , 212 = xy , 22 y . Mostrar que 11 + 22 = 11
+ 22 .

Hiptesis: Considerar caso de deformacin plana.


Solucin:
Utilizando la ley de transformacin de las componentes de un tensor de segundo orden que
es independiente de una direccin, podemos decir que se cumple que:
11 + 22 11 22
=
11 + cos( 21 ) + 12 sin( 21 ) (7.73)
2 2

11 + 22 11 22
=
11 + cos( 2 2 ) + 12 sin( 2 2 ) (7.74)
2 2

donde 1 = 60 y 2 = 120 . Luego, combinando las expresiones anteriores, eliminamos


12 , resultando:

2
22 = 11 11 = 4,66667 10 4
+ 11
3 2

Una vez obtenido el valor de 22 = 4,66667 10 4 , podemos reemplazar en la ecuacin


(7.73) y obtenemos que:

Universidad de Castilla- La Mancha Draft Por: Eduardo W. V. Chaves (2012)


Ciudad Real - Espaa
7 ELASTICIDAD LINEAL 467

1
xy = 2 12 = (411 11 3 22 ) = 3,46410 10 4
3
12 = 1,73205 10 4
Para obtener 22 , primero determinando el ngulo de giro con respecto a x1 que es
3 = 60 +90 = 150 , resultando:
11 + 22 11 22
22 = + cos( 2 3 ) + 12 sin( 2 3 ) = 0,33333 10 4
2 2
Comprobando as que:
+ 22 = 0,66667 10 4
11 + 22 = 11

Ejemplo 7.24
Considrese una seccin de una presa que presenta el campo de desplazamiento dado por:

y, v

u ( x, y ) = 4 x 2 y 2 + 2 xy + 2

v ( x, y ) = 4 y 2 x 2 + 2 xy + 5 x, u

El material que constituye dicha estructura presenta las siguientes propiedades elsticas:
E = 100 MPa , G = 35,7 MPa , = 0, 4 y est sometido a un nivel de carga tal que se puede
considerar que est en el rgimen de pequeas deformaciones.
Se pide:
a) Obtener el campo de tensin;
b) Demostrar que si se cumplen las ecuaciones de equilibrio para el campo de
desplazamiento dado.
Solucin:
a) Clculo de las componentes del tensor de deformacin:
u v u v
x = = 8 x + 2 y ; y = = 8 y + 2 x xy = + =0
x y y x
Luego, las componentes del tensor de deformacin quedan:
8 x + 2 y 0 0
ij = 0 8 y + 2 x 0
0 0 0

b) Para una presa, como ya hemos visto, podemos analizarla segn la aproximacin del
estado de deformacin plana:

Universidad de Castilla- La Mancha Draft Por: Eduardo W. V. Chaves (2012)


Ciudad Real - Espaa
468 PROBLEMAS RESUELTOS DE MECNICA DEL MEDIO CONTINUO

x
1 0 x
E
y = (1 + )(1 2 ) 1 0 y
xy 1
0 0 xy
2
0,6 0,4 0 8 x + 2 y
= 357 ,1428 0,4 0,6 0 8 y + 2 x MPa
0 0 0,3 0

x 4 x 2 y

y = 357 ,1428 2 x 4 y MPa
xy 0

E
z =
(1 + )(1 2 )
( )
x + y = 357 ,1428 [( 8 x + 2 y ) + ( 8 y + 2 x )]

Las ecuaciones de equilibro quedan:


x xy xz
+ + + b x = 0
x y z
4 + 0 + 0 + 0 0 No cumple
xy y yz

x
+
y
+
z
+ b y = 0 0 4 + 0 + 0 0 No cumple


z
yz z 0 + 0 + +0=0
xz + + + b z = 0 z
x y z

Lo que indica que el campo de desplazamientos dado no cumple las ecuaciones de


equilibrio.

Ejemplo 7.25

Se considera un prisma cuadrangular regular cuyo material tiene de mdulo de elasticidad


E = 27,44 10 5 N / cm 2 y coeficiente de Poisson = 0,1 . La longitud del lado de la seccin
recta es a = 20cm . En ambas bases del prisma se colocan dos placas perfectamente lisas y
rgidas, de peso despreciable, unidas entre s mediante cuatro cables de seccin A1 = 1cm 2 y
mdulo de elasticidad E1 = 19,6 10 6 N / cm 2 de longitudes iguales a la altura del prisma
l = 1m , simtricamente dispuesto, como indica en la Figura 7.11.
Sobre dos caras laterales opuestas del prisma se aplica una fuerza de compresin uniforme
p = 7350 N / cm 2 . Se pide calcular:
1. Tensin C en los cables;
2. Tensiones principales en el prisma;
3. Variacin de volumen experimentada por el prisma.

Universidad de Castilla- La Mancha Draft Por: Eduardo W. V. Chaves (2012)


Ciudad Real - Espaa
7 ELASTICIDAD LINEAL 469

a) Configuracin de referencia
b) Configuracin deformada
z

l p
p p

l = 1m

a
y
a

Figura 7.11
Solucin:
Verifiquemos que el cable y el prisma tienen que deformarse, segn direccin z , de igual
manera.
Pz = Cz
En el cable se cumple que:
C
C = E1 Cz Cz =
E1
El campo de tensiones en el prisma vienen dados por:

0 0 0
P
ij = 0 p 0

0 0 4 C A1
a 2
Deformacin del prisma segn direccin z :

Pz =
1
[ (
z x + y =
1
)] 4 c A1
+

p
E E a2
Aplicando que Pz = Cz :
Pz = Cz
1 4 c A1
+ p = C
E a2 E1
Tras algunas manipulaciones algebraicas obtenemos la tensin en el cable:

Universidad de Castilla- La Mancha Draft Por: Eduardo W. V. Chaves (2012)


Ciudad Real - Espaa
470 PROBLEMAS RESUELTOS DE MECNICA DEL MEDIO CONTINUO

E1 pa 2 0,1 19,6 10 6 7350 20 2 N


c = = = 4900 2
( Ea 2 + 4 E1 A1 ) 5 2
(27,44 10 20 + 4 19,6 10 1)6
cm
La tensin normal segn direccin z en el prisma queda:
4 C A1 4 4900 1 N
Pz = 2
= 2
= 49 2
a 20 cm
0 0 0
N
ijP = 0 7350 0
cm 2
0 0 49
Variacin de volumen en el prisma:
V = V V0
donde V = I es la deformacin volumtrica lineal (pequeas deformaciones):
x + y + z
V = I = x + y + z = (1 2 ) = 2,12857 10 3
E
y V0 = 4 10 4 cm 3 es el volumen del prisma, resultando:
V = V V0 = (2,12857 10 3 )(4 10 4 ) = 85,1428cm 3

Ejemplo 7.26
Dos paraleleppedos iguales del mismo material y de dimensiones a b c , se colocan a
uno y otro lado de una placa lisa rgida adosados a ella por sus caras a c , de tal forma que
sus ejes de simetra perpendiculares a dichas caras sean coincidentes. Ambos
paraleleppedos, junto con la placa, se introducen en una ranura de anchura igual a dos
veces la longitud de la arista b ms el espesor de la placa. Las paredes de la ranura son
planas, rgidas y perfectamente lisas.
Se aplican respectivamente a los bloques en sus caras superiores y perpendiculares a ellas
fuerzas uniformemente repartidas p1 y p 2 por unidad de superficie.
Conociendo el mdulo de elasticidad E y el coeficiente de Poisson , se pide calcular:
a) Las tensiones principales en ambos bloques
b) Las variaciones de longitud de las aristas de los bloques.

Universidad de Castilla- La Mancha Draft Por: Eduardo W. V. Chaves (2012)


Ciudad Real - Espaa
7 ELASTICIDAD LINEAL 471

a
p1 p2

1 2

c
y

b b

x
Figura 7.12.

Solucin:
Prisma 1
(x1) = 0 ; (y1) ; (z1) = p1

Prisma 2
(x2) = 0 ; (y2) ; (z2) = p 2

Por compatibilidad de tensin:


(y1) = (y2) = y

(y1) + (y2 ) = 0


1 (1)
E
[ ( 1
)] [
y (x1) + (z1) + (y2 ) (x2 ) + (z2 ) = 0
E
( )]
[ (1)
] [
y z + y z = 0 (2)
]
[ + p ] + [
y 1 y + p2 = 0 ]
Resultando
( p1 + p 2 )
y =
2
Prisma 1:
( p1 + p 2 )
(x1) = 0 ; (y1) = ; (z1) = p1
2
Prisma 2:

Universidad de Castilla- La Mancha Draft Por: Eduardo W. V. Chaves (2012)


Ciudad Real - Espaa
472 PROBLEMAS RESUELTOS DE MECNICA DEL MEDIO CONTINUO

( p1 + p 2 )
(x2 ) = 0 ; (y2) = ; (z2 ) = p 2
2
Las deformaciones en cada prisma viene dadas por:
Prisma 1:

(x1) =
1 (1)
E
[ (
x (y1) + (z1) =

2E
)]
[( p1 + p 2 ) + 2 p1 ]
(y1)
1
[ (
= (y1) (x1) + (z1) =
E

2E
)]
( p1 p 2 )

(z1)
1
[ (
= (z1) (x1) + (y1) =
E
)]
1 2
2E
[
( p1 + p 2 ) 2 p1 ]
Prisma 2:

(x2 ) =
E
[
1 ( 2)
(
x (y2 ) + (z2 ) = )]

2E
[( p1 + p 2 ) + 2 p 2 ]
(y2 )
1
[ (
= (y2) (x2 ) + (z2 ) =
E
)]

2E
( p 2 p1 )

(z2 )
1
[ (
= (z2) (x2 ) + (y2 ) =
E
)]
1 2
2E
[
( p1 + p 2 ) 2 p 2 ]

Variacin de las aristas:


Prisma 1 Prisma 2

a a
a (1) = (x1) a = [( p1 + p 2 ) + 2 p1 ] a ( 2 ) = (x2) a = [( p1 + p 2 ) + 2 p 2 ]
2E 2E
b b (7.75)
b (1) = (y1) b = ( p1 p 2 ) b ( 2 ) = (y2 ) b = ( p 2 p1 )
2E 2E
c (1) = (z1) c =
2E
[
c 2
( p1 + p 2 ) 2 p1 ] c ( 2 ) = (z2 ) c =
2E
[
c 2
( p1 + p 2 ) 2 p 2 ]

Ejemplo 7.27
Una presa de gravedad de perfil triangular est construida mediante hormign de peso
5
especfico ( es el peso especfico del agua), siendo su forma y dimensiones
2
transversales las indicadas en la Figura 7.13. La solucin de tensiones (campo de tensin)
para este problema de deformacin plana es conocida y viene dada por:
11 = x 2

22 = ( x1 3 x 2 )
2
12 = x1
1
Considerar: Coeficiente de Poisson: = ; Mdulo de elasticidad longitudinal E .
4
Se pide:

Universidad de Castilla- La Mancha Draft Por: Eduardo W. V. Chaves (2012)


Ciudad Real - Espaa
7 ELASTICIDAD LINEAL 473

a) Representacin grfica de las fuerzas de superficies que debe ejercer el terreno sobre el
lado AB , para que la solucin indicada sea la correcta;
b) Tensiones Principales en los puntos A y B . A partir del crculo del Mohr en tensiones,
obtener los valores extremos de las tensiones en los respectivos puntos.
c) Obtener el campo de deformacin en la presa.

O x1

45 = g
g -aceleracin de la gravedad
- densidad de masa
h
kg m N
[ ] = 3 2
= 3
5 m s m

2

A B

x2
Figura 7.13.
NOTA: Aunque en la literatura se denomina de peso especfico, en realidad es el
r r r
mdulo de la fuerza msica por unidad de volumen, i.e. = p = b = g , donde b es la

fuerza msica por unidad de masa b = [r ] N m


= 2 . Recordar que, en el Sistema Internacional
kg s
de Unidades el trmino especfico se refiere siempre a por unidad de masa, que no es el caso
de , el trmino correcto sera densidad de peso, por decir algo, ya que en el SI el trmino
densidad siempre se refiere a por unidad de volumen.
Solucin:
a) Campo de tensin y de deformacin en la presa:
x 2 x1 0 11 12 0

ij = x1 ( x1 3 x 2 ) 0 ; ij = 12 22 0
2
33 0 0 0
0 0

Obtenemos la fuerza de superficie a travs del vector traccin t (n) = n . Para el lado AB

tenemos como normal el vector n i = [0,1,0] :


t 1 ( AB ) x 2 x1 0 0 x1
( AB )
t 2 = x1 ( x1 3 x 2 ) 0 1 = ( x1 3 x 2 )
t ( AB ) 2 2
3 0 0 33 0 0

Universidad de Castilla- La Mancha Draft Por: Eduardo W. V. Chaves (2012)


Ciudad Real - Espaa
474 PROBLEMAS RESUELTOS DE MECNICA DEL MEDIO CONTINUO

Fuerza de superficie en la base de la presa:

h h
A B A B
3h
h 2 h

( AB ) ( AB )
t1 (segn direccin x1 ) t2 (segn direccin x 2 )

O x1

45

5

2

B
A

t (n)
x2

b) Fijemos que 33 ya es una tensin principal. Partiendo de = Tr ( )1 + 2 podemos


obtener 33 :
ij = Tr ( ) ij + 2 ij 33 = Tr ( ) 33 + 2 33 33 = Tr ( )

A continuacin determinamos Tr ( ) . Para ellos hacemos el doble producto escalar de


= Tr ( )1 + 2 con el tensor identidad de segundo orden, resultando:
: 1 = Tr ( )1 : 1 + 2 : 1
Tr ( ) = 3Tr ( ) + 2 Tr ( ) = [3 + 2 ]Tr ( )
Tr ( ) + 22 + 33
Tr ( ) = = 11
3 + 2 3 + 2
Luego la componente 33 queda definida como:

Universidad de Castilla- La Mancha Draft Por: Eduardo W. V. Chaves (2012)


Ciudad Real - Espaa
7 ELASTICIDAD LINEAL 475


33 = Tr ( ) = (11 + 22 + 33 )
3 + 2

33 33 = (11 + 22 )
3 + 2 3 + 2

33 1 = (11 + 22 )
3 + 2 3 + 2

33 = (11 + 22 )
2( + )
Reemplazando los valores de 11 , 22 , obtenemos que:

33 = (11 + 22 ) = x 2 + ( x1 3x 2 ) = [x1 5 x 2 ] = [x1 5 x 2 ]
2( + ) 2 2 8

donde hemos considerado que = .
2( + )
El estado tensional en el punto A( x1 = 0; x 2 = h) viene dado por:

x 2 x1 0 h 0 0 1 0 0
3h 3
ij( A) = x1 ( x1 3 x 2 ) 0 = 0 0 =0 0 h
2 2 2
5h 5
0

0 [x1 5 x 2 ] 0 0 0 0
8 8 8
Fijemos que estas componentes ya son las tensiones principales en el punto A .
Crculo de Mohr en tensiones en el punto A :

S ( h)

S max = 0,4375

1,5 1 0,625 N (h)

El estado tensional en el punto B( x1 = h; x 2 = h) viene dado por:



x 2 x1 0 h h 0 1 1 0

ij( B ) = x1 ( x1 3 x 2 ) 0 = h (h 3h) 0 = 1 1 0 h
2 2 1
0 0
0

0 [x1 5 x 2 ] 0 0 [h 5h]
2
8 8

Universidad de Castilla- La Mancha Draft Por: Eduardo W. V. Chaves (2012)


Ciudad Real - Espaa
476 PROBLEMAS RESUELTOS DE MECNICA DEL MEDIO CONTINUO

Las tensiones principales en el punto B( x1 = h; x 2 = h) vienen dadas por:


1 1
=0 (1 ) 2 1 = 0 (1 ) 2 = 1 ( 1 ) = 1
1 1
1 = 2

2 = 0

S ( h)

S max = 1

2 0,5 0 N (h)

c) Podemos obtener la expresin del campo de deformacin partiendo de la expresin:


= Tr ( )1 + 2 :
= Tr ( )1 + 2
2 = Tr ( )1
1
= Tr ( )1
2 2
Tr ( )
Recordemos que anteriormente hemos obtenido que Tr ( ) = , luego:
3 + 2
1
= Tr ( )1
2 2
1
= Tr ( )1
2 2 (3 + 2 )
Tambin podemos expresar la relacin anterior en funcin de los parmetros E y :
E 1 (1 + )
=G = =
2(1 + ) 2 E
(3 + 2 ) 1 1
E= =
+ (3 + 2 ) E ( + )
1
= =
2 (3 + 2 ) 2 E ( + ) E
Luego:
1
= Tr ( )1
2 2 (3 + 2 )
(1 + )
= Tr ( )1
E E

Universidad de Castilla- La Mancha Draft Por: Eduardo W. V. Chaves (2012)


Ciudad Real - Espaa
7 ELASTICIDAD LINEAL 477

La traza de viene dada por:


Tr ( ) = 11 + 22 + 33
5
= ( x 2 ) + ( x1 3 x 2 ) + ( x1 5 x 2 ) = ( x1 5 x 2 )
2 8 8
1
Pudiendo as obtener las componentes del tensor de deformaciones con = :
4
5 5
ij = ij ( x1 5 x 2 ) ij
4E 32 E

x 2 x1 0 1 0 0
5 5
ij = x1 ( x1 3 x 2 ) 0 ( x1 5 x 2 ) 0 1 0
4E 2 32 E
0 0 1
0

0 [x1 5 x 2 ]

8
1
8 ( x1 3 x 2 ) x1 0
5 1
= x1 (3 x1 + 7 x 2 ) 0
4E 8
0 0 0

Ejemplo 7.28
Un cubo metlico que tiene longitud de arista a = 0,20m se sumerge en el mar a una
profundidad z = 400m .
Conociendo el mdulo de elasticidad longitudinal del metal E = 21 1010 Pa , el coeficiente
de Poisson = 0,3 , calcular la variacin de volumen que experimenta el cubo sumergido.
Considerar la aceleracin de la gravedad igual a g = 10m / s 2 .
OBS.: Aunque la densidad de masa vara con la temperatura, salinidad, y presin
(profundidad) considerar la densidad de masa del agua del mar igual a = 1027 kg / m 3 .

Universidad de Castilla- La Mancha Draft Por: Eduardo W. V. Chaves (2012)


Ciudad Real - Espaa
478 PROBLEMAS RESUELTOS DE MECNICA DEL MEDIO CONTINUO

Solucin:
Debido a la profundidad y a las dimensiones del cubo podemos tomar como una buena
aproximacin que todo el cubo est sometido a una misma presin.

h = 400m p

p
p

Figura 7.14.

La presin podemos obtener a partir de la primera ley de Newton F = ma = V g (peso de


la columna de agua) y dividiendo por el rea:
F V g Ah g kg m kg m
p= = = = gh = 1027 3 10 2 400m = 4,108 10 6 2 2 = 4,108 10 6 Pa
A A A m s m s
Luego, las componentes del tensor de tensiones en el cubo vienen dadas por:
p 0 0 4,108 0 0

ij = 0 p 0 = 0 4,108 0 MPa
0 0 p 0 0 4,108
Como slo tenemos componentes normales de tensin y el material es istropo, slo habr
componentes normales de deformacin e iguales:

z = y = x =
1
[ (
x y + z = )] 1
10
[ 4,108 0,3 ( 4,108 4,108 )] 10 6
E 21 10
Resultando
z = y = x = 7,82 10 6

En pequeas deformaciones la deformacin volumtrica lineal es igual a la traza del tensor


de deformaciones infinitesimal:
V
= DVL V = Tr ( ) V = V0 Tr ( ) = 0,2 3 ( 2,346 10 5 ) = 1,8768 10 7 m 3
V0

donde hemos considerado que Tr ( ) = 2,346 10 5 .

Universidad de Castilla- La Mancha Draft Por: Eduardo W. V. Chaves (2012)


Ciudad Real - Espaa
7 ELASTICIDAD LINEAL 479

Ejemplo 7.29
Un cilindro macizo, de 0,05m de radio de la base y 0,25m de altura, est constituido por
un material elstico lineal, de mdulo de elasticidad longitudinal E = 3 10 4 MPa y
coeficiente de Poisson = 0,2 . Dicho cilindro se sita entre los pistones de una prensa, que
se pueden considerar infinitamente rgidos, y todo ello se encierra en recipiente hermtico,
como se indica en la Figura 7.15.
Se llena el recipiente con aceite, y mediante el mecanismo adecuado, se eleva la presin en
el fluido hasta 15MPa . Haciendo funcionar la prensa, se aplica una fuerza axil total de
F = 2,35619 10 5 N sobre las bases del cilindro. Esta fuerza axil es el resultado debido a la
accin de la prensa ms el producido por la presin del aceite sobre los pistones, y se puede
considerar uniformemente repartido sobre las bases.
Se pide determinar, en un punto genrico del cuerpo:
a) Las componentes del tensor de tensiones;
b) Las componentes del tensor de deformaciones;
c) Las componentes del campo de desplazamientos ( u , v , w ).

0,25m
x x
A A

Corte AA
F

0,1m

Figura 7.15: Ensayo de compresin triaxial.

Solucin:
a) Tensor de tensiones
F 2,35619 10 5
z = = = 30 MPa
A (0,05) 2
x = y = 15MPa

Las componentes del tensor de tensiones son:

Universidad de Castilla- La Mancha Draft Por: Eduardo W. V. Chaves (2012)


Ciudad Real - Espaa
480 PROBLEMAS RESUELTOS DE MECNICA DEL MEDIO CONTINUO

15 0 0

ij = 0 15 0 MPa
0 0 30
b) Para un material elstico, lineal, homogneo e istropo, tensiones normales slo
producen deformaciones normales, luego:
1
[ (
x = E x y + z )]

1
[
y = y ( x + z )
E
]

1
[ (
z = E z x + y )]

Sustituyendo los valores de las variables obtenemos las siguientes componentes para el
Tensor de Deformaciones
2 0 0
ij = 0 2 0 10 4

0 0 8

c) Campo de desplazamientos
Como estamos en pequeas deformaciones se cumplen las siguientes relaciones:
u v w
x = ; y = ; z =
x y z
Integrando y obteniendo los valores de las constantes de integracin obtenemos finalmente
el campo de desplazamientos:
u = 2 10 4 x ; v = 2 10 4 y ; w = 8 10 4 z

Ejemplo 7.30
Un hexaedro regular, de 0,1m de lado, est constituido por un material cuyas propiedades
mecnica viene representadas por las constantes de Lam: = 8333,33MPa ,
= 12500 MPa .
Mediante una mquina de ensayos adecuada se le impone la deformacin representada en la
Figura 7.16, en la cual todas las caras continan siendo planas, las caras AEFB y DHGC
pasan a ser rombos y las restantes continan siendo cuadradas. En este estado se pide
calcular:
a) El campo de desplazamientos;
b) El campo de deformaciones;
c) El campo de tensiones;
d) El tensor de tensiones en el centro del hexaedro;
e) Las deformaciones principales en el centro del hexaedro;
f) Las tensiones principales en el centro del hexaedro;
g) Las acciones ejercida por la mquina de ensayo sobre las caras ABFE y BCGF .

Universidad de Castilla- La Mancha Draft Por: Eduardo W. V. Chaves (2012)


Ciudad Real - Espaa
7 ELASTICIDAD LINEAL 481

H H G G

tg( ) = 0,001
E
E F F

C = C
D = D y

A = A B = B

x
Figura 7.16: Hexaedro deformado.
Solucin:
a) Segn Figura 7.16 podemos verificar que slo habr componentes tangenciales de
deformacin. Adems verificamos tambin que no hubo desplazamientos segn las
direcciones x y z , luego u = 0 , w = 0 . A travs de una analoga de tringulos podemos
sacar el desplazamiento v :
v
tg( ) = 0,001 = v( z ) = 0,001z
z
Campo de desplazamiento:
u = 0

v( z ) = 0,001z
w = 0

b) Teniendo en cuenta las componentes del tensor de deformaciones:
u 1 v u 1 u
1 1 + +
x xy xz x 2 x y 2 x z
2 2
1 1 1 v u v 1 v
ij = xy y yz = + +
2 2 2 x y y 2 y z
1 1
yz z 1 u 1 v
2 xz 2 x + z +
2 y z
2
z
Concluimos que x = y = z = xy = xz = 0 y la componente yz viene dada por:
v w
yz = + = 0,001
z y

Universidad de Castilla- La Mancha Draft Por: Eduardo W. V. Chaves (2012)


Ciudad Real - Espaa
482 PROBLEMAS RESUELTOS DE MECNICA DEL MEDIO CONTINUO

1 1
x 2
xy
2
xz
1 1 0 0 0
ij = xy y
yz = 0 0 0,0005
2 2
1 1 0 0,0005 0
2 xz yz z
2

E E

v( z )
z

c) Campo de tensiones = Tr ( )1 + 2
Considerando Tr ( ) = 0 , = 8333,33MPa , = 12500MPa , resulta:
0 0 0

ij = 2 (12500) 0 0 0,0005 MPa
0 0,0005 0
0 0 0
= 0 0 12,5 MPa

0 12,5 0

e, f) Deformaciones principales:
0,0005 2 = +0,0005
=0 2 = 0,0005 2 = 0,0005
0,0005 3 = 0,0005
Recordemos que en pequeas deformaciones las direcciones principales de tensiones
coinciden con las direcciones principales de deformaciones, luego podemos aplicar la
expresin = Tr ( )1 + 2 en el espacio principal de deformacin:
0 0 0

ij = 2 (12500) 0 0,0005 0 MPa

0 0 0,0005
0 0 0

= 0 12,5 0 MPa
0 0 12,5

Universidad de Castilla- La Mancha Draft Por: Eduardo W. V. Chaves (2012)


Ciudad Real - Espaa
7 ELASTICIDAD LINEAL 483

g) Para obtener la fuerza total en una cara, multiplicamos la fuerza de superficie por el rea
de la respectiva cara. La fuerza de superficie se obtiene a travs de la expresin del vector
tensin t (n) = n . Para la cara ABFE la normal viene dada por n i = [1,0,0] , luego:

t 1 ( ABFE ) 0 0 0 1 0
( ABFE )
t 2 = 0 0 12,5 0 = 0
t ( ABFE ) 0 12,5 0 0 0
3

Para la cara BCGF la normal viene dada por n i = [0,1,0] , luego


t 1 ( BCGF ) 0 0 0 0 0
( BCGF )
t 2 = 0 0 12,5 1 = 0 MPa
t ( BCGF ) 0 12,5 0 0 12,5
3
Si hacemos el mismo procedimiento para las dems caras verificamos que la representacin
de las fuerzas de superficies viene indicada tal y como se muestra en la figura abajo:

z
H H G
G

E E F F

C = C
D = D y

A = A B = B

Ejemplo 7.31

Sobre el prisma recto de la Figura 7.17 actan las fuerzas F1 = 10 N y F2 = 2 N sobre las
10
caras indicadas. Las longitudes de las aristas del prisma son: AB = 4cm , AD = cm ,
3
AA = 2cm . Sabiendo que el material que constituye el prisma tiene como propiedades
N
mecnicas: Mdulo de Young E = 2,5 10 6 2 y coeficiente de Poisson = 0,25 . Y
cm
1
propiedades trmicas = 5 10 8 .
C

Universidad de Castilla- La Mancha Draft Por: Eduardo W. V. Chaves (2012)


Ciudad Real - Espaa
484 PROBLEMAS RESUELTOS DE MECNICA DEL MEDIO CONTINUO

Se pide:
a) Obtener las tensiones principales; b) Obtener las componentes del vector tensin en el
plano . Es en el plano donde acta la mxima tensin tangencial? Justificar la
respuesta. c) Obtener el valor de las fuerzas F1 y F2 que se deben aplicar para que no haya
desplazamiento, segn las direcciones x1 y x2 , cuando el prisma est sometido a una
variacin de temperatura de T = 20 C .

x2 A2

A1
F2
B


D
A
F1
F1
x1

60
A
F2

x3
Figura 7.17.
a) Campo de tensiones
F1
A 0 0
1 1,25 0 0
10 F2 N
A1 = 8,0 , A2 = 4 ij = 0 0 = 0 0,15 0

3 A2 cm 2
0 0 0 0
0 0


Que son las propias tensiones principales.

Universidad de Castilla- La Mancha Draft Por: Eduardo W. V. Chaves (2012)


Ciudad Real - Espaa
7 ELASTICIDAD LINEAL 485

b)
x2
r
t (n)

n
D
A

x1

60
A

x3

3 1
La normal (vector unitario) tiene componentes: n i = ; ; 0 . Luego, el vector
2 2
r
tensin t (n) viene dado por:
r
t (n) = n

t i(n) = ij n j
3

1,25 0 0 2 1,0825
1
= 0 0,15 0 = 0,075

t i(n)
2
0 0 0 0 0


La componente normal:
r
N = t (n) n = t i(n) n i

3

2
1
N = [1,0825 0,075 0] = 0,9
2
0


La componente tangencial:
r 2 r 2
t (n) = 2N + 2S S = t (n) 2N

donde
1,0825
r r (n ) r (n )
= t t = t i t i = [1,0825 0,075 0] 0,075 = 1,1775
2
(n ) (n )
t (n)
0

Universidad de Castilla- La Mancha Draft Por: Eduardo W. V. Chaves (2012)


Ciudad Real - Espaa
486 PROBLEMAS RESUELTOS DE MECNICA DEL MEDIO CONTINUO

Luego:
r 2
S = t (n) 2N = 1,1775 0,9 2 = 0,60621778

Si dibujamos el crculo de Mohr de tensiones


III = 0,15 0 I = 1,25 N ( N / cm 2 )

Verificamos que en cualquier punto del slido la tensin tangencial mxima est en un
2 2
plano definido por la normal n i = ; ; 0 y tiene como valor mximo
2 2
I III
max = = 0,7 > S
2
c) Consideremos el campo de deformaciones:
1+
= Tr ( )1 + T 1
E E
1+
ij = ij Tr ( ) ij + T ij
E E
Para el caso en particular Tr ( ) = 11 + 22 :
0 0 0 11 0 0 1 0 0
0 0 0 = 1 + 0 22
0 + T Tr ( ) 0 1 0
E E
0 0 33 0 0 0 0 0 1

Luego, montamos el siguiente sistema:


1+ 1+
11 = 0 = E 11 + T E Tr ( ) = E 11 + T E (11 + 22 )


= 0 = 1 + + T Tr ( ) = 1 + + T ( + )
22 E
22 E E
22 E
11 22

Resolviendo el sistema anterior, obtenemos que:
ET N
11 = 22 = = 3,33333 2
(1 ) cm
Luego, las fuerzas vienen dadas por:
F1 = 11 A1 = 26,66666 N

F2 = 22 A2 = 44,44444 N

Universidad de Castilla- La Mancha Draft Por: Eduardo W. V. Chaves (2012)


Ciudad Real - Espaa
7 ELASTICIDAD LINEAL 487

7.2 Ejercicios Propuestos

Problema 7.1
En que casos podemos idealizar la estructura como problemas bidimensionales? Escribir
las simplificaciones de cada caso y dar ejemplos.

Problema 7.2
Sea Q e (N) el tensor acstico elstico definido como:
Q e (N) = N C N componente
s Q e jl = N i C ijkl N k

donde C es el tensor elstico para un material elstico lineal e istropo dado por:
C = 1 + ( + )N N cuyas componentes son: C ijkl = ij kl + ( ik jl + il jk ) .

Probar que:
Q e jl = jl + ( + )N j Nl

Problema 7.3
Hacer el planteamiento del problema elstico lineal, es decir, definir las ecuaciones que
gobiernan el problema elstico.

Problema 7.4
5) Definir el Estado de Tensin Plano y el Estado de Deformacin Plano. Para cada
estado, qu simplificaciones son consideradas? Dar ejemplos de estructuras que se pueden
aproximar por cada estado.

Problema 7.5
6) Cules son las hiptesis para el planteamiento de la teora de la elasticidad lineal?

Problema 7.6
Definir el comportamiento elstico lineal, elstico no-lineal y el inelstico. Para cada
rgimen hacer una grfica representativa tensin-deformacin.

Problema 7.7
Definir material Anistropo e istropo. Definir los tipos de simetra que puede presentar el
material.

Universidad de Castilla- La Mancha Draft Por: Eduardo W. V. Chaves (2012)


Ciudad Real - Espaa
488 PROBLEMAS RESUELTOS DE MECNICA DEL MEDIO CONTINUO

Problema 7.8
Consideremos una barra de seccin circular donde aplicamos una fuerza en las
extremidades de misma direccin y sentido contrario, ver Figura 7.18. La barra est
constituida por un material homogneo, elstico-lineal e istropo.

x3
x2
A

F F

x1

Figura 7.18: Sistema de fuerzas I

a) Dado el sistema el sistema de fuerzas I representado por la Figura 7.18. Justificar y


explicar porque este sistema es equivalente al sistema de fuerzas II dado por la Figura 7.19.

x3
x2

F F
= =
A A
x1

Figura 7.19: Sistema de fuerzas II

b) Obtener las componentes del tensor de tensiones de Cauchy para el sistema de fuerzas
II.
c) Verificar si el sistema II verifica las ecuaciones de equilibrio;
d) Demostrara que en la superficie de la barra est libre de fuerzas de superficies.
e) Obtener las componentes del tensor de deformacin infinitesimal y del tensor spin
infinitesimal;
f) Obtener la mxima tensin normal y la mxima tensin tangencial.

Universidad de Castilla- La Mancha Draft Por: Eduardo W. V. Chaves (2012)


Ciudad Real - Espaa
7 ELASTICIDAD LINEAL 489

Problema 7.9
Dar detalles de la reduccin de la matriz elstica orttropa a la matriz istropa

C11 C12 C13 0 0 0


C C22 C23 0 0 0
12
C C23 C33 0 0 0 Matriz Orttropa
[C ] = 13
0 0 0 C44 0 0
0 0 0 0 C55 0

0 0 0 0 0 C66

C11 C12 C12 0 0 0


C C11 C12 0 0 0
12
C C12 C11 0 0 0 Matriz Istropa
[C ] = 12
0 0 0 1
2
(C11 C12 ) 0 0
0 0 0 0 1
(C11 C12 ) 0
2

0 0 0 0 0 1
2
( C11 C12 )

Universidad de Castilla- La Mancha Draft Por: Eduardo W. V. Chaves (2012)


Ciudad Real - Espaa
490 PROBLEMAS RESUELTOS DE MECNICA DEL MEDIO CONTINUO

Universidad de Castilla- La Mancha Draft Por: Eduardo W. V. Chaves (2012)


Ciudad Real - Espaa
11 Fluidos
11.1 Ejercicios Resueltos

Ejemplo 11.1
Justificar si son ciertas o falsas las siguientes afirmaciones:
a) Si el campo de velocidad es estacionario, el campo de aceleraciones tambin lo es;
b) Si el campo de velocidad es uniforme, el campo de aceleraciones es siempre nulo;
c) Si el campo de velocidades es estacionario y el medio es incompresible el campo de
aceleraciones es siempre nulo.

Solucin:
a) Velocidad estacionaria
r r
v ( x , t) r
=0
t
El campo de aceleraciones queda:
r
vi ( x , t )
a i = v&i = + v k v i ,k = v k vi , k
142 t 43
=0i
r r
r r& v ( x , t ) r r r r r r r r
a=v = + xr v ( x , t ) v ( x , t ) = xr v ( x ) v ( x )
t 144244 3
No depende del tiempo

Luego la suposicin (a) es VERDADERA.


r r r
b) Campo de velocidad uniforme implica que v ( x , t ) = v (t ) , luego:
r r r r
r r& v ( x , t ) r r r r v ( x , t )
a=v = + x v ( x , t ) v ( x, t ) =
r
t 1424 3 t
=0

Luego, la suposicin (b) es FALSA.


r r r
c) Campo de velocidad estacionario implica que v = v ( x ) , no es funcin del tiempo, y un
r r
medio incompresible viene caracterizado por xr v ( x , t ) = 0 , con lo cual se concluye que:
r r
r r& v ( x ) r r r r r r r r
a=v = + xr v ( x ) v ( x ) = xr v ( x ) v ( x )
t
Luego, la suposicin (c) es FALSA.
492 PROBLEMAS RESUELTOS DE MECNICA DEL MEDIO CONTINUO

Ejemplo 11.2
Demostrar las ecuaciones de movimiento de Navier-Stokes-Duhem:

v&i = b i p,i + ( * + * )v j , ji + *vi , jj Ecuaciones de movimiento


r r r r de Navier-Stokes-Duhem (11.1)
v& = b xr p + ( * + * ) xr ( xr v ) + * xr 2 v

Solucin:
Las ecuaciones de movimiento de Navier-Stokes-Duhem son combinacin de las
r r
ecuaciones de movimiento xr + b = v& ( ij , j + b i = v&i ) y las ecuaciones
constitutivas:
= p1 + * Tr (D)1 + 2 *D ij = p ij + * ijD kk + 2 *Dij (11.2)

La divergencia del tensor de tensiones de Cauchy ( xr ) puede ser obtenido como sigue:
ij , j = ( p ij + * ijD kk + 2 *Dij ) j = p, j ij + * ijD kk ( ),j
+ 2 *Dij , j
(11.3)
= p,i + * ijD kk , j + 2 *Dij , j = p,i + *D kk ,i + 2 *D ij , j
Notar que, en esta formulacin, estamos considerando un material homogneo, i.e.
*, j = *, j = 0 .
Adicionalmente, considerando que 2D ij = vi , j + v j ,i y 2D kk = v k ,k + v k , k = 2v k ,k , podemos
obtener que:
2D ij , j = vi , jj + v j ,ij = vi , jj + v j , ji D kk , j = vk ,kj (11.4)
con lo cual la ecuacin (11.2) puede ser reescrita como:
(
ij , j = p,i + *D jj ,i + 2 *Dij , j = p, i + *v j , ji + * vi , jj + v j , ji )
(11.5)
( )
= p,i + * + * v j , ji + *vi , jj

Luego, considerando la ecuacin (11.5) dentro de las ecuaciones de movimiento


( ij , j + b i = v&i ), obtenemos las ecuaciones de movimiento de Navier-Stokes-Duhem
para un material homogneo.
NOTA 1: La forma explcita de la ecuacin (11.1) se presenta a continuacin:
( * + * )v j , ji + vi , jj + b i p,i = v&i
( * + * )(v1,1i + v2, 2i + v3,3i ) + * (vi ,11 + vi , 22 + vi ,33 ) + b i p,i = v&i

( * + * )(v1,11 + v2, 21 + v3,31 ) + * (v1,11 + v1, 22 + v1,33 ) p,1 + b1 = v&1


*
( + )(v1,12 + v2, 22 + v3,32 ) + (v2,11 + u 2, 22 + v2,33 ) p, 2 + b 2 = v&2
* *

*
( + )(v1,13 + v2, 23 + v3,33 ) + (v3,11 + v3, 22 + v3,33 ) p,3 + b 3 = v&3
* *

Universidad de Castilla- La Mancha Draft Por: Eduardo W. V. Chaves (2012)


Ciudad Real - Espaa
11 FLUIDOS 493

* v1 v2 v3 2v 2v 2v
( + )
*
+ + + * 21 + 21 + 21 + b1 p,1 = v&1
x1 x1 x2 x3 x x2 x3
1

* v1 v2 v3 2v 2 v2 2 v2
( + )
*
+ + + * 22 + + + b 2 p, 2 = v&2
x2 x1 x2 x3 x x22 x32
1

( * + * ) v1 + v2 + v3 + * v3 + v3 + v3 + b p = v&
2 2 2

x3 x1 x2 x3 x 2 x32
3 ,3 3
1 x22

NOTA 2: Hemos demostrado en el Ejemplo 1.105 (Captulo 1) que se cumple que:


r r r r r
( a) = ( a) 2 a indicial
ilq qjk a k , jl = a j , ji a i , jj
Con eso podemos obtener que
r 2r r r r r
xr ( xr v ) xr v = xr ( xr v ) xr ( xr v ) indicial
vi , jj = v j , ji ilq qjk vk , jl
con lo cual la ecuacin (11.1) puede ser reescrita como:
(* + * )v j , ji + *vi , jj + bi p,i = v&i
(* + * )v j , ji + * (v j , ji ilq qjk vk , jl ) + bi p, i = v&i
(* + 2 * )v j , ji *ilq qjk vk , jl + bi p, i = v&i
cuya ecuacin en notacin tensorial es:
r r r r
(* + * )[ xr ( xr v )] + * [ xr ( xr v )] + b xr p = v&
[ ]
r r r r r r r
(* + * )[ xr ( xr v )] + * xr ( xr v ) xr ( xr v ) + b xr p = v&
[ ]
r r r r r r
(* + 2 * )[ xr ( xr v )] * xr ( xr v ) + b xr p = v&
En el sistema Cartesiano tenemos que:
r
v = vi e i = v1e 1 + v2 e 2 + v3e 3
r r r r v v v v v v
( v ) rot (v ) = (rot (v ) )i e i = 3 2 e 1 + 1 3 e 2 + 2 1 e 3
x2 x3
142
4 r43
4
x3 x1
142
4 r43
4
x1 x2
142
4 r43
4
=(rot (u ) )1 =(rot (u) )2 =(rot (u) )3

r r r (rot (vr ) )3 (rot(vr ) )2


r r
(rot (v ) )1 (rot (v ) )3
r r
(rot (v ) )2 (rot (v ) )1
( v ) = e 1 + e 2 + e 3
x2 x3 x3 x1 x1 x2
r r
(rot (v ) )3 (rot (v ) )2 v2 v1 v1 v3
x2 x1 x2 x3 x3 x1
x 2 x 3
r r
(rot (v ) )1 (rot (v ) )3 v3 v2 v2 v1
[r r r
]
( v ) i = =
x3 x1 x3 x2 x3 x1 x1 x2
r
(rot (v ) ) r
(rot (v ) )1 v v v3 v2

2
1 3
x1 x2 x x
1 3 x1 x2 x2 x3

NOTA 3: Si estamos tratando con material heterogneo, tenemos que:


ij = p ij + * ijD kk + 2 *D ij
ij , j = ( p ij + * ijD kk + 2 *D ij ) , j = p, j ij + (*D kk ), j ij + 2( *D ij ), j
ij , j = p,i + (*D kk ) ,i + 2( *Dij ), j
Teniendo en cuenta que 2D ij = vi , j + v j ,i y D kk = vk ,k , la ecuacin anterior queda:
[
ij , j = p,i + (*D kk ),i + 2( *D ij ) , j = p,i + (*vk ,k ),i + * (vi , j + v j ,i ) , j ]
por lo tanto

Universidad de Castilla- La Mancha Draft Por: Eduardo W. V. Chaves (2012)


Ciudad Real - Espaa
494 PROBLEMAS RESUELTOS DE MECNICA DEL MEDIO CONTINUO

ij , j + bi = v&i [
p, i + (*vk , k ), i + * (vi , j + v j ,i ) , j + bi = v&i] (11.6)
Notar que
r r
vk ,k = Tr (v ) = ( v )
Dv&i vi vi v v v v
v&i = = + v j = i + i v1 + i v2 + i v3 , y sus componentes explcitas son:
Dt t x j t x1 x2 x3
v1 v1 v v
+ v1 + 1 v2 + 1 v3
t x1 x2 x3
v v v v
ai = v&i = 2 + 2 v1 + 2 v2 + 2 v3
t x1 x2 x3
v3 v3 v v
+ v1 + 3 v2 + 3 v3
t x1 x2 x3

[ ]
* (vi , j + v j ,i ) , j =

x j
[
* (vi , j + v j ,i ) ]
=

x1
[
* (vi ,1 + v1,i ) +

x2
] [
* (vi , 2 + v2,i ) +

x3
]
* (vi ,3 + v3,i ) [ ]

[
2 * (v1,1 ) +

] [
* (v1, 2 + v2,1 ) +

]
* (v1,3 + v3,1 )

[ ]
x1 x2 x3
[ ]
(vi , j + v j ,i ) , j =
*
[
(v2,1 + v1, 2 ) +
*
]
2 ( v2 , 2 ) +
*
[ ]
(v2,3 + v3, 2 )
*
[ ]
x1 x2 x3

[
* (v3,1 + v1,3 ) +

]
* (v3, 2 + v2,3 ) + [
]
2 * (v3,3 ) [ ]
x1 x2 x3
Las tres ecuaciones en (11.6), ( i = 1,2,3 ), son explcitamente dadas por:

*

x
[ r
( v ) + ]


x
[
2 * (v1,1 ) +


x
]
* (v1, 2 + v2,1 ) + [


x
] [
* (v1,3 + v3,1 ) + b1 p,1 = v&1 ]
1 1 2 3
*

x
[ r
( v ) +

]
*
x
[
(v2,1 + v1, 2 ) +


x
]
2 * ( v2 , 2 ) +


x
[ ] [
* (v2,3 + v3, 2 ) + b 2 p, 2 = v&2 ]
2 1 2 3
*

x
[ r
( v ) +

]
*
x
[
(v3,1 + v1,3 ) +


x
]
* (v3, 2 + v2,3 ) +


[
x
] [
2 * (v3,3 ) + b3 p,3 = v&3 ]
3 1 2 3

(11.7)

*

x
[ r
( v ) + 2 * (v1,1 ) +


x
] [
* (v1, 2 + v2,1 ) +


x
] [
* (v1,3 + v3,1 ) + b1 p,1 = v&1 ]
1 2 3
*

x
[ r
( v ) + 2 * (v2, 2 ) +


x
]
* (v2,1 + v1, 2 ) + [


x
] [
* (v2,3 + v3, 2 ) + b 2 p, 2 = v&2 ]
2 1 3
*

x
[ r
( v ) + 2 * (v3,3 ) +


x
] [
* (v3,1 + v1,3 ) +


x
] [
* (v3, 2 + v2,3 ) + b3 p,3 = v&3 ]
3 1 2

(11.8)

Universidad de Castilla- La Mancha Draft Por: Eduardo W. V. Chaves (2012)


Ciudad Real - Espaa
11 FLUIDOS 495

NOTA 4: Para obtener las ecuaciones dinmica del movimiento en un fluido en


movimiento en un esfera, tenemos que considerar la rotacin de la Tierra y su curvatura. La
aceleracin para un sistema fijo fue obtenida en el Ejemplo 4.32:
r r r r r r r r
a f = a r + 2( v r ) + T v r + ( x )
r r r r r r r r (11.9)
a r = a f 2( v r ) T v r ( x )
r r r
donde (2( v r )) es el trmino de Coriolis, ( T v r ) es el trmino debido a la curvatura,
r r r
( x ) es la aceleracin centrfuga, y las componentes de estos trmino vienen dadas
por:
2[3 vr 3 cos( ) 3 vr 2 sin( ) ]
r r
2( v r ) i = 2[3 vr1 sin( )]

2[ v
3 r1 cos( ) ]

vr1vr 2 tan( ) + vr1vr 3 (11.10)
T r 1 2
( v r ) i = r vr1 tan( ) + vr 2 vr 3
x
vr21 vr22
r r r r 2r
( ( r )) i = r
r
Luego, si queremos considerar estos trminos, debemos reemplazar a r dentro de las
ecuaciones de Navier-Stokes-Duhem.

Ejemplo 11.3
Considerando

v&i = bi p,i + (* + * )v j , ji + *vi , jj Ecuacin del movimiento


r r r r (11.11)
v& = b xr p + (* + * ) xr ( xr v ) + * xr 2v Navier-Stokes-Duhem

Demostrar la ecuacin de vorticidad:


r

t
r r
[
+ 2 xr ( v ) anti
* r2 r r
]
= 0 Ecuacin de vorticidad
x
(11.12)

r r r r r
donde es el vector vorticidad y viene dado por rot (v ) xr v . ( )
Solucin:
Teniendo en cuenta la derivada material de la velocidad Euleriana:
r r
Dvi vi v i v r r v ( x , t ) r r r r
v&i = + v j = i + vi , j v j a = v& = + xr v ( x , t ) v ( x , t ) (11.13)
Dt t x j t t

Las componentes resultantes de la operacin vi , j v j son las componentes de la siguiente


r r
operacin ( xr v ) v , ver captulo 1 Ejemplo 1.106, y tambin fue demostrado que:

Universidad de Castilla- La Mancha Draft Por: Eduardo W. V. Chaves (2012)


Ciudad Real - Espaa
496 PROBLEMAS RESUELTOS DE MECNICA DEL MEDIO CONTINUO

r r r r r 1 r r
( xr v ) v = ( xr v ) v + xr (v v )
2
r r r 1
= ( xr v ) v + xr (v 2 ) (11.14)
2
r r 1
= v + xr (v 2 )
2
Luego, la aceleracin Euleriana tambin puede ser representada por:
r r
r r r& v ( x , t ) r r 1 r 2
a ( x, t ) = v = + v + x (v ) (11.15)
t 2
cuya expresin ya ha sido demostrado en el Ejemplo 2.22 (captulo 2).
Teniendo en cuenta (11.13) y (11.14), la expresin (11.11) queda:
v&i = bi p,i + (* + * )v j , ji + *vi , jj
r r r r
v& = b xr p + (* + * ) xr ( xr v ) + * xr 2v
r r
v r r 1 r 2 r 2r
+ v + x (v ) = b xr p + (* + * ) xr ( xr v ) + * xr v (11.16)
t 2
r r
v r r 1 r 2 1 (* + * ) r r r * r 2 r r
+ v + x (v ) b + xr p x ( x v ) v =0
t 2 x
A continuacin tomamos el rotacional de la expresin anterior:
r r 1
r v r r 1 (* + * ) r r r * r 2 r r
x + v + xr (v 2 ) b + xr p
r x ( x v ) v = 0 (11.17)
t 2 x
Recordar del captulo 1 Vol.1 que se cumple que:
[ ]
r r r r r r r
xr xr (v 2 ) = 0 , xr [ xr p ] = 0 , xr [ xr ( xr v )] = 0 , donde hemos aplicado
la definicin que el rotacional del gradiente de un escalar resulta el vector nulo;

r
r
r
[ r r
]r r r r
[r r
] r r r
xr ( xr v ) v = ( xr v )( xr v ) + xr ( xr v ) v xr v ( xr v )
;
r r r r r r r r
xr [ v ] = ( xr v ) + [ xr ] v xr v

r
[2r
]
r r r
[ r 2
r r
] 2 r
xr xr v = xr xr ( xr v ) = xr xr v = xr ; [ ]
r r

r v r r r
x =
r x v = [ ; ]
t t t
r
Considerando que el campo b es conservativo, y teniendo en cuenta que el
rotacional de cualquier campo vectorial conservativo es igual al vector nulo
r r r
xr b = 0 .
Teniendo en cuenta todo lo anterior, la relacin (11.17) resulta:
r
r r r r r r * 2 r
r
+ ( xr v ) + ( xr ) v xr v xr = 0 (11.18)
t
Fijemos que se cumplen las siguientes relaciones:

Universidad de Castilla- La Mancha Draft Por: Eduardo W. V. Chaves (2012)


Ciudad Real - Espaa
11 FLUIDOS 497

(v i j ), i = vi , i j + vi j , i v i , i j = (vi j ), i vi j , i
(11.19)
(v i j ), j = v i , j j + vi j , j vi , j j = (v i j ), j v i j , j = (vi j ), j

que es lo mismo en notacin tensorial que:


r r r r r r
( xr v ) = xr [ v ] ( xr ) v
r r r r r r r (11.20)
( xr v ) = xr [v ] ( xr )v = xr [v ]
r

donde hemos aplicado la definicin que la divergencia del rotacional de un vector es nulo,
r r r
es decir, xr = xr ( xr v ) = 0 . Teniendo en cuenta (11.20), la expresin (11.18)
queda:
r
r r r r r r * r2 r r
+ ( xr v ) + ( xr ) v xr v =0
t x
r
r r r r r r r r * r2 r r
+ xr [ v ] ( xr ) v + ( xr ) v xr [v ] =0
t x
r
r r r r * r2 r r
+ xr [ v ] xr [v ] =0 (11.21)
t x
r
r r r r * 2 r
r
+ xr [ v v ] xr = 0
t
r
[
]
r r *
2 r
r
+ 2 xr ( v ) anti xr = 0
t
Demostrando as la ecuacin de vorticidad (11.12).

Universidad de Castilla- La Mancha Draft Por: Eduardo W. V. Chaves (2012)


Ciudad Real - Espaa
498 PROBLEMAS RESUELTOS DE MECNICA DEL MEDIO CONTINUO

Ejemplo 11.4
r
Hallar la fuerza de traccin E que acta en la superficie cerrada S que contiene al
volumen V de un fluido Newtoniano donde la viscosidad volumtrica es nula.

r
n t (n )

Solucin:
dE i = t i(n ) dS
La fuerza de traccin total viene dada por la siguiente integral:



E i = t i( n ) dS = ij n j dS = ij , j dV (11.22)
S S V

donde se ha empleado la relacin ij n j = t i( n ) .


2
Si la viscosidad volumtrica es nula, entonces * = 0 * = * (condicin de Stokes).
3
Teniendo en cuenta la ecuacin constitutiva de un fluido Newtoniano y la condicin de
Stokes obtenemos que:
ij = p ij + * ij D kk + 2 *D ij
2 *
= p ij ij D kk + 2 *D ij
3
D
= p ij + 2 * D ij kk ij
1442443 3
Dijdev

ij = p ij + 2 *D ijdev

Reemplazando la ecuacin constitutiva anterior en la expresin (11.22) obtenemos que:


E i = ( p ij + 2 *D ijdev ) n j dS
S

Aplicando el Teorema de Gauss:

Universidad de Castilla- La Mancha Draft Por: Eduardo W. V. Chaves (2012)


Ciudad Real - Espaa
11 FLUIDOS 499

( p + 2 D ) dV
* dev
Ei = ij ij ,j
V

= ( p + 2 D ) dV
,j ij
* dev
ij , j
V


= ( p ,i + 2 *D ijdev, j ) dV
V

donde hemos tenido en cuenta que *,j = 0 j , es decir * es un campo escalar homogneo
(material homogneo). La expresin anterior en notacin tensorial queda:

[ ]
r
E= r
x p + 2 * xr (D dev ) dV (11.23)
V

Ejemplo 11.5
Considrese un fluido en reposo de densidad de masa f . Demostrar el Principio de
Arqumedes: Todo cuerpo sumergido en un fluido en reposo experimenta un empuje hacia arriba igual
al peso del volumen del fluido desalojado.
Si el cuerpo tiene densidad de masa s y el campo de fuerzas msicas viene dado por
b i = g i 3 . Obtener la fuerza resultante que acta en el cuerpo y la aceleracin.
Solucin:
r
[
En el Ejemplo 11.15 hemos demostrado que E = xr p + 2 * xr (D dev ) dV . Si el fluido ]
est en reposo tenemos que D dev = 0 y la presin termodinmica es igual a la presin
hidrosttica, p = p 0 . Luego, quedamos con:

r
E= [
V
r
x p 0 ]dV (11.24)

r
E

V -volumen
p0 n

x3 r
Ws
x2

x1

Universidad de Castilla- La Mancha Draft Por: Eduardo W. V. Chaves (2012)


Ciudad Real - Espaa
500 PROBLEMAS RESUELTOS DE MECNICA DEL MEDIO CONTINUO

El peso del volumen del fluido desalojado viene dado por:


r r

= f bdV
f
W (11.25)
V

Aplicando la ecuacin de equilibrio:


r r
xr + f b = 0 ij , j + f b i = 0 i
r
xr = f b ij , j = f b i
r (11.26)
xr ( p 0 1) = f b ( p 0 ij ), j = f b i
r
xr p 0 = f b p 0,i = f b i

Teniendo en cuenta (11.24) y (11.25), concluimos que:


r r r

= f bdV = xr p 0 dV = E
f
W (11.27)
V V

Demostrando as el principio de Arqumedes.


El peso del cuerpo, de densidad de masa s , que est sumergido en el lquido viene dado
por:
r r
W s = s bdV
V

La fuerza resultante que acta en el cuerpo es:


r r r
R = E + Ws
r r

= f bdV + s bdV
V

V

r

= ( s f )bdV
V

Cuyas componentes son:



0

V
V

Ri = ( s f )b i dV = g ( s f ) i 3 dV =

0

g ( f s )dV
V

Verificando as que: si el cuerpo tiene densidad de masa menor que la densidad de masa del
r r
lquido, por ejemplo si el cuerpo es un gas, el cuerpo asciende, i.e. f > s R > 0 , y caso
r r
contrario en cuerpo desciende. Si tenemos en cuenta que R = m s a , donde m s es la masa
total del cuerpo sumergido, podemos obtener la aceleracin del cuerpo como:
s g ( f
s)
g ( s )dV g ( s)
f f s
dV dV
R3 s s
g ( f
s)
a3 = =V =V = V
=
ms ms ms ms s

Universidad de Castilla- La Mancha Draft Por: Eduardo W. V. Chaves (2012)


Ciudad Real - Espaa
11 FLUIDOS 501

NOTA: Es interesante observar que si el medio ( f ) es tal que f = 0 tenemos que


a 3 = g , es decir, la aceleracin es independiente de la masa. Comprobando as, como hizo
Galileo, que a travs de experimento sencillo verific que cuerpo en cada libre era
independiente de la masa. Por ejemplo, en la luna donde podemos considerar que la
densidad de masa del aire es igual a cero, dos cuerpos con distintas masas en cada libre, e.g.
una pluma y un martillo, tendrn la misma aceleracin y alcanzarn la superficie de la luna
en el mismo tiempo.

Ejemplo 11.6
Probar que el tensor desviador de tensiones dev es igual dev , donde ij = p ij + ij .
Solucin
Si kk = 3 p + kk

ijdev = ij
kk
ij = p ij + ij
( 3 p + kk ) = kk = dev
ij ij ij ij
3 3 3

Ejemplo 11.7
Deducir la ecuacin unidimensional de continuidad de masa para el flujo de un fluido
incompresible no viscoso a travs de un tubo. Considere V el volumen comprendido entre
dos secciones transversales arbitrarias A y B .
B
n B
r
vB

n A
V
r
vA

Solucin:
Para un medio incompresible, la densidad de masa es independiente del tiempo & = 0 , y
r
adems teniendo en consideracin la ecuacin de continuidad & + v i ,i = ( xr v ) = 0 , se
puede decir que:
r
xr v = 0 v k ,k = 0 (11.28)

Tomando para un volumen V :


r
r
x v dV =0 v k , k dV =0 (11.29)
V V

Aplicando el teorema de Gauss:


r
v n dS = 0
S
v
S
k dS
kn =0 (11.30)

Universidad de Castilla- La Mancha Draft Por: Eduardo W. V. Chaves (2012)


Ciudad Real - Espaa
502 PROBLEMAS RESUELTOS DE MECNICA DEL MEDIO CONTINUO

Luego:
rA rB
v n A dS + v n B dS = 0
SA SB

Velocidad uniforme y perpendicular a las secciones SA y SB


r r
v A = v A n A ; v B = v B n B

Reemplazando la velocidad en la integral, resulta:


v A n A n A dS + v B
n
B
n B dS = 0
A B
S S

v AS A = vBS B

Ejemplo 11.8
El campo de velocidad de un gas en movimiento a travs de una tubera, cuyo eje
prismtico es x 2 , se define mediante sus componentes:

v1 = 0 ; v 2 = 0,02 x 2 + 0,05 ; v3 = 0

kg
Cuando el gas pasa por x 2 = 0 la densidad de masa es igual a 1,5 . Hallar para
m3
x 2 = 5m .
Solucin:
r r r
El campo de velocidad es estacionario, v = v ( x ) . De la ecuacin de continuidad de masa:
r r
+ xr ( v ) = 0 xr ( v ) = 0
t
r
Luego, podemos concluir que v es una constante:
( v 2 ) x2 = 0
= ( v 2 ) x2 =5

v 2 ( x 2 = 0) = 0,02 0 + 0,05 = 0,05 y v 2 ( x 2 = 5) = 0,02 5 + 0,05 = 0,15 con eso:


( v 2 ) x2 = 0
= ( v 2 ) x2 =5

kg
1,5 0,05 = 0,15 ( x 2 = 5) = 0,5
m3
Solucin alternativa:
r
xr ( v ) = 0 indicial
( v i ) ,i = ,i v i + v i ,i = 0

v v v v
vi + i = v1 + v2 + v3 + 1 + 2 + 3 = 0
x i xi x1 x 2 x 3 x1 x 2 x 3
Resultando:

Universidad de Castilla- La Mancha Draft Por: Eduardo W. V. Chaves (2012)


Ciudad Real - Espaa
11 FLUIDOS 503


(0,02 x 2 + 0,05) + (0,02) = 0
x 2
0,02
= x
(0,02 x 2 + 0,05) 2
Integrando la expresin anterior, obtenemos que:
Ln = Ln(0,02 x 2 + 0,05) + LnC =
C
Ln = Ln
(0,02 x 2 + 0,05)
C
=
(0,02 x 2 + 0,05)
Condiciones de contorno, x 2 = 0 = 1,5 , con lo cual obtenemos que C = 0,075 :
0,075 x2 = 5 0,075 kg
= = = 0,5 3
(0,02 x 2 + 0,05) (0,02 5 + 0,05) m

Ejemplo 11.9
Las componentes del tensor de tensin en un punto de un fluido Newtoniano, con una
viscosidad volumtrica nula, son:
6 2 1
ij = 2 9 4 Pa
1 4 3

Determinar las componentes del tensor de tensin viscoso.


Solucin:
En el caso de viscosidad volumtrica nula (condicin de Stokes) tenemos p = p = p 0 , y
adems se puede obtener que:
ij = p ij + ij
2 *
* = * + =0
3
ii = 3 p

p=
ii
=
( 6 9 3) = 6
3 3
Luego:
ij = ij + p ij
6 2 1 6 0 0 0 2 1
= 2 9 4 + 0 6 0 = 2 3 4 Pa

1 4 3 0 0 6 1 4 3

Ejemplo 11.10
kk
Determinar las condiciones bajo las cuales la presin normal media p = = m es
3
igual a la presin termodinmica p para un fluido Newtoniano.

Universidad de Castilla- La Mancha Draft Por: Eduardo W. V. Chaves (2012)


Ciudad Real - Espaa
504 PROBLEMAS RESUELTOS DE MECNICA DEL MEDIO CONTINUO

Solucin:
Fue deducido que:
1 kk
ijdev = 2 *D ijdev ; kk = p + k *D ii ; = p = p + *D kk
3 {3
p

As, para que la igualdad p = p se cumpla, hay que cumplir que:


D ii = 0 2
* = 0 ; ; * = *
Tr (D) = 0 3

Ejemplo 11.11
kk
Determinar las condiciones bajo las cuales la presin normal media p = es igual a la
3
presin termodinmica p , para un fluido Newtoniano.
Solucin:
s ij = 2 *Dij
kk = 3 p + 3 *Dkk
kk
* = 0 p= (11.31)
3
2 * 2
* = * + =0 * = *
3 3
Dkk = 0

Ejemplo 11.12
Un fluido perfecto y barotrpico tiene una ecuacin cintica de estado definida por
p
=0 + , donde k es constante. Obtener el campo de presin (distribucin de presin)
k
en rgimen cuasiesttico (aceleracin nula) bajo accin del campo gravitatorio
b i = [0 0 g ] .
T

Solucin:
Ecuacin constitutiva de un fluido perfecto:
= p1
Las ecuaciones del movimiento quedan:
r r
xr + b = v& indicial
ij , j + b i = v&i = 0 i
{
= 0i
( p ij ), j + b i = 0 i (11.32)
p, j ij + b i = 0 i
r r
xr p + b = 0 tensorial
p, i + b i = 0 i

Considerando el vector de fuerzas msicas b i = [0 0 g ] concluimos que:

Universidad de Castilla- La Mancha Draft Por: Eduardo W. V. Chaves (2012)


Ciudad Real - Espaa
11 FLUIDOS 505

p p
(i = 1) + b1 = 0 = 0 p = p ( x1 , x 2 , x3 )
x 1 x 1
p p
p, i + b i = 0 i (i = 2) + b2 = 0 = 0 p = p ( x1 , x 2 , x3 ) (11.33)
x 2 x 2
p dp ( x3 )
(i = 3) x + b 3 = 0 dx + g = 0
3 3

Con lo cual hemos concluimos que el campo de presin es slo funcin de la coordenada
x 3 , i.e. p = p( x3 ) .
El hecho de que sea un fluido barotrpico, eso implica que la densidad de masa es
nicamente funcin de la presin = ( p ) . Esta relacin es precisamente la ecuacin
cintica del enunciado:
p
= ( p) =0 +
k
Luego:
dp( x3 )
+ g = 0
dx3
dp( x3 ) p
+ 0 + g = 0 (11.34)
dx3 k
dp( x3 ) g
+ p = 0 g
dx3 k
La solucin de esta ecuacin diferencial es la suma de la solucin homognea y una
particular:
g
dp( x 3 ) g ( x3 )
Solucin homognea: + p = 0 p = Cexp k
dx3 k
Solucin particular: p = k 0
Luego:
g
( x )
p = Cexp k 3 k 0

Ejemplo 11.13
Un gas perfecto es un fluido ideal e incompresible en el cual en la ausencia de fuente de
calor, el movimiento es barotrpico, la presin es proporcional a , donde es una
constante y > 1 . Demostrar que cuando r = 0 (fuente interna de calor), la energa interna
especfica para un gas perfecto viene dada por:
1 p
u= + constante
( 1)
Solucin:
Para el problema propuesto, la ecuacin de energa se resume a:
r
u& = : D xr q + r = : D
u& : D = 0

Universidad de Castilla- La Mancha Draft Por: Eduardo W. V. Chaves (2012)


Ciudad Real - Espaa
506 PROBLEMAS RESUELTOS DE MECNICA DEL MEDIO CONTINUO

Para un gas perfecto la tensin es un tensor esfrico y viene dado por:


( p ) = p1
donde p es la presin termodinmica. Luego, la ecuacin de energa queda:
u& : D = 0
u& + p1 : D = 0
u& + pTr (D) = 0
r
u& + p xr v = 0
Para un movimiento barotrpico la energa interna especfica es una funcin de la densidad
de masa, u = u ( ) , luego:
r
u& + p xr v = 0
u r
& + p xr v = 0

D r r
Teniendo en cuenta la ecuacin de continuidad de masa + xr v = 0 & = xr v ,
Dt
la ecuacin de energa queda:
u r
& + p xr v = 0

u r r
xr v + p xr v = 0

u r
2 + p xr v = 0
12
0 3

con lo cual hay que cumplir que:


u u p
2 + p=0 = 2

Como la presin es proporcional a , podemos decir que p = p ( ) = k , donde k es


una constante de proporcionalidad, luego:
u u k
2 + p=0 = = k ( 2 )
2
Integrando la expresin anterior, obtenemos que:
k
u= + constante
( 1)
1 p
= + constante
( 1)
Ejemplo 11.14
r
Un fluido se mueve alrededor de una esfera de radio R con velocidad v , siendo sus
componentes en coordenadas esfricas ( r , , ) dadas por:

Universidad de Castilla- La Mancha Draft Por: Eduardo W. V. Chaves (2012)


Ciudad Real - Espaa
11 FLUIDOS 507

R 3 3R
v r = c 3 + 1 cos()
2r 2r
R 3
3R (11.35)
v = c 3 + 1 sin()
4r 4r
v = 0

donde c es una constante positiva.


Es un movimiento isocrico? Probar la respuesta.
r
Nota: Dado un vector u , la divergencia de este vector en coordenadas esfricas es:
r r u 1 u 1 u cot() 2
div u xr u = r + + + u + u r
r r r sin() r r
Solucin: Para demostrar que un movimiento es isocrico tenemos que probar que
r
xr v = 0
Obtenemos las siguientes derivadas:
v r R 3 3R 3R 3 3R
= c 3 + 1 cos() = c + 2 + 1 cos()
r r 2r 2r 2r
4
2r

v R 3 3R R 3 3R
= c +
1 sin( ) = c 3 + 1 cos()
4r 3 4r 4r 4r
Sacamos ahora la divergencia de la velocidad:
r v 1 v 1 v cot() 2
xr v = r + + + v + v r

r r r sin() { r r
=0

3R 3
3R 1 R 3 3R
= c 4
+ 2
+ 1 cos() + c


4r 3 + 4r 1 cos() +
2 r 2 r r
cos() 1 R 3 3R 2 R 3 3R
+ c 3 + 1 sin() + c 3 + 1 cos()
sin() r 4r 4r r 2r 2r
Simplificando obtenemos que:
r
xr v = 0
Comprobando que es un movimiento isocrico.

Ejemplo 11.15
El fluido barotrpico del interior de la tubera de la figura tiene por ecuacin cintica de
estado:

p = Ln ; ( y 0 constantes)
0
p - presin, - densidad de masa.
Calcular en rgimen estacionario la presin de salida p ( 2) en funcin de las dems variables
de la figura. Justificar la frmula empleada.

Universidad de Castilla- La Mancha Draft Por: Eduardo W. V. Chaves (2012)


Ciudad Real - Espaa
508 PROBLEMAS RESUELTOS DE MECNICA DEL MEDIO CONTINUO

p (1)
p( 2)
v (1)
v( 2)
S (1)
S (2)

p - presin, v - velocidad, S - rea de la seccin.

Solucin:
Segn el principio de conservacin de masa:
D
Dt V
dV = 0

Dada una propiedad ( x , t ) se cumple que:


D r D ( x, t ) DdV
Dt V
( x , t )dV =
V
Dt
dV + ( x , t )
Dt
D ( x, t ) r r
= dV + ( x , t ) xr v ( x , t )dV
V
Dt
D ( x, t ) r r
= + ( x , t ) xr v ( x , t ) dV
V
Dt
( x , t ) r r r r
= + xr ( x , t ) v ( x , t ) + ( x , t ) xr v ( x , t ) dV
V
t
( x , t )
(( x, t )v ( x, t )) dV
r r
=
V
t
+ xr

( x , t ) r r
=
dV + xr [( x, t )v ( x, t )]dV
V
t V

( x , t ) r r
=
V
t
dV + [( x, t )v ( x, t )] n dS = 0
S

haciendo = ( x , t ) obtenemos que:


D r ( x , t ) r r
( x , t )dV = dV + [ ( x , t )v ( x , t )] n dS = 0

Dt V V
t S

( x , t )
Aplicando la condicin de rgimen estacionario = 0 , resultando as que:
t

Universidad de Castilla- La Mancha Draft Por: Eduardo W. V. Chaves (2012)


Ciudad Real - Espaa
11 FLUIDOS 509

r r r r r r
[ ( x)v ( x)] n dS = 0
S

S
[ ( x )v ( x )] n dS + [ ( x )v ( x )] n dS = 0
S
(1) (2)

S (1) v (1) dS +
S
( 2 ) v ( 2 ) dS =0
(1) (2)

(1) v (1) S (1) + ( 2 ) v ( 2) S ( 2) = 0


Resultando que:

(1) v (1) S (1) = ( 2) v ( 2) S ( 2) (11.36)

Recordar que q = v es el flujo de masa que tiene por unidad [q] =


r r r kg
.
m2s
Introduciendo la ecuacin cintica de estado se obtiene la siguiente expresin:
p p
p
p = Ln = Ln exp = ( x ) = 0 exp
0 0 0
Luego:
(1) v(1) S (1) = ( 2) v( 2) S ( 2)
p ( 1) p( 2 )


0 exp
v(1) S (1) = 0 exp
v( 2 ) S ( 2 )
p ( 2 ) p (1 )
v(1) S (1)

exp
=
v( 2 ) S ( 2 )
p( 2 ) p(1) v(1) S (1) (11.37)
= Ln
v( 2 ) S ( 2 )

v(1) S (1)
p( 2 ) p(1) = Ln
v( 2 ) S ( 2 )

v(1) S (1)
p( 2 ) = p(1) + Ln
v( 2 ) S ( 2 )

NOTA: El caudal, muchas veces representado por Q , es el flujo total especfico, i.e.:
r r r r
q dS v dS r r m3
Q= = = v dS Caudal (11.38)
S
S
S s

q r r
Verificamos las unidades [Q ] =
dS kg m 3 2 m 3
= 2
S m s kg
m =
s
. En este ejemplo hemos

obtenido que (1) v (1) S (1) = ( 2) v ( 2) S ( 2) , que puede ser reescrito como:
(1) v (1) S (1) = ( 2) v ( 2) S ( 2) (1) Q(1) = ( 2) Q( 2)
Para el caso particular de un medio incompresible, (1) = ( 2) , resulta que:
v (1) S (1) = v ( 2) S ( 2) Q(1) = Q( 2 ) (ver Ejemplo 11.7)

Universidad de Castilla- La Mancha Draft Por: Eduardo W. V. Chaves (2012)


Ciudad Real - Espaa
510 PROBLEMAS RESUELTOS DE MECNICA DEL MEDIO CONTINUO

Ejemplo 11.16
Partiendo de la ecuacin de Navier-Stokes-Duhem, obtener la ecuacin de Bernoulli:

p v2
gh + + = constante Ecuacin de Bernoulli (11.39)
2

Hiptesis: Fluido no viscoso, incompresible. Campo de velocidad estacionario e irrotacional.


Solucin:
r
Considerando el medio incompresible ( xr v ) = 0 , y no viscoso (* = * = 0) , la ecuacin
de Navier-Stokes-Duhem se resume a:
v&i = bi p,i + (* + * )v j , ji + *vi , jj
r r r r
v& = b xr p + (* + * ) xr ( xr v ) + * xr 2v (11.40)
r r
v& = b xr p
Que es la ecuacin de Euler del movimiento. La derivada material de la velocidad, ver
expresin (11.15), se resume a:
r
r v r r 1 r 2 1
v& = + v + x (v ) = xr (v 2 )
t 2 2
r
v r
donde hemos considerado el campo de velocidad estacionario = 0 e irrotacional
t
r r r r r
xr v = rot v = = 0 . Con eso la ecuacin (11.40) puede ser reescrita como:
r 1 r 2 r 1 r
xr (v 2 ) = b xr p x (v ) b + xr p = 0 (11.41)
2 2
Consideremos que la fuerza msica (campo conservativo) podemos representarla por
r
b = xr , donde es un potencial y adems considerando que el campo de densidad de
p 1
masa es homogneo se cumple que xr = xr p . Luego, (11.41) queda:

p v2 p v2
xr + + = 0i
+ + = constante (11.42)
2 2

Considerando que el potencial dado por = gh , donde g es la aceleracin de la gravedad


y h la altura piezomtrica, obtenemos la conocida ecuacin de Bernoulli:
p v2
gh + + = constante
2

v 2 p N m 3 Nm J m 2
Verificando las unidades [gh] = = = 2 = = = , que es la unidad de
2 m kg kg kg s 2
energa especfica, es decir, unidad de energa por unidad de masa.
Es interesante resaltar que la ecuacin de Bernoulli es la aplicacin de la conservacin de la
energa y se supone que en el sistema no hay prdida de energa:

Universidad de Castilla- La Mancha Draft Por: Eduardo W. V. Chaves (2012)


Ciudad Real - Espaa
11 FLUIDOS 511

energa constante
energa en A energa en B

p
A
p

v2 B

2 A
v2
gh A A 2 B

h B gh B

Ejemplo 11.17
Se considera un fluido perfecto e incompresible que circula por el canal de la figura en
rgimen estacionario. Determinar el valor de H .
v (1) = 1m / s

h(1)
v( 2 ) = 2 m / s

H
h( 2)

Hiptesis: No se considera prdida de energa.

Solucin:
Ecuacin de continuidad:
v (1) 1
v (1) h(1) = v ( 2 ) h( 2) h( 2 ) = h(1) = h(1)
v( 2) 2
Ecuacin de Bernoulli:
v (21)
( H + h(1) ) + 0 +
2g v (22 ) v (21) 1 3
H = h( 2 ) h(1) + H = h(1) +
v (22) 2g 2 2g
h( 2 ) + 0 +
2g

Universidad de Castilla- La Mancha Draft Por: Eduardo W. V. Chaves (2012)


Ciudad Real - Espaa
512 PROBLEMAS RESUELTOS DE MECNICA DEL MEDIO CONTINUO

Ejemplo 11.18
Un depsito circular de gran dimetro que est lleno de agua que vierte por un pequeo
orificio lateral situado a una altura H por debajo del nivel del agua del depsito. Si el
caudal vertido es Q , obtener el dimetro D del orificio.

Hiptesis: Considerar que H no


A vara con el tiempo (rgimen
estacionario). Considerar que en la
H seccin BB la presin en el flujo
B es igual a la presin atmosfrica:

( p patm ) patm

C B

C
B Presin Presin

Seccin CC Seccin BB

Solucin:
El agua podemos considerar como un fluido perfecto e incompresible. Aplicando la
Ecuacin de Bernoulli:
p v2
z+ + = ctte
g 2g
donde se cumple que:
p atm
Punto A H+ +0
g
2
v ( B ) = 2 gH
p v( B )
Punto B 0 + atm +
g 2 g
Teniendo en cuenta que el caudal viene dado por Q = v ( B ) S ( B ) , concluimos que:

D 2 4Q
Q = v ( B ) S ( B ) = 2 gH D=
4 2 gH

Universidad de Castilla- La Mancha Draft Por: Eduardo W. V. Chaves (2012)


Ciudad Real - Espaa
11 FLUIDOS 513

Ejemplo 11.19
Considrese un tubera donde se ha introducido un tubo de pitot tal y como se indica en la
Figura 11.1. Obtener la velocidad en el punto 1 en funcin de h(1) y h( 2) . Considerar que
no hay prdida en el sistema.

h( 2)

h(1)

v(1) v( 2) = 0
1 2
h p(1) p( 2 )

Figura 11.1: Tubo de pitot


Solucin:
Aplicando la ecuacin de Bernoulli entre los puntos 1 y 2 , hay que cumplir que:
p (1) v (21) p( 2) v (22 )
gh + + = gh + +
2 2
p (1) v (21) p ( 2)
+ =
2
2( p ( 2 ) p (1) )
v (1) =

Considerando que las presiones en los puntos 1 y 2 vienen dadas respectivamente por:
p (1) = gh(1) ; p ( 2 ) = gh( 2 )

Con lo cual la velocidad v (1) viene definida por:

2( p ( 2) p (1) ) 2( gh( 2 ) gh(1) )


v (1) = = = 2 g (h( 2 ) h(1) )

Universidad de Castilla- La Mancha Draft Por: Eduardo W. V. Chaves (2012)


Ciudad Real - Espaa
514 PROBLEMAS RESUELTOS DE MECNICA DEL MEDIO CONTINUO

Ejemplo 11.20
Considrese un fluido no viscoso e incompresible, y caracterizado por presentar el campo
de velocidad estacionario, irrotacional, e independiente de x3 . Expresar las ecuaciones de
gobierno del problema a travs de un potencial de velocidad y de lneas de corrientes .
Solucin:
Potencial de velocidad: En este ejemplo podemos representar el campo de velocidad a travs de
r
un potencial , i.e. v = xr . Con eso estamos considerando que el campo de velocidad es
r r r r r
conservativo, y como consecuencia el rotacional es igual a cero, i.e. xr v = rot v = = 0 .
Recordar que, un campo cuyo rotacional es cero no necesariamente implica que el campo
sea conservativo, pero todo campo conservativo el rotacional es igual a cero.
Observemos que la velocidad tiene la misma direccin que xr , y es normal a las
isosuperficies, i.e. normal a las superficies de = ctte .
Lnea de corriente: Recordar que una lnea de corriente ( ) es la curva en el instante t donde
la tangente a esta curva (en cualquier punto) es paralela a la velocidad. Dos lneas de
corrientes no pueden cruzarse. Si el campo de velocidad es estacionario la lnea de corriente
coincide con la trayectoria de la partcula.
Volumen de control
(5) = ctte

r r ( 4) = ctte
v ( x)
t Lneas de corriente

( 2) = ctte
xr
(1) = ctte

r
x

Partiendo de la definicin de diferencial total d y de gradiente xr obtenemos la


r
relacin d = xr dx .
r
v = xr
+ d
xr

= ctte d = 0
en la lnea de corriente

Universidad de Castilla- La Mancha Draft Por: Eduardo W. V. Chaves (2012)


Ciudad Real - Espaa
11 FLUIDOS 515

r
Observar que se cumple que xr xr = 0 . El diferencial total dx en la lnea de corriente
en un punto tiene la misma direccin que la velocidad en este punto. Con eso se cumple
que:
r r r
dx v = 0
En componentes cartesianas:
e 1 e 2 e 3
r r r
dx v = dx1 dx 2 dx3 = 0
v1 v2 v3
r
= (v3 dx 2 v 2 dx3 )e 1 + (v 3 dx1 v1 dx3 )e 2 + (v 2 dx1 v1 dx 2 )e 3 = 0
Componentes:
(v3 dx 2 v 2 dx3 ) 0
(dx v ) i = (v 3 dx1 v1 dx3 ) = 0
r r

(v 2 dx1 v1 dx 2 ) 0

Para este ejemplo el campo de velocidad es independiente de x3 , es decir, estamos en el


caso plano x1 x 2 (2D). Con eso quedamos solo con:
0 0
r r
( dx v ) i = 0 = 0

(v 2 dx1 v1 dx 2 ) 0

Con eso concluimos que:


v 2 dx1 v1 dx 2 = 0 (11.43)
Teniendo en cuenta que en una lnea de corriente se cumple que = ctte d = 0 y
r
adems aplicando la definicin d = xr dx , obtenemos que:
r
d = xr dx indicial
d = ,i dxi = 0
d = ,1 dx1 + , 2 dx 2 + ,3 dx3 = 0

d = dx1 + dx 2 + dx3 = 0
x1 x 2 x3
Para el caso 2D tenemos que:

dx1 + dx 2 = 0 (11.44)
x1 x 2
Si comparamos las relaciones (11.43) y (11.44) concluimos que:

v1 = ; v2 = (11.45)
x 2 x1

r
1) Partiendo que el fluido es incompresible: ( xr v ) = 0 obtenemos que:
v1 v 2 v3 2D v1 v 2
v i ,i =
+ + =0 + =0
x1 x 2 x3 x1 x 2
r
Teniendo en cuenta que v = xr , concluimos que:

Universidad de Castilla- La Mancha Draft Por: Eduardo W. V. Chaves (2012)


Ciudad Real - Espaa
516 PROBLEMAS RESUELTOS DE MECNICA DEL MEDIO CONTINUO

v1 v 2 2 2
+ =0 + =0 2xr = 0 (11.46)
x1 x 2 x12 x 22
r r r r r
2) Partiendo del hecho que el fluido es irrotacional xr v = rot v = = 0 obtenemos que:

e 1 e 2 e 3
r r r r
rot (v ) = xr v = = ijk v k , j e i = 0
x1 x 2 x3
v1 v2 v3 (11.47)
v v v v v v r
= 3 2 e 1 + 1 3 e 2 + 2 1 e 3 = 0
x 2 x3 x3 x1 x1 x 2
Luego:
v 3 v 2

x 2 x 3 0
v1 v3
= 0

x3 x1 0
v v1
2

x1 x 2
Como estamos en el caso 2D, la expresin anterior se resume a:
v 2 v1
=0
x1 x 2
Teniendo en cuenta las relaciones (11.45) concluimos que:
v 2 v1 2 2
=0 + =0 2xr = 0
x1 x 2 x1 x 2
Con lo cual el problema queda planteado a travs de las relaciones:

2xr = 0 ; 2xr = 0 (11.48)

Universidad de Castilla- La Mancha Draft Por: Eduardo W. V. Chaves (2012)


Ciudad Real - Espaa

Bibliografia

ASARO, R.J. & LUBARDA, V.A. (2006). Mechanics of solids and materials. Cambridge University
Press, New York, USA.
BATRA, R. C. (2006). Elements of Continuum Mechanics. John Wiley & Sons Ltd., United
Kingdom.
CASANOVA, J.C. (1993). Ejercicios de elasticidad. Editorial UPV.
CHADWICK, P. (1976). Continuum mechanics concise theory and problems. George Allen & Unwin
Ltd.Great Britain.
CHAVES, E.W.V. (2009). Mecnica del medio continuo: Modelos Constitutivos. CIMNE, Barcelona,
Espaa.
CHAVES, E.W.V. (2007). Mecnica del medio continuo: Conceptos bsicos. CIMNE, Barcelona,
Espaa, (1 Edicin (2007), 2 Edicin (2010), 3 Edicin(2012)).
GOICOLEA, J.M. Mecnica del medio continuo web: http://w3.mecanica.upm.es/mmc-ig/
HOLZAPFEL, G.A. (2000). Nonlinear solid mechanics. John Wiley & Sons Ltd. England.
MASE, G.E. (1977). Teora y problemas de mecnica del medio continuo. McGraw-Hill, USA.
OLIVER, X. & AGELET DE SARACBAR, C. (2000). Mecnica de medios continuos para ingenieros.
Ediciones UPC, Barcelona, Espaa.
OLIVER, X. & AGELET DE SARACBAR, C. (2000). Cuestiones y problemas de mecnica de medios
continuos. Ediciones UPC, Barcelona, Espaa.
ORTIZ BERROCAL, L. (1985). Elasticidad. E.T.S. de Ingenieros Industriales. Litoprint. U.P.
Madrid.
PARKER, D.F. (2003). Fields, Flows and Waves: An introduction to continuum models. Springer-
Verlag London, UK.

Вам также может понравиться